You are on page 1of 666

TEST BANK

to accompany
Stearns/Adas/Schwartz/Gilbert

WORLD CIVILIZATIONS
The Global Experience
Single Volume Edition
Fifth Edition

John Lyons
Joliet Junior College

New York Boston San Francisco


London Toronto Sydney Tokyo Singapore Madrid
Mexico City Munich Paris Cape Town Hong Kong Montreal

This work is protected by United States copyright laws and is provided solely for the use of instructors in
teaching their courses and assessing student learning. Dissemination or sale of any part of this work
(including on the World Wide Web) will destroy the integrity of the work and is not permitted. The work
and materials from it should never be made available to students except by instructors using the
accompanying text in their classes. All recipients of this work are expected to abide by these restrictions
and to honor the intended pedagogical purposes and the needs of other instructors who rely on these
materials.

Test Bank to accompany Stearns/Adas/Schwartz/Gilbert, World Civilizations: The Global Experience, Single
Volume Edition, Fifth Edition
Copyright 2007 Pearson Education, Inc.
All rights reserved. Printed in the United States of America. Instructors may reproduce portions of this book for
classroom use only. All other reproductions are strictly prohibited without prior permission of the publisher, except
in the case of brief quotations embodied in critical articles and reviews.
ISBN: 0-321-44257-1
1 2 3 4 5 6 7 8 9 10OPM09 08 07 06

Contents
Chapter 1 The Neolithic Revolution and the Birth of Civilization

Chapter 2 The Rise of Civilization in the Middle East and Africa

15

Chapter 3 Asias First Civilizations: India and China

31

Chapter 4 Unification and the Consolidation of Civilization in China

47

Chapter 5 Classical Civilization in the Eastern Mediterranean: Persia and Greece

64

Chapter 6 Religious Rivalries in Indias Golden Age

81

Chapter 7

97

Rome and its Empire

Chapter 8 The Peoples and Civilizations of the Americas

113

Chapter 9 The Spread of Civilization and the Movement of Peoples

129

Chapter 10 The End of the Classical Era: World History in Transition,


200-700 C.E.

146

Chapter 11 The First Global Civilization: The Rise and Spread of Islam

164

Chapter 12 Abbasid Decline and the Spread of Islamic Civilization to South and
Southeast Asia

182

Chapter 13 African Civilizations and the Spread of Islam

199

Chapter 14 Civilization in Eastern Europe: Byzantium and Orthodox Europe

217

Chapter 15 A New Civilization Emerges in Western Europe

235

Chapter 16 The Americas on the Eve of Invasion

253

Chapter 17 Abbasid Decline and the Spread of Islamic Civilization to South and
Southeast Asia

270

Chapter 18 The Spread of Chinese Civilizations: Japan, Korea, and Vietnam

287

Chapter 19 The Last Great Nomadic Challenges: From Chinggis Khan to Timur

304

Chapter 20 The West and the Changing World Balance

321

Chapter 21 The World Economy

333

Chapter 22 The Transformation of the West, 1450-1750

349

Chapter 23 The Rise of Russia

366

Chapter 24 Early Latin America

380

Chapter 25 Africa and the Africans in the Age of the Atlantic Slave Trade

397

Chapter 26 The Muslim Empires

414

Chapter 27 Asian Transitions in an Age of Global Change

431

Chapter 28 The Emergence of Industrial Society in the West, 1750-1914

448

Chapter 29 Industrialization and Imperialism: The Making of the European


Global Order

462

Chapter 30 The Consolidation of Latin America, 1830-1920

478

Chapter 31 Civilizations in Crisis: The Ottoman Empire, The Islamic Heartlands


and Qing China

495

Chapter 32 Russia and Japan: Industrialization Outside the West

512

Chapter 33 Descent into the Abyss: World War I and the Crisis of the European
Global Order

530

Chapter 34 The World Between the Wars: Revolutions, Depression, and


Authoritarian Response

546

Chapter 35 A Second Global Conflict and the End of the European World Order

565

Chapter 36 Western Society and Eastern Europe in the Decades of the Cold War

580

st

Chapter 37 Latin America: Revolution and Reaction in the 21 Century

596

Chapter 38 Africa, the Middle East, and Asia in the Era of Independence

612

Chapter 39 Rebirth and Revolution: Nation Building in East Asia and the
Pacific Rim

628

Chapter 40 The End of the Cold War and the Shape of a New Era: World
History 1990-2006

646

Chapter 41 Globalization and Resistance: World History, 1990-2006

657

Chapter1 TheNeolithicRevolutionandtheBirth
ofCivilization
Multiple-ChoiceQuestions
1) Whichofthefollowingisthemostadvancedstageofhumanevolution?
A) HomoErectus
B) HomoAfricanus
C) HomoHabilis
D) Homosapiens
Answer: D
Diff:2
PageRef:9
Topic:Introduction
Skill:Conceptual

2) Thecreationofagriculturalsurplusesresultedin
A) socialstratification.
B) anindustrialrevolution.
C) matrilocalpatternsofmarriage.
D) alackofsocialstratification.
Answer: A
Diff:2
PageRef:18
Topic:Introduction
Skill:Conceptual

3) Theriseoffarmingtookplaceinthe
A) PaleolithicEra.
B) BronzeAge.
C) NeolithicEra.
D) IronAge.
Answer: C
Diff:1
PageRef:17
Topic:Introduction
Skill:Factual

4) TheNeolithicorNewStoneAgelastedfrom
A) 3000to500B.C.E.
B) 8000to3500B.C.E.
C) 12,000to8000B.C.E.
D) 4000to3000B.C.E.
Answer: B
Diff:3
PageRef:14
Topic:Introduction
Skill:Factual

5) Theagriculturalrevolutionallowedwhichofthefollowingtransformationsinhumanculture
totakeplace?
A) Spreadofregularlycultivatedfields
B) Domesticationofanimals
C) Developmentoftowns
D) Alloftheabove
Answer: D
Diff:2
PageRef:17
Topic:Introduction
Skill:Conceptual

6) ThePaleolithicorOldStoneAgehadendedby
A) 5000B.C.E.
B) 8000B.C.E.
C) 3000B.C.E.
D) 15000B.C.E.
Answer: D
Diff:1
PageRef:10
Topic:HumanLifeintheEraofHuntersandGatherers
Skill:Factual

7) WhichofthefollowingattributeswasNOTanadvantageheldby Homosapiens?
A) Opposablethumbs
B) Alargerbrain
C) Superiorspeedandbodystrength
D) Erectposturewhichfreedhands
Answer: C
Diff:1
PageRef:9
Topic:HumanLifeintheEraofHuntersandGatherers
Skill:Conceptual

8) WhatwasoneofthetechnologicaladvancesofthePaleolithicpeoples?
A) Stonetools
B) Caves
C) Monumentalarchitecturalstructures
D) Bronzetools
Answer: A
Diff:1
PageRef:11
Topic:HumanLifeintheEraofHuntersandGatherers
Skill:Factual

9) GiventhelocationandsubjectmatterofPaleolithiccavepaintings,itislikelythattheearlyart
served
A) torelievetheotherwisedrabinteriorsofcaves.
B) asmapstolocategameherds.
C) religiousorritualpurposes.
D) toindicatethelimitedlevelofthinkingofPaleolithicmenandwomen.
Answer: C
Diff:2
PageRef:13
Topic:HumanLifeintheEraofHuntersandGatherers
Skill:Conceptual

10) Thecontinentonwhichhumansoriginatedwas
A) Asia.
B) Africa.
C) Europe.
D) Australia.
Answer: B
Diff:1
PageRef:11
Topic:HumanLifeintheEraofHuntersandGatherers
Skill:Factual

11) BythelatePaleolithicAge,humanshadcolonized
A) allofthecontinentsoftheEasternHemisphere.
B) Africa,Europe,andAsia.
C) allofthecontinentsexceptAustralia.
D) allofthecontinentsexceptAntarctica.
Answer: D
Diff:2
PageRef:11
Topic:HumanLifeintheEraofHuntersandGatherers
Skill:Conceptual

12) MosthumansocietiesinthePaleolithicAgeconsistedof
A) urbanizedcivilizations.
B) smallgroupsofhuntersandgatherers.
C) sedentaryagriculturalgroups.
D) cave-dwellingbands.
Answer: B
Diff:3
PageRef:11
Topic:HumanLifeintheEraofHuntersandGatherers
Skill:Conceptual

13) Huntingandgatheringbandsnormallyconsistedofgroupsofmenandwomennumbering
A) between20and30.
B) between30and50.
C) between50and100.
D) between100and500.
Answer: A
Diff:3
PageRef:12
Topic:HumanLifeintheEraofHuntersandGatherers
Skill:Factual

14) Inhuntingandgatheringbands,labor
A) wassharedequallybyallmembersofthegroup.
B) fellentirelytodominantmaleswithinthegroup.
C) wasdividedaccordingtogender.
D) fellentirelytofemaleswithinthegroup.
Answer: C
Diff:2
PageRef:12
Topic:HumanLifeintheEraofHuntersandGatherers
Skill:Conceptual

15) WhichofthefollowingwasNOTalikelycontributionbyfemalestohuntingandgathering
bands?
A) ApplicationofMedicinalPlants
B) Givingbirthtomaintainthesizeoftheband
C) Huntinggame
D) Gatheringoffoodforbasicsubsistence
Answer: C
Diff:1
PageRef:12
Topic:HumanLifeintheEraofHuntersandGatherers
Skill:Conceptual

16) Twooftheareaswherehumansdevisedmoreintensivehuntingandgatheringpatternsthat
permittedestablishmentofpermanentsettlementswere
A) thechichimeccultureofMexicoandthepastoralsettlementofCatalHuyuk.
B) centralRussiaandtheNatufiancomplexoftheJordanrivervalley.
C) theNatufiancomplexoftheJordanrivervalleyandthePeruviancomplexof
Mesoamerica.
D) centralRussiaandthePeruviancomplexofMesoamerica.
Answer: B
Diff:2
PageRef:13-14
Topic:HumanLifeintheEraofHuntersandGatherers
Skill:Factual

17) TheprobablecauseofthedeclineoftheNatufiancomplexafter9000B.C.E.was
A) aclimaticchangethatcausedtheregiontobecomemorearid.
B) aninvasionbybarbarians.
C) anaturaldisaster,possiblyanearthquake.
D) anunknownepidemicdisease.
Answer: A
Diff:2
PageRef:15
Topic:HumanLifeintheEraofHuntersandGatherers
Skill:Conceptual

18) Thetermforyoungmengoingtolivewiththeirwivesfamiliesis
A) matrilineal.
B) matrilocal.
C) patrilineal.
D) patrilocal.
Answer: B
Diff:2
PageRef:14
Topic:HumanLifeintheEraofHuntersandGatherers
Skill:Factual

19) Thetermforfamilydescentandinheritancetracedthroughthefemalelineis
A) matrilineal.
B) matrilocal.
C) patrilineal.
D) patrilocal.
Answer: A
Diff:2
PageRef:14
Topic:HumanLifeintheEraofHuntersandGatherers
Skill:Factual

20) Lifestylesofintensivehuntingandgatheringgroupslivinginpermanentlocationsremained
precariousbecause
A) theyfailedtodevelopbettersheltersormoresecuresuppliesoffood.
B) theyfailedtoincreasethesizeofthehuntingandgatheringbands.
C) theyremainedvulnerabletoattacksfromnomadicgroups.
D) theyweredependentonparticularanimalsandplantsthatcoulddisappearifchangesin
theclimateoccurred.
Answer: D
Diff:3
PageRef:15
Topic:HumanLifeintheEraofHuntersandGatherers
Skill:Conceptual

21) AllofthefollowingfactorsmayhaveledtotheadoptionofsedentaryagricultureEXCEPT
A) lesslaborwasrequiredforsedentaryagriculturethanforhuntingandgathering.
B) climaticchangesleadingtochangesinthegrazinglocationsofgameanimals.
C) climaticchangesleadingtochangesinplantecology.
D) anincreaseinhumanpopulation.
Answer: A
Diff:2
PageRef:15
Topic:AgricultureandtheOriginsofCivilization
Skill:Conceptual

22) Thetransitiontosedentaryagricultureisreferredtoas
A) thePaleolithicrevolution.
B) theBronzeAgerevolution.
C) theMesolithicrevolution.
D) theNeolithicrevolution.
Answer: D
Diff:1
PageRef:15
Topic:AgricultureandtheOriginsofCivilization
Skill:Factual

23) ThefirstgrainsdomesticatedintheMiddleEastwere
A) wheatandrice.
B) wheatandoats.
C) milletandrice.
D) wheatandbarley.
Answer: D
Diff:2
PageRef:15
Topic:AgricultureandtheOriginsofCivilization
Skill:Factual

24) Whichofthefollowingwerethemostdifficultanimalstodomesticate?
A) HornedCattle
B) Pigs
C) Dogs
D) SheepandGoats
Answer: A
Diff:2
PageRef:16
Topic:AgricultureandtheOriginsofCivilization
Skill:Factual

25) WhichofthefollowingwasNOTanadvantagegainedfromthedomesticationofanimals?
A) Expandedchoiceofmaterialsforclothing
B) Materialsforboats
C) Additionalsourcesofprotein
D) Animalpowerforfarming
Answer: D
Diff:2
PageRef:16
Topic:AgricultureandtheOriginsofCivilization
Skill:Conceptual

26) PastoralismhasNOTthrivedinwhichofthefollowingareas?
A) ThesavannazoneofeastandsouthAfrica
B) Australia
C) CentralAsia
D) TheSudanicbeltsouthoftheSaharadesert
Answer: B
Diff:1
PageRef:16
Topic:AgricultureandtheOriginsofCivilization
Skill:Factual

27) Thegreatestchallengetosedentaryagriculturewasaformofsocialorganizationbasedon
intensiveherdingknownas
A) huntingandgathering.
B) pastoralism.
C) civilization.
D) bands.
Answer: B
Diff:2
PageRef:16
Topic:AgricultureandtheOriginsofCivilization
Skill:Factual

28) WhichofthefollowingregionsdidNOToriginateanagriculturalsystem?
A) Egypt
B) Australia
C) TheHuanghevalleyofChina
D) TheMiddleEast
Answer: B
Diff:3
PageRef:17
Topic:AgricultureandtheOriginsofCivilization
Skill:Conceptual

29) AgriculturalsystemsintheMiddleEast,China,andtheAmericaswereall
A) basedonthesamegrains wheatandbarley.
B) basedonacombinationofdomesticatedplantsandanimals.
C) basedondifferentdomesticatedplantsineachregion.
D) dependentonpastoralformsofsocialorganization.
Answer: C
Diff:2
PageRef:16-17
Topic:TheNeolithicRevolution
Skill:Conceptual

30) Thetransformationsassociatedwiththeagrarianrevolutionoccurred
A) onlyintheMiddleEast,theareawhereagriculturewasfirstinitiated.
B) onlyinAfrica,theareawherehumansfirstdeveloped.
C) onlyintheEasternHemisphere.
D) inalloftheworldwherecivilizationsemerged.
Answer: D
Diff:2
PageRef:17
Topic:TheNeolithicRevolution
Skill:Conceptual

31) TheNeolithicrevolutioncausedthepopulationofhumansto
A) declineasfewerpeoplewereneededtoproducemorefood.
B) staythesameasfewpeoplebecamesedentary.
C) abandonhuntingandgatheringasameansofsubsistence.
D) increasefrom8millionto60or70million.
Answer: D
Diff:2
PageRef:17
Topic:TheNeolithicRevolution
Skill:Conceptual

32) Whichofthefollowingtechnologicalinnovationswasassociatedwiththetransitionto
sedentaryagriculturalcommunities?
A) Giggingsticks,axes,andplows
B) Fire
C) Wheeledvehicles
D) Steel
Answer: A
Diff:2
PageRef:18
Topic:TheNeolithicRevolution
Skill:Conceptual

33) Insedentaryagriculturalcommunities,socialdistinctionswereheightened,
A) andorganizationintodistinctclassesoccurred.
B) butwell-definedsocialstratificationandclassidentitywasnon-existent.
C) buttherewaslittledivisionoflaborandoccupationalspecificity.
D) butnopoliticalelitesemerged.
Answer: B
Diff:3
PageRef:20
Topic:TheNeolithicRevolution
Skill:Conceptual

34) Incomparisontothepositionofwomeninhuntingandgatheringsocieties,thesocialstatusof
womeninsedentaryagriculturalcommunities
A) improved.
B) stayedaboutthesame.
C) allowedthemtomonopolizethereligiousandpoliticalelites.
D) declined.
Answer: D
Diff:2
PageRef:20
Topic:TheNeolithicRevolution
Skill:Conceptual

35) TotheChinese,abarbarianwasanyonewho
A) wasraciallydifferent.
B) didnotspeakChineseoradoptChineseways.
C) didnotfarm.
D) wasnotamemberofthepoliticalandreligiouselite.
Answer: B
Diff:2
PageRef:22
Topic:InDepth:TheIdeaofCivilizationinWorldHistoricalPerspective
Skill:Conceptual

36) Theconceptofbarbarians
A) wasstrictlyaChineseidea.
B) wasonlydevelopedinnineteenth-centuryEuropeanculture.
C) wascommonlyusedtodistinguishbetweencosmopolitan,urban -focusedculturesand
nomadicpeoples.
D) wasdroppedinmoderncultures.
Answer: C
Diff:2
PageRef:22-23
Topic:InDepth:TheIdeaofCivilizationinWorldHistoricalPerspective
Skill:Conceptual

37) Peoplereferredtoasbarbarianswereoften
A) membersofurbanizedcultures.
B) membersofhunter-gathererbands.
C) sedentaryagriculturalists.
D) pastoralherdsmen.
Answer: D
Diff:1
PageRef:22-23
Topic:InDepth:TheIdeaofCivilizationinWorldHistoricalPerspective
Skill:Conceptual

38) Theconceptofcivilizationbasedonracialcharacteristicswas
A) commonamongtheChinese.
B) developedbythinkersinwesternEuropeintheeighteenthandnineteenthcenturies
C) adoptedbytheRomansandtheGreeks.
D) nevertakenseriously.
Answer: B
Diff:2
PageRef:23
Topic:InDepth:TheIdeaofCivilizationinWorldHistoricalPerspective
Skill:Conceptual

39) Racialconceptsofcivilizationwereutilizedtojustify
A) nineteenth-centuryimperialism.
B) theestablishmentoftheShangdynastyinChina.
C) theRomanempire.
D) theenslavementofconqueredpeoplesintheancientworld.
Answer: A
Diff:1
PageRef:23
Topic:InDepth:TheIdeaofCivilizationinWorldHistoricalPerspective
Skill:Factual

10

40) Combinationsoftheideas,objects,andpatternsofbehaviorthatresultfrom
humansocialinteractionarereferredtoas
A) culture.
B) society.
C) civilization.
D) socialstratification.
Answer: A
Diff:3
PageRef:23
Topic:InDepth:TheIdeaofCivilizationinWorldHistoricalPerspective
Skill:Conceptual

41) Byabout7000B.C.E.,techniquesofagriculturalproductionintheMiddleEasthadreacheda
levelthat
A) permittedtheestablishmentofthefirsttowns.
B) permittedtheestablishmentofhugecities.
C) forcedareturntohuntingandgathering.
D) allowedmostpeopletoengageinotheroccupations.
Answer: A
Diff:2
PageRef:20
Topic:TheFirstTowns:SeedbedsofCivilization
Skill:Conceptual

42) IndenselypopulatedMiddleEasternagriculturalsettlements,occupationalspecializationand
political-militaryelites
A) advancedsignificantly.
B) failedtodevelop.
C) wereretardedbythegeneralfailureoforganizedreligion.
D) remainedatthelevelofhuntingandgatheringsocieties.
Answer: A
Diff:2
PageRef:20
Topic:TheFirstTowns:SeedbedsofCivilization
Skill:Conceptual

11

43) WhichofthefollowingdevelopmentswasNOTakeyingredientofearlyMiddleEastern
civilization?
A) Theexistenceofspecializednon-farmingproducers
B) Theexistenceofnon-farmingpoliticalandreligiouselites
C) Craftssuchaspotteryandmetalworking
D) Heavyrelianceonpastoralism
Answer: D
Diff:1
PageRef:20-21
Topic:TheFirstTowns:SeedbedsofCivilization
Skill:Conceptual

44) Jericho,oneoftheearlytowncenters,waslocated
A) inEgypt.
B) inAsiaMinor.
C) intheJordanrivervalley.
D) intheHuangherivervalley.
Answer: C
Diff:1
PageRef:20
Topic:TheFirstTowns:SeedbedsofCivilization
Skill:Factual

45) TheeconomyofJerichowasbasedprimarilyon
A) huntingandtrade.
B) pastoralism.
C) themanufactureofflinttools.
D) cultivationofbarleyandwheat.
Answer: D
Diff:2
PageRef:20
Topic:TheFirstTowns:SeedbedsofCivilization
Skill:Factual

46) CatalHuyuk,oneoftheearlyagriculturalcommunities,waslocated
A) inEgypt.
B) insouthernTurkey.
C) intheJordanrivervalley.
D) intheHuangherivervalley.
Answer: B
Diff:1
PageRef:21
Topic:TheFirstTowns:SeedbedsofCivilization
Skill:Factual

12

47) IncomparisontoJericho,CatalHuyukhadapopulation
A) thatwaslargerandmorediversified.
B) one-halftoone-thirdaslarge.
C) thatwasaboutthesame.
D) nearly10timesaslarge.
Answer: A
Diff:3
PageRef:21
Topic:TheFirstTowns:SeedbedsofCivilization
Skill:Factual

48) ThemanyreligiousshrinesatCatalHuyukindicatetheexistenceof
A) humansacrificeonamassivescale.
B) therejectionoffemalefertilitycults.
C) apowerfulpriesthood.
D) povertyamongthecitizensofthetown.
Answer: C
Diff:2
PageRef:21
Topic:TheFirstTowns:SeedbedsofCivilization
Skill:Conceptual

49) WhichofthefollowingwasNOTatransformationassociatedwiththefourthmillennium
B.C.E.(4000to3000B.C.E.)?
A) Increaseduseoftheplow
B) Theharnessingofanimalpoweronallcontinents
C) Useofbronzeforweapons
D) Thedevelopmentofwriting
Answer: B
Diff:3
PageRef:21-22
Topic:Conclusion:TheWatershedoftheFourthMillenniumB.C.E.
Skill:Conceptual

50) WhatwastheheartoftheNeolithicRevolutionthatbecamethebasisforthespreadof
humansocieties?
A) Innovativetechnologiesandmodesofagrarianproduction
B) Religion
C) Hunting
D) Architecture
Answer: A
Diff:3
PageRef:23
Topic:Conclusion:TheWatershedoftheFourthMillenniumB.C.E.
Skill:Conceptual

13

EssayQuestions
1) WhatwerethemostsignificanthumanachievementsbeforetheNeolithicperiod?
Answer: Developmentoffire,tools,language,art,agricultureandtrade.Thecolonizationof
mostareasoftheworldandtheestablishmentofsmallcommunities.
Diff:2
PageRef:8-15
Topic:HumanLifeintheEraofHuntersandGatherers
Skill:Conceptual

2) ComparethePaleolithicAge(theOldStoneAge)andtheNeolithicAge(theNewStoneAge)
intermsofmeansofsubsistenceandsocialorganization.
Answer: Paleolithic:organizationinbands;dependenceonhuntingandgathering;some
intensivehuntingandgatheringcommunities.Neolithic:domesticationofplantsand
animals;creationofsedentaryagriculturalcommunities;moresocialstratificationand
occupationalspecialization.
Diff:2
PageRef:8-20
Topic:AgricultureandtheOriginsofCivilization
Skill:Conceptual

3) ComparethestatusofwomeninthePaleolithicAgewiththatofwomeninthesedentary
agriculturalcommunitiesoftheNeolithicera.
Answer: Paleolithic:Highsocialstatus;significanteconomiccontributiongathering,medicinal,
reproductive;centerofreligiousfertilitycults.Neolithic:reducedstatus;males
dominatedmostofproductiveeconomy,politicalandreligiouselites.
Diff:2
PageRef:9-20
Topic:TheNeolithicRevolution
Skill:Conceptual

4) Definecivilization.Howhastheconceptofcivilizationchangedovertheages?
Answer: Civilization:SeedefinitioninTerms.Changes:originallyaculturaldefinition,favored
urban-focusedsocieties;seventeenthandeighteenthcenturies,creationofhierarchiesof
civilization;nineteenthcenturycreationofraciallybasedhierarchyofcivilization.
Diff:2
PageRef:22-23
Topic:In-Depth:TheIdeaofCivilizationinWorldHistoricalPerspective
Skill:Conceptual

5) HowdidtheexistenceofsedentaryagriculturaltownssuchasJerichoandCatalHuyuk
contributetothedevelopmentofcivilization?
Answer: Setbasicpatternsforfortification(walledcities);layout planneddomestic
architecture,ritualcenters;existenceofpowerfulpoliticalandreligiouselites;existence
oftrade;existenceofsocialstratificationandeconomicspecialization.
Diff:2
PageRef:20-23
Topic:TheFirstTowns:SeedbedsofCivilization
Skill:Conceptual

14

Chapter2 TheRiseofCivilizationintheMiddleEast
andAfrica
Multiple-ChoiceQuestions
1) AtwhatdatedidthefirstcivilizationsemergeintheMiddleEast?
A) 2000B.C.E.
B) 1000B.C.E.
C) 5000B.C.E.
D) 4000B.C.E.
Answer: D
Diff:2
PageRef:27
Topic:Introduction
Skill:Factual

2) ThereasonthatcivilizationappearedatanearlydateintheMiddleEastwasthat
A) settledagriculture,dependentonorganizedirrigationsystems,hademergedthere.
B) therewasahigherconcentrationofhumansinthisregionthananywhereelse.
C) therewerenohuntingandgatheringsocietiesintheregion.
D) itwastheonlyregionintheworldtodevelopsedentaryagriculture.
Answer: A
Diff:1
PageRef:27
Topic:TheMiddleEastby4000B.C.E.
Skill:Conceptual

3) Alongwhichriverdidthefirstcivilizationemerge?
A) PersianGulf
B) Huanghe
C) Nile
D) TigrisandEuphrates
Answer: D
Diff:2
PageRef:28
Topic:TheMiddleEastby4000B.C.E.
Skill:Conceptual

15

4) Whichofthefollowingfacilitatedthedevelopmentofthefirstcivilizations?
A) Irrigationsystems
B) Supplementaryhuntingandgathering
C) Monumentalarchitecture
D) Roadsandtransportationsystems
Answer: A
Diff:2
PageRef:27
Topic:TheMiddleEastby4000B.C.E.
Skill:Factual

5) Whatwasthefirstrivervalleycivilization?
A) Egypt
B) Sumer
C) Mesoamerica
D) Anatolia
Answer: B
Diff:1
PageRef:27
Topic:Introduction
Skill:Factual

6) Technologicalinnovationsoccurringbetween6000and4000B.C.E.preparedthewayfor
civilizationby
A) leadingtoanindustrialrevolution.
B) ensuringmoreconsistentsurplusesoffoodandnecessaryproducts.
C) allowingtheeliminationofhumanlaborintheproductionoffood.
D) removingthenecessityofsocialstratification.
Answer: B
Diff:2
PageRef:28
Topic:TheMiddleEastby4000B.C.E.
Skill:Conceptual

7) WhichofthefollowingwasNOTatechnologicalinnovationleadingtocivilization?
A) Wheel
B) Useofbronze
C) Writing
D) Useofanimalbones
Answer: D
Diff:2
PageRef:27
Topic:TheMiddleEastby4000B.C.E.
Skill:Conceptual

16

8) ThefirstcivilizationtoemergeintheMiddleEastwascreatedbythe
A) Assyrians.
B) Egyptians.
C) Babylonians.
D) Sumerians.
Answer: D
Diff:1
PageRef:27
Topic:CivilizationinMesopotamia
Skill:Factual

9) Before3000B.C.E.oneaccomplishmentoftheSumerianswasthe
A) conquestofEgypt.
B) establishmentofacentralizedmonarchy.
C) developmentofreligiouscentersandtemplecomplexes.
D) dominationofthenorthernTigris-Euphratesvalley.
Answer: C
Diff:2
PageRef:30
Topic:CivilizationinMesopotamia
Skill:Conceptual

10) WhichofthefollowingisNOTanaccuratedescriptionoftheSumerianformofwriting?
A) Itwasdoneonclaytablets.
B) Itwasusedtokeeplistsofgoodsandreceipts.
C) Priestsandofficialsdevelopedanear-monopolyoftheskill.
D) Itnevermadethetransitionfrompictogramstophoneticsymbols.
Answer: D
Diff:2
PageRef:29
Topic:CivilizationinMesopotamia
Skill:Conceptual

11) Sumerianwritingiscalled
A) hieroglyphics.
B) LinearA.
C) cuneiform.
D) pictographic.
Answer: C
Diff:1
PageRef:29
Topic:CivilizationinMesopotamia
Skill:Factual

17

12) TheprincipleofpoliticalorganizationinSumeriancivilizationwas
A) tightlycontrollednation-states.
B) tightlyorganizedcity-states.
C) huntingandgatheringbands.
D) sedentaryagriculturalvillages.
Answer: B
Diff:2
PageRef:29
Topic:CivilizationinMesopotamia
Skill:Conceptual

13) TheSumeriangovernmentstookoverallofthefollowingpoliticalfunctionsEXCEPT
A) thecreationofauniformcodeoflaw.
B) thedefinitionofboundaries.
C) theregulationofreligionandjustice.
D) defenseandwarfare.
Answer: A
Diff:2
PageRef:29
Topic:CivilizationinMesopotamia
Skill:Conceptual

14) ThescopeofSumeriangovernmentwasoftenmore
A) imperialthanregional.
B) republicanthanmonarchic.
C) democraticthanoligarchic.
D) regionalthanimperial.
Answer: D
Diff:2
PageRef:29
Topic:CivilizationinMesopotamia
Skill:Conceptual

15) UrbanpopulationsinSumeriancitiesrangedfrom
A) 5000to10,000.
B) 10,000to30,000.
C) 10,000to50,000.
D) 10,000to70,000.
Answer: D
Diff:2
PageRef:29
Topic:CivilizationinMesopotamia
Skill:Factual

18

16) TheSumerianscreatedamoreelaboratecultureincludingtheworldsfirstheroicepic,
A) theAeneid.
B) theEpicofGilgamesh.
C) theIliad.
D) theGreatFlood.
Answer: B
Diff:1
PageRef:29
Topic:CivilizationinMesopotamia
Skill:Factual

17) Whatisanimism?
A) Amassivetower
B) Templeswithstatuesandpaintedfrescos
C) Theideasthatadivineforcelieswithinnaturalobjects
D) Mathematicaladvances
Answer: C
Diff:2
PageRef:30
Topic:CivilizationinMesopotamia
Skill:Conceptual

18) ZigguratswereSumerian
A) wheeledvehicles.
B) irrigationsystems.
C) slaves.
D) towers.
Answer: D
Diff:1
PageRef:30
Topic:CivilizationinMesopotamia
Skill:Factual

19) WhichofthefollowingstatementsaboutSumerianreligioniscorrect?
A) TheSumeriansbelievedinasinglegodwhoweighedallsoulsatdeath.
B) TheSumeriansbelievedinajoyousafterlifefortherighteous.
C) SomeofthebasicelementsofSumerianreligioncontinuedtohaveforceinJudaism,
Christianity,andIslam.
D) Sumerianreligionrejectedanimisminfavorofabstractanddistantdeities.
Answer: C
Diff:3
PageRef:30
Topic:CivilizationinMesopotamia
Skill:Conceptual

19

20) WhatwasdistinctiveaboutSumeriancivilizationatthetime?
A) Socialstratificationandagriculturalsurpluses
B) Aclearly-definedgovernmentandlargecities
C) Occupationalspecializationandpatriarchy
D) Pastoralismandmonumentalarchitecture
Answer: B
Diff:3
PageRef:31
Topic:CivilizationinMesopotamia
Skill:Conceptual

21) WhichstatementistrueconcerningHammurabisLawCode?
A) Everyonewasequalbeforethelaw
B) Menandwomenfacedthesamepunishmentsforcommittingthesamecrimes
C) ReligiousormagicalbeliefsdonotplayaroleintheCode
D) Punishmentswereoftenharsh
Answer: D
Diff:2
PageRef:34
Topic:CivilizationinMesopotamia
Skill:Conceptual

22) AllofthefollowingwereimportantcontributionsofSumerianscienceEXCEPT
A) asystemofnumbersbasedonunitsof12,60,and360.
B) chartsofmajorconstellations.
C) theearliestanatomicalatlases.
D) acommitmenttoobservationandabstractthought.
Answer: C
Diff:3
PageRef:29
Topic:CivilizationinMesopotamia
Skill:Conceptual

20

23) Whichofthefollowingstatementsconcerningtherelationshipofcivilizationandwritingis
correct?
A) Writingallowedforthekeepingofcomplexrecordsandtheestablishmentofformal
bureaucracies.
B) Writingwasmoreimportanttothedevelopmentofcivilizationthansedentary
agriculture.
C) Mostcivilizationsdevelopedwithoutwriting.
D) Nosophisticatedcivilizationdevelopedwithoutasystemofwriting.
Answer: A
Diff:2
PageRef:31
Topic:CivilizationinMesopotamia
Skill:Conceptual

24) WhichofthefollowingisNOTadrawbacktocivilization?
A) Aggressivebehaviorandwarfare
B) Greaterinequalitybetweenmenandwomen
C) Strictersocialstratificationintoclassesorcastes
D) Decreasedwealthandfoodsurpluses
Answer: D
Diff:2
PageRef:31
Topic:CivilizationinMesopotamia
Skill:Conceptual

25) ThefirstrulertocreateanempireinMesopotamiawas
A) GilgameshofErech.
B) DumuziofUr.
C) SargonIofAkkad.
D) NebuchadnezzarofBabylon.
Answer: C
Diff:1
PageRef:31
Topic:CivilizationinMesopotamia
Skill:Factual

21

26) By2000B.C.E.Mesopotamia
A) wascentralizedinasingle,powerfulempireundertheBabylonians.
B) wasconqueredbytheHittites.
C) hadrevertedtoregionalcity-states.
D) hadregressedtothelevelofhuntingandgatheringsocieties.
Answer: C
Diff:3
PageRef:32
Topic:CivilizationinMesopotamia
Skill:Conceptual

27) ThepoliticalunityofallofMesopotamiawasfirstachievedin1800B.C.E.bythe
A) Assyrians.
B) Sumerians.
C) Hittites.
D) Babylonians.
Answer: D
Diff:2
PageRef:32
Topic:CivilizationinMesopotamia
Skill:Factual

28) OneofthemostsignificantaccomplishmentsofHammurabiwasthe
A) conquestofEgypt.
B) codificationofthelawsofMesopotamia.
C) constructionofthemonumentalarchitectureofHattusas.
D) destructionoftheBabylonianempire.
Answer: B
Diff:1
PageRef:33
Topic:CivilizationinMesopotamia
Skill:Factual

22

29) WhichofthefollowingstatementsabouttheBabylonianempireismostaccurate?
A) Althoughwarlike,theBabylonianswereculturallyprimitive.
B) OfallthesuccessorsoftheSumerians,theBabyloniansconstructedthemostelaborate
culture.
C) TheBabyloniansmadesomecontributionsinliterature,butwerelesscapablein
astronomyandmathematics.
D) Babylonianinfluencewaslimitedbecauseofitsfailuretoestablishacommoncultural
zoneinMesopotamia.
Answer: B
Diff:3
PageRef:33
Topic:CivilizationinMesopotamia
Skill:Conceptual

30) WhichofthefollowingpeopleswasresponsiblefortheconquestoftheBabylonianempireca.
1600B.C.E.?
A) Sumerians
B) Assyrians
C) Hittites
D) Egyptians
Answer: C
Diff:1
PageRef:35
Topic:CivilizationinMesopotamia
Skill:Factual

31) Egyptiancivilizationdevelopedalongwhichriver?
A) TheNile.
B) TheRedSea.
C) TheTigrisandEuphrates
D) TheHuanghe
Answer: A
Diff:2
PageRef:35
Topic:AncientEgypt
Skill:Conceptual

23

32) ThefirstpharaohofEgyptwas
A) Amenhotep.
B) Gilgamesh.
C) Akhenaton.
D) Narmer.
Answer: D
Diff:1
PageRef:36
Topic:AncientEgypt
Skill:Factual

33) WhichofthefollowingwascharacteristicofthepowerofthepharaohinEgyptiancivilization?
A) Therulerhadauthoritylimitedtohisowncity-state
B) Thepharaohspowerwastruncatedbythelackofaformalbureaucracy
C) HewasregardedasagodwithpowertoassureprosperityandcontroltheNile
D) Thepharaohsauthoritywaslimitedbyindependentregionalgovernors
Answer: C
Diff:2
PageRef:36
Topic:AncientEgypt
Skill:Conceptual

34) WhichofthefollowingwasNOTcharacteristicofEgyptianreligiousbeliefconcerningdeath?
A) Constructionofmassivefuneralmonuments
B) Mummification
C) Deathritualsthatextendedorganizationtotheafterlife
D) Deathritualsthatconsignedmostmembersofsocietytoendlessdarkness
Answer: D
Diff:1
PageRef:36-37
Topic:AncientEgypt
Skill:Conceptual

35) TheEgyptianformofwritingiscalled
A) cuneiform.
B) hieroglyphic.
C) LinearA.
D) cursive.
Answer: B
Diff:1
PageRef:37
Topic:AncientEgypt
Skill:Factual

24

36) OneofthemajordifferencesbetweenEgyptianliterarytraditionandMesopotamianliterary
workswas
A) thelackofanepicliterarytraditioninEgypt.
B) thelackofanepicliterarytraditioninMesopotamia.
C) thelesspictographicnatureofEgyptianwriting.
D) theEgyptianuseofclaytablets.
Answer: A
Diff:2
PageRef:37
Topic:AncientEgypt
Skill:Conceptual

37) ThepillarofEgyptianculturewas
A) astronomy.
B) mathematics.
C) religion.
D) appliedtechnology.
Answer: C
Diff:2
PageRef:37
Topic:AncientEgypt
Skill:Conceptual

38) ThefairlysteadydeclineofEgyptiancivilizationbeganaround
A) 2000B.C.E.
B) 1500B.C.E.
C) 1150B.C.E.
D) 900B.C.E.
Answer: C
Diff:3
PageRef:38
Topic:AncientEgypt
Skill:Factual

39) ComparedtoMesopotamiancivilization,Egyptiancivilizationwas
A) lessstableduetomanyforeignincursions.
B) lessstableduetomanypoliticalchanges.
C) morestableduetofewforeignincursions.
D) morestableduetobrutalrepressivemeasures.
Answer: C
Diff:2
PageRef:39
Topic:AncientEgypt
Skill:Conceptual

25

40) WhichofthefollowingstatementsbestsummarizestheeconomicdifferencesbetweenEgypt
andMesopotamia?
A) UnlikeMesopotamia,Egyptwasheavilydependentontrade.
B) Mesopotamiancivilizationhadnomerchantclass.
C) Egyptiancivilizationhadnofarming.
D) UnlikeEgypt,Mesopotamiawasheavilydependentontrade.
Answer: D
Diff:3
PageRef:40
Topic:EgyptandMesopotamiaCompared
Skill:Conceptual

41) Whichofthefollowingstatementsbestexpressesoneofthemostdistinctivesimilaritiesof
EgyptianandMesopotamiancivilizations?
A) Bothwerestronglystratifiedwithdistinctiveclassesofnobles,priests,peasants,and
slaves.
B) Bothhadreligionsthatfeaturedemphasisonpreparationforandritualsrelatingtothe
afterlife.
C) Bothfeaturedstrongregionalgovernmentattheexpenseofcity-states.
D) Neitherculturewasabletoachieveadvancesinscienceortechnology.
Answer: A
Diff:3
PageRef:40
Topic:EgyptandMesopotamiaCompared
Skill:Conceptual

42) Agriculturalsocietiesinwhichmencontrolledmostorallpropertywerenormally
A) patriarchal.
B) martrilineal.
C) matriarchal.
D) sociallyunstratified.
Answer: A
Diff:1
PageRef:38
Topic:EgyptandMesopotamiaCompared
Skill:Conceptual

26

43) Whydidsexualinequalityexistinsomanysocieties?
A) Womenhadnoreligiousroles
B) Thestatusofwomenreflectedthevalueoftheirlaborinagriculturalsociety
C) Nosocietiescontinuedtopracticematrilineardescentandinheritance
D) Therewerenofemalerulersinancientsocieties
Answer: B
Diff:2
PageRef:38-39
Topic:EgyptandMesopotamiaCompared
Skill:Conceptual

44) WhatwasthefirstAfricanstateotherthanEgypttoemerge?
A) Minoan
B) Phoenicia
C) Assyria
D) Kush
Answer: D
Diff:1
PageRef:40
Topic:CivilizationCentersinAfricaandtheEasternMediterranean
Skill:Factual

45) WhichofthefollowingwasNOTproducedbyKushiteculture?
A) Hieroglyphicwriting
B) Ironmaking
C) Bronzeweapons
D) Strongmonarchies
Answer: C
Diff:2
PageRef:41
Topic:CivilizationCentersinAfricaandtheEasternMediterranean
Skill:Conceptual

46) ThemostsignificantcontributionoftheearlyJewishstatetoWesterncivilizationwas
A) itssenseofadivineplan.
B) itsuseofelaborateritualsconcerningdeath.
C) itsemphasisondivinekingship.
D) thedevelopmentofastrongmonotheisticreligion.
Answer: D
Diff:1
PageRef:42
Topic:CivilizationCentersinAfricaandtheEasternMediterranean
Skill:Conceptual

27

47) WhichofthefollowingpeoplessuccessfullycolonizedthewesternMediterranean?
A) Minoans
B) Hittites
C) Phoenicians
D) Assyrians
Answer: C
Diff:1
PageRef:43
Topic:CivilizationCentersinAfricaandtheEasternMediterranean
Skill:Factual

48) Whichofthefollowingstatementsmostaccuratelydescribestheimpactofthe
Indo-EuropeansontheancientcivilizationsoftheMiddleEast?
A) TheyrapidlyadaptedtoMesopotamiancultureandsohadminimalimpact.
B) Theycameinsuccessivewaves,disruptedthecivilizationsofEgyptandMesopotamia,
andintroducedtheuseofiron
C) AlthoughtheIndo-Europeaninvasionshadatemporaryimpact,theancientcivilizations
soonrecovered
D) TheIndo-EuropeanimpactwaslimitedtoAsiaMinorandtheGreekmainland
Answer: B
Diff:3
PageRef:43
Topic:TheEndoftheEarlyCivilizationPeriod
Skill:Conceptual

49) WhichofthefollowingwasNOTaheritageoftheancientcivilizations?
A) ThestoryoftheGreatFlood
B) Architecturalforms
C) Matriarchy
D) Useofmoney
Answer: C
Diff:3
PageRef:43
Topic:Conclusion:TheIssueofHeritage
Skill:Conceptual

28

50) WhichofthefollowingwereNOTlatercivilizationsinfluencedbytheMesopotamians?
A) India
B) Greece
C) Christianity
D) Islam
Answer: A
Diff:3
PageRef:43
Topic:Conclusion:TheIssueofHeritage
Skill:Conceptual

EssayQuestions
1) WhydidcivilizationemergeintheTigris-Euphratesvalley?WhatwasthenatureofSumerian
civilization?
Answer: Creationoflarge-scaleirrigationprojects;technologicaladvances;introductionof
writing;politicalorganizationintocity-states;creationofelaboratecultureepic
literature,astronomy,andscience.
Diff:2
PageRef:27-30
Topic:CivilizationinMesopotamia
Skill:Conceptual

2) HowdidSumeriansocietyextendtheconceptofcivilization?HowsignificantwereSumerian
contributions?
Answer: Establishmentoforganizedgovernments;cities,whichallowedamassingofwealthand
power,exchangeofideas,technologicaldevelopment,specializationinmanufacture
andtrade;writing,whichallowedrecords,developmentofbureaucracies,more
elaborateintellectuallife,readiertransmissionofvaluesystems.
Diff:2
PageRef:30-31
Topic:CivilizationinMesopotamia
Skill:Conceptual

3) CompareandcontrastEgyptianandMesopotamiancivilization.
Answer: Similarities:emphasisonsocialstratification,culturaldevelopmentinscience,
conservatismtowardsocialchange;contrasts:differentformsofgovernment,Egypt
lackedliterarytradition,Egypthadmoremonumentalarchitecture,moretechnological
advanceinMesopotamia,statusofwomenhigherinEgypt,Egyptmorestable.
Diff:2
PageRef:39-40
Topic:CivilizationinMesopotamia
Skill:Conceptual

29

4) InwhatwaysweretheKush,MinoansandPhoeniciansinfluencedbyEgyptand
Mesopotamia?
Answer: Kush:politicalorganization,writingtakenfromEgypt;Minoans:tookwriting,
monumentalarchitecture,politicalorganizationfromEgypt;Phoenicians:writingfrom
Mesopotamia,andEgyptiannumberingsystem.
Diff:3
PageRef:40-43
Topic:CivilizationCentersinAfricaandtheEasternMediterranean
Skill:Conceptual

5) WhatwasthetechnicalandculturalheritageofthecivilizationsoftheancientNearEast?
Answer: Technical:writing,calendars,scientificandastronomicalknowledge,irrigation,useof
iron,potterswheel,wheeledvehicles,useofmoney,writtenlawcodes;cultural:story
ofGreatFlood,vocabulary,musicinstrumentsandscales,architecturalforms,political
forms,distinctivecosmology,patriarchalsociety.
Diff:2
PageRef:43-44
Topic:Conclusion:TheIssueofHeritage
Skill:Conceptual

30

Chapter3 AsiasFirstCivilizations:IndiaandChina
Multiple-ChoiceQuestions
1) WhatwasoneofthestrongestsimilaritiesbetweentheEgyptian,Mesopotamian,Harappan,
andChinesecivilizations?
A) Locatedneargreatriversystems
B) Lackofawrittenlanguage
C) City-statepoliticalorganization
D) Lackofcentralizedauthority
Answer: A
Diff:1
PageRef:46
Topic:Introduction
Skill:Conceptual

2) WhatwasoneofthegreatestdifferencesbetweenHarappancivilizationand
Chinesecivilization?
A) Useofwrittenlanguage
B) Twogreatcapitals
C) Failuretoprovidethebasisforacontinuouscivilization
D) Significanceofshamansandpriests
Answer: C
Diff:2
PageRef:48
Topic:Introduction
Skill:Conceptual

3) TheHarappancivilizationdevelopedalongwhatrivervalleysystem?
A) Nile
B) Indus
C) Yellow
D) Ganges
Answer: B
Diff:1
PageRef:49
Topic:TheIndusValleyandSouthAsianCivilization
Skill:Factual

31

4) SeasonalwindsthatcarryraintotheIndiansubcontinentarecalled
A) typhoons.
B) hurricanes.
C) Saharahighs.
D) monsoons.
Answer: D
Diff:2
PageRef:49
Topic:TheIndusValleyandSouthAsianCivilization
Skill:Factual

5) WhatwasthefirstcivilizationtoemergeintheIndiansubcontinent?
A) Aryan
B) Harappan
C) Shang
D) Assyrian
Answer: B
Diff:1
PageRef:49
Topic:TheIndusValleyandSouthAsianCivilization
Skill:Factual

6) WhichmountainrangecarvedouttheIndusRiversystem?
A) TheAndes
B) TheKhingan
C) TheVindhya
D) TheHimalayas
Answer: D
Diff:3
PageRef:49
Topic:TheIndusValleyandSouthAsianCivilization
Skill:Factual

7) WhatagriculturalproductsdidHarappancivilizationcultivate?
A) Wheat,rye,andpotatoes
B) Wheat,corn,andpotatoes
C) Wheat,rye,andcorn
D) Wheat,rye,andpeas
Answer: D
Diff:2
PageRef:50
Topic:TheIndusValleyandSouthAsianCivilization
Skill:Conceptual

32

8) ThetwomajorcitiesofHarappancivilizationwereHarappaand
A) Loyang.
B) Nineveh.
C) Mohenjo-Daro.
D) Memphis.
Answer: C
Diff:1
PageRef:49
Topic:TheIndusValleyandSouthAsianCivilization
Skill:Factual

9) WhichareasdidHarappancivilizationtradewith?
A) MesopotamiaandwesternEurope
B) MesopotamiaandChina
C) ChinaandwesternEurope
D) WesternEuropeandtheAmericas
Answer: B
Diff:2
PageRef:50-51
Topic:TheIndusValleyandSouthAsianCivilization
Skill:Conceptual

10) WhichofthefollowingcharacteristicswasNOTtypicalofthecapitalsofHarappan
civilization?
A) Massivecitywalls
B) Plannedlayoutofsquaregridpattern
C) Variationindomesticarchitecture
D) Citadelsforadministrationanddefense
Answer: C
Diff:2
PageRef:50
Topic:TheIndusValleyandSouthAsianCivilization
Skill:Conceptual

11) AllofthefollowingwerecharacteristicofHarappancitadelsEXCEPT
A) publicbathingareas.
B) placesofworship.
C) talltowersorziggurats.
D) livingquartersfortheelite.
Answer: C
Diff:2
PageRef:50
Topic:TheIndusValleyandSouthAsianCivilization
Skill:Conceptual

33

12) Abouttheonlysimilaritybetweenthecitadelsanddomesticarchitecturewas
A) ornateexternaldecoration.
B) lackofstandardization.
C) constructionofwoodandstone.
D) presenceofbathingfacilities.
Answer: D
Diff:2
PageRef:50-51
Topic:TheIndusValleyandSouthAsianCivilization
Skill:Conceptual

13) WhichofthefollowingisNOTcharacteristicoftheHarappaneconomy?
A) Irrigationsystems
B) Absenceofevidencefortrade
C) Avariedagriculturalsystembasedonwheat,rye,peas,andrice
D) Cultivationofcotton
Answer: B
Diff:2
PageRef:50-51
Topic:TheIndusValleyandSouthAsianCivilization
Skill:Conceptual

14) WhywastheHarappanmaterialculturerelativelyunsophisticated?
A) Theydidnotestablishcommercialrelationshipswithothercivilizations.
B) Theylackedthetypeofcentralizedgovernmentthatcoulddisseminatetechnological
improvements.
C) Despitecontactswithothercivilizations,theywereintenselyconservativeandresistant
tochange.
D) Constantnaturaldisasters(floods)hinderedtechnologicalinnovation.
Answer: C
Diff:3
PageRef:51
Topic:TheIndusValleyandSouthAsianCivilization
Skill:Conceptual

15) WhatappearstohavebeenthefocusofHarappanreligion?
A) Fertility
B) Warfare
C) Theafterlife
D) Monotheism
Answer: A
Diff:2
PageRef:51
Topic:TheIndusValleyandSouthAsianCivilization
Skill:Conceptual

34

16) WhatgroupappearstohavebeeninchargeofHarappansociety?
A) Themilitaryelite
B) Acommercialelite
C) Astrictlyfemaledynasty
D) Aneliteofpriests
Answer: D
Diff:2
PageRef:51
Topic:TheIndusValleyandSouthAsianCivilization
Skill:Factual

17) WhichofthefollowingwasNOTalikelycauseofthedemiseofHarappancivilization?
A) Suddendefeatatthehandsofnomadicinvaders
B) Naturaldisasters(floods)
C) Changesintheclimate
D) Lossofcontrolbytheelite
Answer: A
Diff:3
PageRef:51
Topic:TheIndusValleyandSouthAsianCivilization
Skill:Conceptual

18) TheAryansweremembersofwhatlinguisticgroup?
A) Egyptian
B) Semitic
C) Indo-European
D) Hamitic
Answer: C
Diff:2
PageRef:52
Topic:TheAryanIncursionsandEarlyAryanSociety
Skill:Factual

19) WhatwasSanskrit?
A) AnAryangod
B) Alanguage
C) AnAryanleader
D) Religioustexts
Answer: B
Diff:2
PageRef:52
Topic:TheAryanIncursionsandEarlyAryanSociety
Skill:Factual

35

20) TheVedaswereAryan
A) hymnsoriginallyusedforanimalsacrifices,entertainment,andhistory.
B) twinfemaledeitiesdedicatedtofertilityrites.
C) twinmaledeitiesdedicatedtowarandhorsemanship.
D) horsescriticaltotheirmilitarydominanceoverthenativeHarappancivilization.
Answer: A
Diff:2
PageRef:52
Topic:TheAryanInvasionsandEarlyAryanSociety
Skill:Conceptual

21) ThechiefdeityoftheAryanswas
A) Indra,godofbattleandlightning.
B) anunnamedmalegodwithlonghorns.
C) Tian,godofHeaven.
D) Veda,goddessoffertility.
Answer: A
Diff:1
PageRef:52
Topic:TheAryanIncursionsandEarlyAryanSociety
Skill:Factual

22) TheonetechnologyinwhichtheAryanssurpassedtheHarappanswas
A) agriculturaltools.
B) domesticarchitecture.
C) monumentalarchitecture.
D) weapons.
Answer: D
Diff:1
PageRef:53
Topic:TheAryanIncursionsandEarlyAryanSociety
Skill:Conceptual

36

23) WhatsortofrelationshipsdidtheAryansmaintainwiththeconqueredpopulation?
A) TheAryansimmediatelyincorporatedallmembersofconqueredgroupswhowere
willingtofightintheirarmies.
B) Theconqueredpeoples,calledDasas,wereenslavedandtheAryansprohibitedallsocial
relationshipswiththem.
C) BecausetheAryanslackedasystemofsocialstratification,theconqueredpeopleswere
soonincorporatedinAryansociety.
D) TheconqueredpeoplewerepermittedtoenterallranksofAryansocietyexceptthe
priesthood.
Answer: B
Diff:3
PageRef:53
Topic:TheAryanIncursionsandEarlyAryanSociety
Skill:Conceptual

24) HowdidtheAryanlevelofsocialintegrationcomparetothatofHarappan
civilization?
A) TheAryansofferedamorecentralizedtraditionofabsolutestatehood.
B) TheAryanswerelesswarlikeandthusdependedmoreonapriesthoodthanthe
Harappans.
C) TheAryansfailedtosurpasstheleveloftribalorganizationandremainednomadic
pastoralists.
D) TheAryanshadalesscentralizedtraditionoftribalorganizationandthenlatercreated
smallkingdoms.
Answer: D
Diff:3
PageRef:55
Topic:TheAryanIncursionsandEarlyAryanSociety
Skill:Conceptual

25) IntheearlydaystheAryanbandsweredividedintohowmanysocialgroups?
A) 12
B) Nine
C) Therewerenodistinctsocialgroups
D) Three
Answer: D
Diff:1
PageRef:53
Topic:TheAryanIncursionsandEarlyAryanSociety
Skill:Factual

37

26) WhichofthefollowingwasNOTavarna inAryansociety?


A) Artisans
B) Brahmans
C) Warriors
D) Peasants
Answer: A
Diff:3
PageRef:53
Topic:TheAryanIncursionsandEarlyAryanSociety
Skill:Conceptual

27) WhichofthefollowingstatementsmostaccuratelyreflectsthenatureofAryanhouseholds?
A) Althoughpatriarchalinauthority,bothmatrilocalmarriagepatternsandmatrilineal
descentwerecustomary.
B) Aryanhouseholdswerereflectiveoftheirtribalbackgroundandlackedwell -defined
authoritystructures.
C) Aryanhouseholdsreflectedmaledominancefeaturingpatriarchalauthority,patrilocal
marriagepatterns,andpatrilinealdescent.
D) Aryanhouseholdspracticedpatriarchalauthoritypatternsandpatrilocalmarriage
customs,buttraceddescentthroughthefemaleline.
Answer: C
Diff:3
PageRef:54
Topic:TheAryanIncursionsandEarlyAryanSociety
Skill:Conceptual

28) WhatwasthenatureofAryanreligion?
A) Monotheistic,dedicatedtothegodVeda
B) Polytheistic,featuringprimarilyfemalefertilitydeities
C) Polytheistic,featuringprimarilymaledeitiesrelatedtowar,fire,andrain
D) Polytheistic,featuringprimarilyanimaldeities
Answer: C
Diff:2
PageRef:54
Topic:TheAryanIncursionsandEarlyAryanSociety
Skill:Conceptual

38

29) WhichofthefollowingrepresentstheprimaryconcernofAryanReligion?
A) Thenatureofthesoul
B) Thefateofthedeadandthequalityoftheafterlife
C) Thepurposeofcreation
D) Praiseofdeitiesandritualsofsacrifice
Answer: D
Diff:2
PageRef:54
Topic:TheAryanIncursionsandEarlyAryanSociety
Skill:Conceptual

30) WhatwastheculturalrelationshipbetweenHarappancivilizationandtheAryans?
A) TheAryansadoptednearlyalltheculturalattributesoftheHarappansincluding
urbanizationandpoliticalcentralization.
B) TheAryansrapidlyassimilatedtheHarappanagriculturaltechniquesincludingmassive
irrigationprojects,butrejectedallelse.
C) TheAryansacceptedtheHarappanreligiouspantheon,butwereunabletoachievethe
technologicalproficiencyoftheHarappans.
D) TherewaslittlecontinuitybetweenHarappanandAryancultures.
Answer: D
Diff:3
PageRef:55
Topic:TheAryanIncursionsandEarlyAryanSociety
Skill:Conceptual

31) Chinesecivilizationbeganwithinwhatrivervalleysystem?
A) HuangheorYellow
B) Yangtze
C) Indus
D) Ganges
Answer: A
Diff:1
PageRef:55
Topic:ABendintheRiverandtheBeginningsofChina
Skill:Factual

39

32) Oneoftheprimaryfeaturesthatencouragedthedevelopmentofsedentaryagricultureinthe
OrdosbulgeofChinawastheexistenceof
A) loess,anextremelyfertilesoil.
B) regularandadequateannualrainfall.
C) mountainsthatsurroundedtheregiononallsides.
D) broaddeltaregionscreatedbythemouthoftheriversystem.
Answer: A
Diff:2
PageRef:55
Topic:ABendintheRiverandtheBeginningsofChina
Skill:Conceptual

33) Whichofthefollowingcultureswasbasedprimarilyonhuntingandfishingsupplementedby
shiftingcultivation?
A) Yangshao
B) Longshan
C) Shang
D) Zhou
Answer: A
Diff:2
PageRef:55
Topic:ABendintheRiverandtheBeginningsofChina
Skill:Conceptual

34) WhichofthefollowingwasafeatureoftheLongshanculture?
A) Sedentarycultivationofriceasthemajorcrop
B) Permanent,walledvillages
C) Centralizedmonarchy
D) Theabsenceofariversystem
Answer: B
Diff:3
PageRef:56
Topic:ABendintheRiverandtheBeginningsofChina
Skill:Conceptual

35) LikeHarappancivilization,earlyChinesecivilizationemphasizedtheimportanceof
A) concernfortheafterlife.
B) oraclesanddivinationbyanimalbones.
C) irrigation.
D) completestandardizationofsociety.
Answer: C
Diff:3
PageRef:56
Topic:ABendintheRiverandtheBeginningsofChina
Skill:Conceptual

40

36) WhichofthefollowingstatementsconcerningtheChinesekingdomofXiaiscorrect?
A) Xiahadahighlycentralizedpoliticalsystemcenteredinasinglecapitalinthewestern
portionoftheOrdosbend.
B) LikeAryansociety,thekingdomofXiaconsistedoflooselyorganizedmilitarytribes.
C) TherearenoarchaeologicalremainsassociatedwithXia,andthekingdommaybe
mythical.
D) ThekingdomofXiaemergedfromthevassalstatesoftheShang.
Answer: C
Diff:3
PageRef:56
Topic:ABendintheRiverandtheBeginningsofChina
Skill:Conceptual

37) WhatdynastyestablishedthefirstactualkingdomofancientChina?
A) Xia
B) Shang
C) Han
D) Ming
Answer: B
Diff:1
PageRef:56
Topic:ABendintheRiverandtheBeginningsofChina
Skill:Factual

38) WhichofthefollowingwasNOTadistinctivefeatureofearlyChinesecivilizationunderthe
Shang?
A) Cookingvesselsandcuisine
B) Matrilineallinesofdescent
C) Useofsilkfabrics
D) Divinationbasedoncrackedanimalbones
Answer: B
Diff:2
PageRef:57
Topic:ABendintheRiverandtheBeginningsofChina
Skill:Conceptual

41

39) WhichofthefollowingstatementsistrueaboutShangwarfare?
A) TheShangwerewarlikenomads.
B) Shangbattleshingedonhand-to-handcombatbetweenchosenchampions.
C) TheShangoftenfoughtonhorseback.
D) Alloftheabove.
Answer: D
Diff:3
PageRef:56
Topic:ABendintheRiverandtheBeginningsofChina
Skill:Conceptual

40) WhichofthefollowingmostaccuratelydescribesthenatureofShanggovernment?
A) Shangmonarchsruledasdespotsaidedonlybyasizeablebureaucracy.
B) Shanggovernmentwasessentiallytribalwithlittlesocialstratificationoreconomic
specialization.
C) TheShanggovernmentwasbasedoncity-statessimilartothoseofMesopotamia.
D) TheShangruledthroughvassalretainerswhoreceivedcontroloverlandandpeasantsin
returnforpaymentoftributetotheking.
Answer: D
Diff:2
PageRef:57
Topic:ABendintheRiverandtheBeginningsofChina
Skill:Conceptual

41) ThefamilypatternmostcommonamongtheeliteduringtheShangperiodinChinawas
A) thepatriarchalextendedfamily.
B) thematrilinealextendedfamily.
C) thepatriarchalnuclearfamily.
D) thematrilinealnuclearfamily.
Answer: A
Diff:2
PageRef:57
Topic:ABendintheRiverandtheBeginningsofChina
Skill:Conceptual

42) Unliketheelite,thefamilypatternforpeasantsduringtheShangperiodwas
A) thepatriarchalextendedfamily.
B) thematriarchalextendedfamily.
C) thepatriarchalnuclearfamily.
D) thematriarchalnuclearfamily.
Answer: C
Diff:2
PageRef:57
Topic:ABendintheRiverandtheBeginningsofChina
Skill:Conceptual

42

43) WhichofthefollowingmostaccuratelydescribesthedeathritualsforShangroyalty.
A) LikeMesopotamianmonarchs,theShanghadnoparticularfascinationwiththeafterlife.
B) TheShangpracticedritualcremationoftheirroyalty.
C) LiketheEgyptians,Shangmonarchswereburiedwiththeirwives,servants,retainers,
andwarcaptives.
D) ShangroyaltyweresetadriftontheHuangheinshipsthathadbeensetafire.
Answer: C
Diff:2
PageRef:57
Topic:ABendintheRiverandtheBeginningsofChina
Skill:Conceptual

44) DuringtheShangera,thepracticeofwritingevolvedfrom
A) commercialpractices.
B) ritualdivinationusinginscribedbonesorshells.
C) directcommunicationwithEgypt.
D) thecreationofdetailedtributelists.
Answer: B
Diff:1
PageRef:57
Topic:ABendintheRiverandtheBeginningsofChina
Skill:Factual

45) WhywasChinesewritingreferredtoasthekeytothecreationofadistinctiveChinese
identity?
A) TheChineseweretheonlycivilizationtoevolvetheirwritingsystemfrompictographs.
B) TheChineseweretheonlycivilizationtoutilizetheirwritingforcommerce.
C) TheChinesewritingsystemencouragedalmostuniversalliteracyamongChinesepeople.
D) Thewrittenlanguageservedtounitethevariousethnicgroupswhowereabletoidentify
themselvesthroughwritingasculturallyChinese.
Answer: D
Diff:3
PageRef:58
Topic:ABendintheRiverandtheBeginningsofChina
Skill:Conceptual

43

46) TheZhoupeoplewhoeventuallyoverthrewtheShangwere
A) probablyTurkic-speakingpeoplewhomigratedfromCentralAsia.
B) arelatedfamilyofvassalretainers.
C) ethnicallyMongolpeoplefromthemountainousregionnorthoftheOrdosbend.
D) anoffshootoftheAryanmigrationfromtheGangesrivervalleyoftheIndian
subcontinent.
Answer: A
Diff:2
PageRef:59
Topic:TheDeclineoftheShangandtheZhouEra
Skill:Conceptual

47) WhichofthefollowingstatementsisNOTaccurateconcerningtheZhouexerciseofauthority
throughthefeudalsystem?
A) ThefeudalsystemoftheZhouwasmoretightlycontrolledthanthatoftheShang.
B) WhereastheShanghadutilizedrelativesasvassalretainers,theZhoupreferredloyal
militaryretainers.
C) TheZhouformallyannexedalloftheregionsbelongingtotheirvassals.
D) TheZhoutightlycontrolledthesuccessiontovassalpositionsatalllevels.
Answer: B
Diff:3
PageRef:59
Topic:TheDeclineoftheShangandtheZhouEra
Skill:Conceptual

48) TherationalizationfortheoverthrowoftheShangbasedontheprincipleofdivinejudgment
wascalled
A) Tiansjustice.
B) Judgmentofthegods.
C) theDivineWind.
D) theMandateofHeaven.
Answer: D
Diff:1
PageRef:59-60
Topic:TheDeclineoftheShangandtheZhouEra
Skill:Factual

44

49) Iftheappealtoasupernaturalsourceoflegitimationenhancedtheabsoluteauthorityofthe
ruler,whatwasitschiefdrawback?
A) Italsostrengthenedtheclassofpriestswhocametodominatetheroyalcourts
B) Itestablishedthatthesupremepoliticalauthoritywasgrantedbytherulersvassals
C) IttiedtheZhoumonarchstoanincreasinglyconservativereligioninthefaceof
aggressivedemandsforreligiousreform
D) Itimpliedthatrulerswhofailedintheirdutieshadlosttheirheavenlyjustificationand
couldbeoverthrown
Answer: D
Diff:3
PageRef:60
Topic:TheDeclineoftheShangandtheZhouEra
Skill:Conceptual

50) WhatwasthesignificanceofthethegreatmigrationoftheChinesepopulationthatoccurred
duringtheZhouperiod?
A) Themigrationbroughtthemonsoon-wateredagriculturalregionsoftheYangtzeriver
valleyintoChinesehands.
B) ThemigrationtookChinesecivilizationnorthwardintotheareasofMongolia.
C) ThemigrationledtodirectcontactsbetweenChinesecivilizationandthePhoenicians.
D) ThemigrationseparatedChinesecivilizationintofivedistinctlydifferentculturalgroups.
Answer: A
Diff:2
PageRef:64
Topic:TheDeclineoftheShangandtheZhouEra
Skill:Conceptual

EssayQuestions
1) WhatwerethemainfeaturesofHarappansociety?
Answer: Majorcities,centralizedadministrationbasedontwocapitals,polytheisticreligion,
overseastrade,anddevelopedwritingsystem.
Diff:2
PageRef:46-52
Topic:TheIndusValleyandSouthAsianCivilization
Skill:Conceptual

2) ContrastAryanculturewiththeHarappancivilizationthatprecededit.Considerpolitical
organization,ethniccomposition,andmaterialculture.
Answer: Politicalorganization:notascentralizedasHarappancivilization,organizedintotribes
andsmallkingdomsdominatedbywarriors;ethniccomposition:Aryanswere
Indo-EuropeanmigrantsintoIndusandGangesrivervalleysystemswhereas
HarappanswereMediterraneanskeletaltypes;materialculture:failedtoreachthatof
Harappans,nomonumentalarchitecture,villagesofthatchandwood,noapparent
interestinsculptureorpainting.
Diff:2
PageRef:52-55
Topic:TheAryanIncursionsandEarlyAryanSociety
Skill:Conceptual

45

3) CompareandcontrastthepoliticalorganizationofShangandZhouChina.
Answer: Shang:organizedintokingdomwiththeoreticallysubjectvassalstatesruledbyroyal
retainers;originalvassalsweredescendantsoforiginalrulingfamiliesofsmall
kingdomsthatprecededShang;vassalsgivencontrolovercommonersintheirregions
fromwhichtheyobtainedtributesentontoroyalfamily.Zhou:alsoafeudal
organization,butmorecloselycontrolledbyroyalfamily;vassalretainersoftenrelated
toroyalfamilies;kingsformallyannexedallvassalstatesandcontrolledsuccessionof
vassals;vassalsrequiredtogiveformaloathsofallegiance;bureaucracydeveloped.
Diff:2
PageRef:59-62
Topic:TheDeclineoftheShangandtheZhouEra
Skill:Conceptual

4) GiventhedevelopmentofmoreabsolutemonarchyinassociationwithZhoufeudalism,what
werethedevelopmentsthatworkedagainstthecontinuationofpoliticalcentralizationin
China?
Answer: ConceptofMandateofHeaven:gaverulersdivinejustificationforrule,butimpliedthat
rulerswhofailedintheirmissionhadlostmandateandcouldbeoverthrown,i.e.
justifiedpoliticalrevolution;growthofprofessionalbureaucracy:begantocontrol
administrativeprocedures,alsosomecontrolofland,offeredalternativetomilitary
vassalsofmonarch.
Diff:2
PageRef:59-65
Topic:TheDeclineoftheShangandtheZhouEra
Skill:Conceptual

5) CompareandcontrastHarappancivilizationwithChinesecivilization.
Answer: Harappansociety:basedonirrigationsystem(deterioratedunderAryans);developed
independently;centralizedadministrationbasedontwocapitals;developedwriting
system;littlecontinuityafterHarappandeclineChinese:developedindependently;
basedonrivervalleyirrigationsystem;centraladministration(undertwocapitalsin
Zhouperiod);developedwritingsystem;unlikeHarappaabletowithstandsuccessive
migrationsandretainculture;greatersenseofcontinuity.
Diff:2
PageRef:46-5255-62
Topic:TheDeclineoftheShangandtheZhouEra
Skill:Conceptual

46

Chapter4 UnificationandtheConsolidationof
CivilizationinChina
Multiple-ChoiceQuestions
1) AstheZhouabilitytocontroltheirvassalsdecreased,Chinaentered
A) theeraoftheMongoldynasty.
B) alongperiodofpoliticalconflictandsocialturmoil.
C) theeraoftheShangdynasty.
D) aperiodofDoricinvasions.
Answer: B
Diff:2
PageRef:77
Topic:Introduction
Skill:Factual

2) Whichofthefollowingwasaresultoftheperiodofpoliticalconfusionfollowingthefallofthe
Zhoudynasty?
A) Chinesecivilizationfailedtoproduceanotherdynastyforcenturies.
B) TheshiceasedtoplayasignificantroleinChinesegovernment.
C) FeudalismbecamethedominantformofpoliticalorganizationinChinesesociety.
D) Philosopherssoughttofindwaystoendtheconflictandcreatemorepermanentand
unifiedpoliticalsystems.
Answer: D
Diff:2
PageRef:77
Topic:Introduction
Skill:Conceptual

3) WhoweretheShi?
A) Militarycommanders.
B) Peasants.
C) Thescholargentry.
D) Thefamilyoftheemperor.
Answer: C
Diff:2
PageRef:74
Topic:PhilosophicalRemedies
Skill:Conceptual

47

4) WhichofthefollowingwasNOTcharacteristicoftheperiodofwarfarefollowingthedecline
oftheZhou?
A) Failureofthemerchantclass
B) Neglectofpublicworkssuchasdikes,canals,androads
C) Increasedtaxationandconscriptionofthepeasantry
D) SpreadofdiseaseasarmiesroamedthroughoutChina
Answer: A
Diff:3
PageRef:77
Topic:PhilosophicalRemedies
Skill:Conceptual

5) Duringmostofhislife,KungFuziorConfucius
A) servedtheHanemperorsastheirchiefcourtadvisor.
B) advocatedabolitionoftheshiasdangerousopponentsofacentralizedstate.
C) wanderedsearchingfortheidealruler.
D) remainedatopamountainincontemplativeseclusion.
Answer: C
Diff:2
PageRef:78
Topic:PhilosophicalRemedies
Skill:Conceptual

6) LoyaldisciplescollectedConfuciusssayingsinthe
A) KungMao.
B) Analects.
C) BookofLordShang.
D) BookofDao.
Answer: B
Diff:1
PageRef:78
Topic:PhilosophicalRemedies
Skill:Factual

7) WhichofthefollowingmostaccuratelydescribesConfucius?
A) Anactiveadministratorandadvisor
B) AreligiousteachersimilartotheJewishprophetsandBuddha
C) Emperor
D) Asocialphilosopherobsessedwiththeneedfororderandharmony
Answer: D
Diff:2
PageRef:78
Topic:PhilosophicalRemedies
Skill:Conceptual

48

8) IntheopinionofConfucius,thewisemenbestabletoadministerthegovernmentcouldonly
comefromthe
A) militaryvassals.
B) shi.
C) eunuchs.
D) familiesoftheemperors.
Answer: B
Diff:1
PageRef:78
Topic:PhilosophicalRemedies
Skill:Conceptual

9) AccordingtoConfucius,forwhatreasonshouldsuperiormenrule?
A) Toenrichtheirfamiliesandearndistinction
B) Toestablishthegloryoftheemperor
C) Toestablishthegloryoftheregionalaristocracy
D) Toservesocietyasawhole
Answer: D
Diff:2
PageRef:78
Topic:PhilosophicalRemedies
Skill:Conceptual

10) WhichofthefollowingwasNOToneofthetiesofloyaltyandobediencethatheldtogether
Confuciussconceptofsocialorder?
A) Father/son
B) Merchant/peasant
C) Husband/wife
D) Ruler/subject
Answer: B
Diff:2
PageRef:78
Topic:PhilosophicalRemedies
Skill:Conceptual

49

11) WhichofthefollowingrepresentsthefundamentalbeliefofMencius,oneofConfuciuss
disciples?
A) Humanswerebynatureinclinedtogoodnessandoughttoberuledinthatfashion.
B) Humanswereinclinedtobelazyandevilandoughttoberuledstrictly.
C) Humansshouldretreatfromsocietyandseekonenesswithnature.
D) Governmentshouldberigorousandbasedonstrictlawsharshlyexecuted.
Answer: A
Diff:3
PageRef:78
Topic:PhilosophicalRemedies
Skill:Conceptual

12) WhichofthefollowingrepresentstheviewsofSunzi,oneofConfuciussdisciples?
A) Humanswerebynatureinclinedtogoodnessandoughttoberuledinthatfashion.
B) Humanswereinclinedtobelazyandevilandoughttoberuledbyauthoritarian
government.
C) Humansoughttoretreatfromsocietyandseekonenesswithnature.
D) Governmentshouldberigorousandbasedonstrictlawsharshlyexecuted.
Answer: B
Diff:3
PageRef:79
Topic:PhilosophicalRemedies
Skill:Conceptual

13) WhichofthefollowingrepresentsthepoliticalviewpointoftheLegalists?
A) Humanswereinclinedtogoodnessandoughttoberuledwithcompassion.
B) Humanswereinclinedtobelazyandevilbutoughttobegovernedwithasenseof
compassion.
C) Humansshouldretreatfromsocietyandseekonenesswithnature.
D) Governmentshouldberigorousandbasedonstrictlawsharshlyexecuted.
Answer: D
Diff:3
PageRef:78-79
Topic:TriumphoftheQin
Skill:Conceptual

50

14) WhichofthefollowingrepresentsthephilosophicalviewpointofLaozi?
A) Humanswereinclinedtogoodnessandoughttobegovernedinacompassionate
fashion.
B) Humanswereinclinedtobelazyandevilandoughttobegovernedrigorously.
C) Humansoughttoretreatfromsocietyandseekonenesswithnature.
D) Governmentoughttobeauthoritarianandbasedonstrictlawsharshlyexecuted.
Answer: C
Diff:3
PageRef:79
Topic:PhilosophicalRemedies
Skill:Conceptual

15) ThephilosophyespousedbyLaoziwascalled
A) Confucianism.
B) Daoism.
C) Legalism.
D) Shintoism.
Answer: B
Diff:2
PageRef:79
Topic:PhilosophicalRemedies
Skill:Factual

16) WhatphilosopherbelievedthatagoodmanwasonewhoTreatshisbettersasbetters,Wears
anairofrespect,Whoinservingfatherandmother,Knowshowtoputinhiswholestrength.
A) Confucius
B) Mencius
C) Sunzi
D) Laozi
Answer: A
Diff:3
PageRef:78
Topic:PhilosophicalRemedies
Skill:Conceptual

51

17) Whichphilosophersaid,Itakenoactionandthepeoplearereformed.Ienjoypeaceandthe
peoplebecomehonest.Idonothingandpeoplebecomerich.Ihavenodesiresandpeople
returntothegoodandsimplelife?
A) Confucius
B) Mencius
C) Sunzi
D) Laozi
Answer: D
Diff:3
PageRef:80
Topic:PhilosophicalRemedies
Skill:Conceptual

18) Whichphilosophersaid,Thenatureofmanisevil:hisgoodnessisacquired.Hisnaturebeing
whatitis,manisborn,first,withadesireforgain.Ifthisdesireisfollowed,strifewillresult
andcourtesywilldisappear?
A) Confucius
B) Mencius
C) Sunzi
D) Laozi
Answer: C
Diff:3
PageRef:80
Topic:PhilosophicalRemedies
Skill:Conceptual

19) Whichphilosophersaid,Personalcultivationbeginswithpoetry,ismadefirmwithrulesof
decorum,andisperfectedbymusic?
A) Confucius
B) Mencius
C) Sunzi
D) Laozi
Answer: A
Diff:3
PageRef:80
Topic:PhilosophicalRemedies
Skill:Conceptual

52

20) Whichphilosophersaid,Whenitislefttofollowitsnaturalfeelingshumannaturewilldo
good.ThatswhyIsayitisgood.Ifitbecomesevil,itisnotthefaultofmansoriginal
capability?
A) Confucius
B) Mencius
C) Sunzi
D) Laozi
Answer: B
Diff:3
PageRef:80
Topic:PhilosophicalRemedies
Skill:Conceptual

21) TheQinstatewasregardedasbarbarianatleastinpartbecause
A) itwaslocatedinthenewlydevelopedsouthernregionsofChinaneartheYangzeriver
valley.
B) itrefusedtheserviceoftheshi.
C) theQinoriginatedaspastoralnomadsentrustedwithdefendingthewesternfrontierof
China.
D) itwaslocatedinthelesscivilizedeasternportionsofChina.
Answer: C
Diff:2
PageRef:80
Topic:TriumphoftheQin
Skill:Conceptual

22) TheQinweresuccessfulasaresultofsignificantsocialandpoliticalchanges.Whichofthe
followingwasNOTareformoftheQinrulers?
A) Freeingofthepeasantsfrombondagetolocallords
B) Creationofabureaucracyof?shi?toreplacetheweakenedlocalvassals
C) Conscriptingthefreedpeasantstoenlargetheirarmies
D) Creatingarmiescomposedalmostentirelyoffootsoldiers
Answer: D
Diff:2
PageRef:80
Topic:TriumphoftheQin
Skill:Conceptual

53

23) ThefounderoftheLegalistschoolofpoliticalphilosophywrotethe
A) Analects.
B) BookofLordShang.
C) SacredSoilofChina.
D) SayingsofShiHuangdi.
Answer: B
Diff:3
PageRef:81
Topic:TriumphoftheQin
Skill:Factual

24) WhichofthefollowingwasNOTapoliticalviewpointoftheLegalists?
A) ThepowerofChinasrulersshouldbeabsolute.
B) Thestatesmainconcernwastoenforcestrictlawswithharshpunishments.
C) Therulershouldpromotethewelfareofthepeople.
D) Themajorobjectiveoftherulerwastoenhancethestrengthandwealthofthestate.
Answer: C
Diff:3
PageRef:81
Topic:TriumphoftheQin
Skill:Conceptual

25) ShiHuangdi
A) promotedConfucianphilosophy.
B) attemptedtoalleviatetheharsheraspectsofLegalistpoliticalphilosophy.
C) believedintheessentialgoodnessofhumans.
D) wassoconcernedaboutcontrollingideasintheQinstatethatheproposedtheburningof
allbooksotherthanLegalisttractsandafewotherofficialvolumes.
Answer: D
Diff:2
PageRef:81
Topic:TriumphoftheQin
Skill:Conceptual

26) TheemperorresponsibleforthereunificationofChinaafterthecollapseoftheZhouwas
A) HanWudi.
B) HanFeizi.
C) ShiHuangdi.
D) ShangYang.
Answer: C
Diff:2
PageRef:81
Topic:TriumphoftheQin
Skill:Factual

54

27) WhichofthefollowingstatementsbestrepresentsShiHuangdisapproachtothevassalstates
andregionalaristocracy?
A) Hedestroyedmostofthevassalsofthepreviousdynastyandappointedhisrelativesto
ruleintheregions.
B) Hestrengthenedthehandoftheregionalvassalstodestroythegrowingpowerofthe shi.
C) ShiHuangdiwasneverabletocurbtheauthorityofthevassalstatesandwasrestricted
torulinginQin.
D) HedestroyedtheregionalvassalsandreplacedthemwithQinbureaucrats.
Answer: D
Diff:2
PageRef:81
Topic:TriumphoftheQin
Skill:Conceptual

28) ShiHuangdiwasfamedforhispublicbuildingprojects.Whichofthefollowingishismost
famousmonumentalstructure?
A) TheroyalpalaceatBeijing
B) TheGreatWallofChina
C) TheShintotempleatLoyang
D) TheConfucianlibraryatXianyang
Answer: B
Diff:1
PageRef:82
Topic:TriumphoftheQin
Skill:Factual

29) WhatwasthecapitalcityoftheQinempire?
A) Loyang
B) Anyan
C) Beijing
D) Xianyang
Answer: D
Diff:2
PageRef:81
Topic:TriumphoftheQin
Skill:Conceptual

55

30) WhichofthefollowingstatementsbestdescribesthepoliticalphilosophyofShiHuangdiand
hisgovernment?
A) Theywereopposedtoanypoliticalphilosophiesotherthantheofficiallyapproved
Legalistdoctrines.
B) TheQingovernmentwasessentiallyDaoist.
C) ShiHuangdiandhisadvisorsfollowedthepoliticalphilosophyofMencius,oneof
Confuciussdisciples.
D) ShiHuangdisgovernmentappliedtheconceptsofmanypoliticalphilosophiesincluding
ConfucianismandDaoism.
Answer: A
Diff:2
PageRef:81-82
Topic:TriumphoftheQin
Skill:Conceptual

31) TheoverthrowoftheQindynastywasbegunby
A) theregionalvassalsoftheQin.
B) shiadvisorswithintheimperialgovernment.
C) outragedDaoistpoliticalphilosophers.
D) twodisgruntledpeasantswhohadbeenconscriptedforabuildingproject.
Answer: D
Diff:2
PageRef:82
Topic:TriumphoftheQin
Skill:Factual

32) WhatwasthemostsignificantaccomplishmentoftheQindynasty?
A) Thestrengtheningofthearistocracyagainsttheshi
B) TheunificationofChinaunderashibureaucracy
C) Thereorderingoftheregionalstatestostrengthenthefeudalsystem
D) Theirabilitytolastforfourcenturies
Answer: B
Diff:2
PageRef:82
Topic:TriumphoftheQin
Skill:Conceptual

56

33) WhichofthefollowingworkswaswrittenbySunzi?
A) Analects
B) BookofLordShang
C) TheArtofWar
D) Mahabharata
Answer: C
Diff:1
PageRef:84
Topic:TriumphoftheQin
Skill:Factual

34) WhichofthefollowingwasNOTachangeinwarfarebroughtaboutbythephilosophyof
Sunzi?
A) Bluffs,saboteurs,andthreatswereemployedbeforearmiesweresenttowar.
B) Shibureaucracieswereexcludedfromtheprocessofwarfare.
C) Weatherconditionsandtheadvantagesofterraindeterminedthetimeandplaceof
battle.
D) Disciplineandregularformationsreplacedindividualcombatastheruleformilitary
engagement.
Answer: B
Diff:2
PageRef:84-85
Topic:TriumphoftheQin
Skill:Conceptual

35) ThefounderoftheHandynastywas
A) Sunzi.
B) HanFeizi.
C) HanWudi.
D) LiuBang.
Answer: D
Diff:1
PageRef:83
Topic:TheHanDynasty
Skill:Factual

57

36) WhowerethemostformidableexternalenemiesoftheHandynasty?
A) TheHsiung-nunomads.
B) Theshi.
C) TheKoreans.
D) TheVietnamese.
Answer: A
Diff:2
PageRef:83
Topic:TheHanDynasty
Skill:Conceptual

37) Whichofthefollowingstatementsmostaccuratelydescribestheterritorialexpansionofthe
Handynasty?
A) TheHanwereunabletoexpandtheirterritoriesduetoconstantwarswiththerulersof
thestates.
B) TheHanactuallylostterritoriestotheraidsoftheHsiung-nuandothernomadic
invaders.
C) TheHantemporarilydefeatedtheHsiung-nuanddramaticallyexpandedChinese
territorytotheeastandsouth.
D) TheHanwereabletoexpandtheirterritoriesbyconqueringtheGuptacivilizationof
India.
Answer: C
Diff:2
PageRef:83
Topic:TheHanDynasty
Skill:Conceptual

38) WhichofthefollowingmostaccuratelydescribesthepoliticalphilosophyoftheHan?
A) TheHancontinuedtheLegalistpoliticalphilosophyoftheQindynasty.
B) TheHansuppressedtheLegalistsandadoptedthepoliticalphilosophyofLaozi.
C) TheHansuppressedtheLegalistsandadoptedthepoliticalphilosophyofConfucius.
D) TheHansuppressedtheLegalistsandadoptedthepoliticalphilosophyofShangYang.
Answer: C
Diff:2
PageRef:84-85
Topic:TheHanDynasty
Skill:Conceptual

58

39) WhichofthefollowingstatementswasNOTanaspectofthe shi administrationduringthe


Handynasty?
A) Confucianteachingsbecameinstitutionalizedwiththecreationofanimperialuniversity
B) KnowledgeofConfucianteachingsbecameessentialtoemploymentandpromotionin
theHanadministration
C) Formalexaminationsforgovernmentpositionswereestablishedatthebeginningofthe
lastcenturyB.C.
D) Legalcodesweremadeevenharsher
Answer: D
Diff:2
PageRef:85
Topic:TheHanDynasty
Skill:Conceptual

40) DuringtheHandynasty,theshievolvedasaresultofintermarriagewithothersocialgroups
intothe
A) scholar-gentry.
B) educatedpeasantry.
C) merchantelite.
D) feudalaristocracy.
Answer: A
Diff:2
PageRef:86
Topic:TheHanDynasty
Skill:Factual

41) WhatwasthestatusofwomenduringtheHandynasty?
A) WomenenjoyedequalstatuswithmalesduringtheHandynasty.
B) DespitetheConfucianrequirementforfemaledeferencetomales,womenduringthe
Hanhadmorefreedomthanduringlaterdynasties.
C) DespitetheConfucianrequirementformaledeferencetofemales,womenhadless
freedomthanduringlaterdynasties.
D) Thedemeanedstatusofwomenwasrepresentedbytheirinabilitytoshareinfamily
property.
Answer: B
Diff:2
PageRef:87
Topic:TheHanDynasty
Skill:Conceptual

59

42) WhichofthefollowingstatementsconcerningthepeasantryduringtheHandynastyistrue?
A) NopeasantsheldlandduringtheHandynasty.
B) PeasantsduringtheHandynastywerefreedfromconscriptionforpublicworksand
militaryservice.
C) Manypeasantshadlittleornolandandworkedforwell -to-dolandlords.
D) ThepeasantryenjoyedgeneralprosperityduringtheHandynasty.
Answer: C
Diff:2
PageRef:88
Topic:TheHanDynasty
Skill:Conceptual

43) DuringtheHandynasty,theemperorslived
A) inthestateofQinonthewesternfrontier.
B) inanimperialagriculturalvillageinthecountryside.
C) inawalledprecinctjustoutsidethecapitalcity.
D) intheforbiddencity,awalledprecinctwithinthecapitalcity.
Answer: D
Diff:1
PageRef:88
Topic:TheHanDynasty
Skill:Factual

44) WhichofthefollowingstatementsconcerningthearchitectureoftheHancapitalcityisNOT
true?
A) Nearlyallbuildingswithinthecapitalweremadeofbrickofastandardizedsizeand
construction.
B) Mostofthebuildingswithinthecapitalcitywereorientedtowardthesouth.
C) Thearistocratsandthescholar-gentrylivedinanimperialcomplexjustoutsidethe
imperialpalaces.
D) Mosthouseswereroofedinglazedtileswithupturnededges.
Answer: A
Diff:3
PageRef:89
Topic:TheHanDynasty
Skill:Conceptual

60

45) WhichofthefollowingstatementsmostaccuratelydescribesthedegreeofurbanizationinHan
China?
A) ThecapitalatXianyangmayhavenumberedasmanyas250,000people,buttherewere
fewothercities.
B) Chinamayhavebeenthemosturbanizedcivilizationintheworldwithmanylargecities
numberinginthethousands.
C) AsaresultoftheconstantwarfareduringtheHandynasty,fewcitieswereableto
survive.
D) Asidefromwalledagriculturalcomplexesdominatedbytheregionalaristocracy,there
werefewwalledcities.
Answer: B
Diff:2
PageRef:89
Topic:TheHanDynasty
Skill:Conceptual

46) Whichofthefollowingstatementsisprobablytrue?
A) Thescholar-gentryheldmerchantsingreatesteem.
B) Themerchantclassvirtuallydisappearedasaresultofthediminutionoftrade.
C) Chinawasthemosttechnologicallybackwardofalltheclassicalcivilizations.
D) Chinawasthemosttechnologicallyinnovativeandadvancedofalltheclassical
civilizations.
Answer: D
Diff:2
PageRef:89
Topic:TheHanDynasty
Skill:Conceptual

47) WhichofthefollowingwasNOTatechnologicalinnovationoftheHanperiodinChina?
A) Brushpenandpaper
B) Ruddersandcompassesforimprovednavigation
C) Gunpowderandcannon
D) Advancedminingtechniques
Answer: C
Diff:1
PageRef:90
Topic:TheHanDynasty
Skill:Conceptual

61

48) ChineseartduringtheHanclassicalperiod
A) concentratedonthedevelopmentofmonumentalsculpture.
B) wasuninspiredandgenerallyofpoorquality,duetotheChineseconcentrationonthe
sciences.
C) wasmostadvancedintheareaofpainting.
D) waslargelydecorativeoftenreflectingthegeometricprecisionofChinesecalligraphy.
Answer: D
Diff:2
PageRef:91
Topic:TheHanDynasty
Skill:Conceptual

49) WhatfamilyrelatedtotheHanemperorsbymarriagetemporarilysucceededinoverthrowing
theHanin9C.E..?
A) Shang
B) Qin
C) Wang
D) Zhou
Answer: C
Diff:1
PageRef:91
Topic:TheHanDynasty
Skill:Factual

50) TheperiodofthelaterHandynastywaspoliticallytroubledbystrugglesforpowerbetween
A) thefamiliesoftheimperialwivesandthefeudallordsofthestates.
B) thefamiliesoftheimperialwives,thescholar-gentry,andtheimperialeunuchs.
C) theemperorsandtheGuptacivilizationofIndia.
D) thescholar-gentryandtheregionalkings.
Answer: B
Diff:2
PageRef:91
Topic:TheHanDynasty
Skill:Conceptual

EssayQuestions
1) WhatisthebasisofConfucianteachings?
Answer: ObsessedbyneedtorestoreorderandharmonytoChina;governmenttobebasedon
adviceofsuperiormenidenticalwithshi;superiormencreatedbyeducationtoserve
allofChinesesociety;socialharmonydependentoneachpersonacceptinghissocial
positionandperforminghissocialtask.
Diff:2
PageRef:78-81
Topic:PhilosophicalRemedies
Skill:Conceptual

62

2) HowdidtheideasofConfuciusandDaoismdiffer?Whichhadagreaterimpactonlater
Chinesepoliticalphilosophy?
Answer: Confucius:strongstate,educatedshouldrule,socialharmonybasedonobedienceand
deference,andvirtue.Daoism:retreatintonature,nonaction,andselfexamination
ratherthanpoliticalpower.Confuciusspoliticalphilosophywaslaterincorporatedinto
Handynasty.
Diff:2
PageRef:77-79
Topic:PhilosophicalRemedies
Skill:Conceptual

3) Despitethebrevityofthedynasty,theQinestablishedthefoundationsofaunitedChina.
Whatwerethesesignificantpoliticalandsocialreforms?
Answer: Destructionofregionalkingdomsandfeudallordships;establishmentofcentralized
governmentunderadministrativebureaucracystaffedbyshi;standardizationof
currency,weights,andmeasures;constructionofcanals,roads,irrigationsystemsthat
tiedallofChinatogether.
Diff:2
PageRef:80-83
Topic:TriumphoftheQin
Skill:Conceptual

4) InwhatwayswasHanChinasimilartoQinChina?Inwhatwaysweretheydifferent?
Answer: Similarities:destructionoffeudallordsinpreferenceforshi regionalgovernors;
emphasisoncentralizationandunification.Differences:institutionalizationof shi
creationofstateuniversity,establishmentofstateandregionalcivilserviceexams;
evolutionofshitoscholar-gentryasaresultofintermarriagewithregionallandholders;
shiftfromLegalistpoliticalphilosophytothatofvariousschoolsofConfucius.
Diff:2
PageRef:80-92
Topic:TheHanDynasty
Skill:Conceptual

5) Whatisthecycleofdynasticgovernmentthattheauthorssuggestwasestablishedduringthe
Handynasty?
Answer: Localrebellionsculminatedintheoverthrowofthepreviousdynasty;centuryof
dynasticsuccess;afterperiodofdecline,oneortwoemperorstemporarily
re-establisheddynasticsuccess;twocenturiesafterthefoundationofthedynasty,
prolongeddeclinesetintypifiedbyforeigninvasions,rebellion,andcorruptionincivil
life;subsequentoverthrowofthedynasty.
Diff:2
PageRef:91-92
Topic:TheHanDynasty
Skill:Conceptual

63

Chapter5 ClassicalCivilizationintheEastern
Mediterranean:PersiaandGreece
Multiple-ChoiceQuestions
1) WhatwastheGreekwordforcity-stategovernment?
A) Minoan
B) Achaean
C) Polis
D) Mycenae
Answer: C
Diff:1
PageRef:98
Topic:Introduction
Skill:Conceptual

2) WhatdoesHellenismmean?
A) derivedfromtheGreek
B) imperialgovernment
C) Greekphilosophy
D) city-state
Answer: A
Diff:3
PageRef:96
Topic:GreeceasaClassicalCivilization
Skill:Conceptual

3) HowdidGreekandChinesecivilizationdiffer?
A) TheChineseformsofpoliticalorganizationweregenerallylesscentralizedthanthoseof
Greekcivilization.
B) TheChinesewereunabletodevelopsophisticatedphilosophicalsystemsofthought.
C) TheGreekswereunabletodevelopsophisticatedphilosophicalsystemsofthought.
D) TheGreekformsofpoliticalorganizationweregenerallylesstidythanthoseofChinese
civilization.
Answer: D
Diff:3
PageRef:96
Topic:GreeceasaClassicalCivilization
Skill:Conceptual

64

4) ThepersonresponsibleforthecreationoftheunifiedPersianempirearound550B.C.was
A) Hammurabi.
B) AlexandertheGreat.
C) Xerxes.
D) CyrustheGreat.
Answer: D
Diff:1
PageRef:96
Topic:ThePersianEmpire
Skill:Factual

5) WhichofthefollowingstatementsisNOTtrueconcerningthePersianreligionof
Zoroastrianism?
A) itwasanpolytheistic
B) itfeaturedapantheonofmaleandfemaledeitieswhoactedverymuchlikehumans
C) therighteouslivedonafterdeathintheHouseofSong
D) Zoroastersawlifeasastrugglebetweentwocosmicforcesofgoodandevil
Answer: B
Diff:2
PageRef:97
Topic:ThePersianEmpire
Skill:Conceptual

6) WhichofthefollowingdescriptionsofGreekpoliticsafter800B.C.E.isNOTaccurate?
A) Greekpoliticswasusuallylocalized
B) rulebyaristocratswasmostcharacteristic,butperiodsoftyrannywerealsoimportant
C) duringmuchofitsformativeperiod,GreecewasdominatedbythePersians
D) theGreekssketchedadynamicversionofdemocracy
Answer: C
Diff:2
PageRef:97
Topic:ThePoliticalCharacterofClassicalGreece
Skill:Conceptual

7) WhichofthefollowingwasnotaGreekcitystate?
A) Istanbul
B) Athens
C) Sparta
D) Olympia
Answer: A
Diff:2
PageRef:98
Topic:GreeceasaClassicalCivilization
Skill:Factual

65

8) WhoseemstohavewrittentheIliad andtheOdyssey?
A) Solon
B) Plato
C) CirustheGreat
D) Homer
Answer: D
Diff:2
PageRef:97
Topic:ThePoliticalCharacterofClassicalGreece
Skill:Conceptual

9) TheGreekalphabetwasadaptedfromthe
A) Egyptians.
B) Macedonians.
C) Chalcedonians.
D) Phoenicians.
Answer: D
Diff:1
PageRef:97
Topic:ThePoliticalCharacterofClassicalGreece
Skill:Factual

10) ThemythosofGreekculturewasestablishedbythewideacceptanceoftwogreatpoems,the
A) GilgameshandtheArtofWar.
B) AnalectsandtheAeneid.
C) IliadandtheOdyssey.
D) MetamorphosesandtheOdyssey.
Answer: C
Diff:1
PageRef:97
Topic:ThePoliticalCharacterofClassicalGreece
Skill:Factual

11) TheGreekgovernmentsoftheperiodimmediatelyafter800B.C.E.largelyconsistedof
A) regionalkingdoms.
B) aunifiedempireunderasingleruler.
C) city-states.
D) feudalvassalslooselycontrolledbyasingleruler.
Answer: C
Diff:2
PageRef:98
Topic:ThePoliticalCharacterofClassicalGreece
Skill:Conceptual

66

12) Intheearlieststagesofcity-statedevelopment,thepoleis werelargelyruledby


A) tyrants.
B) aristocrats.
C) smallfarmers.
D) kings.
Answer: B
Diff:2
PageRef:98
Topic:ThePoliticalCharacterofClassicalGreece
Skill:Conceptual

13) Whatwerethecausesforthedisputesthataroseafter700B.C.inthecity -statesofGreece?


A) aseriesofforeigninvasions
B) commercialexpansionandtheproblemsofsmalllandownersintheneweconomy
C) adecreaseinthenumberofavailableslavesduetoeconomicreverses
D) theimpositionofauniversalincometaxonallGreeks
Answer: B
Diff:2
PageRef:98
Topic:ThePoliticalCharacterofClassicalGreece
Skill:Conceptual

14) Whichofthefollowingstatementsbestdescribesthepoliticalchangethattookplaceinmany,
butnotall,Greekcity-statesby500B.C.?
A) Mostabandonedaristocraticgovernmentinfavorofcentralizedmonarchies.
B) Therewasnoobservablechange,astheinstitutionsandpersonnelofgovernment
remainedmuchastheywerein800B.C.
C) Therewasagrowingspiritofloyaltytothecommunityratherthantoindividuals,
democratizationofmilitaryservice,andagreatercommitmenttopublicservice.
D) Allpoleisbannedthearistocracyfromfurtherparticipationintheaffairsofthecity-states
andinitiatedtrulydemocraticgovernments.
Answer: C
Diff:2
PageRef:98
Topic:ThePoliticalCharacterofClassicalGreece
Skill:Conceptual

67

15) WhichofthefollowinggroupswereNOTexcludedfromparticipationintheAthenian
democracyofthefifthcenturyB.C.?
A) women
B) foreigners
C) slaves
D) aristocrats
Answer: D
Diff:2
PageRef:99
Topic:ThePoliticalCharacterofClassicalGreece
Skill:Conceptual

16) TheAthenianpracticeofexilingpotentialthreatstothedemocraticstructureofthecity-state
wascalled
A) ostracism.
B) treason.
C) banallordship.
D) cynicism.
Answer: A
Diff:2
PageRef:100
Topic:ThePoliticalCharacterofClassicalGreece
Skill:Factual

17) WhichofthefollowingstatementsaccuratelydescribesadifferencebetweenclassicalChinese
approachtopoliticsandtheapproachofHellenicGreecetopolitics?
A) ChinaplacedlessemphasisonhierarchyandobediencetoauthoritythandidtheGreeks.
B) Greekpoliticsemphasizedasingle,centralizedsystemofauthority.
C) TheGreeksplacedmoreemphasisonparticipationandlessonbureaucracy.
D) TheChineselackedthebureaucracythatmadedemocraticgovernmentinGreece
possible.
Answer: C
Diff:2
PageRef:100
Topic:ThePoliticalCharacterofClassicalGreece
Skill:Conceptual

68

18) WhatwasthenameofthewarbetweenSpartaandAthens?
A) thePeloponnesianWars
B) thePersianWars
C) theOlympicGames
D) theoracleatDelphi
Answer: A
Diff:2
PageRef:102
Topic:ThePoliticalCharacterofClassicalGreece
Skill:Conceptual

19) TheAthenianempireofthefifthcenturyB.C.grewfromanallianceofcity -statescalledthe


A) PeloponnesianLeague.
B) LeagueofCorinth.
C) LombardLeague.
D) DelianLeague.
Answer: D
Diff:2
PageRef:101
Topic:ThePoliticalCharacterofClassicalGreece
Skill:Factual

20) ThevictorinthePeloponnesianWarwas
A) Athens.
B) Sparta.
C) Persia.
D) Thebes.
Answer: B
Diff:1
PageRef:102
Topic:ThePoliticalCharacterofClassicalGreece
Skill:Factual

21) IntheaftermathofthePeloponnesianWar,by355B.C.thepoliticalstructureofGreece
A) wascompletelydominatedbySparta.
B) fellunderthecontrolofAthensandarenewedAthenianEmpire.
C) hadbecomeakingdomruledbyPhilipofMacedon.
D) hadreturnedtoasettingofindependent,disorganizedcity-states.
Answer: D
Diff:2
PageRef:102
Topic:TheHellenisticPeriod
Skill:Conceptual

69

22) WhichofthefollowingstatementsaboutthekingdomofMacedonisNOTtrue?
A) MacedonlaynorthofGreece,butitsinhabitantsdidspeakGreekandwereinterestedin
Greekculture.
B) Macedonia,likemainlandGreece,wasdividedintonumerouscity-states.
C) MacedonpossessedastrongermilitaryforceafterthereignofKingPhilipII(359 - 336
B.C.)
D) PhilipIIbeganaseriesofmilitarycampaignsagainstthecity-statesoftheGreek
mainland.
Answer: B
Diff:2
PageRef:102-3
Topic:TheHellenisticPeriod
Skill:Conceptual

23) ThepersonresponsiblefortheconquestofthePersianempirewas
A) PhilipII.
B) PericlesofAthens.
C) AgesilausofSparta.
D) AlexandertheGreat.
Answer: D
Diff:1
PageRef:103
Topic:TheHellenisticPeriod
Skill:Factual

24) WhichofthefollowingstatementsmostaccuratelyreflectsAlexandersplansforhisnew
empire?
A) HeplannedtotransplantGreekcultureinPersiawhileatthesametimewipingoutall
tracesofeasternculture.
B) HeplannedtotransplantAsianculturalandpoliticalinstitutionstoGreecewhileatthe
sametimewipingoutalltracesofHellenicculture.
C) AsaMacedonian,AlexanderdisdainedbothGreekandAsiancultureaseffeminateand
plannedtoimposeMacedonianculturalvaluesinallhisdomains.
D) AlexanderwantedtocombineGreekculturewithAsianculturetocreateanew,
Hellenisticculture.
Answer: D
Diff:2
PageRef:103
Topic:TheHellenisticPeriod
Skill:Conceptual

70

25) WhichofthefollowingwasNOToneoftheregionaldynastiesthatwereestablishedafter
Alexandersdeath?
A) Ptolemies
B) Antigonids
C) Abbassids
D) Seleucids
Answer: C
Diff:2
PageRef:103
Topic:TheHellenisticPeriod
Skill:Factual

26) WithAlexandersdeath,theGreekinfluencewithintheregionaldynasties
A) wanedasGreeksincreasinglyreturnedtotheGreekmainland.
B) wanedasGreekswereexpelledfrompositionsinthegovernmentandcommerceand
werereducedtoslavery.
C) continuedasGreeksmovedintogovernmentjobsandmerchantpositions.
D) grewgreaterasGreeksmonopolizedallimportantpoliticalposts.
Answer: C
Diff:3
PageRef:104
Topic:TheHellenisticPeriod
Skill:Conceptual

27) OneofthemajorpoliticaldifferencesbetweenHellenicGreeceandHellenistickingdomswas
that
A) city-statesceasedtoexistinthedynastickingdoms.
B) Hellenisticgovernmentcenteredonmilitaryempiresratherthancity-states,eventhough
thelattercontinuedtoexist.
C) therehadbeennokingsinHellenicGreece,buttheydominatedtheHellenisticsuccessor
states.
D) theHellenistickingdomsproducedmoreenduringpoliticalprinciples.
Answer: B
Diff:2
PageRef:104
Topic:TheHellenisticPeriod
Skill:Conceptual

71

28) WhichofthefollowingstatementsmostaccuratelycomparestheGreekreligiontothatofthe
IndianandChinesecivilizations?
A) Greekreligionwasavastlymoresophisticatedbeliefbasedonmoralpreceptsanda
strongattachmenttoanafterlife.
B) GreekreligionwasmorestronglyspiritualthaneithertheIndianorChinesereligions.
C) TheGreeksfailedtodevelopareligionofanykind.
D) Greekreligionwasderivedfromabeliefinthespiritsofnatureelevatedintoacomplex
setofgods.
Answer: D
Diff:2
PageRef:105
Topic:GreekandHellenisticCulture
Skill:Conceptual

29) TheauthorssuggestthatthemysteryreligionsofGreecewhichofferedamorespiritual
approachtoreligionparalleledtheroleofwhatphilosophicalapproachinChina?
A) Confucianism
B) Legalism
C) Daoism
D) Buddhism
Answer: C
Diff:2
PageRef:105
Topic:GreekandHellenisticCulture
Skill:Conceptual

30) WhichofthefollowingwasNOTaGreekphilosopher?
A) Plato
B) Socrates
C) Aristophanes
D) Aristotle
Answer: C
Diff:1
PageRef:105
Topic:GreekandHellenisticCulture
Skill:Factual

72

31) WhichofthefollowingstatementsbestdescribesthefunctionofphilosophyinGreekculture?
A) itsoughttodeviseasystemofrewardsandpunishmentsintheafterworld
B) itestablishedethicalsystemsbasedonrationaldefinitionsofrightandwrong
C) itsetupethicalsystemsbasedonthestrugglebetweenthecosmicforcesofgoodand
evil
D) philosophywaslargelyignoredinHellenicsociety
Answer: B
Diff:2
PageRef:105
Topic:GreekandHellenisticCulture
Skill:Conceptual

32) Platobelievedthattruewisdomcouldonlycomefromanunderstandingofthe
A) perfectforms.
B) Analects.
C) materialworld.
D) properworkingsofdemocracy.
Answer: A
Diff:2
PageRef:105
Topic:GreekandHellenisticCulture
Skill:Conceptual

33) WhichofthefollowingstatementsmostaccuratelydescribestheGreekcontributiontoscience?
A) TheGreekswereunabletomakeanyadvancesovertheMiddleEasterncivilizations.
B) TheGreeksstronglyemphasizedempiricalresearchinscience,andmostoftheir
contributionsweretheresultofempiricalevidence.
C) TheGreekswereabletomakeadvancesinmedicineandscience,butnotinmathematics
andgeometry.
D) TheGreekswereprimarilyinterestedincreatingsweepingtheoriesbasedonabeliefin
theinherentrationalismofthecosmos.
Answer: D
Diff:2
PageRef:106
Topic:GreekandHellenisticCulture
Skill:Conceptual

73

34) Indrama,theGreeksplacedgreatestemphasison
A) tragedy.
B) comedy.
C) romance.
D) theepic.
Answer: A
Diff:1
PageRef:107
Topic:GreekandHellenisticCulture
Skill:Factual

35) Whichofthefollowingdramatistswrotedramaticcomedy?
A) Aeschylus
B) Sophocles
C) Aristophanes
D) Euripides
Answer: C
Diff:2
PageRef:107
Topic:GreekandHellenisticCulture
Skill:Factual

36) WhichofthefollowingstatementsmostaccuratelydescribesGreekarchitecture?
A) Greekarchitecturelackedalmostentirelyanymonumentalbuilding.
B) MostGreekmonumentalbuildingwasexecutedinbricksofstandardizedformwith
little,ifany,decoration.
C) Greekmonumentalarchitecturewasalmostentirelydecorativeandhadnoutilitarian
purpose.
D) Greekmonumentalarchitecture,oftensquareorrectangularinshapeanddecoratedwith
columns,wasintendedforavarietyofpublicuses.
Answer: D
Diff:2
PageRef:108
Topic:GreekandHellenisticCulture
Skill:Conceptual

74

37) WhichofthefollowingwasNOTaGreekarchitecturalstyle?
A) Corinthian
B) Doric
C) Theban
D) Ionian
Answer: C
Diff:1
PageRef:108
Topic:GreekandHellenisticCulture
Skill:Factual

38) WhichGreekscientistsuggestedthattheearthwasatthecenteroftheuniverse?
A) Socrates.
B) Plato.
C) Ptolemy.
D) Sophocles.
Answer: C
Diff:2
PageRef:110
Topic:GreekandHellenisticCulture
Skill:Conceptual

39) Whichofthefollowingstatementsmostaccuratelydescribestheculturalgapbetween
educatedeliteandthemassesinHellenicandHellenisticculture?
A) Therewaslessgapbetweenelitesandthemassesintermsofaunifiedculturethanin
eitherChineseorIndiancivilization.
B) TherewereagreaternumberofsharedassumptionsbetweenvariouslevelsofHellenic
andHellenisticsocietiesthanineitherChinaorIndia.
C) Thoughplaysandotherartformswerewidelyshared,theemphasisonphilosophy,and
thefactthatphilosophywasclosedtomostordinarypeople,createdatensionbetween
theeliteandthemasses.
D) TheriseofthekingdomsafterAlexandersdeathservedtomodifythefailureofthe
city-statestocreateanationalculturalsystembasedonsharedculturalattributes.
Answer: C
Diff:3
PageRef:109
Topic:GreekandHellenisticCulture
Skill:Conceptual

75

40) HowdidtheliteraryproductionofHellenisticGreekculturecomparetothatoftheHellenic
period?
A) Whilehistoriesceasedtobewrittenandtheepicdeclined,HellenisticGreecesawthe
zenithofdramaticproduction.
B) ForthefirsttimesincetheHomericperiod,newGreekepicswerewidelydisseminated.
C) LevelsofliteracyfellthroughouttheGreekworld,andmostoftheHellenicworkswere
lost.
D) Historicalinformationwaselaborated,andhistoricalbiographycameintoitsown.
Answer: D
Diff:2
PageRef:107
Topic:GreekandHellenisticCulture
Skill:Conceptual

41) Whichofthefollowingstatementmostaccuratelydescribesthenatureofscientific
achievementduringtheHellenisticperiod?
A) Scientificadvancewasmostpronouncedinastronomyandgeography.
B) ScientificstudywasnotsupportedbytheregionaldynastiesafterAlexandersdeathand
generallydeclined.
C) Inthefieldsofbiologyandmedicine,manynewadvancesweremade.
D) AfterthethirdcenturyB.C.,therewasarenaissanceofHellenisticscientificadvancethat
setnewstandardsforempiricalresearch.
Answer: A
Diff:2
PageRef:110
Topic:GreekandHellenisticCulture
Skill:Conceptual

42) WhichofthefollowingrepresentsadifferencebetweentheeconomyofGreeceandChina?
A) Chinawasnotanagriculturaleconomy.
B) GreecedependedmoreonslavelaborthandidChina.
C) Chinaseconomypaidlessattentiontoimprovingmanufacturingtechnology.
D) GreecefailedtocolonizeasdidChinaduringtheclassicalperiod.
Answer: B
Diff:2
PageRef:111
Topic:PatternsofGreekandHellenisticSociety
Skill:Conceptual

76

43) ThebulkoftheGreekpopulation
A) livedinthemajorurbancentersandweredependentonimportedfood.
B) weremerchantswhohadlimitedconnectionswiththeruralcountryside.
C) wereartisanswholivedinsmallvillagesspreadthroughoutthecountryside.
D) wasruralandlivedinfarmingvillagesspreadthroughoutthecountryside.
Answer: D
Diff:2
PageRef:110
Topic:PatternsofGreekandHellenisticSociety
Skill:Conceptual

44) WhichofthefollowingstatementsconcerningclassicalGreekslaveryisNOTtrue?
A) Slavesweremainlyacquiredasaresultofwar.
B) Between80,000and100,000peopleinAthensinthe5thcenturywereslaves.
C) Slaverywasanimportantingredientoftheeconomy.
D) Slaverywasbasedonrace.
Answer: D
Diff:2
PageRef:111
Topic:PatternsofGreekandHellenisticSociety
Skill:Conceptual

45) InGreekcivilization,women
A) wereconsideredthefullequalsofmales,despitethepatriarchalnatureofthehousehold.
B) wereprobablynotasoppressedasChinesewomen,butwereclearlyregardedasinferior
tomales.
C) couldholdpublicoffice,ownproperty,andconductbusinessintheirownnames.
D) wereregardedasequalintermsofthelaw,butwereregardedasinferiorculturally.
Answer: B
Diff:2
PageRef:112
Topic:PatternsofGreekandHellenisticSociety
Skill:Conceptual

77

46) WhichofthefollowingstatementsabouthomosexualityinGreekcultureismostaccurate?
A) HomosexualityinGreekculturewasstrictlyforbiddenasasocietaltaboo.
B) Homosexualrelationshipsoftenappearedbetweenyoungpeopleandtheirmentors,
particularlyamongmales.
C) HomosexualitywassowidelypracticedamongtheGreeksthatitdetractedfromtheir
devotiontomarriageandthehousehold.
D) HomosexualitywaspermittedinmostpartsofGreeksocietyexceptamongtheSpartans
whereitwasstrictlyforbiddenamongthemilitarycitizens.
Answer: B
Diff:2
PageRef:113
Topic:PatternsofGreekandHellenisticSociety
Skill:Conceptual

47) HowdidattitudestowardwomeninHellenisticGreececomparewiththoseinHellenic
Greece?
A) Therewaslittleornochangeinattitudestowardwomen.
B) ThepositionofwomeninHellenisticGreecedeterioratedundertheregionaldynasties.
C) WomeninHellenisticGreeceachievedfullparitywithmales.
D) ConditionsforwomeninHellenisticGreeceimprovedsomewhat,althoughtheyfailedto
achieveequality.
Answer: D
Diff:2
PageRef:112-113
Topic:PatternsofGreekandHellenisticSociety
Skill:Conceptual

48) WhowasthemotherofaHellenistickinginSpartawhowillinglyservedasahostagetohelp
formanalliancewithamorepowerfulstate?
A) Cratesiclea
B) Portia.
C) Calpurnia
D) Thesmophoria
Answer: A
Diff:2
PageRef:113
Topic:PatternsofGreekandHellenisticSociety
Skill:Conceptual

78

49) Oftheapproximately270,000peopleinfifthcenturyAthens,abouthowmanywereslaves?
A) 80,000to100,000
B) 250,000
C) 30,000to70,000
D) fewerthan10,000
Answer: A
Diff:3
PageRef:111
Topic:PatternsofGreekandHellenisticSociety
Skill:Factual

EssayQuestions
1) DescribeGreekclassicalcivilization?
Answer: ExtendedoverbroadregioninMediterranean,althoughnotpoliticallyunified;
demonstratednewpoliticalandculturalcapacitiesnewphilosophies,newscientific
andmathematicaladvances,elaborateformsoftrade
Diff:2
PageRef:97-100
Topic:GreeceasaClassicalCivilization
Skill:Conceptual

2) Whatwasthepolis?Whatwasitsmostcommonearlyformat?
Answer: ThepoliswastheGreekcity-state,themostcommonformofpoliticalorganizationin
Greekcivilization;consistedofurbancenterwithruralhinterland;onlyexercised
regionalauthority.Mostcommonearlyformwasaristocraticoligarchy;councilsplayed
greatroleincity-stategovernment;warriorsandfarmersmettodiscussissuesin
councils.
Diff:2
PageRef:97-100
Topic:ThePoliticalCharacterofClassicalGreece
Skill:Conceptual

3) Inwhatwayswerethefragmentedcity-statesofGreeceabletoworktogether?
Answer: AllparticipatedincolonizationofMediterraneantorelievepopulationpressureandto
openupnewsuppliesofgrain;commonlyparticipatedinsomerituals theOlympic
GamesandtherespectfortheoracleofApolloatDelphi;militarycooperationtodefeat
theinvasionsofthePersianempirein490and480B.C.
Diff:2
PageRef:100-102
Topic:ThePoliticalCharacterofClassicalGreece
Skill:Conceptual

4) HowdidthepoliticalstructureofHellenisticGreececontrastwiththatofHellenicGreece?
Answer: HellenicGreecetypifiedbycity-states;theycontinuedtoexistinHellenisticGreece,but
dominatedbyregionaldynasties;temporarycreationofgreatHellenisticempireunder
AlexandertheGreatfragmentedintoregionaldynastiesafterhisdeath.
Diff:2
PageRef:102-103
Topic:TheHellenisticPeriod
Skill:Conceptual

79

5) HowwasGreekpoliticsdifferentfromChina?
Answer: TheGreeksweredividedintocitystatesnotunifiedintoapoliticalempireasinChina;
GreeksemphasizedparticipationinpubliclifetheChinesedidnot,andtheGreekshad
lessemphasisonhierarchyandobediencethaninChina.
Diff:2
PageRef:100-102
Topic:Conclusion:AComplexLegacy
Skill:Conceptual

80

Chapter6 ReligiousRivalriesandIndiasGoldenAge
Multiple-ChoiceQuestions
1) Tribesofnomadicwarriorsthatprovidedthefoundationforpost-Harappancivilizationin
Indiaoriginallysettled
A) inthefoothillsoftheHimalayaMountains.
B) alongtheGangesrivervalley.
C) alongtheMahabhartarivervalley.
D) inthefoothillsoftheZagrosMountains.
Answer: A
Diff:2
PageRef:118
Topic:TheAgeofBrahmanDominance
Skill:Factual

2) Whichofthefollowingstatementsbestdescribestheearlykingdomsestablishedinnorthern
India?
A) Mostofthemwererepublicsruledcollectivelybyacouncilmadeupofthefreewarrior
elite.
B) LiketheGreekmainland,thekingdomswereactuallycity-statesruledbyaristocratic
councils.
C) Theearlykingdomswerebasedonastronglycentralizedmonarchywithwell
establishedbureaucracies.
D) Earlypoliticalorganizationamongthekingdomsamountedtolittlemorethanhunting
andgatheringgroups.
Answer: A
Diff:2
PageRef:119
Topic:TheAgeofBrahmanDominance
Skill:Conceptual

3) Whichofthefollowingstatementsbestdescribesthepositionofthebrahminsintheearly
AryankingdomsofnorthernIndia?
A) Brahminsdominatedallaspectsoflifeastutorsoftheprincesandadvisorstotheking.
B) BrahminswereexcludedfromtheAryanssocietyingeneral.
C) Thewarrioreliteswerecarefultokeepthepowerofthebrahminsincheckandfostereda
healthyskepticismoftheirreligiouspowers.
D) Therulersofthekingdomswerethemselvesbrahmanswhopracticedavarietyofrituals
intendedtomaintaintheircentralizedcontrolovertheirkingdoms.
Answer: C
Diff:2
PageRef:119
Topic:TheAgeofBrahmanDominance
Skill:Conceptual

81

4) AfterthespreadofAryankingdomstotheGangesriverplain,howdidthepolitical
organizationofthenewkingdomscomparetothatoftheolderones?
A) Thenewkingdomswerelesscentralized.
B) Thenewkingdomswererepublicsruledcollectivelybyacouncilmadeupofthefree
warriorelite.
C) Thenewkingdomsweremorecentralizedandauthoritarianwithhereditarymonarchs
andpowerfulbrahminpriests.
D) Thenewkingdomswerebasedoncity-stateorganizationsimilartotheGreekmainland.
Answer: C
Diff:2
PageRef:119
Topic:TheAgeofBrahmanDominance
Skill:Conceptual

5) WhatwasthepositionofthebrahminsinthekingdomsoftheGangesplainandsouthern
India?
A) Thepowerofthebrahminswascarefullycheckedbythewarrioreliteandthekingsof
thesouthernkingdoms.
B) Brahminsformedthedynastiesofthesouthernkingdomsandexercisedallauthority
withoutchallenge.
C) ThesouthernkingdomsrapidlyconvertedtoBuddhismanddiminishedtheauthorityof
thebrahmins.
D) Aseducators,bureaucrats,andreligiousauthorities,thebrahminsoftenheldmore
powerthanthemembersofthewarriorelitewhoservedaskings.
Answer: D
Diff:2
PageRef:119
Topic:TheAgeofBrahmanDominance
Skill:Conceptual

6) Whatwasthekeyfunctionthatassuredtheauthorityofthebrahminsinthesouthern
kingdoms?
A) Onlythebrahminswerecapableofperformingtheritualsandsacrificesthatobligedthe
godstointerveneinhumanaffairs.
B) Asthewarriorelite,thebrahminscontrolledthemilitaryaffairsofthesouthern
kingdoms.
C) AsthecommercialclassofIndia,thebrahminswereentirelyresponsibleforthe
economicprosperityofthesouthernkingdoms.
D) Thebrahminclassprovidedthemembersofallofthedynastiesofthesouthern
kingdoms.
Answer: A
Diff:2
PageRef:119-120
Topic:TheAgeofBrahmanDominance
Skill:Conceptual

82

7) WhatwasthedegreeofurbanizationinthesouthernkingdomsoftheGangesplain?
A) AswastypicalofAryanculture,thesouthernkingdomshadvirtuallynocitiesortowns.
B) Somesmallurbancentersgrewuparoundthepalaces,buttherewasnocommerceor
manufacturingassociatedwiththem.
C) Townsappearedaroundthekingdomscapitalsandalsoasmanufacturingandtrading
centersalongtheGangesRiver.
D) Ascity-states,thesouthernkingdomswereentirelybasedonanurbanculturethat
rivaledtheurbanizationofChina.
Answer: C
Diff:2
PageRef:120
Topic:TheAgeofBrahmanDominance
Skill:Conceptual

8) WhatwasthestatusofmerchantsinthesouthernkingdomsoftheGangesplain?
A) LikeChina,merchantsamongtheAryankingdomsweredespisedand,although
wealthy,heldlowsocialstatus.
B) Thelackofcommercewithinthesouthernkingdomsleftthesmallmerchantclasspoor
andlow-caste.
C) Therewasnosocialclassorvarna formerchantswhowerelumpedtogetherwithartisans
andpeasants.
D) Thegreatwealthamassedbymerchantsallowedthemtowinaprominentplaceinthe
Indiansocialhierarchy.
Answer: D
Diff:2
PageRef:120
Topic:TheAgeofBrahmanDominance
Skill:Conceptual

9) WhatwastheIndiannameforthecategoriesinthesocialorder?
A) Aryans
B) Brahmins
C) Varnas
D) Untouchables
Answer: C
Diff:2
PageRef:121
Topic:TheAgeofBrahmanDominance
Skill:Factual

83

10) WhatdeterminedapersonsplacewithintheIndiansocialhierarchy?
A) Wealth
B) Positionwithinthegovernment
C) Thedegreetowhichtheoccupationwasconsideredpolluting
D) Hisreligiouspiety
Answer: C
Diff:2
PageRef:121
Topic:TheAgeofBrahmanDominance
Skill:Conceptual

11) ThecasteattheverybottomoftheIndiansocialhierarchywasthe
A) brahmins.
B) artisans.
C) peasants.
D) untouchables.
Answer: D
Diff:2
PageRef:121
Topic:TheAgeofBrahmanDominance
Skill:Factual

12) WhichofthefollowingwasNOTameansofdistinguishingbetweencastegroups?
A) Diet
B) AbilitytobatheintheGanges
C) AbilitytoreadtheVedas
D) Intermarriagewithothercastes
Answer: B
Diff:2
PageRef:122
Topic:TheAgeofBrahmanDominance
Skill:Conceptual

13) Thecastepositionandlifepathdeterminedbyapersonsbirthwascalledones
A) varna.
B) ramayana.
C) dharma.
D) mantra.
Answer: C
Diff:2
PageRef:123
Topic:TheAgeofBrahmanDominance
Skill:Factual

84

14) Thetransferofapersonssoulfromonebodytoanotherafterdeathiscalled
A) Kamasutra.
B) suttee.
C) mitigation.
D) transmigration.
Answer: D
Diff:1
PageRef:123
Topic:TheAgeofBrahmanDominance
Skill:Factual

15) Whichofthefollowingdeterminedthesortofpersonthatasoulwouldbeattachedtoinits
nextreincarnation?
A) Dharma
B) Karma
C) Wealth
D) Mandala
Answer: B
Diff:2
PageRef:123
Topic:TheAgeofBrahmanDominance
Skill:Factual

16) Howdidtheclassicalcivilizationsviewofsocialorganizationdifferfromthatofmodern
Westerncivilization?
A) Classicalcivilizationshadpoorlydefinedsocialgroupings.
B) Classicalcivilizationsfeltthatsocialmobilitywascriticaltothewell-beingofsociety.
C) Modernsocietiesarepredicatedonsocialmobilitybasedontalentandtheworkethic.
D) Modernsocietieslackanykindofsocialgroupingsdesignedtoimplystatus.
Answer: C
Diff:2
PageRef:122-23
Topic:TheAgeofBrahmanDominance
Skill:Conceptual

17) ThetwogreatepicsofthepreimperialperiodinIndiawerethe
A) VedasandtheUpanishads.
B) MahabharataandtheRamayana.
C) Kamasutra andtheMandala.
D) ChandraguptaandtheBindusara.
Answer: B
Diff:3
PageRef:124
Topic:TheAgeofBrahmanDominance
Skill:Factual

85

18) WhatwasthehouseholdstructureofpreimperialIndia?
A) Nuclearfamiliesforallcastegroups
B) Extendedfamiliesforallcastegroups
C) Extendedhouseholdsforhigh-castegroups,nuclearhouseholdsforlower-castegroups
D) Nuclearhouseholdsforhigh-castegroups,extendedfamiliesforlower-castegroups
Answer: C
Diff:2
PageRef:124
Topic:TheAgeofBrahmanDominance
Skill:Conceptual

19) WhatwasthestatusofwomeninthepreimperialperiodinIndia?
A) Withinthefamilytheywereclearlysubordinatetomen,butenjoyedgreater
occupationalopportunitiesthanwasthecasebythelastcenturiesB.C.E.
B) Womenwereclearlysubordinatewithinthehouseholdandwereunabletoholdany
occupationsoutsideofit.
C) Womenenjoyedgreatfreedomwithinthehousehold,butwereunabletoholdany
occupationsoutsideofit.
D) Womenenjoyedequalitywithinthehousehold,butwererestrictedfromreadingthe
sacredtexts.
Answer: A
Diff:2
PageRef:124
Topic:TheAgeofBrahmanDominance
Skill:Conceptual

20) WhenwerethetwogreatIndianepics,theMahabharata andtheRamayanawritten?


A) Before1500B.C.E.
B) Between1200and900B.C.E.
C) Between900and500B.C.E.
D) InthelastcenturiesB.C.E.
Answer: D
Diff:2
PageRef:124
Topic:TheAgeofBrahmanDominance
Skill:Factual

86

21) WhichofthefollowingstatementsconcerningthelifeofBuddhaisNOTtrue?
A) Buddhawasbornintooneofthebrahminclans.
B) Buddhawassupposedlythesonandheirofalocalruler.
C) Buddharenouncedhisclaimstosucceedhisfatherandbecameawanderingholyman.
D) BuddhareachedenlightenmentwhilesittingunderaBotree.
Answer: A
Diff:2
PageRef:125
Topic:ReligiousFermentandtheRiseofBuddhism
Skill:Factual

22) TheBuddhiststateofenlightenmentiscalled
A) dharma.
B) karma.
C) Nirvana.
D) Kamasutra.
Answer: C
Diff:1
PageRef:126
Topic:ReligiousFermentandtheRiseofBuddhism
Skill:Factual

23) WhichofthefollowingwasNOTateachingofBuddha?
A) Themomentwearebornwebegintodie.
B) Attachmentstoimpermanentthingsoftheworldarethesourceofsuffering.
C) Theroadtoenlightenmentbeginswithritualsacrificestothegods.
D) Onceenlightenmentisattained,theindividualisreleasedfromsuffering.
Answer: C
Diff:3
PageRef:125-26
Topic:ReligiousFermentandtheRiseofBuddhism
Skill:Conceptual

24) Afterhisdeath,Buddha
A) wasworshippedasadivinity.
B) ascendedintoheavenfromamountaininnorthernIndia.
C) reappearedreflectedinthewatersoftheGangesRiver.
D) waslargelyforgotten.
Answer: A
Diff:2
PageRef:126
Topic:ReligiousFermentandtheRiseofBuddhism
Skill:Conceptual

87

25) HowdidthemonksdevotedtoBuddhismchangehisteachingsintheyearsafterBuddhas
death?
A) TheyofferedmiraculoustalesofBuddhaslife,equatedNirvanawithheaven,and
stressedthesalvationistqualitiesofthenewreligion.
B) TheysoughttodownplayBuddhaasadeity,attemptedtolimitadmissiontothereligion
totheupper-castegroups,andgainedthecooperationofthebrahmins.
C) Theybegantoemphasizeritualsacrificesofanimalsasameansofgainingtheconfidence
ofthepeople.
D) ThemonksendedtheBuddhasemphasisonmeditationandtaughtthatallmenwere
condemnedtoendlessreincarnation.
Answer: A
Diff:2
PageRef:126
Topic:ReligiousFermentandtheRiseofBuddhism
Skill:Conceptual

26) WhatwastheattitudeofBuddhismtowardthecastesystem?
A) WhiletheBuddhistsdidnotregardsocialstratificationascriticaltothefaithful,they
onlyacceptedmembersoftheupper-castegroupsasmonks.
B) Buddhistsrejectedthecastesystemandadmitteduntouchablesandwomenasmembers
ofthefaith.
C) TheBuddhistsacceptedthecastesystementirelyandincorporateditintotheirreligion.
D) TheBuddhistsrecognizedonlywealthasameansofdefiningonessocialposition.
Answer: B
Diff:2
PageRef:127
Topic:ReligiousFermentandtheRiseofBuddhism
Skill:Conceptual

27) WhichofthefollowingwasNOTaBuddhistchallengetotheauthorityofthebrahmins?
A) TheBuddhafavoredintrospectionandself-masteryoverritualandsacrifice.
B) TheBuddharejectedtheVedas asdivinelyinspiredteaching.
C) TheBuddharejectedbothkarma andreincarnation.
D) TheBuddharejectedthelifestylesofthebrahminsbasedonworldlypowerand
authority.
Answer: C
Diff:2
PageRef:127
Topic:ReligiousFermentandtheRiseofBuddhism
Skill:Conceptual

88

28) ThepersonresponsiblefortheGreekinvasionoftheIndusrivervalleywas
A) PhilipIIofMacedon.
B) AlexandertheGreat.
C) Dionysos.
D) PericlesofAthens.
Answer: B
Diff:1
PageRef:127
Topic:ReligiousFermentandtheRiseofBuddhism
Skill:Factual

29) TheGreekinvasionofnorthwesternIndialastedfrom
A) 496to480B.C.E.
B) 354to350B.C.E.
C) 327to324B.C.E.
D) 153to149B.C.E.
Answer: C
Diff:2
PageRef:127
Topic:ReligiousFermentandtheRiseofBuddhism
Skill:Factual

30) WhichofthefollowingwasNOTanimpactoftheGreekinvasionofnorthwesternIndia?
A) ItstimulatedtradebetweenIndiaandtheWest.
B) GreekastronomicalandmathematicalideasenteredIndia.
C) IndianreligiousphilosophyimpactedtheStoicsandmysteryreligionsofWestern
civilizations.
D) AcentralizedGreekstatewascreatedthatdominatedtheIndusrivervalleyforfour
centuries.
Answer: D
Diff:2
PageRef:127
Topic:ReligiousFermentandtheRiseofBuddhism
Skill:Conceptual

31) WhatwasthefirstimperialdynastyofclassicalIndia?
A) Kushanas
B) Hamdanids
C) Mauryas
D) Guptas
Answer: C
Diff:2
PageRef:127
Topic:ReligiousFermentandtheRiseofBuddhism
Skill:Factual

89

32) ThefirstrulerofimperialIndiaduringtheclassicalperiodwas
A) Ashoka.
B) Bindusara.
C) ChandraguptaMaurya.
D) BindaPasha.
Answer: C
Diff:2
PageRef:127
Topic:ReligiousFermentandtheRiseofBuddhism
Skill:Factual

33) HowdidtheMauryanempireofChandraguptacomparetotheAryankingdomsthat
precededit?
A) Althoughcalledanempire,itwaslittlemorethanatribute-collectingpatchworkofpetty
rulers.
B) ItretainedtherepublicanaspectsofthefirstAryankingdomswithelectedmonarchsand
warriorcouncils.
C) WhileitdidsucceedindominatingmostofnorthwesternIndia,itneverconqueredthe
kingdomsofsouthernIndia.
D) Itwasvastlymorecentralizedthantheprecedingkingdomswithalargestandingarmy
andadministratorsinplaceoftheregionallords.
Answer: D
Diff:2
PageRef:128
Topic:ReligiousFermentandtheRiseofBuddhism
Skill:Conceptual

34) WhichofthefollowingstatementsconcerningtheruleofAshokaisNOTtrue?
A) HeconvertedtoHinduism
B) Hecompletedtheconquestsbegunbyhisfatherandgrandfather
C) Hisearlycareerwaspredicatedonviolence,cruelty,andwarfare
D) Hisreligiousconversionledtoadrasticchangeinhisrulingmethods
Answer: A
Diff:2
PageRef:128
Topic:ReligiousFermentandtheRiseofBuddhism
Skill:Conceptual

90

35) WhichofthefollowinggroupsviewedAshokasreignasathreat?
A) Buddhistmonks
B) Merchants
C) Peasants
D) Brahmins
Answer: D
Diff:2
PageRef:128
Topic:ReligiousFermentandtheRiseofBuddhism
Skill:Factual

36) WhichofthefollowinggroupswasNOToffendedbyAshokasadministrativepractices?
A) Brahmins
B) Warriorfamiliesthathadonceruledsmallstates
C) Buddhistmonks
D) Thepopulaceasawhole
Answer: C
Diff:2
PageRef:128
Topic:ReligiousFermentandtheRiseofBuddhism
Skill:Conceptual

37) WhichgroupsdidAshokassocialpoliciestendtobenefit?
A) Brahminsandwarriors
B) Merchants,women,andartisans
C) Peasants
D) Untouchables
Answer: B
Diff:2
PageRef:128-29
Topic:ReligiousFermentandtheRiseofBuddhism
Skill:Conceptual

38) Buddhistartandarchitecturewerepreservedinthestoneshrinesbuilttoholdrelicsofthe
Buddhaandcalled
A) mandalas.
B) Kamasutra.
C) Vedas.
D) stupas.
Answer: D
Diff:1
PageRef:129
Topic:ReligiousFermentandtheRiseofBuddhism
Skill:Factual

91

39) WhatwastheresultofBuddhistmissionaryeffortsduringthereignofAshoka?
A) Despitemajorefforts,BuddhismfailedtospreadmuchbeyondtheIndiansubcontinent.
B) ThroughtheremainingGreekinfluenceinnorthwesternIndia,Buddhismspreadtothe
areasaffectedbyGreekculture.
C) BuddhismspreadfirsttoSriLankaandtheHimalayankingdomsandfromthereto
centralandsoutheasternAsia.
D) BuddhismsspreadwaslimitedtothepastoralnomadsofthecentralAsiasteppes.
Answer: C
Diff:2
PageRef:129
Topic:ReligiousFermentandtheRiseofBuddhism
Skill:Conceptual

40) BywhatdatehadtheMauryaempiredisappeared?
A) 350B.C.E.
B) 185B.C.E.
C) 9C.E.
D) 185C.E.
Answer: B
Diff:3
PageRef:129
Topic:ReligiousFermentandtheRiseofBuddhism
Skill:Factual

41) WhyweretheBuddhistsvulnerabletotherestorationofbrahminicauthority?
A) TheimperialdynastiesfromtheMauryasonhadremainedsteadfastlyloyaltothe
brahmins.
B) TheBuddhistmonksbecameincreasinglyisolatedfromtherestoftheIndianpopulation.
C) Buddhismhadneverexpandedmuchbeyondthekingdomsinthefoothillsofthe
Himalayas.
D) Buddhismonlyappealedtotheupper-casteclassesofIndiansociety.
Answer: B
Diff:2
PageRef:131
Topic:BrahmanicalRecoveryandtheSplendorsoftheGuptaAge
Skill:Conceptual

92

42) WhichofthefollowingwasNOTatacticutilizedbythebrahminstomaketheirreligionmore
accessibletoallIndians?
A) Theyreducedtheemphasisonlargesacrificesandstressedtheimportanceofsmall
offeringsoffoodandprayers.
B) Templesdevotedtospecialgodsallocatedtospecificgroupsservedasafocusfor
popularworship.
C) Womenwerepermittedtoparticipateindevotionalcultsaspoetsandsingers.
D) HinduismclearlyrejectedanyinclusionofBuddhistbeliefsinthebrahmanicreligion.
Answer: D
Diff:2
PageRef:131-32
Topic:BrahmanicalRecoveryandtheSplendorsoftheGuptaAge
Skill:Conceptual

43) WhateconomicfactorweakenedtheholdofBuddhismontheIndianpopulation?
A) Theimprovingstatusofthepeasantrymadethemlesslikelytobeinterestedina
salvationistreligion.
B) Theuntouchablesweredecimatedbyaseverefamine,thusremovingaclassassociated
withBuddhism.
C) ThefailureoftheRome-Chinatradeaxisweakenedthemerchants,whohadbeenoneof
thechiefbenefactorsoftheBuddhistmonasteries.
D) Theincreasingwealthofallsocialclassesreducedthestrictnessofsocialdivisionwithin
thecastesystemandunderminedoneoftheattractionsofBuddhism.
Answer: C
Diff:2
PageRef:132
Topic:BrahmanicalRecoveryandtheSplendorsoftheGuptaAge
Skill:Conceptual

44) HowdidtheGuptaempirecompareintermsofpoliticalorganizationtotheMauryaempire?
A) Theirdomainswerenotnearlyaslarge,nordidtheyexercisesuchcentralizedcontrol
overthepettystates.
B) TheGuptaempirewasmuchlargerthantheMauryanandalsomorecentralized.
C) TheGuptaempirewasbasedontheconceptoflinkedcity -statesunliketherepublican
basisoftheMauryanempire.
D) TheGuptaemperorspossessedamuchmoreprofessionalizedbureaucracyonthemodel
oftheChinesescholar-gentry.
Answer: A
Diff:2
PageRef:132-33
Topic:BrahmanicalRecoveryandtheSplendorsoftheGuptaAge
Skill:Conceptual

93

45) WhatwasthestatusofthebrahminsundertheGuptas?
A) TheGuptasconvertedtoBuddhismandreducedtheinfluenceofthebrahmins.
B) ThebrahminsrecoveredsomeofthegroundlosttoBuddhistmonks,butwereunableto
achievetheirformerdominance.
C) ThebrahminsonceagainservedastheliterateadministratorsoftheGuptas,buttheir
influencewaslimitedtotheimperialcourt.
D) ThebrahminsrecoveredtheirformerpositionsofdominancethroughoutIndiansociety
asteachers,politicaladministrators,andreligiousauthorities.
Answer: D
Diff:2
PageRef:133
Topic:BrahmanicalRecoveryandtheSplendorsoftheGuptaAge
Skill:Conceptual

46) HowwasthesocialhierarchyofIndiatransformedduringthereignoftheGuptas?
A) Ingeneralrestrictionsagainstlow-castegroupsandwomenbecameharsherandmore
pervasive.
B) Whilerestrictionsagainstuntouchablesbecamemorestrict,womenachievedgreater
equalityduringtheGuptaperiod.
C) Brahminsandotherupper-castegroupslosttheirclearsocialdistinctivenessasIndian
societybecamemoreopentosocialmobility.
D) Thecasteswereabandonedandreplacedbythetripartitesystemofgeneralvarnas.
Answer: A
Diff:2
PageRef:134
Topic:BrahmanicalRecoveryandtheSplendorsoftheGuptaAge
Skill:Conceptual

47) WhatwasthetitleoftheIndianhandbookforyoungmenincludingtipsonhygiene,
grooming,etiquette,andlove-making?
A) Ramayana
B) Kamasutra
C) Mahabharata
D) Arthashastra
Answer: B
Diff:1
PageRef:135
Topic:BrahmanicalRecoveryandtheSplendorsoftheGuptaAge
Skill:Factual

94

48) WhatwasthenatureoftheeconomyduringtheGuptaperiod?
A) ThecontinuedprosperityoftheChina-Romeaxisallowedthecontinueddevelopment
ofinternationaltrade.
B) TheIndianeconomyenteredaperiodofseveredepressionduringwhichmosttradeand
manufacturingceased.
C) AlthoughtheChina-Rometradeaxisdeclined,Indiaremainedthehubofthegreat
IndianOceantradingnetwork.
D) TechnologicalcruditylimitedtheabilityofIndianmanufacturingandagricultural
productivitytoexpand.
Answer: C
Diff:2
PageRef:137
Topic:BrahmanicalRecoveryandtheSplendorsoftheGuptaAge
Skill:Conceptual

EssayQuestions
1) WhatwerethesourcesofbrahminicpowerintheearlyAryankingdomsoftheGangesplain?
Answer: Controllededucationofroyalfamilies,asliterateclassdominatedbureaucracyand
administration;onlyclassthatknewreligiousritesthatconferreddivinestatusonkings;
hadsoleabilitytomediatebetweengodsandhumans ,offeredsacrificesbasedonVedas;
properexecutionofritualsobligedgodstograntwishes.
Diff:2
PageRef:119-125
Topic:TheAgeofBrahmanDominance
Skill:Conceptual

2) Discussthestructureofthecastesystemasitdevelopedfrom1000B.C.E.totheendofthe
Guptaperiod.
Answer: Originallyonlythreebroadvarnas: warriors,brahmins,andpeasants;underkingdoms
ofGangesplainsocialsystembecamemorerigidandmorecomplex; varnasdividedinto
castes;attopbrahmans,warriors,andmerchants;nextpeasantsandartisans;atbottom
theuntouchables;statusbasedondegreeofpollutionrequiredinoccupationofones
caste;bytimeofGuptas,restrictionsagainstlowercastesbecameharsherandmore
strict;statusofwomenalsodeclinedbyGuptaperiod.
Diff:2
PageRef:119-137
Topic:TheAgeofBrahmanDominance
Skill:Conceptual

3) HowdidtheriseofBuddhisminthe5thcenturyB.C.E.challengethedominanceofthe
brahmans?
Answer: MonasticorganizationsassociatedwithBuddhismprovidedaviablealternativeto
brahminpriesthoodandwerenotdependentonsocialstatus;Buddhismquestioned
authorityofVedasonwhichbrahminauthoritydepended;challengedefficacyof
sacrifices;Buddhistssoughttoestablishsocialmobilitybyallowingmenandwomenof
allcastesintothemonasteries.
Diff:2
PageRef:125-127
Topic:ReligiousFermentandtheRiseofBuddhism
Skill:Conceptual

95

4) HowdidtheMauryanempirerepresentasignificantbreakwiththepasttraditionofIndian
civilization?
Answer: MauryanemperorscreatedfirstcentralizedempireinIndiancivilization;basedon
standingarmy,professionalizedadministration,replacementoflocalrulersby
representativesofemperor;alsobrokewithtraditionofbrahminicdominanceby
conversionofAshokatoBuddhism;displacedbrahminsaschiefadministratorsand
imperialadvisors;forbadeanimalsacrificesonwhichbrahminicritualsdepended;
growthofwealthandinfluenceofmerchants.
Diff:2
PageRef:127-129
Topic:ReligiousFermentandtheRiseofBuddhism
Skill:Conceptual

5) CompareandcontrastthepoliticalandreligiousorganizationoftheGuptaempirewiththe
Mauryaempire.
Answer: TheGuptaempirewasnottrulycentralized;pettyrulersremainedincontrolofregional
statesinreturnforpayingtributetotheemperor;restorationofdominanceofbrahmins;
returntorevitalizedHinduismrestoredreligiousauthorityofbrahmins;alsoreturned
asregionaladvisorsandteachersofrulers;Buddhismgraduallylostinfluenceasa
majorreligioninIndia.
Diff:2
PageRef:127-137
Topic:BrahmanicalRecoveryandtheSplendorsoftheGuptaAge
Skill:Conceptual

96

Chapter7 RomeanditsEmpire
Multiple-ChoiceQuestions
1) RomeparalleledGreeceinallofthefollowingwaysEXCEPTwhichofthefollowing?
A) Economicstructure
B) Theabilitytoestablishalong-lastingempire
C) Culture
D) Politics
Answer: B
Diff:2
PageRef:142
Topic:Introduction
Skill:Conceptual

2) WhichNorthAfricancity-statedidtheRomanstwicedefeatinthe3rdcenturyBCE?
A) Athens
B) Sparta
C) Venice
D) Carthage
Answer: D
Diff:2
PageRef:142
Topic:Introduction
Skill:Conceptual

3) WhendidthepeoplewhobecametheRomansmigratetotheItalianpeninsula?
A) 2500B.C.E.
B) 1000B.C.E.
C) 1500B.C.E.
D) 2000B.C.E.
Answer: B
Diff:2
PageRef:142
Topic:TheDevelopmentofRomesRepublic
Skill:Factual

97

4) ThegroupthatfirstruledtheRomanswerethe
A) Etruscans.
B) Aryans.
C) Harappans.
D) Varangians.
Answer: A
Diff:2
PageRef:142
Topic:TheDevelopmentofRomesRepublic
Skill:Factual

5) TheRomanaristocracyrebelledagainstEtruscankingsandexpelledthemaround
A) 1000B.C.E.
B) 550B.C.E.
C) 510B.C.E.
D) 476C.E.
Answer: C
Diff:1
PageRef:142
Topic:TheDevelopmentofRomesRepublic
Skill:Factual

6) WhichofthefollowingofficesandinstitutionswasNOTpartoftheRomanrepublic?
A) Senate
B) Consuls
C) Kings
D) Tribunes
Answer: C
Diff:2
PageRef:142-143
Topic:TheDevelopmentofRomesRepublic
Skill:Factual

7) Howwerethetwochiefexecutives(consuls)chosen?
A) Itwasaninheritedposition.
B) Theyweredirectsuccessorstothetwokingswhohadruledpreviously.
C) OnewaschosenfromeachofthehalvesofthecityofRome.
D) Theywereelectedbyanannualassembly.
Answer: D
Diff:2
PageRef:143
Topic:TheDevelopmentofRomesRepublic
Skill:Conceptual

98

8) InwhatwaywastheRomanrepublicsimilartoGreekpoliticalconcepts?
A) TheofficeoftribunewasderivedfromGreekprecedents.
B) Officersheldtheirpositionscommonlyforlife.
C) Theconstitutionbalancedvariousinterests,butreliedheavilyonthearistocracy.
D) TheconstitutionwasentirelydemocraticontheAthenianmodel,andfeaturedelection
bylot.
Answer: C
Diff:2
PageRef:143
Topic:TheDevelopmentofRomesRepublic
Skill:Conceptual

9) InwhatwaydidRomansocietydifferfromtheGreeks?
A) TheRomanagriculturaleconomyfeaturedmanylargelandholders.
B) TheRomansdidnotdependonslavery.
C) TherewasonlyaverysmallurbanpopulationinRomansociety.
D) TheRomansemphasizedclientagerelationshipsbetweenlandlordsandlessercitizens.
Answer: D
Diff:2
PageRef:143
Topic:TheDevelopmentofRomesRepublic
Skill:Conceptual

10) Intheprocessofmilitaryexpansion,RomeschiefrivalinthewesternMediterraneanwas
A) Syracuse.
B) theGermans.
C) Sparta.
D) Carthage.
Answer: D
Diff:1
PageRef:144
Topic:TheDevelopmentofRomesRepublic
Skill:Factual

11) WhatwereRomesconflictswithCarthageforcontrolofthewesternMediterraneancalled?
A) MacedonianWars
B) JugurthineWars
C) PunicWars
D) PropontineWars
Answer: C
Diff:1
PageRef:144
Topic:TheDevelopmentofRomesRepublic
Skill:Factual

99

12) WhowasthegreatCarthaginiangeneralduringtheSecondPunicWar?
A) AlexandertheGreat
B) Hannibal
C) Tiberius
D) AttilatheHun
Answer: B
Diff:2
PageRef:144
Topic:TheDevelopmentofRomesRepublic
Skill:Conceptual

13) DuringtheRepublicanera,whatwasthetypicalattitudeoftheRomanstowardareas
conqueredoutsideofItaly?
A) ResidentsweremadefullcitizensofRome.
B) Allresidentswerereducedtoslavery.
C) OpponentsofRomanrulewerepunishedseverelyandtherewererepeatedatrocities.
D) TreatmentofconqueredterritoriesintheeasternMediterraneanwasbrutal,but
treatmentofthoseinthewesternMediterraneanresultedingrantsofcitizenship.
Answer: C
Diff:2
PageRef:144
Topic:TheDevelopmentofRomesRepublic
Skill:Conceptual

14) WhichofthefollowingwasNOTaresultoftheimperialexpansionoftheRomanrepublic?
A) Newtradeandwealthenrichedtheupperclassesandopenedagulfbetweenrichand
poor.
B) ItbecameincreasinglydifficulttomanageRomesbalancedrepublicanconstitution.
C) Slaveryincreased.
D) CentralItalybecameincreasinglydevotedtothecommercialproductionofgrain.
Answer: D
Diff:2
PageRef:144
Topic:TheDevelopmentofRomesRepublic
Skill:Conceptual

100

15) OneofthechiefresultsofthechangeintheagriculturaleconomyoftheRomanrepublic
followingimperialexpansionwas
A) theincreasingwealthofthesmallfarmer.
B) thedisappearanceofslaveryfromtheItalianpeninsula.
C) thestrengtheningofthetiesofclientagethatexistedinthecountryside.
D) themigrationofdebt-riddenfarmersintoRome,swellingthepopulationtoover1
million.
Answer: D
Diff:2
PageRef:144
Topic:TheDevelopmentofRomesRepublic
Skill:Conceptual

16) WhichofthefollowingpoliticalfiguresattheendoftheRomanrepublicwasNOTageneral?
A) TiberiusGracchus
B) Marius
C) Sulla
D) Pompey
Answer: A
Diff:1
PageRef:145
Topic:TheDevelopmentofRomesRepublic
Skill:Factual

17) WhatwasMariusschiefcontributiontothepoliticalconflictattheendoftherepublic?
A) Heintroducedaprogramoflandredistribution.
B) HeproposedextendingcitizenshiptotheItalianallies.
C) Hebeganusingpaidvolunteersinhisarmies.
D) HedefeatedPompeyandputanendtotherepublic.
Answer: C
Diff:2
PageRef:145
Topic:TheDevelopmentofRomesRepublic
Skill:Conceptual

18) WhatopponentdidJuliusCaesardefeatinordertotakecontroloftherepublic?
A) GaiusGracchus
B) Sulla
C) Marius
D) Pompey
Answer: D
Diff:2
PageRef:145
Topic:TheDevelopmentofRomesRepublic
Skill:Factual

101

19) WhichofthefollowingwasNOTachangebroughtaboutduringtheruleofJuliusCaesar?
A) TheSenatewasabolished.
B) CaesarbroughtmoreItaliansandotherprovincialsintopoliticsattheexpenseofold
Romanfamilies.
C) Caesarwasnameddictatorforlife.
D) Caesarnamedmanynewofficialsandjudges.
Answer: A
Diff:2
PageRef:145-46
Topic:TheDevelopmentofRomesRepublic
Skill:Conceptual

20) After13yearsofcivilwar,whosucceededJuliusCaesar?
A) MarkAnthony
B) Pompey
C) Octavian
D) Brutus
Answer: C
Diff:1
PageRef:146
Topic:TheDevelopmentofRomesRepublic
Skill:Factual

21) WhichofthefollowingstatementsmostaccuratelydescribesCicero?
A) HewasaradicalpoliticianwhosupportedJuliusCaesarandwhoarguedforthe
destructionoftherepublic.
B) Hewasaconservativepoliticianwhofavoredretentionoftherepublic.
C) CiceroarguedstronglyfortheexclusionofGreekinfluencesfromRomancultureand
literature.
D) CicerosupportedOctavianandemergedasamajorfigureduringtheearlyRoman
empire.
Answer: B
Diff:2
PageRef:145
Topic:TheDevelopmentofRomesRepublic
Skill:Conceptual

102

22) WhatwastheprimarydifferencebetweenRomanandGreekreligion?
A) RomanreligionfeaturedmonotheismincontrasttotheGreekpantheonofgods.
B) Thenamesofthegodsandgoddesseswerechanged.
C) Greekgodsandgoddessesweremorelikelytogetinvolvedinthelivesofmortals.
D) GreekreligionfeaturedmonotheismincontrasttotheRomanpantheonofgods.
Answer: B
Diff:2
PageRef:146
Topic:RomanCulture
Skill:Conceptual

23) WhatwasthemajordifferencebetweenRomanandGreekarchitecturalforms?
A) Romeabandonedmonumentalarchitecturealmostentirelyforbuildingsofsmallerscale.
B) RomeabandonedtheGreekdesignsandintroducedlessornate,morefunctional
architecturaldesigns.
C) Romemadeengineeringadvancesthatallowedconstructionofbuildingsofgreatersize.
D) RomanarchitecturewasentirelyItalianinoriginandowednothingtoGreekinfluence.
Answer: C
Diff:2
PageRef:146
Topic:RomanCulture
Skill:Conceptual

24) WhichofthefollowingRomanliteraryfigureswasNOTpatronizedbyAugustus?
A) Ovid
B) Livy
C) Vergil
D) Horace
Answer: A
Diff:2
PageRef:148
Topic:RomanCulture
Skill:Factual

25) WhatwasVergilsepicpoemcelebratingthefoundationofRome?
A) Metamorphosis
B) ArsAmoris
C) Decameron
D) Aeneid
Answer: D
Diff:3
PageRef:148
Topic:RomanCulture
Skill:Factual

103

26) WhichofthefollowingstatementsconcerningHoraceistrue?
A) Asadevotedrepublican,hispoetryattackedthenewdynastyoftheRomanempire.
B) HerejectedtheGreekpoeticmetersandestablishedapurelyRomanpoeticstyle.
C) Althoughinitiallyarepublican,heshiftedsupporttoAugustusandcelebratedthe
emperorandhisfamily.
D) HewasbestknownforhisepicpoemscelebratingthefoundationofRome.
Answer: C
Diff:2
PageRef:148
Topic:RomanCulture
Skill:Conceptual

27) Whatwastheattitudeoftheimperialadministrationtolocalgovernmentswithintheempire?
A) Localgovernmentswerereplacedbymilitarygarrisonsdirectlyresponsivetothecentral
government
B) Localgovernmentswerereplacedbyprofessionalbureaucratsformallytrainedat
imperialuniversities.
C) Localgovernmentswerehandedovertorelativesoftheimperialfamilywhoruledas
vassalsoftheempire.
D) Localgovernmentsweregrantedconsiderableautonomy,andinsomecaseswhole
kingdomswerepreserved.
Answer: D
Diff:2
PageRef:148
Topic:TheInstitutionsofEmpire
Skill:Conceptual

28) WhatwastheRomanempiresattitudetowardextensionofcitizenship?
A) Unliketherepublic,theempireextendedcitizenshiptoimportantgroupsoutsideof
RomeandtheItalianpeninsula.
B) Liketherepublic,theempirerestrictedcitizenshiptotheoriginaltribesofthecityof
Rome.
C) TheempireextendedcitizenshiptothealliesoftheItalianpeninsula,butnofarther.
D) ThecitizenshipoftheRomanempirewasactuallysmallerthanthatoftherepublic.
Answer: A
Diff:2
PageRef:150
Topic:TheInstitutionsofEmpire
Skill:Conceptual

104

29) WhatwasAugustusattitudetowardtherepublicaninstitutionsafterhisestablishmentofthe
empire?
A) Augustusmodeledtheempireontherepublic,retainingthefullauthorityofthe
republicaninstitutions.
B) Augustustotallyrevisedthegovernmentandeliminatedalloftheinstitutionsandoffices
oftherepublic.
C) Augustusretainedtheofficeofconsulandtribune,buteliminatedtheSenateasa
meaningfulpartoftheempire.
D) Augustusretainedmanyoftheinstitutionsandofficesoftherepublic,includingthe
Senate,buttookactualpowerinhisownhands.
Answer: D
Diff:2
PageRef:149
Topic:TheInstitutionsofEmpire
Skill:Conceptual

30) WhichofthefollowingwasNOTareforminstitutedbyAugustusduringtheearlyempire?
A) Revisingthelawtostrengthenfamilystability
B) Banningmysteryreligions
C) ReplacingthetraditionalRomanceremonieswithnewritualsemphasizingthecultofIsis
D) Institutingmoralreforms
Answer: C
Diff:2
PageRef:149
Topic:TheInstitutionsofEmpire
Skill:Conceptual

31) WhatwasoneofthemajorpoliticalflawsoftheRomanempire?
A) Thebalancedconstitutionwasdifficulttomaintainintimesofeconomicstress.
B) Theabsenceofmilitaryauthoritymadetheempirevulnerabletoattackfromoutside.
C) Theemperorsauthoritywassharplylimitedbytheconsulsandtribunes,particularlyin
theareaoflegislation.
D) Theemperorsneverestablishedaclearlineofsuccession,leavingdecisionstothearmy.
Answer: D
Diff:2
PageRef:149
Topic:TheInstitutionsofEmpire
Skill:Conceptual

105

32) Whichemperorextendedtheempiretoitsgreatestterritorialdimensions?
A) Trajan
B) Commodus
C) Augustus
D) Pompey
Answer: A
Diff:2
PageRef:149
Topic:TheInstitutionsofEmpire
Skill:Factual

33) Whichofthefollowingdatesrepresentsthemomentatwhichtheempirereacheditspinnacle
andbegantodecline?
A) 30C.E.
B) 120C.E.
C) 180C.E.
D) 476C.E.
Answer: C
Diff:3
PageRef:149
Topic:TheInstitutionsofEmpire
Skill:Factual

34) WhichofthefollowingwasNOTaninnovationoftheRomanempire?
A) Regulationofcommerceinordertoensureadequatesuppliesoffood
B) Eliminationofthemilitary
C) Buildingaqueductstocarrywatertocities
D) Theestablishmentofsubstantialtaxrevenues
Answer: B
Diff:2
PageRef:149
Topic:TheInstitutionsofEmpire
Skill:Conceptual

106

35) WhichofthefollowingstatementsmostaccuratelydescribestheRomanimperialattitude
towardreligion?
A) Theempirerequiredallcitizenstofollowthestateritualsandbannedallotherformsof
religion.
B) Theempirequicklymovedawayfromthetraditionalreligiousceremoniesandadopted
themysteryritualsofthecultofIsisasthestatereligion.
C) From30C.E.to324C.E.theofficialimperialpolicywastolerationofallreligions
regardlessoftheirteachings.
D) Theempiretoleratedmostreligionswhosememberswerewillingtogiveatleastformal
acceptanceofthetraditionalstatereligionandthecultoftheemperors.
Answer: D
Diff:2
PageRef:149
Topic:TheInstitutionsofEmpire
Skill:Conceptual

36) WhatwasRomesgreatestpoliticalcontribution?
A) Romanlaw
B) Abureaucracyselectedthroughanexaminationsystem
C) Democracy
D) Centralizedgovernment
Answer: A
Diff:2
PageRef:150
Topic:TheInstitutionsofEmpire
Skill:Conceptual

37) Whichofthefollowingstatementsaboutmobilitywithinthesocialhierarchiesoftheclassical
civilizationsisNOTcorrect?
A) Chinesesocialorganizationofferedlimitedopportunitiesforsocialadvancement
throughtheuniversityandexaminationsystem.
B) Mediterraneansocialorganizationemphasizedaristocracybutrecognizedthe
importanceofacquiredwealth.
C) Ofthethreeclassicalcivilizations,Chinesesocialorganizationofferedtheleast
opportunityformobility.
D) Indiascastesystemofferedsomeopportunityformobilityinthesensethatthestatusof
entirecastesoccasionallyroseandfellinthesocialorder.
Answer: C
Diff:2
PageRef:152-53
Topic:TheInstitutionsofEmpire
Skill:Conceptual

107

38) WhatwasthebasicstructureoftheRomanhousehold?
A) Itwassurprisinglymatriarchalwiththewifeassuminglegalresponsibilityforall
membersofthehousehold.
B) Itwasstrictlypatriarchalwithfemalemembersofthehouseholdmoreharshlyoppressed
thaninGreekcivilization.
C) Itwasstrictlypatriarchal,butwomenenjoyedgreaterfreedomofactionthanineither
Greek,Chinese,orIndiancivilization.
D) TheRomanhouseholdwaspatriarchalinstructure,butfollowedmatrilinealpatternsof
inheritance.
Answer: C
Diff:2
PageRef:150
Topic:TheEvolutionofRomesEconomicandSocialStructure
Skill:Conceptual

39) Whichofthefollowingstatementsconcerningtheagriculturaleconomyofthe
RomanempireisNOTtrue?
A) Theclassoffreefarmersdeclined.
B) Slaveryspreadsteadily.
C) RomaneconomywaslessdependentonslavelaborthanGreece.
D) Slavesstaffedthelargeagriculturalestatesalongwithpaidlaborersandtenantfarmers.
Answer: C
Diff:2
PageRef:152
Topic:TheEvolutionofRomesEconomicandSocialStructure
Skill:Conceptual

40) Notonlydidtheexpansionofslaveryleadtothedeclineofthefreefarmersbutalsoit
influencedthegrowthof
A) themilitary.
B) technologicaldevelopmentinagriculture.
C) aprofessionalizedbureaucracy.
D) technologicaldevelopmentinmanufacturing.
Answer: A
Diff:2
PageRef:151
Topic:TheEvolutionofRomesEconomicandSocialStructure
Skill:Conceptual

108

41) WhichofthefollowingstatementsmostaccuratelycharacterizesthenatureoftheRoman
empireseconomy?
A) Theempirewasasingle,unifiedeconomicstructure.
B) Small,independentfarmsweretheruleoutsideofItaly.
C) MoreactivemerchantgroupsexistedinGreeceandwesternAsiathanelsewhere.
D) UrbanizationoutsideofItalywasextremelylimited.
Answer: C
Diff:3
PageRef:152
Topic:TheEvolutionofRomesEconomicandSocialStructure
Skill:Conceptual

42) Christianityemergedasareformmovementwithinwhatreligion?
A) Judaism
B) Mithraism
C) Zoroastrianism
D) Buddhism
Answer: A
Diff:1
PageRef:154
Topic:TheOriginsofChristianity
Skill:Factual

43) WhowasthefounderofChristianity?
A) PaulofTarsus
B) JesusofNazareth
C) SimonMagus
D) HerodofJudaea
Answer: B
Diff:1
PageRef:154
Topic:TheOriginsofChristianity
Skill:Factual

44) Bythe4thcenturyC.E.,whatpercentageoftheresidentsoftheRomanempirewereChristian?
A) 10percent
B) 20percent
C) 40percent
D) 70percent
Answer: A
Diff:3
PageRef:154
Topic:TheOriginsofChristianity
Skill:Factual

109

45) WhatChristianofficialscametoholdapositionnotunlikethatofRomesprovincial
governors?
A) Bishops
B) Presbyters
C) Deacons
D) Acolytes
Answer: A
Diff:2
PageRef:155
Topic:TheOriginsofChristianity
Skill:Factual

46) WhichofthefollowingwasNOTacontributionofPaultoChristianity?
A) HeusedtheGreeklanguage,thedominantlanguageofthedayintheeastern
Mediterranean.
B) PaulcreatedaChristiantheologyasasetofintellectualprinciplesthatgeneralizedthe
messageofJesus.
C) Paulemphasizedtheequalityofwomenwithmen.
D) HeexplainedChristianbeliefsintermsthatGreco-Romanculturecouldunderstand.
Answer: C
Diff:2
PageRef:155
Topic:TheOriginsofChristianity
Skill:Conceptual

47) WhatwastheRomanimperialgovernmentsattitudetowardChristianity?
A) TheRomangovernmentsystematicallypersecutedChristiansonaconstantbasisuntil
the4thcenturyC.E.
B) TheRomangovernmentembracedChristianityastheofficialstatereligioninthe2nd
centuryC.E.
C) WhilesomeemperorschosetopersecutetheChristians,persecutionwasnotconstant.
D) TherewerenopersecutionsoftheChristiansastheofficialpolicyoftheRomanempire
wasreligioustoleration.
Answer: C
Diff:2
PageRef:155
Topic:TheOriginsofChristianity
Skill:Conceptual

110

48) WhichofthefollowingheritagesoftheMediterraneanclassicalcivilizationhadtheleast
impactonsuccessivewesterncivilizations?
A) traditionsofscienceandphilosophy
B) politicaltheoryanddefinitionsofcitizenship
C) Christianity
D) thedominantGreco-Romanreligion
Answer: D
Diff:2
PageRef:156
Topic:Conclusion:TheMediterraneanHeritage
Skill:Conceptual

EssayQuestions
1) WhatweretheoriginsofRomancivilization?
Answer: RomansocietyinfluencedbyGreeksocialandeconomicstructure(commercial
agricultureoflargeestatesdominatedbyaristocracy,dependenceonslavelabor,
dominanceofaristocracy);andbyGreekpoliticalstructuressuchascity-state
governmentinbothcivilizations;pantheonofgodssimilar.EtruscansruledRomebut
wereoverthrownandRomewasruledasarepublic.
Diff:2
PageRef:141-143
Topic:Introduction
Skill:Conceptual

2) HowdidrepublicanRomesterritorialexpansionchangeRomansocietyandpolitics?
Answer: Wealthyandaristocraticlandownersbenefittedfinanciallywhilesmalllandholders
sufferedcreatingagulfbetweenrichandpoor;clientagerelationshipsdeteriorated;
developmentofcommercialagriculturebasedonoliveandgrape;displacedclassof
smallfarmersmovedtocities;dramaticincreaseinslaveryaslaborsource;riseofa
businessclass(alsoassociatedwithtaxcollection);socialtensionsproducedpolitical
unrest;greaterdifficultyinmaintainingbalancedconstitutionofrepublic.
Diff:2
PageRef:143-146
Topic:TheDevelopmentofRomesRepublic
Skill:Conceptual

3) WhatwasthenatureoftheRomanimperialformofgovernment?Whathappenedtothe
institutionsoftherepublic?
Answer: Romanempirecenteredallauthorityinemperor;institutionsofrepublicwereretained,
butlostallrealpower;imperialgovernmentallowedsubstantiallocalautonomyto
regionalandlocalgovernments;greatestinnovationofempirewascreationofunified
legalsystemapplicabletoallcitizens;extensionoflawsupplementedbyopeningof
citizenshiptomanygroupsoutsideofItalianpeninsula;curiousfailureofempireto
establishaprincipleofsuccessionledtoarmyassourceofpower.
Diff:2
PageRef:144-148
Topic:TheInstitutionsofEmpire
Skill:Conceptual

111

4) HowdidthesocialorganizationofclassicalMediterraneansocietycomparetothatoftheother
classicalcivilizationsinIndiaandChina?
Answer: Allbasedonrelativelyrigidsocialsystempredicatedonsocialinequality;elitesdiffered
ineachcivilization:Indiapriests(brahmins),Chinabureaucrats(scholar-gentry),
Mediterranearistocrats;mobilityvariedineachcivilizationfromvirtuallynomobilityin
theIndiancastesystemtorecognitionofacquiredwealthintheMediterranean;lowest
ordersalsovaried:untouchablesinIndia,slavesinMediterranean;alsomeansof
enforcingsocialordervaried.
Diff:2
PageRef:153
Topic:TheInstitutionsofEmpire
Skill:Conceptual

5) WhataccountsforthesuccessfulspreadofChristianityintheRomanempire?
Answer: Appealedtopoorandslaves;filledspiritualneednotmetbyofficialstatereligions;
benefittedfrompositiveaspectsofempirepoliticalstability,systemof
communication,abletomodelchurchgovernmentafterthatofempire;roleofPaulin
translatingChristianityintotermsunderstandableinGreco-Romanculture.
Diff:2
PageRef:154-155
Topic:TheOriginsofChristianity
Skill:Conceptual

112

Chapter8 ThePeoplesandCivilizationsoftheAmericas
Multiple-ChoiceQuestions
1) Whichofthefollowingstatementsconcerningthecomparisonofdevelopmentsinthe
Americaswiththatofotherclassicalcivilizationsistrue?
A) NocivilizationsemergedintheAmericasthatwereinanywaycomparabletothoseof
AsiaandtheMediterranean.
B) Americancivilizationslackedthesystemsofwritingandtheurbanismfoundinother
civilizations.
C) AmericancivilizationsdiddisplaysomeparallelswithdevelopmentsinAsiaandnorth
Africa.
D) AmericancivilizationsparalleledtheclassicalelementsofAsiaandtheMediterranean
almostexactly.
Answer: C
Diff:2
PageRef:161
Topic:Introduction
Skill:Conceptual

2) WhatwasoneofthemajordifferencesbetweentheresourcesavailableintheAmericasand
thoseinAsiaandtheMediterranean?
A) AmericanIndiansdomesticatedplantswhosecaloricoutputwasgreaterthananyofthe
grainsraisedinEuropeorAsia.
B) Americancivilizationneverdevelopedthemeanstoconstructthetypesofmonumental
architecturefoundinothercivilizations.
C) Americancivilizationhadgreateraccesstoanimalpowerthandidthecivilizationsofthe
OldWorld.
D) Duetothedifficulttopographyinwhichtheydeveloped,Americancivilizationsdidnot
establishsedentaryagriculture.
Answer: A
Diff:2
PageRef:161
Topic:Introduction
Skill:Conceptual

113

3) Accordingtomostarcheologists,howdidHomosapiens gettotheAmericas?
A) VarioustypesofHomosapiens developedintheAmericas.
B) TheycrossedalandbridgefromAsiaduringthelasticeage.
C) TheycrossedfromAsia,probablyJapan,inboats.
D) TheycrossedfromAsiaduringapluvialperiod(warmerclimateandheavierrainfall).
Answer: B
Diff:1
PageRef:161
Topic:OriginsofAmericanSocieties
Skill:Factual

4) WhataccountsforthemigrationofHomosapiens toNorthAmerica?
A) AdroughtmadeagricultureimpossibleinthenorthernAsiansteppes.
B) MigrationsofpastoralnomadsintotheAsiansteppesforcedtheindigenouspopulation
toleave.
C) TheAsiansteppescouldnotsupportthepopulation,forcingmigrationtonewareas.
D) Migranthuntersprobablyfollowedlargegameanimals.
Answer: D
Diff:2
PageRef:161
Topic:OriginsofAmericanSocieties
Skill:Conceptual

5) ForwhatdateistherearchaeologicalevidenceofhumanpresenceintheAmericas?
A) 20,000B.C.E.
B) 10,000B.C.E.
C) 9500B.C.E.
D) 5000B.C.E.
Answer: C
Diff:2
PageRef:161
Topic:OriginsofAmericanSocieties
Skill:Factual

114

6) WhydidthegreatmammalherdsoftheAmericasdiminishandinsomecasesdisappear
entirely?
A) ThereneverwereanylargemammalsintheAmericas.
B) Inadditiontoclimaticchange,theymayhavebeenhuntedtoextinction.
C) Archaeologicalevidencepointstowidespreaddisease,perhapsassociatedwithclimatic
change.
D) Closedofffromotherherds,inbreedingresultedinweakeningofthespeciesuntilthey
succumbedtogeneticfailure.
Answer: B
Diff:2
PageRef:162
Topic:OriginsofAmericanSocieties
Skill:Conceptual

7) WhatisthelinguisticpatternforAmericanIndians?
A) AllIndianlanguagesarederivedfromfewparentstocks,indicatingacommonmigration
stock.
B) ThereareonlytwomajorIndianlanguagegroups,suggestingalimiteddiversityamong
theearlymigrantstotheAmericas.
C) Therewere2000Indianlanguagesinthe16thcenturyderivedfromanumberofparent
languagestocks,suggestingculturaldiversityamongmigrants.
D) TherearethousandsofIndianlanguages,noneofthembearinganyresemblancetoany
other.
Answer: A
Diff:2
PageRef:162
Topic:OriginsofAmericanSocieties
Skill:Conceptual

8) WhichofthefollowingstatementsconcerningAmericancontactswiththeOldWorldpriorto
thefifteenthcenturyiscorrect?
A) ThereareOldWorldartifacts, Japanesepottery, thathavebeendiscoveredinPeruvian
urbancultures.
B) ThereisnothingtosuggestanycontactwithEuropeanorAsiancivilizations.
C) Thereiscircumstantialevidence, similaritiesinpotterydesign,similarplants,that
suggestscontact,butnophysicalevidenceofanykind.
D) AmericanwritingsystemscanbedirectlyassociatedtoChineseideographicalsystems.
Answer: C
Diff:2
PageRef:162
Topic:OriginsofAmericanSocieties
Skill:Conceptual

115

9) WhichofthefollowingwasNOTadisadvantageforAmericanswhentheyeventuallydid
comeincontactwithothercivilizations?
A) Asimplematerialculturepoorinmetallurgicalandweapon-makingskills
B) Alackoflargemammalsasaproteinsourceandapowersource
C) AnisolationfromdiseasescommontoEuropeans,thuslackofimmunities
D) Afailuretodevelopsophisticatedpoliticalsystems
Answer: D
Diff:2
PageRef:162
Topic:OriginsofAmericanSocieties
Skill:Conceptual

10) WhichofthefollowingbestdescribesArchaicculturesintheAmericas?
A) Theyincreasinglydependedonthegatheringofwildfruitsandotherplantfoods,made
baskets,andutilizedgrindingtoolstopreparetherootsandplantstheycollectedfor
food.
B) Archaicculturesweretypifiedbyhuntingandgatheringbandsthatcontinuedtofollow
theherdsoflargemammalsaslateas3000B.C.E.
C) Archaicculturesconstructedmajorurbancentersthatservedaprimarilyreligious
functionsupportedbyagriculturalpopulationsinsurroundingregions.
D) Theyconsistedofsmallvillagesofmulti-storyadobebuildingsshelteredfromthe
weatherandattackbysurroundingcliffs.
Answer: A
Diff:2
PageRef:163
Topic:OriginsofAmericanSocieties
Skill:Conceptual

11) WhatistheearliestdateofevidenceforcultivationinhighlandPeru?
A) 10,000B.C.E.
B) 7000B.C.E.
C) 4000B.C.E.
D) 2000B.C.E.
Answer: B
Diff:2
PageRef:163
Topic:OriginsofAmericanSocieties
Skill:Factual

116

12) WhatstaplecropofAmericanagriculturewasdomesticatedincentralMexicoby4000B.C.E.?
A) Potatoes
B) Manioc
C) Wheat
D) Maize
Answer: D
Diff:1
PageRef:163
Topic:OriginsofAmericanSocieties
Skill:Factual

13) Howwidelydistributedwastheagriculturalproductionofmaize?
A) ThroughoutMesoamerica
B) ThroughoutSouthAmerica
C) ThroughoutMesoamericaandSouthAmerica
D) ThroughoutMesoamerica,SouthAmerica,andNorthAmerica
Answer: D
Diff:1
PageRef:163
Topic:OriginsofAmericanSocieties
Skill:Factual

14) WhatwasthestaplecropdevelopedinthelowlandareasofSouthAmerica,particularlyinthe
Amazonbasin?
A) Potatoes
B) Manioc
C) Wheat
D) Maize
Answer: B
Diff:2
PageRef:163
Topic:OriginsofAmericanSocieties
Skill:Factual

15) WhatwasthestaplecropofpeopleinthehighlandregionsofSouthAmerica?
A) Potatoes
B) Manioc
C) Wheat
D) Maize
Answer: A
Diff:2
PageRef:163
Topic:OriginsofAmericanSocieties
Skill:Factual

117

16) WhichofthefollowingwerenotdevelopedintheAmericas?
A) Thewheel
B) Theplow
C) Iron
D) Alloftheabove
Answer: D
Diff:2
PageRef:162
Topic:OriginsofAmericanSocieties
Skill:Conceptual

17) WhichofthefollowingwasNOTtypicalofthelevelofsocialorganizationdescribedasa
chiefdom?
A) Socialstratificationwithpriests,nobles,andcommoners
B) Centraltownsfeaturingtemplecomplexes
C) Theabsenceofclanorganization
D) Hereditaryrulerscalledchieftains
Answer: C
Diff:2
PageRef:164-64
Topic:OriginsofAmericanSocieties
Skill:Conceptual

18) WhichofthefollowingstatementsmostaccuratelydescribesthegeographyofMesoamerica?
A) Mostoftheregionisdesertwithasmalltropicalzoneinthesouth.
B) Fromthetropicallowlandsofthenorththelandfallsagaintoanaridzoneinthesouth.
C) Mesoamericaisacomplexpatchworkofzonesthataredividedverticallyintocooler
highlands,tropicallowlandsandcoasts,andanintermediatetemperatezone.
D) Mostoftheregionisaflatfeaturelessplaincoveredbyavirtuallyimpenetrablesod.
Answer: C
Diff:2
PageRef:165
Topic:SpreadofCivilizationinMesoamerica
Skill:Conceptual

19) AtapproximatelywhatdatedidthedomesticationofplantsbegininMesoamerica?
A) 10,000B.C.E.
B) 8000B.C.E.
C) 5000B.C.E.
D) 2000B.C.E.
Answer: C
Diff:3
PageRef:165
Topic:SpreadofCivilizationinMesoamerica
Skill:Factual

118

20) ThefirstcivilizationtoariseinMesoamericawasthatofthe
A) Olmecs.
B) Zapotecs.
C) Toltecs.
D) Aztecs.
Answer: A
Diff:2
PageRef:166
Topic:SpreadofCivilizationinMesoamerica
Skill:Factual

21) InwhatregionofMesoamericadidtheOlmeccivilizationoriginate?
A) TropicalGuatemala
B) Thehighlandsofthecentralplateau
C) NearTeotihuacan
D) OnthesoutheasterncoastofMexico
Answer: D
Diff:3
PageRef:165
Topic:SpreadofCivilizationinMesoamerica
Skill:Factual

22) WhatwerethemostimpressiveOlmecsitesinMesoamerica?
A) MonteAlban
B) SanLorenzoandLaVenta
C) Teotihuacan
D) TihuanacoandHuari
Answer: B
Diff:3
PageRef:166
Topic:SpreadofCivilizationinMesoamerica
Skill:Factual

23) WhatmonumentalsculpturesdidtheOlmeccivilizationproduce?
A) Manioc
B) Calendars
C) Stoneheads
D) Stonehands
Answer: C
Diff:2
PageRef:166
Topic:SpreadofCivilizationinMesoamerica
Skill:Conceptual

119

24) WhatwasthemajorceremonialcenteroftheZapotecculture?
A) MonteAlban
B) LaVenta
C) Teotihuacan
D) Tihuanaco
Answer: A
Diff:3
PageRef:166
Topic:SpreadofCivilizationinMesoamerica
Skill:Factual

25) WhatarethedatesoftheMesoamericanPreclassicperiod?
A) 7000-2000B.C.E.
B) 5000-1000B.C.E.
C) 3000-500B.C.E.
D) 2000-300B.C.E.
Answer: D
Diff:3
PageRef:166
Topic:SpreadofCivilizationinMesoamerica
Skill:Factual

26) WhatarethedatesoftheMesoamericanClassicperiod?
A) 3000-500B.C.E.
B) 2000-300B.C.E.
C) 300B.C.-500C.E.
D) 150-900C.E.
Answer: D
Diff:3
PageRef:166
Topic:SpreadofCivilizationinMesoamerica
Skill:Factual

27) WhatwasthemajorceremonialandpoliticalcenterofthecentralMexicanhighlandsduring
theMesoamericanClassicperiod?
A) MonteAlban
B) LaVenta
C) Teotihuacan
D) Tihuanco
Answer: C
Diff:2
PageRef:167
Topic:SpreadofCivilizationinMesoamerica
Skill:Factual

120

28) WhichofthefollowingattributeswasNOTfoundintheClassiccivilizationofthecentral
valleyofMexico?
A) Alargeurbancenterwithatemplecomplexfeaturingenormouspyramids
B) Apopulationperhapsashighas200,000
C) Evidenceofsocialstratificationconsistingofpriests,nobles,andcommoners
D) Worshipofasinglegod, theFeatheredSerpent
Answer: D
Diff:2
PageRef:167
Topic:SpreadofCivilizationinMesoamerica
Skill:Conceptual

29) WhatcultureemergedduringtheMesoamericanClassicperiodintropicallowlandsofMexico
andGuatemala?
A) Olmecs
B) Mayas
C) Toltecs
D) Zapotecs
Answer: B
Diff:1
PageRef:168
Topic:SpreadofCivilizationinMesoamerica
Skill:Factual

30) WhichofthefollowingcharacteristicsoftheMayancalendarwasdifferentfromother
Mesoamericancalendricalsystems?
A) Recurringcyclesofdifferentlength
B) Cyclesof52yearswereregardedassacred
C) Thelongcountcreatedafixeddateoforigin 3114B.C.
D) Beliefingreatcyclesofcreationanddestructionoftheuniverse
Answer: C
Diff:2
PageRef:168
Topic:SpreadofCivilizationinMesoamerica
Skill:Conceptual

121

31) WhichofthefollowingstatementsabouttheMayancultureisNOTtrue?
A) Theypossessedasophisticatedcalendricalsystembasedonbothasacredcycleandthe
solaryear.
B) Despitetheirtechnologicalsophistication,theMayafailedtodevelopasystemof
writing.
C) TheMayacosmosconsistedofaflatEarthwithdominantdeitiesatitscenterand
cardinalpointstosupporttheheavens.
D) Therewerenumbersofcity-statesinMayancivilizationthatengagedinvirtually
constantwarfare.
Answer: B
Diff:2
PageRef:168
Topic:SpreadofCivilizationinMesoamerica
Skill:Conceptual

32) WhatculturesucceededTeotihuacaninthecentralplateauofMesoamerica?
A) Zapotecs
B) Aztecs
C) Toltecs
D) Mixtecs
Answer: C
Diff:2
PageRef:171
Topic:SpreadofCivilizationinMesoamerica
Skill:Factual

33) WhichpeoplelivedinthecavedwellingsatMesaVerde?
A) Hopewell
B) Mississippian
C) Anasazi
D) Adena
Answer: C
Diff:2
PageRef:174
Topic:ThePeoplestotheNorth
Skill:Factual

122

34) WhichofthefollowingwasNOTaMesoamericancivilization?
A) Zapotecs
B) Olmecs
C) Anasazi
D) Toltecs
Answer: C
Diff:1
PageRef:174
Topic:ThePeoplestotheNorth
Skill:Factual

35) WhatseemstoaccountforthedeclineoftheculturesofsouthwesternNorthAmerica?
A) Epidemicdisease
B) Alongperiodofdroughtinthe13thcentury
C) AninvasionoftheNahuatl-speakingToltecs
D) Widespreadfloodingofthecanyonsandvalleysinwhichtheculturesflourished
Answer: B
Diff:2
PageRef:175
Topic:ThePeoplestotheNorth
Skill:Conceptual

36) Whatarekivas?
A) ShellmiddensassociatedwithArchaichuntingandgatheringcultures
B) MoundsassociatedwithNorthAmericanmound-buildingcultures
C) PyramidsfoundatTeotihuacan
D) CircularpitsusedforceremonialpurposesinAnasaziculture
Answer: D
Diff:2
PageRef:174
Topic:ThePeoplestotheNorth
Skill:Factual

37) WhichofthefollowingpeoplesdidNOTliveintheAndeanregionofSouthAmerica?
A) Hopewell
B) Chavin
C) Nasca
D) Moche
Answer: A
Diff:2
PageRef:175
Topic:TheAndeanWorld
Skill:Factual

123

38) HowdidthetopographyoftheAndeanregioninfluencethedevelopmentofcivilization?
A) TheunsuitabilityoftheAndeanregionforanyformofagriculturelimitedthe
developmentofcivilizationsthere.
B) TheexistenceofpastoralnomadismintheAndeanregionledtoanimpermanent
populationbase,asonthefringesofAsianandMediterraneancivilizations.
C) Thearidcoastalvalleysdemandedirrigation,andthisspurredpopulationgrowthand
socialcomplexityleadingtothedevelopmentofcivilizationsasintheearlyriver -valley
civilizationsofAsiaandtheMiddleEast.
D) Themanyflatplainsprovidedampleopportunitiesforwidespreaddevelopmentof
sedentaryagriculturalcommunities.
Answer: C
Diff:2
PageRef:176
Topic:TheAndeanWorld
Skill:Conceptual

39) WhyistheverticalityofAndeantopographyimportantindeterminingthenatureofcultures
there?
A) BecausemostoftheupperareasoftheAndeanregionwereuninhabitable,itwas
necessaryforpeopletocontrolthelowerelevations.
B) BecausetheAndeanregionisdividedintomicroecologiesvertically,itbecameimportant
forpeopletocontrolasmanydifferentverticalecologicalzonesaspossible.
C) Becauseofthearidityofthelowervalleys,peoplestruggledtocontroltheupperzonesto
establishsedentaryagriculture.
D) BecauseoftheabsenceofdomesticatedanimalsintheAndeanregion,itwasnecessaryto
controlthoseecologicalzonessuitableforagriculture.
Answer: B
Diff:3
PageRef:176
Topic:TheAndeanWorld
Skill:Conceptual

40) WhichcultureisassociatedwiththeFirstHorizoninAndeancivilization?
A) Mochica
B) Tihuanaco
C) Chimu
D) Chavin
Answer: D
Diff:2
PageRef:176
Topic:TheAndeanWorld
Skill:Factual

124

41) WhichofthefollowingwasNOTtypicalofChavinculture?
A) Ceremonialcenterswithlargestonebuildings
B) Simpleformsofirrigation
C) Domesticationofsheep
D) Artisticmotifsassociatedwithacultorsystemofreligiousbeliefs
Answer: C
Diff:3
PageRef:187
Topic:TheAndeanWorld
Skill:Conceptual

42) Whichofthefollowingstatementsmostaccuratelyreflectsthesocialandpoliticalorganization
oftheAndeanregionafterthedeclineofChavincultureca.300B.C.E.?
A) Itwasaperiodofregionalcenterswithoutpoliticalunity,butproducedsomeofthe
Andeanworldsfinestart.
B) Itwasaperiodtypifiedbytheriseofacentralizedempirewithitscenterlocatednear
Cuzcointhemountains.
C) Andeansocietyrevertedtohuntingandgatheringsocialgroupswithlittleevidencefor
sedentaryagriculture.
D) AndeansocietybecamepartofthemoreprosperousempireofTeotihuacanasthe
MexicanstatesuccessfullyextendeditscontroltoSouthAmerica.
Answer: A
Diff:2
PageRef:176
Topic:TheAndeanWorld
Skill:Conceptual

43) In300C.E.theIntermediateHorizonwasassociatedwiththeemergenceofwhattwo
politicalcenters?
A) TeotihuacanandTikal
B) PalenqueandChichenItza
C) MocheandChan
D) TihuanacoandHuari
Answer: D
Diff:2
PageRef:179
Topic:TheAndeanWorld
Skill:Factual

125

44) WhoconqueredtheChimustatein1465?
A) TheHuacas
B) TheAztecs
C) TheMoche
D) TheIncas
Answer: D
Diff:2
PageRef:179
Topic:ThePeoplestotheNorth
Skill:Conceptual

45) ThedeclineoftheIntermediateHorizoninAndeancivilizationcoincidedwithwhateventin
Mesoamericancivilization?
A) TheriseofMayancultureandtheurbantemplecomplexatTeotihuacan
B) TheArchaicperiod
C) TheendoftheClassicperiod
D) TheemergenceofOlmeccivilizationduringthePreclassicperiod
Answer: C
Diff:3
PageRef:179
Topic:TheAndeanWorld
Skill:Factual

46) WhichofthefollowingwasNOTaprincipleofAndeanpolitical,social,andeconomic
organization?
A) controlofanumberofecologicalnichesatdifferentaltitudestoensureself-sufficiency
B) compositionofsocietyintohouseholdswhichtogetherrecognizedsomeformofkinship
intoayllus
C) consolidationintoasinglelanguagegroup
D) theprincipleofreciprocalsocialobligations
Answer: C
Diff:2
PageRef:179
Topic:TheAndeanWorld
Skill:Conceptual

126

47) WhichanimalwasfoundintheAndeanregion?
A) Llamas
B) Buffalo
C) Horses
D) Sheep
Answer: A
Diff:2
PageRef:180
Topic:ThePeoplestotheNorth
Skill:Factual

48) WhichofthefollowingstatementsconcerningAndeanayllus ismostcorrect?


A) AyllusonlyexistedinthemoreprimitivesocialunitstypicalofearlyAndeancivilization.
B) Allmembersofanaylluwereregardedassocialequalswithequalaccesstocommunity
property.
C) Membersofanayllupracticedmarriageendogamy theymarriedoutsidetheirkinship
group.
D) Allmembersofanayllutraceddescentfromacommonancestor.
Answer: D
Diff:2
PageRef:179
Topic:TheAndeanWorld
Skill:Conceptual

49) WhichofthefollowingwasNOTaculturaldifferencebetweenMesoamericanandAndean
civilizations?
A) GreatermetallurgicaltechnologyinAndeancivilization
B) TheexistenceofpastoralisminAndeancivilization
C) TheexistenceofwritinginMesoamericancivilization
D) PoliticalunificationintoregionalempiresinMesoamerica
Answer: D
Diff:2
PageRef:180
Topic:Conclusion:AmericanCivilizations
Skill:Conceptual

EssayQuestions
1) WhatisthegenerallyacceptedtheoryforhowhumansmigratedtotheAmericas?Aboutwhen
didtheyarrive?
Answer: CrossedoverlandbridgefromAsia;bridgecreatedduringlasticeageassealevel
dropped;arrivedinAmericasca.20,000B.C.E.
Diff:1
PageRef:161-162
Topic:Introduction
Skill:Conceptual

127

2) WhendidsedentaryagriculturalsocietiesemergeintheAmericas?Whatwerethestaplesof
Americanagriculture?Howdidsedentaryagricultureaffectsocialandpoliticalorganization?
Answer: Earliestknowncultivationca.7000B.C.E..;plantdomesticationwidespreadby5000
B.C.E..;varietyofcrops,butthreemainstaples:maizeinMesoamericalaterspread
throughoutAmericas,maniocinlowlandSouthAmericaandCaribbean,potatoesin
highlandSouthAmerica;sedentaryagricultureledtoincreasinglycomplexsocial
organization;particularlyassociatedwithriseofirrigationsystemsinMesoamerica
(Olmec,Teotihuacan,Maya)andAndes(ChavinbutmoreparticularlywithTihuanaco
andHuari).
Diff:2
PageRef:162-163
Topic:Introduction
Skill:Conceptual

3) WhatwerethemajorcivilizationsofPreclassicandClassicMesoamerica?Howwerethese
civilizationsorganizedsocially,politically,economically?
Answer: Majorcivilizations:Preclassic Olmec,Classic TeotihuacanandMaya.Allcivilizations
basedonurbanizedcentersthatfeaturedmonumentalarchitecture,oftenpyramids,
devotedtoreligiouscults;authorityofearlypoliticalleadersmayhavebeenbasedon
religion;urbancenterssupportedbyagriculturalhinterlandsdevotedtoirrigatedforms
ofagriculture;evidenceofwidespreadtradenetworksthatdistributedreligiousand
culturalformsoverbroadareas;clearevidenceofsocialstratificationinallcivilizations
priests,nobility,artisans,peasantsorcommoners.
Diff:2
PageRef:165-170
Topic:SpreadofCivilizationinMesoamerica
Skill:Conceptual

4) WhatwerethemajorsimilaritiesanddifferencebetweenMesoamericanandAndean
civilizations?
Answer: Similarities:lackofdomesticatedanimalsandthewheel,politicallyhierarchical,
polytheisticreligiousbeliefs.Differences:pastoralism,metallurgy,butnowritinginthe
Andes.
Diff:2
PageRef:191-192
Topic:TheAndeanWorld
Skill:Conceptual

5) WhatwerethebasesfortheorganizationofsocietyamongtheAndescivilizations?
Answer: Attempttocontrolvarietyofecologicalnichestosecureself-sufficiency;organization
intokinshipgroupsorayllus;marriagewithinayllu;socialstratificationwithinayllu;
kinshipgroupsprovidedbasesforlargerpoliticalconglomerations;societybound
togetherbyreciprocalobligations.
Diff:2
PageRef:178-180
Topic:TheAndeanWorld
Skill:Conceptual

128

Chapter9 TheSpreadofCivilizationsandtheMovement
ofPeoples
Multiple-ChoiceQuestions
1) WhichofthefollowingstatementsconcerningthegeographyofAfricaismostcorrect?
A) ItliesinthetropicsandcontainsmostlyrainforestswiththeexceptionoftheSahara
desert.
B) Itliesmostlyinthetropics,butitssurfaceiscoveredmainlybysavannas,aridplains,
anddeserts.
C) Itliesprimarilywithintemperateclimaticzones,andconsistsmainlyofgrassyplains
brokenbyoccasionalmountainranges.
D) Despiteitslocationinatemperatezone,mostofAfricaisdesert.
Answer: B
Diff:2
PageRef:185
Topic:TheSpreadofCivilizationinAfrica
Skill:Conceptual

2) AfricawascalledtheDarkContinent
A) butwasoftenincontactwithothercivilizationsandcultures.
B) becauseitremainedlargelyisolatedfromoutsidecontactsuntilthenineteenthcentury.
C) becauseitssoilwasconsideredmorefertilethanthatofothercontinents.
D) becauseitremainedwithoutcivilizationuntiltheEuropeanexplorationsofthe19th
century.
Answer: A
Diff:2
PageRef:185
Topic:TheSpreadofCivilizationinAfrica
Skill:Conceptual

3) WhatwastheclimaticchangethataffectedthedistributionofpeoplesinAfrica?
A) Theentirecontinentbecameprogressivelywetterduetoannualincreasesinrainfall
beginninginthe3rdmillenniumB.C.E.
B) IncreasedrainfallproducedwidespreadrainforeststhatstretchedacrossAfricafromthe
AtlanticcoastlinetoEthiopia.
C) AdecreaseinannualrainfallturnedthenorthernareasofthecontinentintotheSahara
desertby3000B.C.E.
D) AdecreaseinannualrainfalldestroyedallofthePaleolithicrainforestsofAfricaand
createdwidespreadsavannas,particularlyinwestAfrica.
Answer: C
Diff:2
PageRef:185-186
Topic:TheSpreadofCivilizationinAfrica
Skill:Conceptual

129

4) Howdidsedentaryagriculturedevelopinsub-SaharanAfrica?
A) Sedentaryagriculturewasdevelopedindependentlyinsub-SaharanAfricabythe
domesticationofrootsandtubersnativetothecontinent.
B) ThenatureofthefirstdomesticatedcropsindicatesthatagriculturereachedAfricafrom
EuropeandtheMediterranean.
C) ThenatureofthefirstdomesticatedcropsindicatesthatagriculturereachedAfricafrom
westAsiaandtheNearEast.
D) SedentaryagriculturewasnotdevelopedinAfricauntilthearrivaloftheEuropeans
duringthe19thcentury.
Answer: C
Diff:2
PageRef:186
Topic:TheSpreadofCivilizationinAfrica
Skill:Conceptual

5) BywhatdatehadsedentaryagriculturebeenintroducedintoAfrica?
A) 7000B.C.E.
B) 3000B.C.E.
C) 300C.E..
D) 1800C.E..
Answer: B
Diff:2
PageRef:186
Topic:TheSpreadofCivilizationinAfrica
Skill:Factual

6) WhatwasthesourceofdomesticatedanimalsinAfrica?
A) Africansdomesticatedcattle,horses,goats,andsheepnativetothecontinent.
B) MostlargemammalsofAfricawereunsuitablefordomesticationandAfricanagriculture
waslargelywithoutapastoralelement.
C) Cattle,sheep,goats,andthecamelwereintroducedtoAfricafromthewestern
Mediterranean.
D) Cattle,sheep,goats,andthecamelwereintroducedfromwesternAsia.
Answer: D
Diff:2
PageRef:186-187
Topic:TheSpreadofCivilizationinAfrica
Skill:Conceptual

130

7) WhatbiologicalfactormadeitdifficulttosustainpastoralisminsomeregionsofAfrica?
A) Tsetsefly
B) Rabies
C) Anthrax
D) Lackofgeneticdiversity
Answer: A
Diff:2
PageRef:187
Topic:TheSpreadofCivilizationinAfrica
Skill:Factual

8) WhatwaspeculiarabouttheAfricanadaptationofmetallurgy?
A) MostAfricanculturesfailedtoadoptmetallurgyandremaineddependentonstonetools
andtechnology.
B) Africanculturesdevelopedbronzetoolmakingindependently,butreceivedironworking
fromthewesternMediterranean.
C) Africanculturesneverpassedbeyondbronzemetallurgy.
D) AfricanculturesreceivedironmetallurgyfromwesternAsia,butneverdeveloped
coppermetallurgy.
Answer: D
Diff:2
PageRef:187
Topic:TheSpreadofCivilizationinAfrica
Skill:Conceptual

9) AtapproximatelywhatdatewasironmetallurgyintroducedintoAfrica?
A) 3000B.C.E.
B) ThelastmillenniumB.C.E.
C) 100C.E.
D) 1250C.E.
Answer: B
Diff:3
PageRef:187
Topic:TheSpreadofCivilizationinAfrica
Skill:Factual

131

10) ThepeopleresponsibleforthegreatpopulationmovementthatoriginatedineasternNigeria
inWestAfricaandspreadthroughoutthecontinentwerethe
A) Almoravids.
B) Bantu.
C) Masai.
D) Maori.
Answer: B
Diff:1
PageRef:188
Topic:TheSpreadofCivilizationinAfrica
Skill:Factual

11) WhatisthemostlikelycauseforthesuccessoftheBantumigration?
A) Theirirontechnologygavethemamilitaryadvantageofstone -usingtechnologiesthey
conquered.
B) Theirorganizationinphalanxesallowedthemtooverwhelmlesswell-organized
societies.
C) Theiruseofmountedcavalryandwarchariotsgavethemamilitaryadvantageoverthe
peoplestheyconquered.
D) TheircontrolofthegoldtradewiththeMediterraneangavethemgreatwealth.
Answer: A
Diff:2
PageRef:188
Topic:TheSpreadofCivilizationinAfrica
Skill:Conceptual

12) Whichofthefollowingstatementsconcerningtheoriginalsocialandeconomicorganizationof
theearlyBantuisNOTcorrect?
A) Theydependedonagricultureandfishing
B) Theydomesticatedgoatsandperhapscattle
C) Theylivedinlargetemplecomplexesfeaturingmonumentalarchitecturewith
ceremonialpurposes
D) Theyorganizedtheirsocietyaroundkinshipgroups
Answer: C
Diff:2
PageRef:188
Topic:TheSpreadofCivilizationinAfrica
Skill:Conceptual

132

13) BywhatdatedidtheBantumigrationreachthesouthernendoftheAfricancontinent?
A) 3000B.C.E.
B) 900B.C.E.
C) 900C.E.
D) thethirteenthcenturyC.E.
Answer: D
Diff:3
PageRef:189
Topic:TheSpreadofCivilizationinAfrica
Skill:Factual

14) WhichofthefollowingwasNOTaparalleldevelopmentinsub-SaharanandEgyptian
cultures?
A) Theideaofthekingasadivinebeingwithspecialpowersovernaturalphenomena
B) Brother-sistermarriageamongrulers
C) Ritualsoftakingauthority
D) Monotheism
Answer: D
Diff:2
PageRef:189
Topic:TheSpreadofCivilizationinAfrica
Skill:Conceptual

15) WhatkingdomsucceededMeroeasthedominantpowerintheEthiopianhighlands?
A) Kush
B) Ghana
C) Mali
D) Axum
Answer: D
Diff:2
PageRef:189
Topic:TheSpreadofCivilizationinAfrica
Skill:Factual

16) WhichofthefollowingistrueaboutthekingdomofAxum?
A) ItconvertedtoIslaminthe7thcenturyC.E.
B) IttradedwithEgyptandeventuallywithRome,ByzantiumandIndia.
C) Ithadnocontactwiththeoutsideworld.
D) ItincorporatedMesopotamiaintoitsempire.
Answer: B
Diff:2
PageRef:192
Topic:TheSpreadofCivilizationinAfrica
Skill:Conceptual

133

17) WhatwasthedominantreligionofAxumafter350C.E..?
A) Christianity
B) Islam
C) Mithraism
D) Shinto
Answer: A
Diff:1
PageRef:192
Topic:TheSpreadofCivilizationinAfrica
Skill:Factual

18) Whatgoodsdidthepeopleofthesavannatradebetweentheforestzoneofthesouthandthe
Mediterranean?
A) Irontotheforestzoneinreturnforivory
B) Horsestotheforestzoneinreturnforirontools
C) Camels,sheep,andgoatstotheforestzoneinreturnforfoodproducts
D) Salttotheforestzoneinreturnforgold
Answer: D
Diff:2
PageRef:192
Topic:TheSpreadofCivilizationinAfrica
Skill:Conceptual

19) Whatearlysub-Saharanempirewasbasedonthecontroloftrans-Saharacommerce?
A) Axum
B) Mali
C) Kush
D) Ghana
Answer: D
Diff:1
PageRef:192
Topic:TheSpreadofCivilizationinAfrica
Skill:Factual

20) After985C.E.whatreligionwasembracedbytheelitesinthewestAfricanstates?
A) Christianity
B) Islam
C) Mithraism
D) Shinto
Answer: B
Diff:1
PageRef:192
Topic:TheSpreadofCivilizationinAfrica
Skill:Factual

134

21) WhatgroupofpeoplewereresponsiblefortheconquestofGhanain1076?
A) Almohades
B) Carthaginians
C) Idrisids
D) Almoravids
Answer: D
Diff:2
PageRef:194
Topic:TheSpreadofCivilizationinAfrica
Skill:Factual

22) ThepeoplewhoinhabitedmostoftheBritishIsleswere
A) Germans.
B) Slavs.
C) Celts.
D) Scythians.
Answer: C
Diff:1
PageRef:197
Topic:TheSlavsandGermansontheNorthernRim
Skill:Factual

23) WhichofthefollowingwasNOTafeatureofCelticsocialandeconomicorganization?
A) Mixedagriculturalandhuntingeconomies
B) Lackofcities
C) WritingsystembasedonearlyIndo-Europeanlanguages
D) Organizationintosmallregionalkingdoms
Answer: C
Diff:2
PageRef:198
Topic:TheSlavsandGermansontheNorthernRim
Skill:Conceptual

24) WhatRomanhistorianwroteconcerningthesocietyoftheGermans?
A) Livy
B) AmmianusMarcellinus
C) ScipioCunctor
D) Tacitus
Answer: D
Diff:2
PageRef:198
Topic:TheSlavsandGermansontheNorthernRim
Skill:Factual

135

25) WhichofthefollowingstatementsconcerningthestatusofwomeninGermanicsocietyismost
accurate?
A) Inasocietythatvaluedmatrilinealties,womenwereregardedashavinganelementof
holinessandthegiftofprophecy.
B) Inthestrictlypatrilinealandpatriarchalsocietytypicalofmostnomadicpastoralists,
womenwereoftentreatedasproperty.
C) WomeninGermanicsocietywerescarcelymorethanslaves.
D) InthestronglymatriarchalsocietyoftheGermans,womenweretheheadsof
households.
Answer: A
Diff:2
PageRef:198
Topic:TheSlavsandGermansontheNorthernRim
Skill:Conceptual

26) WhichofthefollowingstatementsconcerningthenatureofGermanicpoliticalorganization
after200C.E.ismostaccurate?
A) Afterthe2ndcenturyC.E.,Germanictribesfragmentedintokinshipunits.
B) After200C.E.,someGermantribescoalescedintolargertribalconfederationsasthey
learnedtocopyRomanmilitarystructure.
C) After200C.E.,severalGermantribessuccessfullycreatedempiresonthebordersofthe
Romanempire.
D) After200C.E.,allofnorthwesternEuropewassubordinatedtoasingleGermanicempire
withitscapitalatJutland.
Answer: B
Diff:2
PageRef:198
Topic:TheSlavsandGermansontheNorthernRim
Skill:Conceptual

27) WhichofthefollowingwasNOTafeatureofGermanicreligionoutsidetheRomanempire?
A) itwasanimistic
B) Germansworshippedthespiritsofnature
C) horseswerecommonlysacrificedtothegods
D) GermanspracticedmonotheisticworshipofthegodWodin
Answer: D
Diff:2
PageRef:198
Topic:TheSlavsandGermansontheNorthernRim
Skill:Conceptual

136

28) WhichofthefollowingwasNOTareasonfortheGermanmigrationintotheRomanempire?
A) PopulationgrowthinGermaniclands
B) Romesgrowingweakness
C) ACelticinvasiononthewesternflankoftheGermanicregion
D) ThenomadictraditionoftheGermans
Answer: C
Diff:2
PageRef:198
Topic:TheSlavsandGermansontheNorthernRim
Skill:Conceptual

29) BywhattimewasagricultureestablishedinsoutheasternEurope(inwhatisnowmodern
Russia)?
A) 5000B.C.E.
B) 3000B.C.E.
C) 900B.C.E.
D) 900C.E.
Answer: B
Diff:2
PageRef:199
Topic:TheSlavsandGermansontheNorthernRim
Skill:Factual

30) FromwhatlinguisticgroupdidtheSlavscome?
A) Celtic
B) Germanic
C) Indo-European
D) Bantu
Answer: C
Diff:2
PageRef:199
Topic:TheSlavsandGermansontheNorthernRim
Skill:Factual

31) WhichofthefollowingwasNOTtypicalofthetransmissionofChineseculturetoJapan?
A) Transmissionbymerchants
B) Transmissionbystudents
C) TransmissionbyBuddhistmonks
D) Transmissionbymilitaryconquest
Answer: D
Diff:2
PageRef:202
Topic:TheSpreadofChineseCivilizationtoJapan
Skill:Conceptual

137

32) ThetransmissionofChineseculturetoJapanwasmediatedbywhatpeople?
A) Koreans
B) Ainu
C) Vietnamese
D) Hsiung-nu
Answer: A
Diff:2
PageRef:202
Topic:TheSpreadofChineseCivilizationtoJapan
Skill:Factual

33) AtwhatdatehadtheancestorsoftheJapanesepeoplebeguntomigratetotheJapanese
islands?
A) 5000B.C.E.
B) 3000B.C.E.
C) 1000B.C.E.
D) 300C.E.
Answer: A
Diff:2
PageRef:202
Topic:TheSpreadofChineseCivilizationtoJapan
Skill:Factual

34) WhatwastheearlycultureofJapanafterthe3rdmillenniumB.C.E.typifiedbyhuntingand
gatheringanddistinctivepottery?
A) Kemal
B) Shinto
C) Jomon
D) Haiku
Answer: C
Diff:2
PageRef:202
Topic:TheSpreadofChineseCivilizationtoJapan
Skill:Factual

35) WhichofthefollowingwasNOTintroducedintoJapanduringtheYayoiepoch?
A) Wet-riceagriculture
B) Buddhism
C) Ironworking
D) Wheel-turnedpottery
Answer: B
Diff:2
PageRef:203
Topic:TheSpreadofChineseCivilizationtoJapan
Skill:Conceptual

138

36) WhatwasthesocialandpoliticalorganizationofJapanpriortothe5thcenturyC.E.?
A) Japanwasdividedintoregionalkingdomswithautocraticrulerswhodominatedthe
socialorganization.
B) Japanwasorganizedintoasingleempireunderanautocraticrulersupportedbya
powerfulpriesthood.
C) Japanwasdividedintohundredsofclansdominatedbyasmallwarrioraristocracy.
D) Japanesesocietywasunabletomovebeyondhuntingandgatheringbandspriortothe
fifthcenturyC.E.
Answer: C
Diff:2
PageRef:203
Topic:TheSpreadofChineseCivilizationtoJapan
Skill:Conceptual

37) WhatwasthepositionofwomeninearlyJapanesesocietypriortothe5thcenturyC.E.?
A) Womenweretreatedaspropertywithinstronglypatriarchalhouseholds.
B) Althoughlinesofdescentwerematrilineal,Japanesewomenwereclearlysubjecttotheir
husbands.
C) Japanesewomenenjoyedlimitedfreedomtoseekoccupationsoutsidethehousehold,
butwereotherwisesubjecttopatriarchalauthority.
D) Japanesehouseholdswerematriarchalanddominatedbychildbearingwomenwhoalso
playedkeyrolesasreligiousshamans.
Answer: D
Diff:2
PageRef:203
Topic:TheSpreadofChineseCivilizationtoJapan
Skill:Conceptual

38) WhatwasthechiefreligionofearlyJapanpriortothe5thcenturyC.E.?
A) Shinto
B) Christianity
C) Islam
D) Buddhism
Answer: A
Diff:2
PageRef:203
Topic:TheSpreadofChineseCivilizationtoJapan
Skill:Factual

139

39) WhichoftheJapaneseclanssucceededinestablishingthefirstempireontheislands?
A) Ainu
B) Honshu
C) Fuji
D) Yamato
Answer: D
Diff:1
PageRef:204
Topic:TheSpreadofChineseCivilizationtoJapan
Skill:Factual

40) WhatwasthesignificanceoftheJapaneseadoptionofChinesescript?
A) TheJapanesewereableforthefirsttimetodevelopironmetallurgy.
B) TheJapanesewereabletoconquertheChinese.
C) TheYamatowereabletobuildatruebureaucracyandestablishtheircontrolovervassal
clanheads.
D) TheadoptionofChinesescriptmadeiteasierfortheChinesetoconquerJapan.
Answer: C
Diff:2
PageRef:204
Topic:TheSpreadofChineseCivilizationtoJapan
Skill:Conceptual

41) InwhatcenturydidBuddhismreachJapanfromChina?
A) 2ndcenturyB.C.E.
B) 2ndcenturyC.E.
C) 4thcenturyC.E.
D) 6thcenturyC.E.
Answer: D
Diff:2
PageRef:204
Topic:TheSpreadofChineseCivilizationtoJapan
Skill:Factual

140

42) WhichofthefollowingactivitieswasNOTpracticedbyBuddhistmonksfollowingthe
conversionoftheJapanese?
A) Monksservedasadvisorstotheemperorandregionallords.
B) Monksstressedscripturalpassagesthatsupportedrulebyastrongmonarchthrougha
centralizedstate.
C) MonksintroducedtheChinesechroniclesasamodelforthefirsthistoriesofJapan.
D) MonksdestroyedtheShintoshrinesthathadexistedthroughoutJapan.
Answer: D
Diff:2
PageRef:204
Topic:TheSpreadofChineseCivilizationtoJapan
Skill:Conceptual

43) WhichofthefollowingdevelopmentswasNOTaresultoftheincreasingChineseinfluenceon
JapanaftertheintroductionofBuddhism?
A) TheJapanesemerchantclassceasedtoexistasaseparatesocialentity
B) Aclassofmonksandscholarsdeveloped
C) TradewithChinaandKorealedtotheimportationofnewtoolsandtechniques
D) MedicinesandmethodsoftreatingdiseasewereintroducedfromChina
Answer: A
Diff:2
PageRef:205
Topic:TheSpreadofChineseCivilizationtoJapan
Skill:Conceptual

44) WhatimpactdidtheintroductionofChinesehouseholdpatternshaveonJapan?
A) Therewasnoimpact,becausetheJapanesealreadypracticedpatriarchalauthorityand
patrilinealdescent.
B) TheimportationofChinesehouseholdpatternsstrengthenedtheroleofwomeninthe
Japanesehousehold.
C) Japanesewomen,whohadenjoyedgreatauthoritywithinthehouseholdinearlyJapan,
lostgroundwiththeimpositionofpatriarchyandpatrilinealdescent.
D) MatriarchalauthorityremainedunaffectedinJapanesehouseholds,butdescentchanged
frommatrilinealtopatrilinealpatternstypicalofChina.
Answer: C
Diff:2
PageRef:205
Topic:TheSpreadofChineseCivilizationtoJapan
Skill:Conceptual

141

45) WhatgroupwithinJapanwasmostopposedtotheintroductionofChinesecustoms?
A) Theemperors
B) Thewarriorelite
C) Thebureaucracy
D) TheBuddhistmonks
Answer: B
Diff:1
PageRef:205
Topic:TheSpreadofChineseCivilizationtoJapan
Skill:Factual

46) ThefamilyoflanguagestowhichthemigrantsfromAsiatoPolynesiabelongiscalled
A) Indo-European.
B) Chinese.
C) Bantu.
D) Austronesian.
Answer: D
Diff:1
PageRef:206
Topic:TheScatteredSocietiesofPolynesia
Skill:Factual

47) AtwhatdateweretheHawaiianislandssettled?
A) 3000B.C.E.
B) 900B.C.E.
C) 300C.E.
D) 1000C.E.
Answer: C
Diff:2
PageRef:208
Topic:TheScatteredSocietiesofPolynesia
Skill:Factual

142

48) WhichofthefollowingstatementsconcerningHawaiiansocialandpoliticalorganizationis
mostcorrect?
A) TheHawaiianswereabletoformacentralizedmonarchyunderanautocraticrulerby
700C.E.
B) Hawaiiansocietywasdividedintochieflyfamiliesdominatedby aliiwhobasedtheir
authorityonlineage.
C) Hawaiiansocietywasessentiallyasocietyofequalswithritualauthoritybeinggranted
toasingleshaman.
D) Hawaiiansocietyfailedtoprogressbeyondthelevelofhuntingandgatheringband.
Answer: B
Diff:2
PageRef:208
Topic:TheScatteredSocietiesofPolynesia
Skill:Conceptual

49) WhichofthefollowingstatementsconcerningthenatureofMaorisocialandpolitical
organizationismostaccurate?
A) TheMaorisdevelopedacentralizedempireunderasingleautocraticrulersupportedby
aliteratepriesthood.
B) TheMaoriswereunabletoprogressbeyondthelevelofhuntingandgatheringband.
C) Maorisocietywasdividedintochieflyfamiliesdominatedbyalii wholegitimizedtheir
authoritythroughgenealogicaldescent.
D) Maorisocietywastribalwithsubdivisionscalledhapu ledbywarriorchiefs.
Answer: D
Diff:2
PageRef:209
Topic:TheScatteredSocietiesofPolynesia
Skill:Conceptual

50) WhichoftheemergingareaswasNOTincontactwithmoreestablishedcivilizationcenters
andthusdevelopedalessadvancedmaterialculture?
A) Polynesia
B) NorthernEurope
C) Japan
D) Sub-SaharanAfrica
Answer: A
Diff:2
PageRef:210
Topic:Conclusion:TheEmergingCultures
Skill:Conceptual

143

EssayQuestions
1) Towhatextentdoesthedevelopmentofcivilizationinsub -SaharanAfricademonstratethe
significanceofinfluencefromcorecivilizations?
Answer: DevelopmentofsedentaryagriculturereceivedfromwesternAsia;domesticated
animalsimportedfromwesternAsia;ironmetallurgyintroducedfromwesternAsia;
sub-Saharanculturesdevelopedironmetallurgywithoutintermediarystepsofcopper
andbronze.
Diff:2
PageRef:185-188
Topic:TheSpreadofCivilizationinAfrica
Skill:Conceptual

2) DescribetheculturesofnorthernEurope.
Answer: Celts,Germans,Slavsbasedontribalorganizationdevelopedintoregionalkingdoms;
practicedmixedagricultureandhuntingeconomies;materialculturewas
unsophisticatedbutdiddevelopironmetallurgy;lackofmonumentalarchitecture,
urbanization.
Diff:2
PageRef:195-199
Topic:TheSlavsandGermansontheNorthernRim
Skill:Conceptual

3) InwhatwaysdidtheimportationofChinesecultureaffectearlyJapan?
Answer: JapaneseabletobuildtruebureaucracyafterintroductionofChinesescript;ableto
learnfromtextsinscience,religion,art;importationofBuddhismmonksstressed
strongcentralmonarchy;developedmonasteriesandshrines;changeinmatriarchal
householdstoChineseemphasisonpatriarchalauthority;developmentofmerchantsas
separateclass.
Diff:2
PageRef:200-205
Topic:TheSpreadofChineseCivilizationtoJapan
Skill:Conceptual

4) WhatwasthesocialorganizationofthePolynesianpeoples?Howdiditcomparewiththatof
civilizations?
Answer: Hawaiiansocietyhighlystratifiedwithalii ortribalchieftainswhocontrolledsections
ofHawaiianislands;authoritybasedongenealogy;commonersengagedinagriculture;
subjectedtopaymentoftributetoalii.NewZealandsocietyorganizedintotribes
subdividedintohapu;withinhapudividedintoextendedfamilyhouseholds;hapuledby
skilledwarriors;Polynesiansocietieslackedoccupationalspecializationfoundin
civilizations;limitedtechnologicaladvance.
Diff:2
PageRef:206-210
Topic:TheScatteredSocietiesofPolynesia
Skill:Conceptual

144

5) Howwereallofthefringesocietiessimilar?Towhatextentweretheydifferent?
Answer: Alldependedondevelopmentofagriculturalsystemsforevolutionofmorecomplex
socialandpoliticalsystems;eachsocietymaintaineduniqueelementsfromitspast
(JapaneseShintoism,warriororganizationofGermanictribes,languagestructureof
Bantu);allofthesocietieswereinfluencedbycorecivilizationsexceptonePolynesia.
Diff:2
PageRef:209-210
Topic:Conclusion:TheEmergingCultures
Skill:Conceptual

145

Chapter10 TheEndoftheClassicalEra:WorldHistory
inTransition,200-700C.E.
Multiple-ChoiceQuestions
1) WhichofthefollowingwasacommonproblemforallthreeoftheEurasianclassical
civilizations?
A) Totallossofallelementsofcivilization
B) Outsideinvasionsbynomadicpastoralists
C) Failureoftheagriculturalsystems
D) Internalrebellionsofpeasants
Answer: B
Diff:2
PageRef:214
Topic:Introduction
Skill:Conceptual

2) WhichofthefollowingpeoplecontributedtothedeclineinRomancivilization?
A) AttilatheHun
B) Hannibal
C) Pompey
D) Gupta
Answer: A
Diff:2
PageRef:214
Topic:Introduction
Skill:Factual

3) Whichclassicalcivilizationwasthefirsttodecline?
A) HanChina
B) Rome
C) GuptaIndia
D) Theyalldeclinedatthesametime.
Answer: A
Diff:2
PageRef:214
Topic:Introduction
Skill:Conceptual

146

4) WhatwasthecauseofthedeterioratingconditionofthepeasantryinlaterHanChina?
A) Alengthydroughtthatcausedfailureofthecrops
B) ThegrowthofslaveryfollowingtheintroductionofBuddhism
C) Thegrowingpowerofthegreatlandholderstotaxleadingtogrowingserfdom
D) Confucianacceptanceoftheenslavementofthesmallfarmers
Answer: C
Diff:2
PageRef:214
Topic:UpheavalsinEasternandSouthernAsia
Skill:Conceptual

5) TheYellowTurbanswere
A) ConfucianadministratorsintheempireofthelaterHan.
B) BuddhistmissionariesfollowingthefalloftheHan.
C) leadingmembersofthescholar-gentryinHanChina.
D) DaoistleadersofapeasantrevolutionagainsttheHan.
Answer: D
Diff:2
PageRef:215
Topic:UpheavalsinEasternandSouthernAsia
Skill:Factual

6) WhichofthefollowingdevelopmentsdidNOToccurfollowingthefalloftheHandynasty?
A) Chinawasruledforseveralcenturiesbythelandowningclasswhooperatedprivate
armies.
B) SouthernandnorthernChinapulledapartwithgreaterprosperityinsouthernChina.
C) AHsiung-nudynastywasestablishedinnorthernChina.
D) BuddhismenteredChina.
Answer: C
Diff:2
PageRef:215
Topic:UpheavalsinEasternandSouthernAsia
Skill:Conceptual

147

7) WhichofthefollowingstatementsaboutearlyBuddhisminChinaismostaccurate?
A) TheChineseimposedsomeoftheirownvaluesonBuddhismsuchastheimportanceof
maintainingfamiliesandpoliticalloyaltytothestate.
B) TheIndianBuddhistwhoenteredChinaseparatedthemselvesfromChinesetraditions
andestablishedregionalcommunitiesthatremainedindependentoftheChinesestate.
C) ChineseBuddhismborenoresemblancetoIndianBuddhism.
D) DaoistssawtheemergenceofChineseBuddhismasanopportunitytooverthrowthe
fundamentaltrendinChinatowardConfucianism.
Answer: A
Diff:2
PageRef:215
Topic:UpheavalsinEasternandSouthernAsia
Skill:Conceptual

8) HowdidtheemergenceofBuddhisminChinaaffectDaoism?
A) DaoismmergedwithBuddhismtocreateasinglereligiousmovement.
B) DaoismrapidlydisappearedfollowingtheappearanceofBuddhism.
C) TheappearanceofBuddhismledtogreaterformalizationofDaoistdoctrineandmore
effortstoreachthecommonpeople.
D) DaoismbecameanillegalreligionandtheChinesestatemovedrapidlytosuppress
variousDaoistrituals,particularlythosedealingwithmagicandhealing.
Answer: C
Diff:2
PageRef:215
Topic:UpheavalsinEasternandSouthernAsia
Skill:Conceptual

9) WhatdynastysucceededtheHan?
A) Tang
B) Song
C) Sui
D) Qin
Answer: C
Diff:1
PageRef:217
Topic:UpheavalsinEasternandSouthernAsia
Skill:Factual

148

10) WhatwasthecauseofthedemiseoftheSuidynasty?
A) RebellionsbytheYellowTurbans
B) FinancialcollapsecausedbyexpansionintoKoreaandcentralAsia
C) OverthrowbyacombinationoflandholdersandBuddhistmonks
D) InvasionfromsoutheastAsia
Answer: B
Diff:2
PageRef:217
Topic:UpheavalsinEasternandSouthernAsia
Skill:Conceptual

11) WhatgroupwasresponsibleforthedisruptionoftheGuptaempire?
A) Aryans
B) Tamils
C) Germans
D) Huns
Answer: D
Diff:1
PageRef:217
Topic:UpheavalsinEasternandSouthernAsia
Skill:Factual

12) TheimperialstructureofIndiawasbrieflyrestoredin506C.E.duringthereignof
A) ChandraguptaII.
B) Harsha.
C) Chandrasankar.
D) Devi.
Answer: B
Diff:1
PageRef:217
Topic:UpheavalsinEasternandSouthernAsia
Skill:Factual

13) ThenamegiventotheregionalprinceswhogovernedmuchofIndiaaftertheimperialperiod
was
A) emir.
B) Dolmut.
C) Bhoona.
D) Rajput.
Answer: D
Diff:1
PageRef:217
Topic:UpheavalsinEasternandSouthernAsia
Skill:Factual

149

14) WhatwasthestatusofBuddhisminIndiafollowingthefalloftheGuptaempire?
A) ItcontinuedtodeclineinthefaceoftheGuptapreferenceforHinduismandthedislike
ofthesucceedingmilitaryrulersforBuddhistprinciples.
B) Buddhismdeclined,butretainedapresenceinIndiaasitwasadoptedbytheHunnic
invaders.
C) BuddhismwasresurrectedasamajorreligioninIndiabecauseitwasadoptedbymany
oftheregionalleaderswhosoughttobuttresstheirauthority.
D) BuddhismreplacedHinduismastheprimaryreligionoftheIndianpopulation,although
itwasoutlawedbytherulingmilitaryprinces.
Answer: A
Diff:2
PageRef:217
Topic:UpheavalsinEasternandSouthernAsia
Skill:Conceptual

15) ThecultofwhatHindudivinitygainedgreaterpopularityfollowingthefalloftheGupta
empire?
A) Shiva
B) Devi
C) Kali
D) Mandala
Answer: B
Diff:2
PageRef:217
Topic:UpheavalsinEasternandSouthernAsia
Skill:Factual

16) WhatwastheimpactofthereassertionofHinduismonthecastesystemofIndia?
A) Thecastesystemwasabolished.
B) Thecastesystemremainedimportantandunchanged.
C) ThecastesystemwasrestrictedtonorthernIndia.
D) Thecastesystembecamemorecomplex.
Answer: D
Diff:3
PageRef:217
Topic:UpheavalsinEasternandSouthernAsia
Skill:Conceptual

150

17) DuringwhatcenturydidtheArabicarmiesrepresentingIslamreachnorthwesternIndia?
A) 4th
B) 6th
C) 7th
D) 8th
Answer: C
Diff:2
PageRef:218
Topic:UpheavalsinEasternandSouthernAsia
Skill:Factual

18) WhichstatementaboutIndiaseconomicactivityinthewakeofthecollapseoftheGupta
dynastyisNOTtrue?
A) Indiaremainedeconomicallystrong
B) ThedecadesafterthefalloftheGuptassawnewoutreachintrade
C) Tradecollapsed,andIndiawaseconomicallydevastated
D) Bythe8thcentury,IslamiccompetitionbegantoaffectIndiaseconomicposition
Answer: B
Diff:3
PageRef:218
Topic:UpheavalsinEasternandSouthernAsia
Skill:Conceptual

19) HowdoesthedeclineandfalloftheRomanempirecomparechronologicallytothecollapseof
theHanandtheGuptaempires?
A) TheRomanempirecollapsedbeforeeitheroftheothertwo.
B) TheRomanempirecollapsedafterbothoftheothertwo.
C) TheRomanempirecollapsedaftertheHanbutbeforetheGupta.
D) TheRomanempirecollapsedaftertheGuptabutbeforetheHan.
Answer: C
Diff:3
PageRef:218
Topic:TheDeclineandFalloftheRomanEmpire
Skill:Factual

151

20) WhataspectofRomespopulationsizewasasignofitsdecline?
A) Decliningpopulationasbirthratesnolongerkeptpacewithdeathrates
B) RisingpopulationasRomanexpansionledtoincreasingnumberofslaves
C) Decliningpopulationasreligiousconversionledtomanymaleschoosingcelibacyasa
wayoflife
D) Risingpopulationleadingtofoodshortages,particularlyacuteintheeasternhalfofthe
empire
Answer: A
Diff:2
PageRef:219
Topic:TheDeclineandFalloftheRomanEmpire
Skill:Conceptual

21) WhichofthefollowingwasNOTasymptomofRomesdecline?
A) ThebrutalityandarbitrarinessofmanylaterRomanemperors
B) Theincreasingdifficultyoftaxcollecting
C) AgrowingsenseofdespairamongRomancitizens
D) Thecatastrophiceffectsofcontinuingexpansion
Answer: D
Diff:2
PageRef:219
Topic:TheDeclineandFalloftheRomanEmpire
Skill:Conceptual

22) AfterwhatdatedidthesignsofdeclinebecomeevidentwithintheRomanempire?
A) 88C.E.
B) 180C.E.
C) 250C.E.
D) 476C.E.
Answer: B
Diff:2
PageRef:218
Topic:TheDeclineandFalloftheRomanEmpire
Skill:Factual

152

23) WhatimpactdidtheendofRomanexpansionhaveontheeconomyoftheempire?
A) Itpermittedtheempiretofocusitsresourcesoverasmallerterritoryandreducedthe
strainontheimperialeconomy.
B) TheprimaryresultwastherestrictionofRomancommercetotradewithsub-Saharan
Africa.
C) TheeconomysufferedastheRomanslostcontroloverthetradeoftheIndianOcean.
D) TheendofexpansionledtoalaborshortageastheRomaneconomydependedona
steadysupplyofslavelabor.
Answer: D
Diff:2
PageRef:219
Topic:TheDeclineandFalloftheRomanEmpire
Skill:Conceptual

24) WhatwasthegreatestweaknessinthepoliticalconstitutionoftheRomanEmpire?
A) TheRomanarmydependedentirelyonsmallfarmersasconscripts.
B) Therewasnoestablishedprincipleofsuccessiontotheofficeofemperor.
C) TheRomanshadnoestablishedlegalsystem.
D) ThearistocraticSenatecompletelydominatedtheimperialconstitution.
Answer: B
Diff:2
PageRef:219
Topic:TheDeclineandFalloftheRomanEmpire
Skill:Conceptual

25) WhichofthefollowingwasNOTaproblemintheprocessofdeclinewithintheRoman
Empire?
A) Aseriesofplagues
B) Leadpoisoning
C) Risingpopulationleadingtofoodshortages
D) Increasingburdensonthestatesbudgetleadingtogreatertaxation
Answer: C
Diff:2
PageRef:218-219
Topic:TheDeclineandFalloftheRomanEmpire
Skill:Conceptual

153

26) WhatwasthenatureoftheculturallifeoftheRomanempireduringtheperiodofdecline?
A) Despitetheworseningpoliticalsituation,theperiodofdeclinewasoneofthemost
productiveculturalperiodsinRomanhistory.
B) TherewasatotaldisappearanceofanyculturalcreativityinthelaterRomanEmpire.
C) Romanscholarsinthelaterempirewereinnovativeinthefieldsofscience,mathematics,
andtechnology.
D) ThesoleareaofcreativityinlaterRomanculturewasinChristiantheology.
Answer: D
Diff:2
PageRef:220
Topic:TheDeclineandFalloftheRomanEmpire
Skill:Conceptual

27) WhichofthefollowingstatementsconcerningtheRomanupperclassattheendoftheempire
ismostaccurate?
A) TheRomanupperclassvaliantlyattemptedtohaltthedeclineoftheempire,butwas
overwhelmed.
B) BecausetheRomanupperclassmadeupthebulkoftheofficercorpsoftheRoman
armies,theirdeclinewasparticularlycritical.
C) Romesupperclassesbecamesteadilymorepleasure-seekingandindividualistic,
turningawayfromthepoliticaldevotionthathadcharacterizedtherepublic.
D) AloneamongtheRomansocialgroups,theelitewasabletoincreasetheirnumbersand
thusbecamemoredominantinthepoliticalstructure.
Answer: C
Diff:2
PageRef:219
Topic:TheDeclineandFalloftheRomanEmpire
Skill:Conceptual

28) HowdidtheplaguesthatsweptovertheRomanempireinthe2ndcenturyC.E.contributeto
theempiresdecline?
A) Theplagueskilledthelastemperor.
B) Theplaguesdecimatedthepopulationandseverelydisruptedeconomiclife.
C) TheplaguesledtowarwithHanChina.
D) TheplaguescompletelywipedoutthepopulationofRome.
Answer: B
Diff:2
PageRef:219
Topic:TheDeclineandFalloftheRomanEmpire
Skill:Conceptual

154

29) WhatoccurredwithintheagriculturalsystemattheendoftheRomanempire?
A) Thedeclineofslavelaborresultedinthereturntoprimarilysmallindependent
farmsteads.
B) Theincidenceofplaguesdrovepeoplefromthecitiesintothecountrysideandresulted
inanintensificationoftheagriculturaleconomyofthelaterRomanempire.
C) Aslifebecamemoreprecarious,manyfarmerssurrenderedfullcontroloftheirestates
andclusteredaroundtheprotectionoflargelandlords.
D) Thestateundertookcontroloftheagriculturaleconomyandestablishedstatefarms
staffedbyslaves.
Answer: C
Diff:2
PageRef:219-220
Topic:TheDeclineandFalloftheRomanEmpire
Skill:Conceptual

30) Howdidthechangeintheagriculturaleconomyaffectthebalanceofpoliticalpoweratthe
endoftheRomanempire?
A) Itgavepoliticalpowertothelandlordsbutweakenedtheholdoftheemperor.
B) Smallindependentfarmerswereabletorestoretheirpoliticalinfluenceattheexpenseof
thecentralgovernment.
C) Theemperorswereabletoestablishalmostabsolutepoliticalcontrolbecauseofthe
failureofthelargelandholders.
D) Thereductioninthenumberofslavesweakenedthelargelandholderswholostpolitical
influencetoboththecentralgovernmentandthesmallindependentfarmers.
Answer: A
Diff:2
PageRef:219-220
Topic:TheDeclineandFalloftheRomanEmpire
Skill:Conceptual

31) WhatRomanemperorattemptedtoreversethedeclineoftheempirebyadministrativereform
andimprovementinthesystemoftaxcollection?
A) Constantine
B) Commodus
C) SeptimiusSeverus
D) Diocletian
Answer: D
Diff:1
PageRef:220
Topic:TheDeclineandFalloftheRomanEmpire
Skill:Factual

155

32) UnderwhatemperordidChristianityreceiveofficialtolerationofthestate?
A) Constantine
B) Commodus
C) SeptimiusSeverus
D) Diocletian
Answer: A
Diff:1
PageRef:220
Topic:TheDeclineandFalloftheRomanEmpire
Skill:Factual

33) WhenwasthelastRomanemperorinthewesternhalfoftheempiredeposed?
A) 180C.E.
B) 240C.E.
C) 384C.E.
D) 476C.E.
Answer: D
Diff:1
PageRef:220
Topic:TheDeclineandFalloftheRomanEmpire
Skill:Factual

34) WhichpeoplesestablishedkingdomsinmanypartsoftheRomanempireby425?
A) Huns
B) Slavs
C) Germans
D) Hans
Answer: C
Diff:2
PageRef:220
Topic:TheDeclineandFalloftheRomanEmpire
Skill:Factual

156

35) WhatwastheprimaryresultofthefalloftheRomanempire?
A) DespitethecollapseofclassicalMediterraneancivilization,thebasicculturalunityofthe
regionwasretained.
B) ThecollapseoftheRomanempireledtoculturalregressionthroughouttheregionof
Mediterraneancivilization.
C) RomesfallsplittheunityoftheMediterraneanlandsthathadbeenwonthrough
HellenisticcultureandthentheRomanempire.
D) Romesfallwasdevastatinginitseasternhalf,butthecultureofthewesternregionswas
fundamentallyunaffected.
Answer: C
Diff:2
PageRef:220
Topic:TheDeclineandFalloftheRomanEmpire
Skill:Conceptual

36) WhichofthefollowingwasNOTasuccessorwithintheboundariesoftheformerRoman
empire?
A) Byzantineempire
B) Sassanidempire
C) SmallkingdomsinnorthernAfrica
D) GermanickingdomsinnorthwesternEurope
Answer: B
Diff:2
PageRef:220-221
Topic:TheDeclineandFalloftheRomanEmpire
Skill:Factual

37) InwhatregionwasthecultureoftheRomanempiremostcontinuous?
A) Byzantineempire
B) Sassanidempire
C) SmallkingdomsofnorthernAfrica
D) GermanickingdomsofnorthernEurope
Answer: A
Diff:1
PageRef:220
Topic:TheDeclineandFalloftheRomanEmpire
Skill:Conceptual

157

38) InwhatregionwasRomanculturemostseriouslyeroded?
A) Byzantineempire
B) Sassanidempire
C) SmallkingdomsofnorthernAfrica
D) WesternEurope
Answer: D
Diff:1
PageRef:221
Topic:TheDeclineandFalloftheRomanEmpire
Skill:Conceptual

39) Whatwastheimpactofthedeclineofclassicalcivilizationsonthemajorreligions?
A) Themajorreligionsbecamemoreregionalizedinspecificcivilizations.
B) AsidefromtheemergenceofIslam,theothermajorreligionssuffereddecline.
C) Themajorworldreligionsactuallyspreadtonewareasinwhichbeliefsandcultural
allegiancestookonnewterritorialpatterns.
D) HinduismandDaoismsharedinthegeneraldeclineofanimistreligionsduringthefall
ofthemajorcivilizations.
Answer: C
Diff:2
PageRef:222
Topic:DevelopmentandSpreadofWorldReligions
Skill:Conceptual

40) WhichofthefollowingisNOTasimilaritybetweenChristianityandBuddhism?
A) Bothdevelopedanimportantmonasticmovement.
B) Bothstressedthepossibilityofanafterlife.
C) Bothacceptedtheexistenceofholymenwhobuiltupspiritualmeritsthatcouldbe
tappedbyotherbelievers.
D) Bothplacedgreatemphasisonchurchorganizationandstructure.
Answer: D
Diff:2
PageRef:222-223
Topic:DevelopmentandSpreadofWorldReligions
Skill:Conceptual

158

41) WhichofthefollowingrepresentsasignificantdifferencebetweenBuddhismand
Christianity?
A) Existenceofholymen
B) Monasticism
C) Emphasisonpossessionofexclusivetruth
D) Emphasisonpossibilityofafterlife
Answer: C
Diff:2
PageRef:222-223
Topic:DevelopmentandSpreadofWorldReligions
Skill:Conceptual

42) ThewordforBuddhistholymenis
A) buddhas.
B) bodhisattvas.
C) mahayanas.
D) monks.
Answer: B
Diff:1
PageRef:223
Topic:DevelopmentandSpreadofWorldReligions
Skill:Factual

43) TheleaderunderwhoseguidanceChristiansbegantoseethemselvesaspartofanewreligion
was
A) Christ.
B) Peter.
C) Augustine.
D) Paul.
Answer: D
Diff:2
PageRef:224
Topic:DevelopmentandSpreadofWorldReligions
Skill:Factual

159

44) WhatpercentageoftheRomanpopulationhadconvertedtoChristianitybythetimeof
imperialacceptanceofthereligion?
A) Five
B) 10
C) 25
D) 40
Answer: B
Diff:2
PageRef:224
Topic:DevelopmentandSpreadofWorldReligions
Skill:Factual

45) WhywastheorganizationoftheChristianchurchstrongerinthewesternhalfoftheRoman
Empire?
A) ThereweremanymoreChristianstherethanintheeasternhalfoftheempire.
B) Christianitybeganandenjoyeditsinitialsuccessinthewesternhalfoftheempire.
C) ThewesternemperorsweremoreenthusiasticintheirsupportofearlyChristianity.
D) TheChurchwasmorecentralizedinthewest,eventuallyinthepersonofthepope.
Answer: D
Diff:2
PageRef:224
Topic:DevelopmentandSpreadofWorldReligions
Skill:Conceptual

46) ThecomplexdoctrineoftheTrinitywasconsideredattheCouncilof
A) Constantinople.
B) Ephesus.
C) Nicaea.
D) Rome.
Answer: C
Diff:1
PageRef:224
Topic:DevelopmentandSpreadofWorldReligions
Skill:Factual

160

47) WhowasresponsibleforthedevelopmentofChristianmonasticisminthewesternhalfofthe
Romanempire?
A) BenedictofNursia
B) St.Basil
C) MonomachusofSyria
D) Augustine
Answer: A
Diff:2
PageRef:224
Topic:DevelopmentandSpreadofWorldReligions
Skill:Factual

48) InwhatyearwasIslamfounded?
A) 476C.E.
B) 542C.E.
C) 610C.E.
D) 724C.E.
Answer: C
Diff:2
PageRef:225
Topic:DevelopmentandSpreadofWorldReligions
Skill:Factual

49) Whichofthefollowingwasageneraltendencyduringthespreadoftheworldsgreatreligions
attheendoftheclassicalperiod?
A) Theadoptionofpantheonsofgods
B) Theabandonmentofformaltheology
C) Disbeliefintheexistenceofanafterlife
D) Concentrationofasingledivineforceasopposedtoahostofdivinespirits
Answer: D
Diff:2
PageRef:225
Topic:DevelopmentandSpreadofWorldReligions
Skill:Conceptual

161

EssayQuestions
1) What,accordingtotheauthors,definestheendofahistoricalperiodandthebeginningofa
newperiod?Howdoestheendoftheclassicalperiodfitthisdefinition?
Answer: Geographicalshiftsinpoliticalboundaries;newkindsofcontactsamongcivilization
areas;newparallelsinculturaldevelopment.Culturalandpoliticalboundariesended
inclassicalperiodwithHunnicinvasions,Germanicinvasions;outreachofworld
religionsacrosscivilizationboundariese.g.,theemergenceofBuddhisminChinese
culture;importantchangesinbeliefsystemscommontoallcivilizations.
Diff:2
PageRef:222
Topic:Introduction
Skill:Conceptual

2) CompareandcontrastthefallofHanChinawiththatofGuptaIndia.
Answer: Chronologically,HanChinadeterioratedfirst(220C.E.);bothexperiencedforeign
invasionbynomadicpeoples;bothexperienceddisruptionofperiodofempire;both
experiencedintroductionofnewreligionsIslaminIndia,BuddhisminChina;
politicaldisruptionmuchmorelong-lastinginIndiathaninChina.
Diff:2
PageRef:214-218
Topic:UpheavalsinEasternandSouthernAsia
Skill:Conceptual

3) WhatarethecausesofthedeclineandfalloftheRomanempire?
Answer: Endofexpansionreducedimportationofslavelaborandforcedreconstructionoflabor
policiesinbasicindustries;failuretoestablishaprincipleofsuccessiontotheofficeof
emperor;interventionofarmyinpoliticalmatters;plaguedecimatedpopulation
reducingtaxbaseandfurtheraffectinglaborsupply;upperclassesdevotedto
pleasure-seekingratherthanpoliticalservice.
Diff:2
PageRef:218-220
Topic:TheDeclineandFalloftheRomanEmpire
Skill:Conceptual

4) WhatwastheimpactofthefalloftheRomanempireontheintegrityofMediterranean
classicalcivilization?
Answer:
EndofculturalunityinMediterranean;divisionintothreeculturalareas:Byzantine
Empire,smallkingdomsofnorthernAfrica(eventuallyconqueredbyIslam),Germanic
kingdomsofnorthernEurope;greatestculturaldeclineinGermanickingdoms;greatest
retentionofMediterraneancultureinByzantineEmpire.
Diff:2
PageRef:220-221
Topic:TheDeclineandFalloftheRomanEmpire
Skill:Conceptual

162

5) CompareandcontrastBuddhismandChristianity.
Answer: Comparison:bothfeaturedemphasisonotherworldlinessincludingbeliefinafterlife;
bothstressedchiefreligiousfigureasgodorsaviorenablingprocessofsalvation;both
producedmonasticmovements;bothacceptedroleofholymenwhobuiltupsurplus
meritsavailabletootherbelievers.Contrast:Christianityplacedgreateremphasison
churchorganization,missionaryactivity,andpossessionofexclusivetruth.
Diff:2
PageRef:222-223
Topic:DevelopmentandSpreadofWorldReligions
Skill:Conceptual

163

Chapter11 TheFirstGlobalCivilization:TheRiseand
SpreadofIslam
Multiple-ChoiceQuestions
1) Islamoriginatedin
A) Egypt.
B) Turkey.
C) Arabia.
D) Iraq.
Answer: C
Diff:1
PageRef:239
Topic:Introduction
Skill:Factual

2) WhichofthefollowingcontinentswasNOTaffectedbyIslaminthemillenniumafter600
C.E.?
A) Africa
B) Europe
C) Asia
D) SouthAmerica
Answer: D
Diff:1
PageRef:236
Topic:Introduction
Skill:Factual

3) TheArabiccamelnomadswerereferredtoas
A) Hashim.
B) bedouin.
C) mawali.
D) ayan.
Answer: B
Diff:1
PageRef:239
Topic:DesertandTown:TheArabianWorldandtheBirthofIslam
Skill:Factual

164

4) Whichofthefollowingstatementsmostaccuratelydescribestheextentofsedentary
agricultureintheArabianpeninsula?
A) TherewasnosedentaryagricultureintheArabianpeninsulabecauseoftheextreme
aridityoftheclimate.
B) InthefarnorthalongtheborderswiththePersianempiresedentaryagriculturewas
common.
C) OnlyinthecoastalregionsofthefarsouthoftheArabianpeninsuladidextensive
sedentaryagriculturedevelopinancienttimes.
D) Sedentaryagriculturewasdistributedthroughoutthepeninsulaastheresultofthe
constructionofextensiveirrigationsystems.
Answer: C
Diff:2
PageRef:239
Topic:DesertandTown:TheArabianWorldandtheBirthofIslam
Skill:Conceptual

5) WhichofthefollowingculturesfoundintheArabianpeninsulawasmostsignificantin
shapingthedevelopmentofIslam?
A) Bedouin
B) Urban
C) Sedentaryagriculturalvillages
D) Huntingandgathering
Answer: A
Diff:2
PageRef:240
Topic:DesertandTown:TheArabianWorldandtheBirthofIslam
Skill:Conceptual

6) WhichofthefollowingstatementsconcerningbedouinsocietyisNOTaccurate?
A) Bedouinherderslivedinkin-relatedclangroups.
B) Bedouinslivedinhighlymobiletentencampments.
C) Clanswerecommonlycongregatedtogetherinlargertribalgroupings.
D) Arabiansocietyfosteredstrongdependenceonloyaltyandcooperationwithkin.
Answer: C
Diff:2
PageRef:240
Topic:DesertandTown:TheArabianWorldandtheBirthofIslam
Skill:Conceptual

165

7) Leadersofbedouinclanswerecalled
A) shaykhs.
B) wazirs.
C) mawali.
D) dhows.
Answer: A
Diff:1
PageRef:241
Topic:DesertandTown:TheArabianWorldandtheBirthofIslam
Skill:Factual

8) WhatwasthemostimportantcityinnorthernArabia?
A) Medina
B) Kaba
C) Mecca
D) Jerusalem
Answer: C
Diff:2
PageRef:241
Topic:DesertandTown:TheArabianWorldandtheBirthofIslam
Skill:Conceptual

9) Whichofthefollowingstatementsconcerninginter-clanrelationshipsinbedouinsocietyis
mostaccurate?
A) Clanswithinthesametribealmostneverengagedinwarfare,butviolencebetween
differenttribeswascommon.
B) Arabicsocietywastoomobiletoresultinmanycontactsbetweenclans,therefore
violencewasminimal.
C) Inter-clanviolenceovercontrolofwaterandpasturagewascommon.
D) Inter-clanviolencewasregulatedbyauniversallyrecognizedcodeoflawimposedby
theQurayshinMecca.
Answer: C
Diff:2
PageRef:241
Topic:DesertandTown:TheArabianWorldandtheBirthofIslam
Skill:Conceptual

166

10) Whatwastheresultofinter-clanrivalries?
A) Itkeptpopulationdowninaregionthatcouldsupportfewpeople.
B) Itweakenedthebedouinincomparisontoneighboringpeoplesandempires.
C) Itallowedforintermarriagebetweenclangroups,thuspreventingsocialisolation.
D) Itpreventedmobilityandmigrationthatwouldhavedebilitatedeffortsatmorecomplex
socialorganization.
Answer: B
Diff:2
PageRef:241
Topic:DesertandTown:TheArabianWorldandtheBirthofIslam
Skill:Conceptual

11) WhatclanwasresponsibleforthefoundationofMecca?
A) Umayyad
B) Abbasid
C) Aghlabid
D) Almoravid
Answer: A
Diff:1
PageRef:241
Topic:DesertandTown:TheArabianWorldandtheBirthofIslam
Skill:Factual

12) WhatwastheKaba?
A) ThetribethatdominatedMecca
B) ThenamegiventoMuhammadsflightfromMecca
C) TheportofMecca
D) Thereligiousshrinethatwasthefocusofanannualtruce
Answer: D
Diff:1
PageRef:241
Topic:DesertandTown:TheArabianWorldandtheBirthofIslam
Skill:Factual

167

13) WhatwasthemajordifferencebetweenMedinaandMecca?
A) PoliticaldominanceinMedinawascontestedbetweenanumberofJewishandbedouin
tribes.
B) Meccawasestablishedinanoasis,andMedinawasinamountainousregion.
C) Medinawasengagedinlong-distancecaravantrade,whileMeccawasnot.
D) MedinawaslocatedonthewesternsideoftheArabianpeninsula,whileMeccawas
locatedonthePersianGulf.
Answer: A
Diff:2
PageRef:241
Topic:DesertandTown:TheArabianWorldandtheBirthofIslam
Skill:Conceptual

14) Whichofthefollowingstatementsmostaccuratelydescribesthestatusofwomeninbedouin
societypriortoIslam?
A) Womenwereregardedaslittlemorethanpropertywithneitherrightsnorstatus.
B) Descentinbedouintribeswasstrictlypatrilineal.
C) Womenweretheequalofmalesintheruggedsocietyofthedesertbedouin.
D) Womeninpre-Islamicbedouincultureenjoyedgreaterfreedomandhigherstatusthan
thoseoftheByzantineandPersianempires.
Answer: D
Diff:2
PageRef:241
Topic:DesertandTown:TheArabianWorldandtheBirthofIslam
Skill:Conceptual

15) Whatwasthenatureofthematerialcultureofbedouinsociety?
A) Thebedouinsconstructednumeroustemplecomplexesfeaturingmonumental
architectureintheformofpyramids.
B) Althoughtheirnomadiclifestyledidnotpermitthedevelopmentofmonumental
architecturalforms,thebedouinswereskilledpaintersandsculptors.
C) Meccawasamajorcenterforthedevelopmentofartandarchitecture,butthedesert
bedouinproducedlittleofculturalvalue.
D) Exceptinthesedentaryagriculturalcommunitiesofthesouth,therewaslittleartor
architecture;andthechieffocusofculturalcreativitywasoralpoetry.
Answer: D
Diff:2
PageRef:242
Topic:DesertandTown:TheArabianWorldandtheBirthofIslam
Skill:Conceptual

168

16) Whatwasthenatureofpre-Islamicbedouinreligion?
A) MostofthebedouinwereChristians.
B) MostofthebedouinwereJews.
C) Bedouinreligionformostclanswasablendofanimismandpolytheismfocusingonthe
worshipofnaturespirits.
D) ThebedouinwerestrictlymonotheistswhoworshippedAllah.
Answer: C
Diff:2
PageRef:242
Topic:DesertandTown:TheArabianWorldandtheBirthofIslam
Skill:Conceptual

17) Whichofthefollowingrepresentsamountingpressureforchangeinpre -Islamicsociety?


A) Invasionfromsub-SaharanAfrica
B) GreaterByzantineandSassaniancontroloverArabictribesofthepeninsulaandArabic
migrationtoMesopotamia
C) InfluenceofHinduanimism
D) Increasinginfluenceofpolytheism
Answer: B
Diff:2
PageRef:242
Topic:TheLifeofMuhammadandtheGenesisofIslam
Skill:Conceptual

18) WhatwastheclanintowhichMuhammadwasborn?
A) Umayyad
B) Abbasid
C) Almoravid
D) BanuHashim
Answer: D
Diff:1
PageRef:242
Topic:TheLifeofMuhammadandtheGenesisofIslam
Skill:Factual

169

19) WhendidMuhammadreceivethefirstrevelationsthatwereeventuallywrittendowninthe
Quran?
A) 550C.E.
B) 610C.E.
C) 622C.E.
D) 632C.E.
Answer: B
Diff:2
PageRef:243
Topic:TheLifeofMuhammadandtheGenesisofIslam
Skill:Factual

20) WhatwastheinitialresponseoftheUmayyadstoMuhammadsnewfaith?
A) Theyregardedhimasathreattotheirwealthandpowerashequestionedthetraditional
godsoftheKaba.
B) TheysoughttoprotecthimfromaplotonhislifebytheBanuHashim.
C) TheUmayyadsimmediatelyacceptedMuhammadastheirreligiousandpoliticalleader
andthechiefpowerinMecca.
D) TheUmayyadssimplyignoredMuhammadasaninsignificantmemberofapowerless
clan.
Answer: A
Diff:2
PageRef:243
Topic:TheLifeofMuhammadandtheGenesisofIslam
Skill:Conceptual

21) WhichofthefollowingstatementsconcerningMuhammadsflighttoMedinaisNOTcorrect?
A) HefledbecauseofthethreatofassassinationinMecca
B) HefledbecausehewasinvitedtomediateadisputebetweenthetribesofMedina
C) MuhammadfledfromMeccawithnearlyonequarterofthecityspopulation
D) OnceinMedinaheattractednewfollowerstohisfaith
Answer: C
Diff:2
PageRef:243
Topic:TheLifeofMuhammadandtheGenesisofIslam
Skill:Conceptual

170

22) TheUmayyadswerethreatenedbyallofthefollowingdevelopmentsEXCEPT
A) MuhammadsdeclaredaimtodestroyMedina.
B) thedevelopmentofanewreligionunderMuhammadsdirection.
C) thegrowingpowerofMedina.
D) thenewfaiththreatenedtosupplantthegodsoftheKaba.
Answer: A
Diff:1
PageRef:243
Topic:TheLifeofMuhammadandtheGenesisofIslam
Skill:Factual

23) WhatwasthedateofMuhammadsflighttoMedinafromMecca?
A) 570C.E.
B) 610C.E.
C) 622C.E.
D) 635C.E.
Answer: C
Diff:1
PageRef:243
Topic:TheLifeofMuhammadandtheGenesisofIslam
Skill:Factual

24) WhatwastheUmayyadresponsetoMuhammadsmigrationtoMedinaandsubsequent
successthere?
A) TheUmayyadrulersofMeccaignoredMuhammadaslongashewascontenttoremain
inMedina.
B) WarbrokeoutbetweenMeccaandMedinaresultingintheeventualvictoryof
MuhammadandtheMedinaclans.
C) WarbrokeoutbetweenMeccaandMedinaresultingintheeventualvictoryofthe
Umayyads.
D) Reluctantlybutpeacefully,theUmayyadswereconvertedtoIslam.
Answer: B
Diff:2
PageRef:243
Topic:TheLifeofMuhammadandtheGenesisofIslam
Skill:Conceptual

171

25) WhatisthenameofthebookthatcontainsMuhammadsrevelations?
A) Bible
B) ArabianNights
C) Umma
D) Quran
Answer: D
Diff:2
PageRef:243
Topic:TheLifeofMuhammadandtheGenesisofIslam
Skill:Conceptual

26) WhichofthefollowingstatementsconcerningtheethicalsystemofearlyIslamisNOTcorrect?
A) IslamstressedthedignityofallbelieversandtheirequalityintheeyesofAllah.
B) Islamstressedtheresponsibilityofthewealthyandstrongtocareforthepoorandweak.
C) Ataxforcharitywasobligatoryinthenewfaith.
D) TheteachingsoftheProphetandtheQuranwerenotformallyincorporatedintoabody
oflaw.
Answer: D
Diff:2
PageRef:244
Topic:TheLifeofMuhammadandtheGenesisofIslam
Skill:Conceptual

27) WhatwasMuhammadsteachingwithrespecttotherevelationsofothermonotheistic
religions?
A) MuhammadacceptedtheearlierChristianrevelations,butrejectedcompletelyany
influencefromJudaism.
B) MuhammadacceptedtheearlierJudaicrevelations,butrejectedcompletelyany
influencefromChristianity.
C) MuhammadacceptedthevalidityofearlierChristianandJudaicrevelationsandtaught
thathisownrevelationswereafinalrefinementandreformulationofearlierones.
D) Muhammadstressedthatonlyhisownrevelationshadmeritandthatotherswereworks
ofthedevil.
Answer: C
Diff:1
PageRef:245
Topic:TheLifeofMuhammadandtheGenesisofIslam
Skill:Conceptual

172

28) WhichofthefollowingisNOTamongthefivepillarsofIslam?
A) Aconfessionoffaith
B) Hajj
C) Zakat
D) PilgrimagetoMedina
Answer: D
Diff:2
PageRef:245
Topic:TheLifeofMuhammadandtheGenesisofIslam
Skill:Conceptual

29) WhathappenedafterMuhammadsdeathin632?
A) ManyofthebedouintribesrenouncedIslam.
B) IslamceasedtoexistuntilitwasreestablishedundertheUmayyaddynastyatDamascus.
C) Afteralengthyperiodofgrief,thetribesselectedanewleaderbasedontheestablished
principleofsuccessionintheQuran.
D) Amilitarycommander,Khalidibnal-Walid,waschosenasleaderofIslam.
Answer: A
Diff:2
PageRef:246
Topic:TheArabEmpireoftheUmayyads
Skill:Conceptual

30) ThewarstodefeatrivalprophetsandrestoretheunityofIslamwerecalled
A) Ridda.
B) dhow.
C) Karbala.
D) thefirstfitnah.
Answer: A
Diff:2
PageRef:246
Topic:TheArabEmpireoftheUmayyads
Skill:Factual

31) TheofficeofthepoliticalandreligioussuccessorofMuhammadwascalled
A) wazir.
B) dhow.
C) Karbala.
D) Caliph.
Answer: D
Diff:1
PageRef:246
Topic:TheArabEmpireoftheUmayyads
Skill:Factual

173

32) WhichofthefollowingwasNOTareasonfortheearlyexpansionofIslambeyondArabia?
A) Thedesireforbooty
B) Thesenseofcommoncauseandunitedstrength
C) ThedesiretoconvertnewpopulationstoIslam
D) Ameanstoreleasetheenergiesofthebedouintribesagainstothersthanthemselves
Answer: C
Diff:2
PageRef:246-247
Topic:TheArabEmpireoftheUmayyads
Skill:Conceptual

33) WhydidtheArabwarriorsnotwanttoconvertlargenumbersofpeopletoIslam?
A) MuhammadspecificallystatedthatIslamcouldonlybespreadamongtheArabs.
B) Theywouldhavehadtosharetheirbootyandwouldhavelosttaxrevenues.
C) Theylackedthepoliticalorganizationtogovernthemandfearedinsurrectionby
non-Arabs.
D) Conversionwouldhavesloweddowntheprocessofconquest.
Answer: B
Diff:2
PageRef:247
Topic:TheArabEmpireoftheUmayyads
Skill:Conceptual

34) WhichofthefollowingareasoftheByzantineempirewasNOTconqueredbytheMuslimsby
650C.E.?
A) Palestine
B) Egypt
C) Syria
D) AsiaMinor
Answer: D
Diff:2
PageRef:247
Topic:TheArabEmpireoftheUmayyads
Skill:Factual

174

35) WhywastheCaliphUthmandislikedbysomanyArabs?
A) Hehadhaltedtheprocessofexpansionandthusstoppedtheflowofbootytothe
tribesmen.
B) HewasthefirstCaliphtobechosenfromMuhammadsearlyenemies,theUmayyads.
C) HewasnotanArab.
D) HewasafirmsupporterofMuhammadsson-in-lawandnephew,Ali.
Answer: B
Diff:2
PageRef:248
Topic:TheArabEmpireoftheUmayyads
Skill:Conceptual

36) WhatwastheresultofthefirstcivilwarbetweenAliandtheUmayyads?
A) AliwasabletodefeattheUmayyadclanandreducethemtopoliticalinsignificance.
B) Aliwaskilledintheconflict,buthissonHasanwasnamedCaliphandwonagreat
victoryovertheAbbasids.
C) Despiteearlysuccesses,AlisfactiondisintegratedleadingtoanUmayyadvictoryand
Alisassassination.
D) AlisufferedadisastrousmilitarydefeatattheBattleofSiffin,andtheUmayyads
emergedvictorious.
Answer: C
Diff:2
PageRef:248
Topic:TheArabEmpireoftheUmayyads
Skill:Conceptual

37) ThepoliticalandtheologicalfactionwithinIslamthatrecognizedonlyAliandthedescendants
ofthefamilyofMuhammadasrightfulrulerswascalled
A) Shias.
B) Sunnis.
C) Kharij.
D) Fiqhs.
Answer: A
Diff:2
PageRef:248
Topic:TheArabEmpireoftheUmayyads
Skill:Conceptual

175

38) UndertheUmayyads,thepoliticalcenterofIslamshiftedto
A) Baghdad.
B) Merv.
C) Constantinople.
D) Damascus.
Answer: D
Diff:1
PageRef:248
Topic:TheArabEmpireoftheUmayyads
Skill:Factual

39) WhatwasthenatureofcitizenshipwithintheUmayyadempire?
A) AllconvertstoIslam,regardlessoftheirethnicorigins,werefullcitizensandmembersof
theelite.
B) OnlyMuslimArabswerefirst-classcitizens.
C) TheUmayyadsrecognizedallresidentsoftheirempire,whetherMuslimsorpeoplesof
thebookasfullcitizens.
D) ArabsrapidlylosttheirdominanceintheUmayyadempiretothenativeresidentsof
Persia.
Answer: B
Diff:2
PageRef:248
Topic:TheArabEmpireoftheUmayyads
Skill:Conceptual

40) Thetermfornon-ArabMuslimconvertsis
A) jizya.
B) dhow.
C) mawali.
D) dhimmis.
Answer: C
Diff:2
PageRef:249
Topic:TheArabEmpireoftheUmayyads
Skill:Factual

176

41) WhatwastheUmayyadattitudetootherreligions?
A) TheUmayyadssuppressedallreligionswithintheirterritoriesotherthanIslam.
B) TheUmayyadsconvertedtoChristianity,butcontinuedtopermittheopenworshipof
Islam.
C) TheUmayyadsdisplayedtolerancetowardsthereligionsofdhimmipeoples.
D) ChristianityandJudaismweresuppressedasheresies,butothercommunitieswere
permittedtoretaintheirreligions.
Answer: C
Diff:2
PageRef:249
Topic:TheArabEmpireoftheUmayyads
Skill:Conceptual

42) WhatwasthemostsignificantofthetransformationsbroughtaboutbytheAbbasidsriseto
power?
A) ThefinaldefeatoftheByzantineempire
B) TheadmissionofthemawaliasfullmembersoftheIslamiccommunity
C) ThedestructionofabsolutismwithinIslamicgovernment
D) ThedestructionofSunniinfluenceswithinIslam
Answer: B
Diff:2
PageRef:253
Topic:FromArabtoIslamicEmpire
Skill:Conceptual

43) TheAbbasidsmovedthepoliticalcenteroftheirempireto
A) Baghdad.
B) Constantinople.
C) Merv.
D) Damascus.
Answer: A
Diff:1
PageRef:253
Topic:FromArabtoIslamicEmpire
Skill:Factual

177

44) WhatwasthenatureoftheAbbasidgovernment?
A) TheAbbasidsabandonedtheformalityandabsolutismoftheUmayyadsandestablished
anopenandrepresentativegovernment.
B) TheAbbasidsoutdidtheUmayyadsinestablishinganabsolutistgovernment
symbolizedbythegrowingpowersofthewazirsandthesinisterpresenceofthe
executioner.
C) TheAbbasidgovernmentrepresentedareturntotheprinciplesofgovernmentinthefirst
daysoftheOrthodoxCaliphate.
D) TheAbbasidscontinuedthepoliciesoftheUmayyadsvirtuallywithoutchange,
includingthemaintenanceofanexclusivelyArabicelite.
Answer: B
Diff:2
PageRef:253
Topic:FromArabtoIslamicEmpire
Skill:Conceptual

45) WhatwasthenatureoftheeconomyoftheAbbasidperiod?
A) Itwasaperiodofgeneralprosperitytypifiedbyurbangrowthandtherestorationofthe
Afro-Eurasiantradeaxis.
B) Therewasageneralcrisisintheagriculturaleconomyresultingfromtheconstant
warfareoftheperiod.
C) Theeconomicdownturnoftheerawastypifiedbythebreakdowninthetradebetween
theMiddleEastandChina.
D) AlthoughcommercewasgenerallyresuscitatedduringtheAbbasidperiod,artisan
productiondroppedoffsignificantly.
Answer: A
Diff:2
PageRef:254
Topic:FromArabtoIslamicEmpire
Skill:Conceptual

46) Whoweretheayan?
A) Abbasidbureaucrats
B) Africanslaves
C) Freeartisans
D) Arurallandholdingelite
Answer: D
Diff:2
PageRef:255
Topic:FromArabtoIslamicEmpire
Skill:Conceptual

178

47) WhatwasthestatusofartisansinAbbasidcities?
A) Handicraftindustrieswerestaffedbyslavelaborexclusively.
B) ThenumberofartisansdecreasedalongwiththeeconomiccrisisoftheAbbasidperiod.
C) Artisanswerefreemenwhoownedtheirowntoolsandwhoformedguildlike
organizationstonegotiatewages.
D) Artisanswereabletoutilizetheirguildlikeorganizationstoseizepoliticalcontrolofmost
Abbasidtowns.
Answer: C
Diff:2
PageRef:254
Topic:FromArabtoIslamicEmpire
Skill:Conceptual

48) WhatwasthenatureofslaverywithintheAbbasidsocialsystem?
A) AccordingtotheQuran,slaverycouldnotexistinIslam,andtheAbbasidfreedall
formerslaves.
B) Slaverywaslimitedtothenon-ArabconvertstoIslam,anddiedoutduringtheperiodof
theAbbasidempire.
C) Becausemostunskilledlaborwaslefttotheunfree,slavescouldbefoundinboththe
townsandcountrysideoftheAbbasidempire.
D) SlaverywasknowninAbbasidcities,butwasvirtuallyunknowninthecountryside
wheremostlaborwasperformedbyafreepeasantry.
Answer: C
Diff:2
PageRef:254
Topic:FromArabtoIslamicEmpire
Skill:Conceptual

49) WhatwastheprimaryculturalcontributionoftheMuslimsduringtheAbbasidperiod?
A) TheMuslimswereabletorecoverandpreservetheworksoftheancientphilosophersas
wellastransmitfromonecivilizationtoanother.
B) TheMuslimsbecameextraordinarilyadeptatportraiture,focusingondepictionsof
MuhammadandtheearlyCaliphs.
C) AlthoughthematerialcultureoftheAbbasidperiodremainedpoor,Muslimswereable
tomakesomeadvancesinmusic.
D) Islamiclearningwasnecessarilyunique,astheyhadnoaccesstotheancienttraditionsof
philosophyandscience.
Answer: A
Diff:2
PageRef:258
Topic:FromArabtoIslamicEmpire
Skill:Conceptual

179

50) Bytheninthcentury,whathappenedtotheAbbasidcontroloftheempire?
A) TheAbbasidscontinuedtodevelopabsolutisminsuchafashionthattheycontrolleda
largeportionofEurasia.
B) Non-Arabpeoples,particularlytheTurkswhohadconvertedtoIslam,increasingly
dominatedpoliticallife.
C) SuccessivedefeatsoftheAbbasidsbytheByzantineempireledtothecollapseofthe
dynastyandtheemergenceofaGreekhegemony.
D) EventuallythePersianinfluenceintheAbbasidempireresultedintheremovalofthe
capitaltoMervandtheriseofapurelyPersiandynasty.
Answer: A
Diff:2
PageRef:258
Topic:Conclusion
Skill:Conceptual

EssayQuestions
1) DescribethesocialorganizationoftheArabspriortotheintroductionofIslam.
Answer: Basedonkin-relatedclangroupstypicalofnomadicpastoralists;groupedintolarger
tribalunits,butseldomlivedtogether;wealthandstatusbasedonpossessionof
animals,pasturageandwaterrights;slaveryutilized;commonincidenceoffeuds.
Diff:2
PageRef:239-242
Topic:DesertandTown:TheArabianWorldandtheBirthofIslam
Skill:Conceptual

2) CompareandcontrastthestatusofwomeninIslamduringthepre -Islamicperiodandthe
Umayyadempire.
Answer: Womeninpre-IslamiccultureenjoyedgreaterlibertythanthoseofByzantiumor
Persia;playedimportanteconomicroles;insomeclansdescentwasmatrilineal;not
secluded;insomeclansbothmalesandfemalesallowedmultiplemarriages.Umayyad
empire:strengthenednuclearfamilyandlegalrightsofwomen;womenweremore
independentandplayedaprominentroleinIslamandinprofessions.
Diff:2
PageRef:241-242,249-251
Topic:FromArabtoIslamicEmpire
Skill:Conceptual

3) WhywasIslamabletoappealtopeopleofsomanydifferentcultures?
Answer: UniversalelementsinIslam:uniqueformofmonotheismappealedtoother
monotheistictraditions;egalitarianism;legalcodes;strongsenseofcommunityinthe
ummah;MuhammadswillingnesstoacceptvalidityofearlierJudaicandChristian
revelations;appealoffivepillarsoffaith.
Diff:2
PageRef:244-246
Topic:TheLifeofMuhammadandtheGenesisofIslam
Skill:Conceptual

180

4) WhatwastheessentialdisputewithinIslamoverthesuccessionoftheProphet?
Answer: Muhammadneverspecifiedaprincipleofsuccession;immediatesuccessorselected
fromamongfirstconvertstoIslam;debatefollowingmurderofUthmanandselection
ofAli;ShiassupportedonlyfamilialdescendantsoftheProphetasrightfulrulers;
UmayyadsestablishedhereditarydynastyafterdefeatanddeathofAli;Sunnis
supportedconceptofdynasticsuccession.
Diff:2
PageRef:247-248
Topic:TheArabEmpireoftheUmayyads
Skill:Conceptual

5) WhatwasthedifferencebetweentheAbbasidempireandtheUmayyadempire?
Answer: Bothwereessentiallyabsolutistinstructure,buttheAbbasidsintroducedgreater
formalismandamorerigorousbureaucraticstructurefeaturingthewazirs;Abbasid
dynastyoriginallybasedonclaimsofdescentfromfamilyoftheProphet(Shia),but
eventuallymovedtosuppressShiamovements;Abbasidsincorporatedmawalior
non-Arabconvertsintofullcitizenshipandparticipation;shiftofcenterofempireto
capitalatBaghdadinPersia.
Diff:2
PageRef:248-258
Topic:FromArabtoIslamicEmpire
Skill:Conceptual

181

Chapter12 AbbasidDeclineandtheSpreadofIslamic
CivilizationtoSouthandSoutheastAsia
Multiple-ChoiceQuestions
1) WhichofthefollowingwasNOTareasonforthedeclineoftheAbbasiddynastybythe9th
centuryC.E.?
A) Thecollapseofthecities
B) Constantcivilviolencetrainedthetreasury
C) Newconstructiontrainedthetreasury
D) Peasantrisings
Answer: A
Diff:2
PageRef:263-264
Topic:Introduction
Skill:Conceptual

2) WhichofthefollowinggroupsdidNOTrevoltagainstAbbasidrule?
A) Slaves
B) Peasants
C) Sunnis
D) Shias
Answer: C
Diff:2
PageRef:264
Topic:Introduction
Skill:Conceptual

3) HowdidtheCaliphal-MahdiresolvetheproblemofsuccessionintheAbbasiddynasty?
A) Hespecifiedtheruleofprimogeniture, thesuccessionoftheoldestson.
B) HeacceptedtheShiadoctrinethatonlythepersonwiththemostdirectrelationshipto
Muhammadshouldsucceed.
C) HeacceptedtheradicalpropositionthatonlyamanofdemonstrableIslamicpurity
shouldsucceed.
D) Hefailedtoresolvetheproblemofdynasticsuccessionwithdisastrousresults.
Answer: D
Diff:2
PageRef:262
Topic:TheIslamicHeartlandsintheMiddleandLateAbbasidEra
Skill:Conceptual

182

4) Whatwasal-MahdisattitudetowardtheShias?
A) Heviewedthemasopponentsofhisdynastyandattemptedtoeliminatethem.
B) HeappealedtothemoderatefactionsoftheShiastosupporttheAbbasiddynasty.
C) HeacceptedthefundamentaldoctrinesoftheShiasandabdicated.
D) InordertoplacatetheShias,al-Mahdilivedapiouslifeofpovertyandsimplicity.
Answer: B
Diff:2
PageRef:262
Topic:TheIslamicHeartlandsintheMiddleandLateAbbasidEra
Skill:Conceptual

5) WhatwasthefictionalaccountoflifeatthecourtoftheCaliphal-Rashid?
A) Shah-nama
B) TheTreasureofAliBaba
C) Analects
D) TheThousandandOneNights
Answer: D
Diff:1
PageRef:263
Topic:TheIslamicHeartlandsintheMiddleandLateAbbasidEra
Skill:Factual

6) Howdidtheadministrationofal-RashidsetatrendforsubsequentAbbasidrulers?
A) HewasattheoutsetofhisreignheavilydependentonPersianadvisors,apracticethat
becamecommonplacethereafter.
B) Hedividedtheempireintoaseriesofstateseachofwhichelectedrepresentativestoa
CaliphalparliamentatBaghdad.
C) Heremovedallregionalgovernorsandestablishedstrictabsolutismfromthecourtat
Baghdad.
D) Hestressedtheimportanceofpietyandsimplicity,apositionsubsequentlyadoptedby
hissuccessors.
Answer: A
Diff:2
PageRef:263
Topic:TheIslamicHeartlandsintheMiddleandLateAbbasidEra
Skill:Conceptual

183

7) Whatwastheresultofthecivilwarsfollowingthedeathofal -Rashid?
A) TheAbbasiddynastycametoanend.
B) Candidatesforthethronerecognizedtheneedtobuilduppersonalarmies,oftenofslave
soldiers.
C) Privatearmieswereabolishedinanattempttocurtailfutureviolenceoverthe
succession.
D) ThepracticeofelectingCaliphsinMeccawasonceagaininstituted.
Answer: B
Diff:2
PageRef:263
Topic:TheIslamicHeartlandsintheMiddleandLateAbbasidEra
Skill:Conceptual

8) WhichofthefollowingstatementsconcerningthemercenaryarmiesofthelaterAbbasiderais
NOTaccurate?
A) MercenarytroopsbecameadisruptiveforceinthelifeofBaghdadandothercities.
B) Theywereconsistentlyamajorplayerinthefactionalcontestsforcontrolofthecapital
andtheempire.
C) Theyoftenconsistedlargelyofslavetroops.
D) Despitetheirtendencytowardrandomviolence,theyloyallydefendedtheAbbasid
Caliphs.
Answer: D
Diff:2
PageRef:264
Topic:TheIslamicHeartlandsintheMiddleandLateAbbasidEra
Skill:Conceptual

9) WhataccountsforthedisruptionoftheagriculturaleconomyoftheAbbasidempire?
A) Thedeclineofthecitiesledtoafallinthedemandforfoodsuppliesandconsequent
dropsinagriculturalprices.
B) Progressivedesiccationoftheregionledtoadiminutionofthelandavailablefor
agriculture.
C) Spirallingtaxation,thedestructionoftheirrigationworks,andpillagingbymercenary
armiesledtodestructionandabandonmentofmanyvillages.
D) ThegovernmentorderedregionsoftheempirepopulatedbyShiasabandoned.
Answer: C
Diff:2
PageRef:264
Topic:TheIslamicHeartlandsintheMiddleandLateAbbasidEra
Skill:Conceptual

184

10) WhatwastheinnovationoftheAbbasidcourtwithrespecttowomen?
A) Theestablishmentoftheharem
B) Thelegislationofmultiplemarriagesforwomen
C) ThecreationofIslamicnunneries
D) Legislationagainstconcubinageandprostitution
Answer: A
Diff:1
PageRef:264
Topic:TheIslamicHeartlandsintheMiddleandLateAbbasidEra
Skill:Conceptual

11) WhatwastheattitudeoftheAbbasidstowardstheinstitutionofslavery?
A) TheAbbasiddynastyforbadeslaveryingeneral.
B) ThedynastypermittedslaveryofMuslims,butforbadetheenslavementofmembersof
otherreligions.
C) TheAbbasidelitedemandedgrowingnumbersofbothmaleandfemaleslavesfor
concubinesanddomesticservice.
D) ThedynastyforbadeallslaveryexceptforthemercenaryarmiesloyaltotheAbbasids.
Answer: C
Diff:2
PageRef:264
Topic:TheIslamicHeartlandsintheMiddleandLateAbbasidEra
Skill:Conceptual

12) WhywerethepracticesofseclusionandveilingseenasessentialinIslamicsociety?
A) Womenwerebelievedtobepossessedofinsatiablelustfromwhichmenhadtobe
protected.
B) Veilingallowedwomentohidetheiridentitiesandtoavoidtherashofdynasticstrife.
C) Womenwereonlypermittedtoengageinbusinessiftheiridentitieswerehidden.
D) Veilingwasseenasameansofhaltingthepracticeofconcubinage.
Answer: A
Diff:2
PageRef:264
Topic:TheIslamicHeartlandsintheMiddleandLateAbbasidEra
Skill:Conceptual

185

13) WhichofthefollowingstatementsconcerningwomenduringtheAbbasideraismost
accurate?
A) NoIslamicwomenengagedinlabor.
B) Womenoftenmarriedatpuberty,setatagenine.
C) RichwomenhadmanycareeroutletsinIslamiccities.
D) Abbasidwomenhadvastlygreaterfreedomthandidwomeninthefirstcenturyof
Islam.
Answer: B
Diff:3
PageRef:265
Topic:TheIslamicHeartlandsintheMiddleandLateAbbasidEra
Skill:Conceptual

14) WhatwastheregionalsplinterdynastythatcapturedBaghdadin945?
A) Mongols
B) Buyids
C) SeljukTurks
D) Ghazni
Answer: B
Diff:2
PageRef:265
Topic:TheIslamicHeartlandsintheMiddleandLateAbbasidEra
Skill:Factual

15) WhatgroupsuccessfullycapturedBaghdadin1055?
A) Mongols
B) Buyids
C) SeljukTurks
D) Ghazni
Answer: C
Diff:2
PageRef:265
Topic:TheIslamicHeartlandsintheMiddleandLateAbbasidEra
Skill:Factual

16) WhatwasthereligiousaffiliationoftheSeljukTurks?
A) Christian
B) Shia
C) Sufi
D) Sunni
Answer: D
Diff:3
PageRef:265
Topic:TheIslamicHeartlandsintheMiddleandLateAbbasidEra
Skill:Factual

186

17) InwhatyearwasJerusalemcapturedbyChristiancrusaders?
A) 985
B) 1099
C) 1142
D) 1245
Answer: B
Diff:3
PageRef:265
Topic:TheIslamicHeartlandsintheMiddleandLateAbbasidEra
Skill:Factual

18) WhatwastheimpactoftheSeljukconquestofBaghdadontheAbbasidempire?
A) TheempirecontinuedtocrumbleasaresultofthemilitarysuccessesofFatimidEgypt
andtheByzantineempire.
B) TheimpositionofaChristiangovernmentinthenameoftheAbbasidCaliphs
temporarilyrestoredorder.
C) TheSeljuksabandonedtheMiddleEastforfurtherconquestsintheIndiansubcontinent.
D) ItactuallyrestoredtheabilityoftheempiretomeetthechallengesofEgyptandthe
Byzantineempire.
Answer: D
Diff:2
PageRef:265
Topic:TheIslamicHeartlandsintheMiddleandLateAbbasidEra
Skill:Conceptual

19) WhowastheMuslimleaderresponsibleforthereconquestofmostoftheterritoriesbelonging
totheChristiancrusaders?
A) Firdawsi
B) MuhammadibnQasim
C) al-Ghazali
D) Saladin
Answer: D
Diff:2
PageRef:265
Topic:TheIslamicHeartlandsintheMiddleandLateAbbasidEra
Skill:Factual

187

20) WhataccountsforthesuccessoftheFirstCrusade?
A) TheoverwhelmingmilitarysuperiorityofWesternmilitarytechnology
B) ThecontemporaryemergenceoftheChristianSeljukTurksinBaghdad
C) Muslimpoliticalfragmentationandtheelementofsurprise
D) ThesupportandcooperationoftheJewishcommunityoftheHolyLand
Answer: C
Diff:2
PageRef:265
Topic:TheIslamicHeartlandsintheMiddleandLateAbbasidEra
Skill:Conceptual

21) ThefallofAcre,thelastcrusaderstronghold,occurredinwhatyear?
A) 1099
B) 1135
C) 1193
D) 1291
Answer: D
Diff:3
PageRef:265
Topic:TheIslamicHeartlandsintheMiddleandLateAbbasidEra
Skill:Factual

22) WhatwastheimpactoftheCrusadesontheChristianWest?
A) Christiansadoptedmilitarytechniques,words,scientificlearning,andArabicnumerals
amongotherthings.
B) ChristiansrejectedmostMusliminfluence,althoughtheydidgainatasteforMuslim
winesandliquors.
C) TherewasnoMusliminfluenceontheChristianWest.
D) TheCrusadesinterruptedthetradeoftheMediterraneanandcutofftheWestfromIslam
until1293.
Answer: A
Diff:2
PageRef:265-266
Topic:TheIslamicHeartlandsintheMiddleandLateAbbasidEra
Skill:Conceptual

188

23) WhatwastheimpactoftheCrusadesonIslam?
A) TheMuslimsadoptedmilitarytechnology,words,andscientificknowledge,among
otherthingsfromtheWest.
B) Althoughtheyresistedmostinfluence,theMuslimsdidacquireatasteforWestern
cuisine.
C) TherewasminimalWesternimpactonIslam.
D) TheCrusadestemporarilycutoffallexchangebetweentheWestandIslam.
Answer: C
Diff:2
PageRef:267
Topic:TheIslamicHeartlandsintheMiddleandLateAbbasidEra
Skill:Conceptual

24) WhatwasthetrendofurbanizationduringmuchoftheAbbasidempire?
A) BecausetheAbbasidsabandonedBaghdadforothercapitals,citieswithintheempire
tendedtowitheranddie.
B) Successiveinvasionsledtoadeclineinurbanization.
C) Despitepoliticaldisintegrationandadeclineintheagriculturaleconomy,towns
continuedtogrowrapidly.
D) Townsestablishedintheearlyyearsofthedynastywereabletoholdtheirown,but
therewaslittlegrowth.
Answer: C
Diff:2
PageRef:267
Topic:AnAgeofLearningandArtisticAchievement
Skill:Conceptual

25) WhatwastheleveloftradeintheAbbasidempire?
A) Long-distancetradewithAfrica,theMediterranean,India,andChinacontinuedto
flourishdespiteperiodicinterruption.
B) TradewiththeEastgrew,buttheCrusadeseliminatedthewesterntraderoutes.
C) TradewithAfricaandtheMediterraneancontinuedtoexpand,butthewarsinIndia
disruptedtheeasterntraderoutes.
D) Asawhole,long-distancetradealongthetraditionalcaravanroutesvirtuallyceased
duringtheAbbasidempire.
Answer: A
Diff:2
PageRef:267
Topic:AnAgeofLearningandArtisticAchievement
Skill:Conceptual

189

26) WhatwastheprimarywrittenlanguageofthelaterAbbasidcourt?
A) Arabic
B) Persian
C) Latin
D) Greek
Answer: B
Diff:1
PageRef:268
Topic:AnAgeofLearningandArtisticAchievement
Skill:Factual

27) WhatwastheShah-Nama?
A) ThenamegiventothefirstwifeoftheCaliph
B) Thetitleofthecommanderoftheslavemercenaries
C) ThetitlegiventotheoldestsonoftheCaliph
D) ThehistoryofPersiafromcreationtotheIslamicconquests
Answer: D
Diff:1
PageRef:268
Topic:AnAgeofLearningandArtisticAchievement
Skill:Factual

28) WhichofthefollowingwasNOTaliteraryfigureduringtheAbbasidEmpire?
A) Firdawsi
B) Sadi
C) MuhammadibnQasim
D) OmarKhayyam
Answer: C
Diff:2
PageRef:268
Topic:AnAgeofLearningandArtisticAchievement
Skill:Factual

29) WhichofthefollowingstatementsconcerningtheSufimovementwithinIslamismost
accurate?
A) TheSufimovementstressedanincreasinglyrestrictiveconservatismwithinIslam.
B) TheSufimovementstressedwithdrawalfromotherbelieversandisolationintomonastic
communities.
C) TheSufimovementincorporatedmysticismwithatrendtowardevangelism.
D) TheSufiquestionedtheIslamicinterestintheGreektraditionsinscience.
Answer: C
Diff:2
PageRef:270
Topic:AnAgeofLearningandArtisticAchievement
Skill:Conceptual

190

30) Whichofthefollowingstatementsconcerningtheulama ismostaccurate?


A) ItstressedanincreasinglyrestrictiveconservatismwithinIslam,particularlywithrespect
toscientificinquiry.
B) Itstressedwithdrawalfromtherestofthecommunityofbelieversandthecreationof
Islamicmonasteries.
C) ItbroughtmysticismintoIslamandemphasisonevangelism.
D) ItembracedthefullconsequencesoffusingGreekandQuranictraditionswillingly.
Answer: A
Diff:2
PageRef:269-270
Topic:AnAgeofLearningandArtisticAchievement
Skill:Conceptual

31) WhatgroupcapturedBaghdadin1258?
A) Buyids
B) SeljukTurks
C) Crusaders
D) Mongols
Answer: D
Diff:1
PageRef:270
Topic:AnAgeofLearningandArtisticAchievement
Skill:Factual

32) HowdidthepoliticalcenterofIslamchangeaftertheMongolinvasions?
A) BaghdadremainedthecapitalofIslam,butunderthecontrolofsuccessiveMongol
dynasties.
B) ThecenterofIslampassedwiththewithdrawaloftheinvadersintothesteppesof
centralAsia.
C) Baghdadbecameaprovincialbackwater,supplantedbyCairototheeastandsoon
thereafterIstanbultothenorth.
D) ThepoliticalcenterofIslamwasremovedtosub-SaharanAfrica.
Answer: C
Diff:2
PageRef:270
Topic:AnAgeofLearningandArtisticAchievement
Skill:Conceptual

191

33) WhatwasthedifferencebetweentheIslamicinvasionsofIndiaandpreviousincursionsofthe
subcontinent?
A) WiththeMuslims,thepeoplesofIndiaencounteredforthefirsttimealarge-scaleinflux
ofinvaderswithacivilizationassophisticatedastheirown.
B) WiththeMuslims,thepeoplesofIndiaencounteredforthefirsttimeaninvasionfrom
thewestratherthantheeast.
C) TheMuslimswererapidlyabletounifyallofIndiaintoasingleempire.
D) TheMuslims,unlikepreviousinvaders,bypassedtheGangeticplaininpreferencefor
southernIndia.
Answer: A
Diff:2
PageRef:270-271
Topic:TheComingofIslamtoSouthAsia
Skill:Conceptual

34) HowdidIslamandHinduismdiffer?
A) Hinduismstressedtheegalitarianismofallbelievers,whileIslamwasmorerigidin
termsoforthodoxbelief.
B) Islamstressedtheegalitarianismofallbelievers,whileHinduismwasmorerigidin
termsoforthodoxbelief.
C) Islamstressedtheegalitarianismofallbelievers,whileHinduismembraceda
caste-basedsocialsystem.
D) Hinduismstressedtheegalitarianismofallbelievers,whileIslamembraceda
caste-basedsocialsystem.
Answer: C
Diff:2
PageRef:271-272
Topic:TheComingofIslamtoSouthAsia
Skill:Conceptual

35) WhatwasthedateofthefirstMuslimraidsintoIndia?
A) 650
B) 711
C) 750
D) 1243
Answer: B
Diff:2
PageRef:272
Topic:TheComingofIslamtoSouthAsia
Skill:Factual

192

36) HowdidtheMuslimconquerorsofSindtreattheHinduandBuddhistresidentsoftheregion?
A) Allnon-Muslimswereputtodeath
B) HindusandBuddhistsweretreatedasdhimmisorpeoplesofthebook
C) MostoftheMuslimsconvertedtoHinduism
D) ForcedconversionsofthenativepopulationfollowedtheMuslimmilitarysuccesses
Answer: B
Diff:2
PageRef:273
Topic:TheComingofIslamtoSouthAsia
Skill:Conceptual

37) Whatwasthemostcriticalculturaladvanceasaresultoftheincreasedcontactbetween
MuslimsandIndiancivilization?
A) MuslimsadoptedtheHindupantheonofgods.
B) MuslimcommercewasincreasinglydominatedbythemerchantcasteofIndia.
C) MuslimsadoptedthesystemofmathematicalnotationlaterreferredtoasArabic
numerals.
D) MuslimsadoptedthehighlystratifiedsocialsystemcommoninIndiancivilization.
Answer: C
Diff:2
PageRef:273
Topic:TheComingofIslamtoSouthAsia
Skill:Conceptual

38) ThecommanderofthefirstIslamicincursionintotheSindin711was
A) MuhammadibnQasim.
B) IbnBatuta.
C) al-Ghazali.
D) MahmudofGhazni.
Answer: A
Diff:2
PageRef:273
Topic:TheComingofIslamtoSouthAsia
Skill:Factual

193

39) WhatdescendantofaTurkishslavedynastyinAfghanistanledaseriesofexpeditionsinto
Indiatoseizebootyinthe11thcentury?
A) MahmudofGhazni
B) MuhammadibnQasim
C) Hajjaj
D) MuhammadofGhur
Answer: A
Diff:2
PageRef:273
Topic:TheComingofIslamtoSouthAsia
Skill:Factual

40) WhatrulerwasassociatedwiththetransitionfromraidingtotheestablishmentofanIslamic
empireintheIndiansubcontinent?
A) MahmudofGhazni
B) MuhammadibnQasim
C) Hajjaj
D) MuhammadofGhur
Answer: D
Diff:2
PageRef:274
Topic:TheComingofIslamtoSouthAsia
Skill:Factual

41) WherewasthecapitaloftheIslamickingdomestablishedafter1206ontheGangeticplain?
A) Bombay
B) Kanauj
C) Isoleta
D) Delhi
Answer: D
Diff:2
PageRef:274
Topic:TheComingofIslamtoSouthAsia
Skill:Factual

42) WhatgroupswereresponsiblefortheconversionofIndianstoIslam?
A) TradersandSufimystics
B) UlamaandSunnievangelists
C) SoldiersandShiaexiles
D) TheMongolsandSeljukTurks
Answer: A
Diff:2
PageRef:274
Topic:TheComingofIslamtoSouthAsia
Skill:Factual

194

43) WhyweretheSufiseffectivemissionarieswithintheIndiansubcontinent?
A) TheyenjoyedthesupportoftheHinduprincesbecauseoftheirsupportforbrahmin
ritual
B) InbothstyleandmessagetheysharedmuchwithIndianmysticsandwanderingascetics
C) TheyweresupportedbyhugearmiesofArabswhomigratedtoIndiainsearchofland
D) TheyrejectedlowcasteHindusinpreferenceforconvertsamongthebrahminelite
Answer: B
Diff:2
PageRef:274-275
Topic:TheComingofIslamtoSouthAsia
Skill:Conceptual

44) WhatgroupsinIndiaweremostlikelytoconverttoIslam?
A) Brahminsandmerchants
B) Rajaandwarriors
C) MembersoftheadministrativemachineryoftheIslamickingdoms
D) Buddhistsandlow-casteHindus
Answer: D
Diff:2
PageRef:275
Topic:TheComingofIslamtoSouthAsia
Skill:Factual

45) HowdidHinduismrespondtothechallengeofIslam?
A) Hindusabandonedtheiremphasisonmanydeitiesinfavorofmonotheismintheperson
ofShiva.
B) Hindusplacedgreateremphasisonthedevotionalorbhakticcultsofgodsand
goddessessuchasShivaandVishnu.
C) HindusconvertedtoIslaminincreasingnumbers,untilMuslimsoutnumberedHindus
inthesubcontinent.
D) ThebrahminsacceptedIslamasavarietyoforthodoxHindubelief,whileanticipating
theincorporationoftheMuslimimmigrantsintotheIndiancastesystem.
Answer: B
Diff:2
PageRef:275
Topic:TheComingofIslamtoSouthAsia
Skill:Conceptual

195

46) ThespreadofIslamtosoutheastAsiawasdelayeduntilthefalloftheBuddhisttradeempire
of
A) Borneo.
B) Sumatra.
C) theCelebesIslands.
D) Shrivijaya.
Answer: D
Diff:2
PageRef:277
Topic:TheSpreadofIslamtoSoutheastAsia
Skill:Factual

47) Ingeneral,howdidIslamspreadinsoutheastAsia?
A) Portcitieswerepointsofdisseminationtootherlinksintradingnetworks.
B) MostofsoutheastAsiawasconvertedtoIslamafterthemilitaryvictoriesof
Qutb-ud-dinAibak.
C) IslamwascarriedtosoutheastAsiafromChina.
D) TradetosoutheastAsiafromAfricaandPersiaestablishedIslamiccentersonthe
mainlandfromwhichconversiontookplace.
Answer: A
Diff:2
PageRef:277
Topic:TheSpreadofIslamtoSoutheastAsia
Skill:Conceptual

48) WhatwasthenatureofIslamicreligionthatdevelopedinsoutheastAsia?
A) Becausemostofthemissionarieswereulama fromArabia,thereligionmostclosely
resembledIslamaspracticedinthefirstgenerationsafterMuhammad.
B) BecauseIslamcametosoutheastAsiafromIndiaandwasspreadbySufiholymen,it
developedamysticalnaturethatincorporatedmuchofindigenousreligion.
C) BecauseIslamwascarriedtosoutheastAsiafromChina,itboremanyofthe
characteristicsofBuddhism.
D) BecauseIslamwascarriedbyconqueringwarriorsfromIndia,itrejectedthenative
BuddhismandHinduisminpreferenceformoreconservativeIslamicorthodoxy.
Answer: B
Diff:2
PageRef:279-280
Topic:TheSpreadofIslamtoSoutheastAsia
Skill:Conceptual

196

EssayQuestions
1) ExplainthereasonsforthepoliticaldisruptionoftheAbbasidempirepriortotheriseofthe
Buyids.
Answer: Difficultiesofcommunicationandmovingarmiesovergreatdistancesofempire;
tendencyofpopulationstoretainregionalidentities;failuretoestablishprincipleof
successionsatisfactorytoallparties;riseofmercenaryarmiesthatbecamevirtually
independent;failureofagriculturaleconomy.
Diff:2
PageRef:262-265
Topic:TheIslamicHeartlandsintheMiddleandLateAbbasidEra
Skill:Conceptual

2) DescribethesocialpositionofwomenduringtheAbbasidempire.
Answer: Womenwerenowveiledandsecluded;increaseinpatriarchalauthority;onlymales
permittedmultiplemarriages;developmentoftheharem.
Diff:2
PageRef:282
Topic:AnAgeofLearningandArtisticAchievement
Skill:Conceptual

3) DiscussthereligioustrendsoftheAbbasideraandtheirimpactontheexpansionofIslam.
Answer: Sufi:developmentofIslamicmysticism;lessemphasisonstrictinterpretationof
traditionaltextsandlaws;emphasisonemotionalreligiousexperience;servedas
healers;identifiedwithwanderingholymenofBuddhistsandHindus;keyfiguresin
expansionofIslamtosouthandsoutheasternAsia.Ulama:developedincreasingly
conservativeandrestrictivereligiousschools;questioneduseofGreekphilosophyin
scientificschoolsofIslam;emphasizedstrictinterpretationoflawsandreligioustexts.
Diff:2
PageRef:269-270
Topic:AnAgeofLearningandArtisticAchievement
Skill:Conceptual

4) InwhatsensedidtheMuslimincursionsintotheIndiansubcontinentresultinIslamic
influencesonHindus?TowhatextentwereMuslimsaffectedbyIndianculture?
Answer: MuslimsestablishedkingdominGangeticplain,butunabletoruleabsolutelybecause
ofdependenceonlocalHinduadministrators;abletoconvertsomeIndiansBuddhists
andlowcasteHindusprimarilyinnorthwesternandnorthernIndia;Hindusresponded
bystrengtheningbhakticcults;highcastegroupsresistedconversion;Muslimsadopted
socialstratificationofHinducastes,attitudestowardwomen,practicesofHindurulers,
tasteforIndianfoodandgames.
Diff:2
PageRef:270-277
Topic:TheComingofIslamtoSouthAsia
Skill:Conceptual

197

5) BywhatmeansdidIslamspreadtosoutheastAsia?
Answer: IncursionofIslamintosoutheastAsiaalmostentirelyasaresultofestablishmentof
traderoutesfromMuslimportsinIndia;SufimysticsandtraderscarriedIslamtoport
citieswithinsoutheastAsia;fromportcitiesIslamdisseminatedtootherregions;
becauseofIndianandSufibackground,lessrigorousemphasisonstrictinterpretation
oftextsandlaws;moreincorporationofindigenousreligiousbeliefs.
Diff:2
PageRef:277-280
Topic:TheSpreadofIslamtoSoutheastAsia
Skill:Conceptual

198

Chapter13 AfricanCivilizationsandtheSpreadofIslam
Multiple-ChoiceQuestions
1) Between800and1500asthefrequencyandintensityofcontactwiththeoutsideworld
increased,whatwasthechiefimpactonsub-SaharanAfrica?
A) ThearrivalofthePortuguese
B) ThearrivalofChristianity
C) ThearrivalofIslam
D) ThearrivalofChinesemerchants
Answer: C
Diff:2
PageRef:283
Topic:Introduction
Skill:Conceptual

2) WhatwasoneofthemajordifferencesbetweenAfricancivilizationsandotherpostclassical
societies?
A) Africancivilizationsbuiltsomewhatlessclearlyonpriorsocietiesthandidotherpost
classicalsocieties.
B) AfricancivilizationwasalmostentirelydependentonculturalimportationsfromIslam
andtheArabicworld.
C) Priorto800,Africancivilizationshadnopriorcontactswithcivilizationsoutsideofthe
Africancontinent.
D) TherewerenocivilizationsinAfricauntilthepostclassicalperiod.
Answer: A
Diff:2
PageRef:284
Topic:Introduction
Skill:Conceptual

3) WhowasMansaMusa?
A) TheleaderofMali
B) AChristianmissionaryfromPortugal
C) TheleaderofSonghay
D) TheleaderofGhana
Answer: C
Diff:2
PageRef:282
Topic:AfricanSocieties:DiversityandSimilarities
Skill:Conceptual

199

4) Africansocietiesorganizedaroundkinshiporotherformsofobligationandlackingthe
concentrationofpoliticalpowerandauthoritywerereferredtoas
A) stateless.
B) huntingandgatheringbands.
C) Bantuconicalclans.
D) Islamictribes.
Answer: A
Diff:2
PageRef:285
Topic:AfricanSocieties:DiversityandSimilarities
Skill:Conceptual

5) WhatwasthefunctionofsecretsocietiesinAfricanculture?
A) TheysmuggledvaluablegoldacrosstheSaharaandestablishedvitaltraderouteswith
theMediterranean.
B) Becausesecretsocietieswererestrictedtofemales,theypermittedwomentohavean
invisible,butpowerful,roleinpoliticalaffairswithinAfricansocieties.
C) Becausetheirmembershipcutacrosslineagedivisions,theyactedtomaintainstability
withinthecommunityanddiminishclanfeuds.
D) TheyservedasadisruptiveandrevolutionaryforceinAfricansocietyforestallingthe
formationoflargerstates.
Answer: C
Diff:2
PageRef:285
Topic:AfricanSocieties:DiversityandSimilarities
Skill:Conceptual

6) Whichofthefollowingstatementsbestdescribestheindigenousreligionofmuchof
sub-SaharanAfrica?
A) Muchofsub-SaharanAfricawasChristian.
B) Animisticreligion,beliefinthepowerofnaturalforcespersonifiedasdeities,
characterizedmuchofAfrica.
C) AfricanreligionpriortothearrivaloftheMuslimswastypifiedbyanindependentform
ofmonotheismcharacterizedbyworshipinmonumentaltemplecomplexes.
D) Uniquely,Africansocietieslackedreligiousprinciplespriortothearrivalofthe
ChristiansandMuslims.
Answer: B
Diff:2
PageRef:285
Topic:AfricanSocieties:DiversityandSimilarities
Skill:Conceptual

200

7) WhichofthefollowingwasNOTabeliefsharedbypractitionersofmanyindigenousAfrican
religions?
A) Theideaofacreatordeity
B) Theideathatdisastersandillnesseswereproducedbysaints
C) Thevenerationofancestors
D) Theviewthatworkingthelandhadreligioussignificance
Answer: B
Diff:2
PageRef:285
Topic:AfricanSocieties:DiversityandSimilarities
Skill:Conceptual

8) WhichofthefollowingstatementsconcerningtheeconomiesofAfricaisNOTcorrect?
A) NorthAfricawasfullyinvolvedintheMediterraneanandArabeconomicworld.
B) Settledagricultureandironworkinghadbeenestablishedinmanyareasbeforethe
post-classicalperiod.
C) Tradewashandledbyprofessionalmerchants,ofteninkinshipgroupings.
D) Muchoftheregionlackedamarketeconomyandwasbasedonself-sufficient
agriculturalunits.
Answer: D
Diff:2
PageRef:285
Topic:AfricanSocieties:DiversityandSimilarities
Skill:Conceptual

9) WhatmayhavebeenthepopulationofAfricain1500CE?
A) 1-15million
B) 15-30million
C) 30-60million
D) 60-75million
Answer: C
Diff:2
PageRef:286
Topic:AfricanSocieties:DiversityandSimilarities
Skill:Conceptual

201

10) WhatregionofAfricawasfirstconvertedtoIslamby700C.E.?
A) EastAfrica
B) CentralAfrica
C) WestAfrica
D) NorthAfrica
Answer: D
Diff:1
PageRef:286
Topic:AfricanSocieties:DiversityandSimilarities
Skill:Factual

11) ThepuritanicalreformmovementsoftheAlmoravidsandAlmohadisaroseamongwhat
groupsofpeople?
A) TheBerbers,nativedesertdwellersofnorthAfrica
B) TheArabsduringtheconquestofnorthAfrica
C) TheNubiansoftheearlySudan
D) TheinhabitantsoftheempireofMali
Answer: A
Diff:2
PageRef:286
Topic:AfricanSocieties:DiversityandSimilarities
Skill:Factual

12) Whatdoesummamean?
A) Socialequality
B) Africanruler
C) Equalityofallclans
D) Communityofbelievers
Answer: D
Diff:2
PageRef:286
Topic:AfricanSocieties:DiversityandSimilarities
Skill:Conceptual

13) WhatwasthemostimportantChristiankingdominAfrica?
A) Mali
B) Songhay
C) Kongo
D) Ethiopia
Answer: D
Diff:2
PageRef:286
Topic:AfricanSocieties:DiversityandSimilarities
Skill:Factual

202

14) Whichofthefollowingregionswasanimportantpointofinitialculturalcontactbetween
AfricaandIslam?
A) TheAtlanticOcean
B) ThePacificOcean
C) NorthAfrica
D) SouthAfrica
Answer: C
Diff:1
PageRef:286
Topic:KingdomsoftheGrasslands
Skill:Factual

15) TheSahelreferstothe
A) grasslandbeltatthesouthernedgeoftheSaharathatservedasapointofexchange
betweentheforestsofthesouthandnorthAfrica.
B) EastAfricancoastlinethatbecametheprimarypointofcontactforMuslimmerchants
fromIndiaandsoutheastAsiaandAfricantraders.
C) seriesoftradingportsthatrapidlydevelopedalongtheAtlanticcoasttosupportthe
tradeinAfricanslaves.
D) forestzoneofcentralAfricathatremainedfreeofIslamicinfluencelargelybecauseofthe
inabilityofthecameltowithstandtheclimateoftheregion.
Answer: A
Diff:2
PageRef:286
Topic:KingdomsoftheGrasslands
Skill:Conceptual

16) WhatSudanickingdomdeclinedin1076makingwayfornewpoliticalorganizationsinthe
region?
A) Mali
B) Songhay
C) Ghana
D) Axum
Answer: C
Diff:2
PageRef:288
Topic:KingdomsoftheGrasslands
Skill:Factual

203

17) WhichofthefollowingstatementsconcerningtheSudanicstatesofMaliandSonghayismost
accurate?
A) Althoughpowerful,theSudanicstatesneverreachedthelevelofempires.
B) Sudanicstateshadterritorialcoreareasinwhichthepeoplewereofthesameethnic
background,buttheirpowerextendedoversubordinatecommunities.
C) WhatprovidedtheculturalunitynecessaryfortheestablishmentofstatesintheSudan
wastheconversionofmanypeopletoChristianity.
D) TheSudanicstatesweredistinguishedfromotherAfricancivilizationsbythepeculiar
lackoffamilyorclanlineagesasanorganizingprincipleofsociety.
Answer: B
Diff:2
PageRef:288
Topic:KingdomsoftheGrasslands
Skill:Conceptual

18) WhatwasthegeographicallocationoftheempireofMali?
A) BetweentheZambeziandCongoRivers
B) BetweenthecitiesofMogadishuandMombasa
C) BetweentheNigerandSenegalRivers
D) AlongtheNileRivervalley
Answer: C
Diff:2
PageRef:288
Topic:KingdomsoftheGrasslands
Skill:Factual

19) WhatmonarchiscreditedwithbeginningMalinkeexpansionandcreatingtheMaliEmpire?
A) Sundiata
B) MahmudofGhur
C) MansaKankanMusa
D) SunniAli
Answer: A
Diff:1
PageRef:289
Topic:KingdomsoftheGrasslands
Skill:Factual

204

20) WhichofthefollowingwasamajorcityneartheNigerRiver?
A) Bantu
B) IbnBatuta
C) Griot
D) Timbuktu
Answer: D
Diff:2
PageRef:291
Topic:KingdomsoftheGrasslands
Skill:Conceptual

21) TheAfricantradersassociatedwiththeMaliEmpirewerecalled
A) Soninke.
B) Zimbabwe.
C) Jenne.
D) Juula.
Answer: D
Diff:2
PageRef:289
Topic:KingdomsoftheGrasslands
Skill:Factual

22) WhatwasthenatureofurbanizationwithintheMaliEmpire?
A) Asaconquestempire,Malipossessedgarrisoncitiesforitssoldiers,butfailedtodevelop
commercialcenters.
B) MalipossessedportcitiesalongtheNigerRiversuchasJenneandTimbuktuthat
flourishedbothcommerciallyandculturally.
C) ThecitiesofMaliwereessentiallyreligiousandpalacecomplexesthatlacked
populationsofspecialistsotherthanmendevotedtoreligiousobservances.
D) Malifailedtodevelopcitiespriortoitsfall.
Answer: B
Diff:2
PageRef:291
Topic:KingdomsoftheGrasslands
Skill:Conceptual

205

23) WhichofthefollowingstatementsconcerningtheagriculturaleconomyofMaliismost
accurate?
A) Becauseofthepoorsoil,themajorityofthepeopleofMaliabandonedagriculturefor
trade.
B) LandinMaliwasownedcommunallyandfarmedinclangroups.
C) BecauseoftheearlyintroductionofironworkinginMalisculture,theagricultural
economywasabletotakeadvantageofextremelyadvancedtechnology.
D) Giventhedifficultiesofthesoil,periodicdroughts,andthelimitationsoftechnology,the
farmersofMaliwerebarelyabletoprovidethebasicfoodsthatsupportedtheimperial
states.
Answer: D
Diff:3
PageRef:291
Topic:KingdomsoftheGrasslands
Skill:Conceptual

24) Whatwasthesocialandpoliticalfunctionofthegriots?
A) Griotswerereligiousdivinerswhosefunctionwastoforetellthefutureandguidethe
decisionsofkings.
B) GriotsweretheclassesofpeopleoftheconqueststatesoftheMalikingswhowere
consignedtolaborwithintheempiresmines.
C) GriotsmasteredtheoraltraditionsoftheMalinkeandbyknowingthepastwere
consideredexcellentadvisorsofkings.
D) GriotswereMalinkemerchantswhoservedastrademiddlementhroughoutAfrica.
Answer: C
Diff:2
PageRef:289
Topic:KingdomsoftheGrasslands
Skill:Conceptual

25) WhatrulerwasresponsibleforthecreationoftheSonghayEmpire?
A) Sundiata
B) MansaKankanMusa
C) MahmudofGhur
D) SunniAli
Answer: D
Diff:2
PageRef:291
Topic:KingdomsoftheGrasslands
Skill:Factual

206

26) SonghaywasoriginallypartofwhatAfricanempire?
A) Ethiopia
B) Axum
C) Mali
D) Zimbabwe
Answer: C
Diff:1
PageRef:291
Topic:KingdomsoftheGrasslands
Skill:Factual

27) WhatwasthemilitarytitletakenbythelaterMuslimrulersofSonghay?
A) Dhow
B) Askia
C) Sultan
D) Caliph
Answer: B
Diff:1
PageRef:291
Topic:KingdomsoftheGrasslands
Skill:Factual

28) WhatledtothedownfallofSonghay?
A) Thecollapseoftheirrigationsystemonwhichtheagriculturaleconomydepended
B) InvasionbyaMoroccanMuslimarmyequippedwithfirearms
C) InvasionbythePortuguese
D) DefeatandincorporationwithintheMaliEmpire
Answer: B
Diff:2
PageRef:291
Topic:KingdomsoftheGrasslands
Skill:Conceptual

29) FollowingthedeclineofSonghay,smallerstatesdevelopedeastofMaliandSonghayamong
whatpeople?
A) Nok
B) Almoravid
C) Hausa
D) Masai
Answer: C
Diff:3
PageRef:291
Topic:KingdomsoftheGrasslands
Skill:Factual

207

30) WhywasIslamsoreadilyadoptedbyrulerswithintheSudan?
A) TheywereallconqueredbyoverwhelmingMuslimarmiesandforciblyconvertedto
Islam.
B) TheMuslimconceptofarulerwhounitedcivilandreligiousauthorityreinforced
traditionalideasofkingship.
C) TheMuslimconceptofreligiousequalityallowedrulerstodisposeofthetraditional
clansandlineagesofAfrica.
D) Asamonotheisticreligion,IslamwasmuchlikethetraditionalreligionsofAfrica.
Answer: B
Diff:2
PageRef:292
Topic:KingdomsoftheGrasslands
Skill:Conceptual

31) WhatwastherelationshipbetweenIslamandtheindigenousreligionsofAfrica?
A) IslamicteachersattemptedtoeradicatetheanimistindigenousreligionsofAfrica.
B) BecausebothindigenousAfricanreligionandIslamweremonotheistic,thetwobecame
inextricablyintertwined.
C) Islamaccommodatedpaganpracticeandbeliefintheearlystagesofconversion,because
rulershipandauthoritywasstillbasedontheabilitytointercedewithlocalspirits.
D) Islamsuccessfullyovercameindigenousreligiousbeliefs,andalmostallAfricans
convertedtoIslam.
Answer: C
Diff:2
PageRef:292
Topic:KingdomsoftheGrasslands
Skill:Conceptual

32) HowdidcontactwiththeMuslimworldaffecttheAfricanslavetrade?
A) BecauseoftheMuslimemphasisonequalityofallbelievers,earlyMuslimrulers
suppressedtheslavetrade.
B) SlaverywasunknowninAfricansocietyuntiltheMuslimsintroducedit.
C) WiththeMuslimconquestsofnorthAfricaandcommercialpenetrationtothesouth,
slaverybecameamorewidelydiffusedphenomenonandtheslavetradedeveloped
rapidly.
D) DespitetheMuslimacceptanceofslaveryanditswidespreaduseinIslamicsociety
outsideofAfrica,Muslimsgenerallyrefusedtoacceptblackslaves.
Answer: C
Diff:2
PageRef:292
Topic:KingdomsoftheGrasslands
Skill:Conceptual

208

33) HowwastheinstitutionofslaveryviewedinMuslimsociety?
A) Intheory,slaverywasseenasastageintheprocessofconversionofpaganstoIslam.
B) Slaverywasbelievedtobeapermanentconditionthatrenderedtheenslavedincapable
ofenteringheaven.
C) Slaverywasviewedassodemeaningthatthosewhowereenslavedweregoodfor
nothingbeyondlaborinthefieldsorthemines.
D) SlaverywasseenasabhorrentinIslamicsocietybecauseoftheemphasisontheequality
ofallbelievers.
Answer: A
Diff:2
PageRef:292
Topic:KingdomsoftheGrasslands
Skill:Conceptual

34) WhichofthefollowinggroupsdidNOTmigratetothecoastalregionofeastAfrica?
A) Bantu
B) SeaborneimmigrantsfromIndonesiaandMalaya
C) RefugeesfromOmanandthePersianGulf
D) BerbersfromnorthAfrica
Answer: D
Diff:2
PageRef:294
Topic:TheSwahiliCoastofEastAfrica
Skill:Conceptual

35) TheArabictermfortheeastAfricancoastwas
A) Zenj.
B) Dhow.
C) Askia.
D) Mandala.
Answer: A
Diff:1
PageRef:294
Topic:TheSwahiliCoastofEastAfrica
Skill:Factual

209

36) WhatwasthecommonculturaltraitoftheurbanizedtradingportsoftheeastAfricancoast?
A) MembershipintheSoninketribe
B) TheartisticstyleoftheNokculture
C) Bantu-basedandArabic-influencedSwahililanguage
D) AsinglerulingfamilyfromtheMalinketribe
Answer: C
Diff:2
PageRef:294
Topic:TheSwahiliCoastofEastAfrica
Skill:Conceptual

37) WhatwasthenatureoftradewiththetownsofeastAfrica?
A) MosttradewasfocusedoncaravanscrossingtheSaheltonorthAfrica.
B) Tradewaslimitedtolocalcommerceamongthetownsandwiththehinterlandslying
westoftheurbanizedcoast.
C) TherewasactivetradetothePersianGulfandEgypt,butlittlebeyondthosepoints.
D) InternationaltradeflourishedintheurbanizedportsofeastAfrica,includingcommerce
withIndiaandChina.
Answer: D
Diff:2
PageRef:294
Topic:TheSwahiliCoastofEastAfrica
Skill:Conceptual

38) HowdidtheexpansionofIslamaidinthecreationofinternationaltradeontheeastAfrican
coastline?
A) IslamexpandedtoIndiaandsoutheastAsiaprovidingareligiousbondoftrustbetween
thoseregionsandtheconvertedrulersofthecitiesofeastAfrica.
B) BecauseIslamregardedChristiansaspeoplesofthebook,Muslimmerchantscameto
tradeattheChristianizedportsofeastAfrica.
C) TheconnectionwiththeIslamicstatesofnorthAfricapermittedtheurbanizedportsof
eastAfricatotradewidelywithnorthernEurope.
D) ThedirecttraderoutesbetweentheAfricanstatesofwestAfricaandthecoastofeast
AfricastimulatedcommercebetweenthecitiesoftheeastandtheAtlanticOcean.
Answer: A
Diff:2
PageRef:295
Topic:TheSwahiliCoastofEastAfrica
Skill:Conceptual

210

39) TowhatextentdidIslamsuccessfullypenetratethepopulationsofeastAfrica?
A) AsinAsiatheconversionofthecoastalcitiesledtoalmostcompleteconversionofthe
populationslyinginland.
B) MostoftheclansandthemostimportantlineagegroupsoutsideoftheeastAfricancities
wereconvertedtoIslaminordertoparticipateintrade.
C) Islampenetratedverylittleintotheinterioramongthehunters,pastoralists,andfarmers,
andeventheareasnearthetradingtownsremainedrelativelyunaffected.
D) Islamspreadfromthecitiestothehinterlandinsuchawaythateventuallythe
countrysidewasmoreIslamicthanwerethecities.
Answer: C
Diff:2
PageRef:295
Topic:TheSwahiliCoastofEastAfrica
Skill:Conceptual

40) WhatwastheimpactofthePortuguesearrivalonthetradingpatternsoftheeastAfrican
coast?
A) Despitegreatefforttoshiftthefocusoftradeintotheirownhands,thePortuguesewere
neverabletocontrolthetradeonthenorthernSwahilicoast.
B) ThePortuguesearrivaldisruptedthenormaltradelinessoseverelythatAfricantrade
withIndianandsoutheastAsiaceasedtoexistforcenturies.
C) ThePortugueserapidlyusedtheirmilitarysuperioritytocontrolallaspectsoftrade
alongtheeastAfricancoast.
D) ThePortuguesealliedthemselveswithChristiansfromEthiopiainacombinedassaulton
theeastAfricancoastthatresultedinthedestructionoftheurbanizedports.
Answer: A
Diff:2
PageRef:295
Topic:TheSwahiliCoastofEastAfrica
Skill:Conceptual

41) Whatcity-statedidEwuaretheGreatrule?
A) Benin
B) Timbukto
C) Niger
D) Oyo
Answer: A
Diff:1
PageRef:299
Topic:TheSwahiliCoastofEastAfrica
Skill:Factual

211

42) TheDemographicTransitionisoftenassociatedwithwhatchangeinsociety?
A) Themilitarizationofsociety
B) Theadoptionofprivateproperty
C) Industrialization
D) ConversiontoChristianity
Answer: C
Diff:2
PageRef:297
Topic:TheSwahiliCoastofEastAfrica
Skill:Conceptual

43) WhatwasthemajorartisticoutputinBenin?
A) Oilpainting
B) Woodenjewelry
C) Featheredheadcoverings
D) Ivoryandbronzesculptures
Answer: D
Diff:2
PageRef:299
Topic:PeoplesoftheForestandthePlains
Skill:Conceptual

44) WhatwastheformofpoliticalorganizationoftheYorubapeopleofNigeria?
A) TheYorubawereorganizedinsmalltribalvillagesundertheruleofindividual
headmen.
B) TheYorubawereorganizedinanumberofsmallcity -statesundertheauthorityof
regionalkings.
C) TheYorubastatewasstronglycentralizedunderadivineemperorwhoruledfrom
Ile-Ife.
D) TheYorubawerepartoftheMaliEmpire.
Answer: B
Diff:2
PageRef:298
Topic:PeoplesoftheForestandthePlains
Skill:Conceptual

212

45) WhatwastheformofpoliticalorganizationoftheKingdomofKongo?
A) TheKingdomofKongowasaconfederationofsmallerstatesbroughtunderthecontrol
ofthekinganddividedintoeightprovinces.
B) TheKingdomofKongowasorganizedintoanumberofcity -statesruledfromIle-Ife.
C) TheKingdomofKongowaspartoftheMaliEmpire.
D) TheKingdomofKongowasastronglycentralizedempireruledbyadivinekingin
GreatZimbabwe.
Answer: A
Diff:2
PageRef:301
Topic:PeoplesoftheForestandthePlains
Skill:Conceptual

46) WhichofthefollowingstatementsconcerningGreatZimbabweisNOTcorrect?
A) Zimbabweactuallyreferstothestonebuildingsthatweretypicaloftheculture.
B) GreatZimbabwewasboththecapitalofthekingdomandareligiouscenter.
C) GreatZimbabwewasconstructedbyArabMuslimswhoweretradingwiththeBantu
residentsoftheregion.
D) Bythe15thcentury,acentralizedstatehadbeguntoformcenteredonGreatZimbabwe.
Answer: C
Diff:2
PageRef:301
Topic:PeoplesoftheForestandthePlains
Skill:Conceptual

47) TherulerofthekingdomcenteredonGreatZimbabwetookthetitleof
A) Mansa.
B) Askia.
C) Alafin.
D) MweneMutapa.
Answer: D
Diff:2
PageRef:301
Topic:PeoplesoftheForestandthePlains
Skill:Factual

213

48) InwhatregionwastheinfluenceofIslammostprofound?
A) CentralAfrica
B) WestAfricaamongtheYorubaandatBenin
C) Ethiopia
D) SudanandSwahilicoast
Answer: D
Diff:2
PageRef:301
Topic:Conclusion:InternalDevelopmentandExternalContacts
Skill:Factual

49) WhichofthefollowingstatesrepresentsthedevelopmentofBantuconceptsofkingshipand
state-building?
A) Mali
B) KingdomofKongo
C) Songhay
D) Mombassa
Answer: B
Diff:2
PageRef:302
Topic:Conclusion:InternalDevelopmentandExternalContacts
Skill:Conceptual

50) WhichofthefollowingstatementsconcerningtheimpactofIslamonsub-SaharanAfricais
mostaccurate?
A) IslamcutoffnorthAfricafromtheregionsofsub-SaharanAfrica.
B) AlthoughAfricahadneverbeentotallyisolatedfromtheMediterranean,thespreadof
IslambroughtlargeareasofAfricawithintheglobalcommunity.
C) WiththeconversionofregionsofthecontinenttoIslam,Africabecamethecenterofthe
Islamicworld.
D) DespitewidespreadconversionofAfricanstoIslam,thecontinentremainedoutsidethe
tradingsphereoftheIslamicworld.
Answer: B
Diff:2
PageRef:302
Topic:Conclusion:InternalDevelopmentandExternalContacts
Skill:Conceptual

214

EssayQuestions
1) WhatwerethecommonelementsofAfricansocietypriortotheincursionofIslam?
Answer: ExistenceofstatelesssocietiesthroughoutAfricabasedontribalandclanorganizations
typicalofchiefdom;Bantuascommonlanguagebase;widespreadacceptanceofanimist
religionbasedonnaturalforcesanddeitiesassociatedwiththem;significanceofclassof
diviners;emphasisonreligiousqualityofland;venerationofancestors;evidencefor
vitallocaltradenetworks;tendencytoexchangerawmaterialsformanufactured
products.
Diff:2
PageRef:282-286
Topic:AfricanSocieties:DiversityandSimilarities
Skill:Conceptual

2) Bywhatmeanswasconversionofsub-SaharanAfricacarriedout?Whatweretheprimary
avenuesofIslamicentryintosub-SaharanAfrica?
Answer: Primarilyconversionbytradersandmerchantsratherthanbyconquest;threeprimary
pointsofentry,AtlanticOcean,IndianOcean,andtheSahel.
Diff:2
PageRef:286-292
Topic:KingdomsoftheGrasslands
Skill:Conceptual

3) IslamicinfluencewasstrongestintheSudanicstatesandtheSwahilicoast.Compareand
contrastthepoliticalformsoftheseregions.WhatdoesthissuggestaboutthenatureofIslamic
influenceinAfrica?
Answer: Sudanicstates:empiresunderstrongcentralrulerswithprovincialsub-kings;social
stratificationintoclangroupsassociatedwithoccupationalspecialization;primarily
agriculturaleconomy.Swahilicoast:city-statesratherthancentralizedempire;basedon
trade,ofteninternationalinscope.Islamdidnotimposeaformofgovernment;
providedstrengthforindigenousinstitutionsandroyalauthority,legalcode;connected
AfricatoMuslimtradeworld.
Diff:2
PageRef:287-295
Topic:TheSwahiliCoastofEastAfrica
Skill:Conceptual

4) CompareandcontrastthepoliticalformsoftheAfricanstatesnotaffectedbyIslam(Yoruba,
Benin,Kongo,Zimbabwe)withthosethatwere(theSudanicstatesandtheSwahilicoast).
Answer: YorubaandBeninwerebasedoncity-stateorganizationsimilartoSwahilicoast,Kongo
andZimbabwewerelesserimperialformsofgovernmentsimilartotheSudanicstates.
SuggestsagainthatIslamdidnotseriouslyaffectnatureofpoliticaldevelopment.
Diff:2
PageRef:287-301
Topic:PeoplesoftheForestandthePlains
Skill:Conceptual

215

5) GiventhegeographicallocationofthoseAfricanstatesmostaffectedbyIslam,whatwerethe
mostimportantpointsofcontactbetweenAfricanandIslamicsocieties?Whatdoesthis
suggestaboutthenatureofIslamicconversion?
Answer: TheSudanicstatesjustsouthoftheSaharaandtheSwahilicoastweremostaffectedby
Islam;indicatesprimaryimportanceofIndianOceantraderoutesandcommercial
routesconnectingSahelwithMediterranean.Althoughsomeearlyevidenceofmilitary
penetrationofsub-SaharanAfrica(Almoravids,Almohades)fromMediterranean,most
contactprobablythroughtradeandcommerce.
Diff:2
PageRef:287-297
Topic:Introduction
Skill:Conceptual

216

Chapter14 CivilizationinEasternEurope:Byzantium
andOrthodoxEurope
Multiple-ChoiceQuestions
1) WhichofthefollowingisseenasadirectcontinuationoftheeasternportionoftheRoman
Empire?
A) FrankishEmpire
B) OttomanEmpire
C) ByzantineEmpire
D) AbbasidEmpire
Answer: C
Diff:2
PageRef:306
Topic:Introduction
Skill:Conceptual

2) TheByzantineEmpirelastedfrom
A) 500to1450.
B) 200to1200.
C) 300to1700.
D) 700to1650.
Answer: A
Diff:2
PageRef:306
Topic:Introduction
Skill:Factual

3) WhichofthefollowingwasthecapitaloftheByzantineEmpire?
A) Venice
B) Rome
C) Constantinople
D) Athens
Answer: C
Diff:3
PageRef:306
Topic:Introduction
Skill:Factual

217

4) WhatwasthemainreligionoftheByzantineEmpire?
A) Islam
B) Judaism
C) Hinduism
D) Christianity
Answer: D
Diff:2
PageRef:306
Topic:Introduction
Skill:Conceptual

5) WhatcountrywasparticularlyinfluencedbyByzantinecivilization?
A) Italy
B) TheMiddleEast
C) Russia
D) Poland
Answer: C
Diff:1
PageRef:304
Topic:Introduction
Skill:Factual

6) WhichofthefollowingdoesNOTrepresentasimilaritybetweenthespreadofcivilizationin
easternandwesternEurope?
A) CivilizationspreadnorthwardfromaMediterraneanbase.
B) Animismgavewaytomonotheism.
C) Northernpoliticalunitsrapidlydominatedthepoliticalsophisticationofcivilization
areasinAsiaandnorthAfrica.
D) InbothcasesnewlycivilizedareaslookedbacktotheGreco-Romanpast.
Answer: C
Diff:3
PageRef:306
Topic:Introduction
Skill:Conceptual

218

7) Whichofthefollowingrepresentsadifferencebetweenthespreadofcivilizationineastern
andwesternEurope?
A) TheyproduceddifferentversionsofChristianity,culturallyaswellasorganizationally
separate.
B) OnlyeasternEuropedevelopednorth-southcommercialties.
C) Centralizedgovernmentandwell-organizedbureaucracywasmoreafeatureofwestern
EuropethaneasternEurope.
D) EasternEuroperetainedlessfullythecultureofthelaterRomanEmpirethandidthe
West.
Answer: A
Diff:2
PageRef:307
Topic:Introduction
Skill:Conceptual

8) WheredidtheRomansestablishaneasterncapitalinthe4thcenturyCE?
A) Rome
B) Nicaea
C) Constantinople
D) Baghdad
Answer: C
Diff:1
PageRef:307
Topic:TheByzantineEmpire
Skill:Factual

9) TheemperorresponsiblefortheinitialconstructionofConstantinoplewas
A) Constantine.
B) Justinian.
C) Diocletian.
D) Heraklius.
Answer: A
Diff:2
PageRef:307
Topic:TheByzantineEmpire
Skill:Factual

219

10) WhatwasthedifferenceinthemilitaryorganizationofByzantineandwesternRoman
empires?
A) ThewesternRomanEmpiredependedoncitizensoldiersuntilthe5thcentury.
B) TheByzantineEmpirerecruitedmenfromtheMiddleEast.
C) TheByzantineEmpirerecruitedbarbariansalmostexclusivelywhiletheRomanEmpire
oftheWestdependedonIslamicmercenaries.
D) TheByzantineEmpiredependedonthestrengthofConstantinopleswallsanddidnot
recruitanarmy.
Answer: B
Diff:2
PageRef:307
Topic:TheByzantineEmpire
Skill:Conceptual

11) WhatwasthegreatchurchbuiltinConstantinoplebyJustinian?
A) St.Peters
B) TheCathedralofSt.Dimitri
C) HagiaSophia
D) Sts.CyrilandMethodius
Answer: C
Diff:2
PageRef:307
Topic:TheByzantineEmpire
Skill:Factual

12) StartingafterthereignofJustinian,whatwastheofficiallanguageoftheeasternempire?
A) Latin
B) Persian
C) Arabic
D) Greek
Answer: D
Diff:2
PageRef:307
Topic:TheByzantineEmpire
Skill:Factual

220

13) TheByzantineEmpirebegan
A) inthe9thcenturyC.E.,withthemissionaryworkofCyrilandMethodius.
B) inthe4thcenturyC.E.,withtheestablishmentofConstantinopleastheeasterncapitalof
theempire.
C) inthe1stcenturyC.E.,duringthereignofAugustus.
D) inthe5thcenturyC.E.,withthefallofRome.
Answer: B
Diff:2
PageRef:307
Topic:TheByzantineEmpire
Skill:Conceptual

14) WhowasthebrilliantgeneralwhohelpedJustinianachievemilitarygainsinnorthAfricaand
Italy?
A) Belisarius
B) Theodorus
C) Procopius
D) Basil
Answer: A
Diff:3
PageRef:307
Topic:TheByzantineEmpire
Skill:Factual

15) ThenamenormallygiventotheformofChristianitythatemergedintheByzantineEmpire
was
A) RomanCatholicism.
B) Nestorianism.
C) OrthodoxChristianity.
D) Solafideanism.
Answer: C
Diff:1
PageRef:311
Topic:TheByzantineEmpire
Skill:Factual

221

16) WhatEasternemperorwasresponsiblefortheattemptedrestorationofaunitedRoman
Empireafter533?
A) Constantine
B) Justinian
C) Diocletian
D) Theodosius
Answer: B
Diff:1
PageRef:307
Topic:TheByzantineEmpire
Skill:Factual

17) WhichofthefollowingwasNOToneofJustinianspositivecontributionstotheByzantine
Empire?
A) TherebuildingofConstantinople
B) SystematizingoftheRomanlegalcode
C) ThereconquestofGaul
D) TheconstructionofHagiaSophia
Answer: C
Diff:2
PageRef:307
Topic:TheByzantineEmpire
Skill:Factual

18) TheformercourtesanwhowasJustinianswifeandadvisorwas
A) Sophia.
B) Syria.
C) Zoe.
D) Theodora.
Answer: D
Diff:2
PageRef:307
Topic:TheByzantineEmpire
Skill:Factual

222

19) TheSlavickingdomthatparticularlypressedByzantineterritoryintheBalkansafterthe8th
centuryC.E.was
A) Russia.
B) Hungary.
C) Anatolia.
D) Bulgaria.
Answer: D
Diff:1
PageRef:309
Topic:TheByzantineEmpire
Skill:Factual

20) AllofthefollowingwereoutcomesofJustinianswarsofreconquestEXCEPT
A) thepermanentadditionofItalytotheByzantineEmpire.
B) increasedtaxpressuresonthegovernment.
C) militarysuccessesinnorthAfricaandItaly.
D) weakeningoftheempiresdefensesonitseasternfrontiers.
Answer: A
Diff:2
PageRef:308
Topic:TheByzantineEmpire
Skill:Conceptual

21) Afterthe7thcentury,whatgroupposedthegreatestthreattotheeasternfrontiersofthe
ByzantineEmpire?
A) TheSassanidPersians
B) TheGermans
C) TheHuns
D) TheArabMuslims
Answer: D
Diff:2
PageRef:308
Topic:TheByzantineEmpire
Skill:Factual

223

22) WhatwasthetechnologicalinnovationthataidedtheByzantineEmpireinwithstandingthe
MuslimsiegeofConstantinoplein717?
A) Cannon
B) Greekfire
C) Gunpowderrockets
D) Catapults
Answer: B
Diff:1
PageRef:309
Topic:TheByzantineEmpire
Skill:Factual

23) WhichofthefollowingwasaresultoftheconflictbetweentheByzantineEmpireandtheArab
Muslims?
A) TheArabthreattotheByzantineEmpirewaspermanentlyremoved.
B) Thepositionofsmallfarmersintheempirewasweakenedasaresultofheavytaxation,
resultingingreateraristocraticestates.
C) TheByzantineEmpirewasabletorecovertheprovincesofSyriaandEgypt,thus
regainingvaluableagriculturallandandincreasedwealth.
D) ThecommercialsignificanceofConstantinoplewasdestroyedbythe8thcentury,forcing
theByzantineEmpiretodependincreasinglyontradewiththeWest.
Answer: B
Diff:2
PageRef:309
Topic:TheByzantineEmpire
Skill:Conceptual

24) WhichofthefollowingconclusionsthatmightbedrawnabouttheByzantineEmpireasthe
resultofitsconflictswithitsneighborsismosttrue?
A) Despiteallofitsdifficulties,itswarswithitsneighborsdemonstratesthattheempirehad
realcorestrength.
B) Theoutcomeofthewarsdemonstratesthatbythe10thcenturytheempirewas
completelydecadentandincapableofdefendingitself.
C) ThewarswithIslamicpowersdemonstratetheinherentweaknessoftheByzantine
military.
D) TheroleofTheodoraandotherwomeninthewarsdemonstratethattheintrusionof
womenintoByzantinepoliticswashighlydestructive.
Answer: A
Diff:2
PageRef:309
Topic:TheByzantineEmpire
Skill:Conceptual

224

25) Byzantineculturallifecenteredontheseculartraditionsof
A) Islam.
B) Confucianism.
C) ancientEgypt.
D) Hellenism.
Answer: D
Diff:1
PageRef:310
Topic:TheByzantineEmpire
Skill:Factual

26) Whatemperorbecamerenownedinthe11thcenturyastheslayeroftheBulgariansby
defeatingtheBulgariankingdomandrestoringByzantineruleintheBalkans?
A) Justinian
B) Anastasius
C) BasilII
D) MichaelIII
Answer: C
Diff:2
PageRef:309
Topic:TheByzantineEmpire
Skill:Factual

27) InwhichofthefollowingwaysweretheByzantinebureaucracyandtheChinesebureaucracy
similar?
A) therewasanextensivestateexamsysteminboth
B) emperorsplayedlittleroleineithergovernment
C) therewasnolinkageofthebureaucraciestolocaladministration
D) bothbureaucracieswereopentotalentedcommoners,notjustaristocrats
Answer: D
Diff:3
PageRef:309
Topic:TheByzantineEmpire
Skill:Conceptual

225

28) WhichofthefollowingstatementsconcerningtheByzantinebureaucracyisNOTaccurate?
A) Manyoftheofficialsclosesttotheemperorwereeunuchs.
B) Aristocratspredominated,buttherewassomeopennesstotalent.
C) Anelaboratesystemofspiesmaintainedloyaltytothecentralgovernment.
D) Bureaucratsweretrainedtobeliterate,butlackedtheformaltraininginGreekclassics
andphilosophy.
Answer: D
Diff:3
PageRef:309
Topic:TheByzantineEmpire
Skill:Conceptual

29) WhichofthefollowingstatementsconcerningByzantinemilitaryorganizationismost
accurate?
A) Byzantinesoldierswererecruitedalmostexclusivelyfrompeoplesoutsidetheempire.
B) TheByzantineEmpireemulatedthelaterArabicempiresbymakinguseofslavearmies
asabasisforthemilitaryforce.
C) TheByzantineEmpirerecruitedtroopswithintheempirebygrantingheritablelandin
returnformilitaryservice.
D) MilitarycommandwithintheByzantineEmpireremainedinthehandsofthetraditional
aristocracywholivedinConstantinople.
Answer: C
Diff:3
PageRef:310
Topic:TheByzantineEmpire
Skill:Conceptual

30) WhichofthefollowingstatementsconcerningurbanizationwithintheByzantineEmpireis
mostcorrect?
A) Constantinoplecontrolledtheeconomyandgrewtoenormoussize,butothercitieswere
relativelysmall.
B) ConstantinoplebegantodeclineinpopulationinthelateryearsoftheByzantineEmpire
andwassurpassedbythegrowthofotherurbancenters.
C) LikeChina,theByzantineEmpirewasheavilyurbanizedwithmanycitiesnumbering
morethan100,000.
D) MostpeopleintheByzantineEmpirelivedinfivegreatcities:Constantinople,Nicaea,
Smyrna,Rome,andAthens.
Answer: A
Diff:3
PageRef:310
Topic:TheByzantineEmpire
Skill:Conceptual

226

31) WhatweretheprimaryexportsoftheByzantineEmpire?
A) Foodproducts
B) Rawmaterials,suchasmetaloresfromAsiaMinor
C) Luxuryproductssuchassilk,cloth,andcarpets
D) Theempireproducedlittleofsignificanceandwasalmostexclusivelyanimporterof
goods
Answer: C
Diff:2
PageRef:310
Topic:TheByzantineEmpire
Skill:Conceptual

32) WhichofthefollowingstatementsconcerningthemerchantclassoftheByzantineEmpireis
mostaccurate?
A) Becauseofitslackofexportproducts,Byzantinecommercewascontrolledcompletelyby
foreignmerchants.
B) Byzantinemerchants,becauseoftheirwealth,rapidlybecamethemostpowerfulforcein
thegovernmentofConstantinopleandtheempire.
C) TheByzantinemerchantclassneverrecoveredfromthelossofterritoriestotheMuslims
andwasnotasignificantfactorwithintheempire.
D) TherewasalargeandwealthymerchantclassintheByzantineEmpire,butitnever
gainedsignificantpoliticalpowerbecauseofthepowerofthebureaucracy.
Answer: D
Diff:3
PageRef:310
Topic:TheByzantineEmpire
Skill:Conceptual

33) WhichofthefollowingwasNOTastrengthofByzantineculturallife?
A) DomedbuildingsadaptedfromtheRomanstyleofarchitecture
B) Richlycoloredmosaics
C) Innovativeliteraryforms
D) Paintedicons
Answer: C
Diff:2
PageRef:310
Topic:TheByzantineEmpire
Skill:Conceptual

227

34) WhatwastheresultoftheconflictovertheuseofreligiousimagesintheOrthodoxchurch?
A) LiketheMuslims,theOrthodoxchurchbannedsubsequentuseofreligiousimagesin
favorofnon-representationalart.
B) Becauseofthepopularreactioninfavoroficons,theOrthodoxchurchrestoredtheiruse;
butthecloserelationshipbetweenchurchandstatewasbroken.
C) Afteralongandcomplexbattle,iconusewasgraduallyrestored,whilethetraditionof
statecontroloverchurchaffairswasalsoreasserted.
D) Becauseofthestrongresistanceofthemonks,iconusewasrestrictedtothoseregionsof
theempirewherethemonasterieshadlittleinfluence.
Answer: C
Diff:3
PageRef:311
Topic:TheByzantineEmpire
Skill:Conceptual

35) WhichofthefollowingissueswasacauseforthesplitbetweentheRomanCatholicand
Orthodoxchurchesafter1054?
A) TheOrthodoxchurchslackofbishops
B) TheinsistenceofthepatriarchofConstantinopleonsupremacywithinchurchcouncils
C) TheabsenceofmonasticisminRomanCatholicism
D) TheRomanCatholicpracticeofrequiringcelibacyforitspriests
Answer: D
Diff:2
PageRef:310
Topic:TheByzantineEmpire
Skill:Conceptual

36) Inthe11thcentury,whatgroupofpeopleseizedmostoftheAsiaticprovincesofthe
ByzantineEmpire?
A) TheHuns
B) TheMongols
C) TheOttomanTurks
D) TheSeljukTurks
Answer: D
Diff:2
PageRef:312
Topic:TheByzantineEmpire
Skill:Factual

228

37) WhatwastheoutcomeoftheWesternCrusadeof1204?
A) ThecrusaderssucceededintemporarilypushingbacktheTurksandrestoringtheAsiatic
provincesoftheByzantineEmpire.
B) TheCrusadesucceededinestablishingaWesternkingdomintheHolyLand,butfailed
torelievetheAsiaticprovincesoftheByzantineEmpire.
C) ThecrusadersattackedandconqueredConstantinople,temporarilyestablishinga
Westernkingdomthere.
D) TheCrusaderesultedintheestablishmentofaWesternkingdomofBulgariainthe
Balkans.
Answer: C
Diff:2
PageRef:312
Topic:TheByzantineEmpire
Skill:Factual

38) InwhatyeardidtheTurkssuccessfullycaptureConstantinople?
A) 1071
B) 1326
C) 1453
D) 1501
Answer: C
Diff:2
PageRef:313
Topic:TheByzantineEmpire
Skill:Factual

39) WhattwomissionarieswereresponsibleforthecreationofawrittenscriptforSlavic
language?
A) BasilandJohnChrysostom
B) CyrilandMethodius
C) ConstansandMnemosyne
D) JohnandMatthew
Answer: B
Diff:1
PageRef:313
Topic:TheSpreadofCivilizationinEasternEurope
Skill:Factual

229

40) TheSlavicalphabetcreatedbyOrthodoxmissionariestotheSlavsiscalled
A) cursive.
B) Constantinapolitan.
C) Cyrillic.
D) Russo-Slavic.
Answer: C
Diff:1
PageRef:314
Topic:TheSpreadofCivilizationinEasternEurope
Skill:Factual

41) WhichofthefollowingcountrieswasconvertedtoOrthodoxChristianity?
A) Poland
B) Russia
C) Hungary
D) Czechoslovakia
Answer: B
Diff:2
PageRef:314
Topic:TheSpreadofCivilizationinEasternEurope
Skill:Factual

42) ThekingdomofKievinsouthernRussiawasestablishedbytradersfromwhatregion?
A) Scandinavia
B) Czechoslovakia
C) TheBalkans
D) TheByzantineEmpire
Answer: A
Diff:2
PageRef:314
Topic:TheSpreadofCivilizationinEasternEurope
Skill:Factual

43) ThefirstkingofKievanRussiawas
A) Yarolav.
B) VladimirI.
C) BorisGudonov.
D) Rurik.
Answer: D
Diff:1
PageRef:314
Topic:TheSpreadofCivilizationinEasternEurope
Skill:Factual

230

44) WhydidVladimirIpreferOrthodoxChristianitytoRomanCatholicism?
A) Hepreferredtoavoidthepitfallsofthevenerationoficons.
B) HebelievedthatRomanCatholicismimpliedpapalinterference,whileOrthodoxy
embracedthecontrolofthechurchbythestate.
C) HewasnotfamiliarwithRomanCatholicism,becausetheWesternformofChristianity
hadnotpenetratedintoeasternEurope.
D) HedidnotbelieveinclericalcelibacywhichwasrequiredoftheRomanCatholic
priesthood.
Answer: B
Diff:2
PageRef:314
Topic:TheSpreadofCivilizationinEasternEurope
Skill:Conceptual

45) WhichofthefollowingpracticeswasNOTadoptedasafeatureoftheRussianOrthodox
Church?
A) Venerationoficons
B) Anactivemonasticmovement
C) Clericalcelibacy
D) Emphasisonalmsgiving
Answer: C
Diff:2
PageRef:315
Topic:TheSpreadofCivilizationinEasternEurope
Skill:Conceptual

46) WhichoftheKievanprincesissuedaformallegalcode?
A) Rurik
B) Yaroslav
C) Vladimir
D) Boris
Answer: B
Diff:2
PageRef:314
Topic:TheSpreadofCivilizationinEasternEurope
Skill:Factual

231

47) WhatwastheextentofwesternEuropeaninfluenceinKievanRussia?
A) DespitetheadoptionofOrthodoxChristianity,mostoftheecclesiasticalinfluenceswere
RomanCatholic.
B) TherewasdirectWesterninfluenceinRussiainbothmusicandartforms.
C) LikewesternEurope,Russiaalsomovedawayfromthepopularvenerationoficonsand
representationalimages.
D) Russiasreligiouscultureandsocialandeconomicpatternsdevelopedseparatelyfrom
westernEuropes.
Answer: D
Diff:2
PageRef:316
Topic:TheSpreadofCivilizationinEasternEurope
Skill:Conceptual

48) WhatgroupofpeopleswereresponsiblefortheconquestofKievanRussiain1237 -1238?


A) Huns
B) Mongols
C) OttomanTurks
D) SeljukTurks
Answer: B
Diff:2
PageRef:317
Topic:TheSpreadofCivilizationinEasternEurope
Skill:Factual

49) WhichofthefollowingstatementsconcerningtheTatarinvasionofRussiaismostaccurate?
A) TatarcontrolofRussialastedforfourdecades.
B) TheTatarsusedRussiaasaspringboardfortheirsuccessfulinvasionofwesternEurope.
C) TatarsupervisiondidnotdestroyRussianChristianityoranativeRussianaristocracy.
D) TheTatarsrapidlydevisedacloselysupervisedlocaladministrationfortheRussian
cities.
Answer: C
Diff:3
PageRef:317
Topic:TheSpreadofCivilizationinEasternEurope
Skill:Conceptual

232

50) Whatwasthenameofthepaintingsofsaintsandotherreligiousfiguresfoundinthe
ByzantiumEmpire?
A) Trinitypaintings
B) Christianpaintings
C) Filioquepaintings
D) Iconpaintings
Answer: D
Diff:2
PageRef:310
Topic:TheByzantineEmpire
Skill:Factual

EssayQuestions
1) CompareandcontrastthespreadofEuropeancivilizationineasternandwesternEurope.
Answer: Similarities:civilizationsinbothhalvesofEuropemovednorthward;typifiedbyspread
ofmonotheismoveranimism;northernpoliticalunitswerelesscomplexandwell
organizedthanMediterraneancorecivilizations;allnewregionsrecognized
Greco-RomanpastandChristianity.Differences:differentversionsofChristianityin
EastandWest;littlecommercialconnectionbetweeneasternandwesternEurope;
easternEuropemorepoliticallyadvancedthanwesternEurope;easternEuropemore
directheirofRomanEmpire.
Diff:2
PageRef:310-314
Topic:Introduction
Skill:Conceptual

2) WhatwasthepoliticalorganizationoftheByzantineEmpire?
Answer: Emperorheldallpower;viewedasdivinelyordainedruler;supportedbyelaborate
courtritual;governmentinhandsoftrainedbureaucracywitheunuchsinpositions
closesttotheemperors;localadministratorsappointedbycentralbureaucracy;military
recruitedfromempirespopulationbygrantsofheritablelandinreturnformilitary
service;growthofauthorityoflocalmilitarycommandersatexpenseoftraditional
aristocracy.
Diff:2
PageRef:307-308
Topic:TheByzantineEmpire
Skill:Conceptual

3) WhatwerethefactorsinthedeclineoftheByzantineEmpire?
Answer: Seriesofexternalthreattoempire;TurkishinvasionsseizedAsiaticportionsofempire
after1071;reducedfoodsuppliesandtaxbaseofempire;growingeconomicand
politicalpowerofwesternEuropeledtoinroadsonConstantinopleseconomicposition;
WesternCrusadein1204temporarilyconqueredByzantinecapital;riseofindependent
SlavickingdomsinBalkanschallengedByzantineauthoritythere;OttomanTurks
conqueredConstantinoplein1453.
Diff:2
PageRef:312-313
Topic:TheByzantineEmpire
Skill:Conceptual

233

4) InwhatwayswasthecultureofKievanRussiaanextensionoftheByzantineEmpire?
Answer: DevelopmentofOrthodoxchurchinRussiaimposedmuchofByzantineculture;
state-dominatedchurch;venerationoficons;similarartisticemphasisonarchitecture
(domedbuildings)andreligiousimages;statebasedondivinelyauthorizedpowerof
monarchs;codificationoflaws;unabletoduplicateByzantinegovernmentinthesense
thattherewasnocomparabletrainedbureaucracy.
Diff:2
PageRef:313-319
Topic:TheSpreadofCivilizationinEasternEurope
Skill:Conceptual

234

Chapter15 ANewCivilizationEmergesin
WesternEurope
Multiple-ChoiceQuestions
1) ThepostclassicalperiodinWesternhistorybetweenthefalloftheRomanEmpireandthe
15thcenturyisreferredtoasthe
A) MiddleAges.
B) Renaissance.
C) AgeofDiscovery.
D) Baroque.
Answer: A
Diff:1
PageRef:345
Topic:Introduction
Skill:Factual

2) InwhichofthefollowingwayswasthemedievalWestNOTlikeothercivilizations?
A) ThemedievalperiodsawthespreadofcivilizationoutsidetheMediterraneanzonecore
regiontonewareasinnorthernEurope.
B) Newreligiousbeliefsaccompaniedthespreadofcivilization.
C) ThemedievalWestremainedculturallybackward.
D) WesternEuropeparticipatedintheemerginginternationalcommunity.
Answer: C
Diff:2
PageRef:345
Topic:TheFlavoroftheMiddleAges
Skill:Conceptual

3) WhichofthefollowingstatementsconcerningtheimpactofChristianityonpolytheistic
religionsinwesternEuropeismostaccurate?
A) Christianityeradicatedalltracesofthoseearlierreligionsasthenewreligionbecame
universalinwesternEurope.
B) Theprocessofconversionproducedareligiousamalgaminwhichbeliefsinmagicand
supernaturalspiritscoexistedwithChristianity.
C) AlthoughChristianitymadeinroads,manyareasofEuroperetainedpolytheisticbeliefs
andrejectedthenewreligion.
D) Smallislandsofpolytheisticbeliefremained,butChristianityeradicatedbeliefinmagic
andspiritswhereverthenewreligionwasaccepted.
Answer: B
Diff:3
PageRef:345
Topic:Introduction
Skill:Conceptual

235

4) MedievalbackwardnessinwesternEuropeaccountsfor
A) thefailureofthemedievalWesttoestablishcentralizedgovernments.
B) theinabilityoftheWesttoestablishtradecontactswiththelargerworldcommercial
network.
C) therefusaloftheWesttoadopttechnologiesandknowledgefromothercivilizations.
D) theanxietyoftheWestaboutthemorepowerfulMuslimworld.
Answer: D
Diff:2
PageRef:345-346
Topic:TheFlavoroftheMiddleAges
Skill:Conceptual

5) FollowingthefallofRome,wherewasthecenterofthepost-classicalWest?
A) IntheformerRomancolonyofSpain
B) InItaly,particularlyRome
C) InthecentralplainofnorthernEurope: France,theLowCountries,southernandwestern
Germany
D) Greece
Answer: C
Diff:2
PageRef:346
Topic:StagesofPost-ClassicalDevelopment
Skill:Factual

6) Whoweretheinvaderswhodisruptedthedevelopmentofpoliticalinstitutionsinthe
medievalWestuntilthe10thcentury?
A) Muslims
B) Mongols
C) Vikings
D) Chinese
Answer: C
Diff:1
PageRef:346-347
Topic:StagesofPost-ClassicalDevelopment
Skill:Factual

236

7) WhichofthefollowingstatementsconcerningtheintellectualactivityofthemedievalWest
priortothe8thcenturyismostaccurate?
A) ClassicalrationaltraditionswereactivelyunitedwithChristianmysticismtocarveouta
newintellectualworld.
B) Withthefewliteratepeopleconcentratedinmonasteries,littlewasachievedotherthan
copyingoldermanuscripts.
C) Universitiesrapidlycreatedanewintellectualclimateinwhichlogicwasappliedto
mattersofChristiandoctrine.
D) Allliteracyandcontactwiththeancientculturewaslostinthecenturiesfollowingthe
fallofRome.
Answer: B
Diff:2
PageRef:347
Topic:StagesofPost-ClassicalDevelopment
Skill:Conceptual

8) Thesystemthatdescribedeconomicandpoliticalrelationsbetweenlandlordsandtheir
peasantlaborerswascalled
A) manorialism.
B) feudalism.
C) slavery.
D) capitalism.
Answer: A
Diff:1
PageRef:347
Topic:StagesofPost-ClassicalDevelopment
Skill:Factual

9) Agriculturallaborersunderthejurisdictionofaristocraticlandownerswerecalled
A) artisans.
B) guilds.
C) serfs.
D) bourgeoisie.
Answer: C
Diff:1
PageRef:347
Topic:StagesofPost-ClassicalDevelopment
Skill:Factual

237

10) WhichofthefollowingstatementsconcerningtheagriculturallaborersofthemedievalWestis
NOTtrue?
A) Theyreceivedprotectionandtheadministrationofjusticefromtheirlandlords
B) Theywereobligatedtoturnoverpartoftheirgoodstoremainontheland
C) Theyretainedessentialownershipoftheirhouses
D) Theywereslaves
Answer: D
Diff:2
PageRef:347
Topic:StagesofPost-ClassicalDevelopment
Skill:Conceptual

11) Themoldboardwas
A) asystemofjusticecommontothemanorialregimeofthemedievalWest.
B) atechnologicalinnovationaplowthatalloweddeeperturningofthesoil.
C) atechnologicalinnovationawater-drivenmillforgrindinggrain.
D) thepeasantcouncilthatdeterminedthedivisionoflandandlaborinapeasantvillage.
Answer: B
Diff:2
PageRef:347
Topic:StagesofPost-ClassicalDevelopment
Skill:Factual

12) WhichofthefollowingstatementsaboutthemanorialsystemisNOTtrue?
A) Itwastechnologicallysophisticated.
B) IthadoriginatedintheRomanEmpire.
C) Itsobligationsboreheavilyonserfs.
D) Agriculturalproductivitywaslow.
Answer: A
Diff:2
PageRef:347
Topic:StagesofPost-ClassicalDevelopment
Skill:Conceptual

238

13) Whichofthefollowingstatementsconcerningthethree-fieldrotationsystemismostaccurate?
A) Introducedinthe8thcentury,thethree-fieldrotationaddedacrestoproductionby
leavingonlyathirdofthelandunplanted.
B) Thethree-fieldsystemremovedlandfromproductionbyreservingone-thirdforfallow.
C) Thethree-fieldsystemwasrapidlyreplacedafterthe8thcenturybythetwo-field
systemthatofferedgreaterflexibilityintermsofcroprotation.
D) Thethree-fieldsystemremovedfallowfieldsandreplacedthemwithnitrogen-bearing
crops.
Answer: A
Diff:2
PageRef:347
Topic:StagesofPost-ClassicalDevelopment
Skill:Conceptual

14) Relationshipsbetweenmembersofthemilitaryelitebasedonareciprocalexchangeoflandfor
militaryserviceandloyaltywerecalled
A) manorialism.
B) feudalism.
C) capitalism.
D) theguildsystem.
Answer: B
Diff:1
PageRef:351
Topic:StagesofPost-ClassicalDevelopment
Skill:Factual

15) Themembersofthemilitaryelitewhoreceivedlandinreturnformilitaryserviceinthebands
ofthegreaterlordswerecalled
A) fiefs.
B) benefices.
C) vassals.
D) serfs.
Answer: C
Diff:1
PageRef:351
Topic:StagesofPost-ClassicalDevelopment
Skill:Factual

239

16) Whichofthefollowingstatementsaboutfeudalismismostaccurate?
A) Althoughitinhibitedthedevelopmentofstrongcentralstates,somekingswereableto
usefeudalismtobuildtheirownpower.
B) Althoughitprovidedinitialpoliticalstability,feudalismwasrapidlyreplacedbya
westernEuropeanimperialsystem.
C) Feudalismrepresentedonlyabrief,andlargelyunsatisfactory,attempttocreatepolitical
stabilityinwesternEurope.
D) Feudalismproducedcentralizedmonarchiesbythe8thcentury.
Answer: A
Diff:2
PageRef:351
Topic:StagesofPost-ClassicalDevelopment
Skill:Conceptual

17) WhichofthefollowingwasNOTapowerofthepapacyimmediatelyafter500?
A) Theabilitytosenddirectivesandreceiveinformation
B) Regulationofdoctrine
C) Sponsorshipofmissionaryactivity
D) Theappointmentofallbishops
Answer: D
Diff:2
PageRef:347
Topic:StagesofPost-ClassicalDevelopment
Skill:Conceptual

18) WhatFrankishkingwasresponsiblefortheconversionofhispeopletoChristianityinorderto
gainavaguedominationovertheFranks?
A) CharlesMartel
B) Clovis
C) Charlemagne
D) PepinIII
Answer: B
Diff:2
PageRef:347
Topic:StagesofPost-ClassicalDevelopment
Skill:Factual

240

19) WhatbeliefdidtheconversionofGermanickingscreateamongWesternreligiousleaders,
particularlythepope?
A) Thatthechurchwassubordinatetothesecularmonarchs
B) Thatthechurchhadalegitimateauthorityseparatefromandsuperiortothesecular
rulers
C) ThatthechurchshouldavoidconversionofnorthernGermanickings
D) Thatsuchconversionrepresentedadangertothepapalhierarchy
Answer: B
Diff:2
PageRef:347
Topic:StagesofPost-ClassicalDevelopment
Skill:Conceptual

20) BenedictofNursiawasresponsibleforwhataccomplishmentinthe6thcentury?
A) TheconversionoftheFranks
B) Thebanningoflayinvestiture
C) Thecreationofasetofrulesformonasteries
D) TheconquestofConstantinople
Answer: C
Diff:2
PageRef:348
Topic:StagesofPost-ClassicalDevelopment
Skill:Factual

21) WhichofthefollowingwasNOTabenefitofthemonasticmovementinwesternEurope?
A) TheydisciplinedtheintensespiritualityofthemedievalWestinordertopromote
Christianunity.
B) Manymonasterieshelpedimprovethecultivationoftheland.
C) Bycopyingancienttexts,monkspreservedclassicalcultureforlaterintellectualinquiry.
D) Theirpoliticalorganizationprovidedthefoundationforthepoliticalorderestablishedin
France,Germany,andEngland.
Answer: D
Diff:2
PageRef:348
Topic:StagesofPost-ClassicalDevelopment
Skill:Conceptual

241

22) WhatdynastytookovertheFrankishmonarchyinthe8thcentury?
A) Merovingian
B) Hohenstaufen
C) Carolingian
D) Saxon
Answer: C
Diff:1
PageRef:348
Topic:StagesofPost-ClassicalDevelopment
Skill:Factual

23) InwhatyearwasCharlemagneabletoestablishasubstantial,iftemporary,empireinFrance
andGermany?
A) 500
B) 800
C) 900
D) 1000
Answer: B
Diff:2
PageRef:349
Topic:StagesofPost-ClassicalDevelopment
Skill:Factual

24) WhichofthefollowingstatementsconcerningtheHolyRomanemperorsafterthe10th
centuryismostaccurate?
A) TheybuiltupontheCarolingianfoundationstoestablishthemostcentralized
governmentfoundinthemedievalWest.
B) BuildingonafeudalframeworkratherthantheCarolingianGermanicfoundations,the
HolyRomanEmperorscreatedastronglycentralizedgovernment.
C) DiscardingmuchoftheformerCarolingianEmpire,theHolyRomanemperors
reestablishedacentralizedgovernmentinnorthernItaly.
D) TheruleoftheHolyRomanemperorsbecameincreasinglyhollow,becausetheydidnot
buildasolidmonarchyfromregionalfoundations.
Answer: D
Diff:2
PageRef:350
Topic:StagesofPost-ClassicalDevelopment
Skill:Conceptual

242

25) WhichofthefollowingwasNOTapositivedevelopmentthatintroducednewsourcesof
strengthbythe9thand10thcenturiestowesternEurope?
A) Newagriculturaltechniques
B) EndofVikingraids
C) Developmentofimperialgovernment
D) Greaterregionalpoliticalstability
Answer: C
Diff:2
PageRef:350-351
Topic:StagesofPost-ClassicalDevelopment
Skill:Conceptual

26) Whatwastheimpactoftheimprovedeconomyafterthe10thcenturyonthesocialsystemof
westernEurope?
A) Theimprovementsintheagriculturalsystemretardedthedevelopmentoftownsand
restrictedsocialmobility.
B) HarshserfdombecametherulethroughoutwesternEurope.
C) Theincreasedpaceofeconomiclifecreatedalessrigidstructure.
D) Despitetheimprovedeconomy,therigidsocialsystemassociatedwithfeudalism
continuedtodominatewesternEurope.
Answer: C
Diff:2
PageRef:351
Topic:StagesofPost-ClassicalDevelopment
Skill:Conceptual

27) Wherewasthegreatestconcentrationofurbanizationafterthe10thcentury?
A) ItalyandtheLowCountries
B) EnglandandFrance
C) FranceandtheHolyRomanempire
D) EnglandandScandinavia
Answer: A
Diff:2
PageRef:351
Topic:StagesofPost-ClassicalDevelopment
Skill:Factual

243

28) HowdidtheintroductionoffeudalmonarchyintoEnglandcomparetothepolitical
experienceofFrance?
A) Englishfeudalmonarchydevelopedmoregraduallyandslowlyinresponsetothe
improvingeconomy.
B) Englishfeudalmonarchywasintroducedabruptlyafter1066,whileFrenchfeudal
monarchydevelopedmoreslowly.
C) Frenchfeudalmonarchyarosealmostimmediatelyinthe10thcenturyasaresultofthe
defeatoftheNormans.
D) Francefailedtodevelopfeudalmonarchyuntilthe15thcentury.
Answer: B
Diff:2
PageRef:352
Topic:StagesofPost-ClassicalDevelopment
Skill:Conceptual

29) BywhatcenturydidFranceachieveacompletefeudalmonarchy?
A) 10th
B) 11th
C) 13th
D) 15th
Answer: C
Diff:3
PageRef:352
Topic:StagesofPost-ClassicalDevelopment
Skill:Factual

30) WhichofthefollowingregionsachievedfeudalmonarchypriortotheendoftheMiddle
Ages?
A) HolyRomanempire
B) England
C) LowCountries
D) Spain
Answer: B
Diff:2
PageRef:352
Topic:StagesofPost-ClassicalDevelopment
Skill:Factual

244

31) WhichofthefollowingareaswasNOToneoftheregionsintowhichexpansionfromwestern
Europetookplace?
A) EasternGermanyandPoland
B) NorthernAfrica
C) Spain
D) IcelandandGreenland
Answer: B
Diff:2
PageRef:353
Topic:StagesofPost-ClassicalDevelopment
Skill:Factual

32) InwhatyeardidPopeUrbanIIcallfortheFirstCrusade?
A) 1095
B) 1130
C) 1236
D) 1453
Answer: A
Diff:3
PageRef:353
Topic:StagesofPost-ClassicalDevelopment
Skill:Factual

33) WhichofthefollowingdidNOToccurasaresultoftheCrusades?
A) TheKingdomofJerusalemwasestablishedfornearlyacenturyintheHolyLand.
B) TheFourthCrusaderesultedinthetemporaryconquestofConstantinople.
C) TheCrusadeshelpedtoopentheWesttonewculturalandeconomicinfluencesfromthe
MiddleEast.
D) TheCrusadesdemonstratedanewWesternsuperiorityinthewiderworld.
Answer: D
Diff:2
PageRef:354
Topic:StagesofPost-ClassicalDevelopment
Skill:Conceptual

34) ThereformingmonasticordersfoundedinAssisiinthe13thcenturywerecreatedby
A) St.BenedictandClovis.
B) St.ClareandSt.Benedict.
C) StFrancisandCharlemagne.
D) St.FrancisandSt.Clare.
Answer: D
Diff:3
PageRef:354
Topic:StagesofPost-ClassicalDevelopment
Skill:Factual

245

35) PopeGregoryVIIdecreedthepracticeofinvestitureinvalid.Whatwasinvestiture?
A) Thepracticewherebyaristocratsdressedinbishopsrobesandattemptedtoruleintheir
place
B) Thepracticeofstateappointmentofbishops
C) Thepracticeoftryingclericsinsecularcourts
D) Thestatespowertotaxtheclergy
Answer: B
Diff:2
PageRef:355
Topic:StagesofPost-ClassicalDevelopment
Skill:Conceptual

36) TheFourthCrusadewasmanipulatedbymerchantsinVenice,whoturneditintoanattackon
A) Constantinople.
B) Jerusalem.
C) Athens.
D) Alexandria.
Answer: A
Diff:2
PageRef:354
Topic:StagesofPost-ClassicalDevelopment
Skill:Factual

37) Inthe12thcentury,whatParisianscholar , theauthorofYesandNo, utilizedlogictoexamine


ecclesiasticaldoctrine?
A) ThomasAquinas
B) WilliamofOckham
C) PeterAbelard
D) WilliamofSt.Thierry
Answer: C
Diff:2
PageRef:356
Topic:WesternCultureinthePost-ClassicalEra
Skill:Factual

246

38) What12thcenturymonkstressedtheimportanceofmysticalunionwithGodoverlogicand
philosophy?
A) DunsScotus
B) BernardofClairvaux
C) SimeonofDurham
D) MarsiglioofPadua
Answer: B
Diff:2
PageRef:356
Topic:WesternCultureinthePost-ClassicalEra
Skill:Conceptual

39) InwhatwaywastheeducationalsystemofthemedievalWestdifferentfromthatofChina?
A) TheWestabandoneditsclassicalheritage
B) Theuniversitieswerenottiedintoasinglebureaucraticsystem
C) IntheWest,therewerenostatebureaucraciestohireuniversitygraduates
D) TheWestlackedaformalsystemofeducation
Answer: B
Diff:3
PageRef:357
Topic:WesternCultureinthePost-ClassicalEra
Skill:Conceptual

40) TheleadingfigureinthesynthesisofclassicalrationalphilosophywithChristiantheologywas
ateacherattheUniversityofParisinthe13thcentury,
A) ThomasAquinas.
B) WilliamofOckham.
C) PeterAbelard.
D) WilliamofSt.Thierry.
Answer: A
Diff:2
PageRef:357
Topic:WesternCultureinthePost-ClassicalEra
Skill:Factual

247

41) BecauseofitsbaseintheuniversitiesofwesternEurope,thedominantmedievalphilosophical
approachwasreferredtoas
A) existentialism.
B) corporatetheology.
C) socialcontracttheology.
D) scholasticism.
Answer: D
Diff:2
PageRef:357
Topic:WesternCultureinthePost-ClassicalEra
Skill:Factual

42) Duringthe11thcentury,whatnewarchitecturalstylefeaturingpointedarchesandflying
buttressesbecamedominantinwesternEurope?
A) Romanesque
B) Gothic
C) Structuralism
D) Turkic
Answer: B
Diff:1
PageRef:358
Topic:WesternCultureinthePost-ClassicalEra
Skill:Factual

43) WhichofthefollowingdevelopmentswasNOTaresultoftheimprovedeconomyoftheHigh
MiddleAges?
A) Urbangrowthallowedmorespecializedmanufacturingandcommercialactivities,
includingbanking.
B) Somepeasantswereabletothrowoffthemostsevereconstraintsofmanorialism,
becomingalmostfreefarmers.
C) RisingtradepermittedtheredevelopmentofcommercewithintheMediterraneanand
beyond.
D) Conflictsbetweenpeasantsandthelandlordsbecamerare,iftheydidnotdisappear
altogether.
Answer: D
Diff:2
PageRef:359-360
Topic:ChangingEconomicandSocialFormsinthePost-ClassicalCenturies
Skill:Conceptual

248

44) AllofthefollowingwerefunctionsofthemerchantandartisanguildsEXCEPT
A) limitationofmembership.
B) regulationofapprenticeship.
C) ensuringafreemarketeconomy.
D) guaranteeinggoodworkmanship.
Answer: C
Diff:2
PageRef:362
Topic:ChangingEconomicandSocialFormsinthePost-ClassicalCenturies
Skill:Conceptual

45) WhichofthefollowingwasNOTtrueofthecareerofJacquesCoeur?
A) Heusedhiswealthtoarrangeforhis16 -year-oldsontobecomeanarchbishop
B) HediedarichandhonoredadvisortothekingofFrance
C) Hewastortured,admittedtovariouscrimes,andhadhispropertyconfiscated
D) HehadthelargestfleeteverownedbyaFrenchsubject
Answer: B
Diff:2
PageRef:361-362
Topic:ChangingEconomicandSocialFormsinthePost-ClassicalCenturies
Skill:Conceptual

46) WhichofthefollowingwasNOTathreattothesourcesofWesternvitalityattheendofthe
MiddleAges?
A) TheBlackDeath
B) Theincreasinginabilityofagriculturetokeeppacewithpopulationgrowth
C) Theeconomictail-spin
D) Newsocialdisputes,involvingbothpeasantsandlandlordsandartisansandtheir
employees.
Answer: C
Diff:2
PageRef:364
Topic:TheDeclineoftheMedievalSynthesis
Skill:Conceptual

249

47) WhichofthefollowingwasaresultoftheHundredYearsWarduringthe14thand15th
centuries?
A) Kingsreducedtheirrelianceonfeudalforcesinfavorofpaidarmies
B) AnEnglishvictory
C) Mountedknightscontinuedtheirdominanceoverfootsoldiersandarchers
D) Majorbattlesresultedinenormouslossoflifeoverthecourseofthewar
Answer: A
Diff:2
PageRef:364
Topic:TheDeclineoftheMedievalSynthesis
Skill:Conceptual

48) WhichofthefollowingwasNOTtypicalofthechallengestomedievalinstitutionsinthe15th
century?
A) Thelandowningaristocracylostitsdominanceasthechiefmilitaryforce
B) Thebalancebetweenchurchandstatebegantofavorthedominanceofthestate
C) Themedievalintellectualandartisticsynthesiswasbreakingdown
D) AsingleimperialgovernmentreplacedthefragmentedpoliticalstructureoftheMiddle
Ages
Answer: D
Diff:2
PageRef:364-367
Topic:TheDeclineoftheMedievalSynthesis
Skill:Conceptual

49) Whichofthefollowingstatementsconcerningtheimportanceofmedievaleconomicsismost
true?
A) MedievaleconomicthoughtandpracticewasofnoconsequencetolaterWestern
economicthinkersandactors.
B) Medievaleconomicssetamuchmoredirectstageforlaterdevelopmentsthanclassical
economicshaddone.
C) Medievaleconomicssimplyrepeatedthethoughtandpracticeofearliereconomic
thinkers.
D) Medievaleconomics,overall,tendedtodiscouragemerchantactivityandtechnical
innovation.
Answer: B
Diff:3
PageRef:359-360
Topic:Conclusion:ThePost-ClassicalWestandItsHeritage
Skill:Conceptual

250

EssayQuestions
1) WhatwerethepositivesignsofvitalityassociatedwiththeMiddleAges?
Answer: Growthofpopulation,economicproductivity(capitalism);selecteduseof
Greco-Romanheritage;developmentofnewpoliticalandsocialstructures(feudalism);
intellectualcontributionsoftenassociatedwithcreationofWestern-styleuniversities;
newarchitecturalforms(Gothic).
Diff:2
PageRef:323-324
Topic:TheFlavoroftheMiddleAges
Skill:Conceptual

2) Definemanorialismandfeudalism.Howdotheyprovidethebuildingblocksformedieval
politicalstructureandsociety?
Answer: Manorialism:systemthatdescribedeconomicandpoliticalrelationshipsbetween
landlordsandpeasantlaborers.Serfsreceivedprotectionandjusticefromlordsin
returnforlaborandportionofproduce.Feudalism:seriesofrelationshipsbetween
membersofmilitaryelite;greaterlordsprovidedprotectionandlandtovassalsin
returnformilitaryserviceandloyalty.Manorialismprovidedcontextforlocal
communitylife,regionalizedandlocalformsofgovernment;relationshipsamong
landlordsledtobuildingpoliticalblocksofpowerbeyondlocalgovernment.
Diff:2
PageRef:323,327
Topic:StagesofPost-ClassicalDevelopment
Skill:Conceptual

3) WhatwerethecharacteristicsoffeudalmonarchyasdemonstratedinFranceandEngland
between1000and1300?
Answer: France:slowandgradualdevelopmentoffeudalmonarchy;gradualgrowthof
bureaucracyandcourtspecializationupto13thcentury;England:moreimmediate
developmentofcentralizedgovernmentfollowingNormanConquestin1066;
establishedcentralgovernmentwithsheriffsaslocaladministrators;Franceresponded
in13thcenturywithdevelopmentoftaxation,courtsystemtosupportmilitaryaction
againstEnglish.
Diff:2
PageRef:327-328
Topic:StagesofPost-ClassicalDevelopment
Skill:Conceptual

4) HowdidthetheologicaloutlookofwesternEuropechangebetween1000and1400?
Answer: After1000developmentofuniversityledtointellectualinnovation;PeterAbelard
introduceduseoflogictoconsiderdoctrinalissues;resistancefromthoselikeBernardof
Clairvauxwhofavoredmysticismoverrationalism(similartotensionsinIslam);
introductionofclassicalauthorsduring12thcenturyledtogreatersynthesisof
rationalismandtheology;greatestsynthesisachievedbyThomasofAquinas( Summa);
after13thcenturyscholasticismdeteriorated.
Diff:2
PageRef:330-334
Topic:WesternCultureinthePost-ClassicalEra
Skill:Conceptual

251

5) Whatwerethedevelopmentsthatledtoincreasesinmonarchicpowerattheendofthe
MiddleAges?Howwasroyalauthoritylimited?
Answer: Increases:developmentofsmallnationalarmies;growthoftrainedbureaucracies;
abilitytotax;centralizationoflegalcodesandcourtsystems.Limitations:church could
excommunicatekings,limitpowerofcourts;aristocratsdemandedreciprocalauthority
structure;parliamentscreatedin13thcentury,institutionalizedprincipleof
consultation,gainedrighttoapprovetaxation.
Diff:2
PageRef:324-328
Topic:ChangingEconomicandSocialFormsinthePost-ClassicalCenturies
Skill:Conceptual

252

Chapter16 TheAmericasontheEveofInvasion
Multiple-ChoiceQuestions
1) Duringthepostclassicalperiod,societiesintheAmericas
A) remainedinrelativeisolationfromtheothercentersofworldhistory.
B) experiencedtheinitialcontactsthatledeventuallytoEuropeaninvasionoftheNew
World.
C) failedtodevelopimperialformsofgovernment, afailurethatmirroredEuropeansociety.
D) wereunitedunderasinglegovernment.
Answer: A
Diff:2
PageRef:346
Topic:Introduction
Skill:Conceptual

2) WhichofthefollowingwasNOTacharacteristicofAzteccivilizationduringthepostclassic
period?
A) Largecitiesbasedonelaboratepoliticalandeconomicorganization
B) Elaborateculturalsystems
C) Monotheism
D) Highlydevelopedagriculture
Answer: C
Diff:1
PageRef:348
Topic:Introduction
Skill:Conceptual

3) ThenorthernnomadicpeopleswhoenteredcentralMexicofollowingthedeclineof
TeotihuacanandestablishedacapitalatTulain968werethe
A) Aztecs.
B) Incas.
C) Olmecs.
D) Toltecs.
Answer: D
Diff:2
PageRef:346
Topic:PostclassicMesoamerica,1000- 1500
Skill:Factual

253

4) TheAzteccapitalwasestablishedat
A) Teotihuacan.
B) Tula.
C) Tenochtitlan.
D) Twantinsuyu.
Answer: C
Diff:2
PageRef:348
Topic:PostclassicMesoamerica,1000- 1500
Skill:Factual

5) WhatwastherelationshipbetweentheToltecsandtheirpredecessorsincentralMexico?
A) TheformerresidentsofcentralMexicowerewipedoutduringtheToltecinvasions.
B) TheentryoftheToltecsintocentralMexicomarksanabruptbreakinthecultural
developmentoftheregion.
C) TheToltecsadoptedtheanimisticreligionoftheirpredecessors,butfailedtodevelop
citiesorceremonialcenters.
D) TheToltecsadoptedmanyculturalfeaturesfromtheirpredecessorstowhichtheyadded
astrongmilitaryethicandhumansacrifice.
Answer: D
Diff:2
PageRef:347
Topic:PostclassicMesoamerica,1000- 1500
Skill:Conceptual

6) HowdidtheAztecsviewtheculturalachievementsoftheToltecs?
A) Asbarbarianswholackedculture
B) Asslaves,fitonlyforconquest
C) Asthegiversofcivilization
D) Asheretics,whopracticedaforbiddenreligion
Answer: C
Diff:2
PageRef:346-347
Topic:PostclassicMesoamerica,1000- 1500
Skill:Conceptual

254

7) TheToltecEmpirelasteduntilaboutwhatdate?
A) 1000
B) 1150
C) 1236
D) 1434
Answer: B
Diff:2
PageRef:347
Topic:PostclassicMesoamerica,1000- 1500
Skill:Factual

8) AfterthesackofTula,thecenterofpopulationandpoliticalpowerinMexicoshiftedto
A) Yucatan.
B) thevalleyofMexicoandtheshoresofachainoflakesinthatbasin.
C) Teotihuacan.
D) Chimor.
Answer: B
Diff:1
PageRef:347
Topic:PostclassicMesoamerica,1000- 1500
Skill:Factual

9) ThepeoplewhosucceededtheToltecsastherulersofcentralMexicowerethe
A) Olmecs.
B) Maya.
C) Incas.
D) Aztecs.
Answer: D
Diff:1
PageRef:347
Topic:PostclassicMesoamerica,1000- 1500
Skill:Factual

10) AroundwhatyeardidtheAztecsmigratetothecentralvalleyofMexico?
A) 1000
B) 1150
C) 1325
D) 1434
Answer: C
Diff:2
PageRef:347
Topic:PostclassicMesoamerica,1000- 1500
Skill:Factual

255

11) IntheperiodshortlyafterthearrivaloftheAztecsinthevalleyofMexico,whatwasthe
natureofthepoliticalorganizationoftheregion?
A) Imperial
B) Regionalkingdoms
C) City-states
D) Huntingandgatheringgroups
Answer: C
Diff:3
PageRef:348
Topic:PostclassicMesoamerica,1000- 1500
Skill:Factual

12) WhichofthefollowingwasNOToneofthecitieschallengingfordominanceatthetimeofthe
AztecarrivalinthevalleyofMexico?
A) Teotihuacan
B) Azcapotzalco
C) Culhuacan
D) Texcoco
Answer: A
Diff:2
PageRef:348
Topic:PostclassicMesoamerica,1000- 1500
Skill:Factual

13) WhatcitydidtheAztecsestablishca.1325onamarshyislandinLakeTexcoco?
A) Teotihuacan
B) Culhuacan
C) Texcoco
D) Tenochtitlan
Answer: D
Diff:2
PageRef:348
Topic:PostclassicMesoamerica,1000- 1500
Skill:Factual

256

14) InwhatyeardidTenochtitlanemergeasthedominantforceinthecentralplateauthatit
controlledalongwithTexcocoandTlacopan?
A) 1100
B) 1234
C) 1325
D) 1434
Answer: D
Diff:2
PageRef:348
Topic:PostclassicMesoamerica,1000- 1500
Skill:Factual

15) WhichofthefollowingdidNOToccurasaresultoftheAztecrisetopower?
A) Theincreasingdominanceofthenobility
B) Theemergenceofarulerwithsupremepowers
C) Theabandonmentofhumansacrifice
D) ThesubsequentexpansionofpowerandtheboundariesofAzteccontrol
Answer: C
Diff:2
PageRef:348
Topic:PostclassicMesoamerica,1000- 1500
Skill:Conceptual

16) WhatwastheimpactofexpansionandconquestontheAztecsocialsystem?
A) Aztecsocietybecamemorehierarchical.
B) ConquestopenedupAztecsocietytoincursionsbytheindigenouspeopleswhobeganto
formatrainedbureaucracy.
C) AztecsocietywastransformedinthesensethattheMexicaadoptedthesocialpatternsof
theMaya.
D) Despitethestressofwarfareandinvasion,theAztecsocietyremainedremarkably
unchangedbytheprocess.
Answer: A
Diff:2
PageRef:351
Topic:PostclassicMesoamerica,1000- 1500
Skill:Conceptual

257

17) Tlalocwasthegodof
A) corn.
B) love.
C) rain.
D) creation.
Answer: C
Diff:2
PageRef:349
Topic:PostclassicMesoamerica,1000- 1500
Skill:Factual

18) WhichofthefollowingstatementsconcerningAztecreligionismostaccurate?
A) Shortlyafterestablishingtheirempire,theAztecsabandonedallgodsotherthantheir
patron,Huitzilopochtli.
B) Aztecdeitieswerenormallyassociatedonlywithmaleforms.
C) Aztecdeitieswerenumerousandhaddifferentformsormanifestationssomewhatlike
theavatarsoftheHindudeities.
D) TherewaslittleornoanimisminthereligiousworldoftheAztecs.
Answer: C
Diff:2
PageRef:349
Topic:PostclassicMesoamerica,1000- 1500
Skill:Conceptual

19) WhichofthefollowingwasNOToneofthemajorthemesorcultsofAztecreligion?
A) Godsoffertilityandtheagriculturalcycle
B) Creatordeities
C) Godsofwarfareandsacrifice
D) Deitiesdevotedtocontemplationandsalvation
Answer: D
Diff:1
PageRef:349
Topic:PostclassicMesoamerica,1000- 1500
Skill:Conceptual

20) ThecentralfigureofthecultofhumansacrificeandthemostsacreddeityoftheAztecswas
A) Quetzalcoatl.
B) Tlaloc.
C) Tonatiuh.
D) Huitzilopochtli.
Answer: D
Diff:2
PageRef:349
Topic:PostclassicMesoamerica,1000- 1500
Skill:Factual

258

21) WhatwastheAztecviewofhistory?
A) TheybelievedinalinearviewofhistorydedicatedtothepremiseofAztecsuperiority
foreternity.
B) LikeotherMesoamericanpeoples,theAztecsbelievedinacyclicalpatternofrepetitive
destructionsoftheworld.
C) UnlikeotherMesoamericanpeoples,theAztecsrejectedthecyclicalviewofhistoryfora
moremodernhistoricalviewbasedonthehistoryoftheirempire.
D) Becausetheylackedacalendarsystem,theAztecshadnoformalhistoricalviewpoint.
Answer: B
Diff:2
PageRef:350
Topic:PostclassicMesoamerica,1000- 1500
Skill:Conceptual

22) WhichofthefollowingstatementsconcerningtheAzteccapitalofTenochtitlanisNOT
accurate?
A) Itwasbuiltonanislandinthemiddleofalake
B) By1520itprobablyhadapopulationofabout150,000
C) Despitetheexistenceofthousandsofhousesandothervarietiesofdomesticarchitecture,
unlikeotherMesoamericancitiesitlackedtemplesandpalaces
D) ThecitywasaslargeascontemporaryParis
Answer: C
Diff:2
PageRef:344
Topic:PostclassicMesoamerica,1000- 1500
Skill:Conceptual

23) TheAztecinnovationinintensiveagricultureintheaquaticenvironmentsofthelakesof
centralMexicowasthedevelopmentof
A) chinampas.
B) pipiltin.
C) quipus.
D) calpulli.
Answer: A
Diff:2
PageRef:351
Topic:PostclassicMesoamerica,1000- 1500
Skill:Factual

259

24) WhatwasthenatureoftheAztececonomy?
A) TheAztecsfailedtodevelopamerchantclass,soalldistributionofgoodswascarried
outbythestate.
B) TheAztecsdevelopedafreemarketeconomyinwhichalltradewasinthehandsof
specializedmerchants.
C) TheAztecstateredistributedmanygoodsreceivedastribute,buttherewasaspecialized
merchantclassthatalsohandledlong-distancetradeinrarecommodities.
D) TherewaslittletradewithinAztecsociety,asalmostallcommunitieswere
self-sufficient.
Answer: C
Diff:2
PageRef:351
Topic:PostclassicMesoamerica,1000- 1500
Skill:Conceptual

25) OneofthemostpermanentfeaturesofAztecsocietywasitsorganizationintoclansor
A) chinampas.
B) pipiltin.
C) quipus.
D) calpulli.
Answer: D
Diff:2
PageRef:351
Topic:AztecSocietyinTransition
Skill:Factual

26) Amerchantclasswhichspecializedinlong-distancetradeinluxuryitemswasreferredtoas
the
A) chinampas.
B) pochteca.
C) quipus.
D) calpulli.
Answer: B
Diff:2
PageRef:351
Topic:AztecSocietyinTransition
Skill:Factual

260

27) WhichofthefollowingisNOTtrueoftheAztecviewofmarriageandthefamily?
A) Virginityatmarriagewashighlyregardedforyoungwomen
B) Polygamywascommonthroughoutsociety
C) Marriageswereoftenarrangedbetweenlineages
D) Younggirlswereoftentrainedbytheolderwomenofthecalpulli
Answer: B
Diff:2
PageRef:353
Topic:AztecSocietyinTransition
Skill:Conceptual

28) WhilethepositionofAztecwomeninmanywaysparalleledthatofwomeninother
civilizationsatasimilarstageofdevelopment,whatwasthesignificantdifferencebetweenthe
lifeofwomeninMesoamericaandintheMediterraneanworld?
A) WomeninMesoamericaparticipatedfullyinthemilitary.
B) TherewasnopolygamypracticedinMesoamerica.
C) Aztecwomenwereunabletoinheritortopasspropertyontoheirs.
D) ThelimitedtechnologyofMesoamericaconfinedwomentomanymorehoursgrinding
grainforfood.
Answer: D
Diff:2
PageRef:353
Topic:AztecSocietyinTransition
Skill:Conceptual

29) WhatwasthenatureoftheAztecadministrationofsubjectterritories?
A) TheAztecsplacedmembersoftheAztecnobilityasrulersoversubjectpeoples.
B) AllterritoriesconqueredbytheAztecsbecamepartofasingularadministrationrunbya
trainedbureaucracylocatedinTenochtitlan,muchliketheByzantineEmpire.
C) Conqueredterritorieswereoftenleftrelativelyunchangedundertheiroldrulersaslong
astheyrecognizedAztecsupremacyandpaidtribute.
D) TheAztecsestablishedamilitaryadministrationwithsubjectterritoriescontrolledby
regionalgenerals.
Answer: C
Diff:2
PageRef:353
Topic:AztecSocietyinTransition
Skill:Conceptual

261

30) WherewastheIncaEmpirebased?
A) Mexico
B) CentralAmerica
C) TheAndeanhighlands
D) Brazil
Answer: C
Diff:2
PageRef:355
Topic:AztecSocietyinTransition
Skill:Conceptual

31) WhatwasthecapitalofthecoastalkingdomonChimor?
A) Cusco
B) Tenochtitlan
C) LaPaz
D) Chan-Chan
Answer: D
Diff:2
PageRef:355
Topic:AztecSocietyinTransition
Skill:Conceptual

32) WhatwastheprimarydifferenceinthepoliticalsituationbetweentheMesoamericanand
Andeanzonesfollowingthebreakupoftheclassicalstates?
A) InMesoamericatherewasnorealpoliticaldeclineasnewpeoplessimplytookoverthe
institutionsoftheirpredecessors.
B) IntheAndeanzonesanumberofrelativelylargestatescontinuedtobeimportant,rather
thanthebreakdownofpowerthatwastypicalofMesoamerica.
C) TheAndeanpoliticalexperiencelackedthemilitaristicovertonesthataccompaniedthe
breakdownofpowerinMesoamerica.
D) ThetransitionofpowerthattookplaceintheAndeanzonewasaccomplishedby
invasionfromoutsidetheregion.
Answer: B
Diff:2
PageRef:355
Topic:Twantinsuyu:WorldoftheIncas
Skill:Conceptual

262

33) ThemostpowerfuloftheAndeanstatesbetween900and1465followingthedeclineof
TihuanacoandHuariwas
A) Chimor.
B) Titicaca.
C) TopacYupanqui.
D) ChichenItza.
Answer: A
Diff:2
PageRef:355
Topic:Twantinsuyu:WorldoftheIncas
Skill:Factual

34) ThecapitaloftheIncaEmpirewas
A) Cuzco
B) ChanChan
C) Tihuanaco
D) ChichenItza
Answer: A
Diff:1
PageRef:356
Topic:Twantinsuyu:WorldoftheIncas
Skill:Factual

35) TheIncarulerassociatedwiththefirstcreationoftheIncaEmpirein1438was
A) TopacYupanqui.
B) Ahuitzotl.
C) Pachacuti.
D) MoctezumaII.
Answer: C
Diff:2
PageRef:356
Topic:Twantinsuyu:WorldoftheIncas
Skill:Factual

36) WhatdotheauthorssuggestwastheprincipalreasonforIncaconquestandexpansion?
A) Theirneedforhumansacrificialvictims
B) Thepracticeofsplitinheritance
C) TheabsenceofirrigationsystemswithintheIncaEmpire
D) Theexistenceoflong-distancetrademerchantswithintheimperialadministration
Answer: B
Diff:2
PageRef:356
Topic:Twantinsuyu:WorldoftheIncas
Skill:Conceptual

263

37) WhatwastheIncapracticeofsplitinheritance?
A) Onthedeathofthepreviousruler,thethronepassedtotwodescendantsfromtherulers
family.
B) Onthedeathofthepreviousruler,thefamilyswealthwasequallydividedamongall
maleheirs.
C) Onthedeathofthepreviousruler,theinheritancepassedthroughthefamilyofthe
seniorwifetoheroldestbrother.
D) Allpoliticalpowerandtitleswenttotherulerssuccessor,buthiswealthwaskeptinthe
handsofthemaledescendantstosupportthecultofthedeadIncasmummy.
Answer: D
Diff:2
PageRef:356
Topic:Twantinsuyu:WorldoftheIncas
Skill:Conceptual

38) ThereligiouspracticemostcloselyassociatedwiththestateandthepersonoftheIncain
Andeancivilizationwasthecultof
A) humansacrifice.
B) theraingod.
C) thesun.
D) Quetzalcoatl.
Answer: C
Diff:2
PageRef:356
Topic:Twantinsuyu:WorldoftheIncas
Skill:Factual

39) WhichofthefollowingwasutilizedintheIncaEmpire,butNOTbytheAztecs?
A) Asemi-divineemperor
B) Extensiveuseofcolonization
C) Useoflocalrulersinexchangeforrecognitionofsovereignty
D) Identificationofthenobilitywiththeadministrativeandmilitaryfunctionsofthestate
Answer: B
Diff:2
PageRef:358
Topic:Twantinsuyu:WorldoftheIncas
Skill:Conceptual

264

40) WhichofthefollowingexistedintheAztecEmpire,butNOTintheIncaEmpire?
A) Atributesystem
B) Redistributionoftributebythestate
C) Extractionoflaborforuseontempleprojects
D) Anextensivemerchantclass
Answer: D
Diff:2
PageRef:358-359
Topic:Twantinsuyu:WorldoftheIncas
Skill:Conceptual

41) WhatwastheAndeanprincipleofinheritance?
A) Matrilineal,inheritancepassinginthefemaleline
B) Patrilineal,inheritancepassinginthemaleline
C) Paralleldescent,orinheritancepassinginboththemaleandfemalelines
D) Everythingpassingtothemonarch
Answer: C
Diff:2
PageRef:358
Topic:Twantinsuyu:WorldoftheIncas
Skill:Factual

42) Tamboswere
A) laborturnsexactedbytheIncaauthorities.
B) knottedstringsusedforrecordkeeping.
C) localrulers.
D) waystationsalongroadsintheIncaEmpire.
Answer: D
Diff:2
PageRef:358
Topic:Twantinsuyu:WorldoftheIncas
Skill:Factual

43) TheIncanobilitywere
A) drawnfrom10royalayllusandthecityofCuzco.
B) drawnfromthenobleayllusoftheconqueredpopulation.
C) notdistinguishedfromthecommonersbyappearanceordress.
D) oftencommonerswhodistinguishedthemselvesinbattle.
Answer: A
Diff:2
PageRef:359
Topic:Twantinsuyu:WorldoftheIncas
Skill:Conceptual

265

44) WhichofthefollowingrepresentsaculturaldifferencebetweentheIncasandAztecs?
A) Monumentalarchitecture
B) Nopracticaluseofthewheel
C) Lackofawritingsystem
D) Beautifulpotteryandcloth
Answer: C
Diff:2
PageRef:360-361
Topic:Twantinsuyu:WorldoftheIncas
Skill:Conceptual

45) Intermsoftheintegrationofacentralizedempire,howdidtheIncasandAztecscompare?
A) TheAztecsandIncasbothmadelittleattempttointegrateconqueredterritoriesand
permittedself-ruleinreturnforloyaltyandtribute.
B) TheAztecsweremoreadvancedthantheIncasinconsolidatingtheirgovernmentintoan
integratedunit.
C) TheIncasattemptedtocreateanover-archingpoliticalstateandmadeconscious
attemptstointegratetheirempireasaunit,whiletheAztecsdidlessinthisregard.
D) BoththeIncasandtheAztecscreatedfullyintegratedempirescompletewithcentral
bureaucraciesandmilitaryadministrativeunitsthatcontrolledallconqueredregions.
Answer: C
Diff:2
PageRef:361
Topic:Twantinsuyu:WorldoftheIncas
Skill:Conceptual

46) WhichofthefollowingrepresentsasignificantdifferencebetweentheAndeanand
Mesoamericancivilizations?
A) Climateandgeography
B) Kin-basedsocialgroups
C) Sedentaryagriculturalsystems
D) Theexistenceofanobility
Answer: A
Diff:2
PageRef:361-362
Topic:Twantinsuyu:WorldoftheIncas
Skill:Conceptual

266

47) WhichofthefollowingstatementsaboutthepopulationoftheAmericasismosttrue?
A) ThepopulationoftheAmericasiseasytocalculate.
B) NorthAmericawasmoredenselypopulatedthanMesoamericaortheAndes.
C) ThepopulationoftheAmericaswasnearlythesameasthatofcontemporaryEurope
(notincludingRussia).
D) Theearly20th-centuryestimateof8.4millionstillseemsthemostaccurate.
Answer: C
Diff:2
PageRef:362
Topic:TheOtherIndians
Skill:Conceptual

48) By1500,agriculturewas
A) largelydiffusedthroughouttheAmericas,althoughnotalwaysinsedentaryagricultural
communities.
B) virtuallyunknownoutsideoftheAndeanandMesoamericancivilizationzones.
C) practicedinsedentarycommunitiesthroughouttheAmericas.
D) practicedinsedentarycommunitiesandbynomadicpastoraliststhroughoutthe
Americas.
Answer: A
Diff:2
PageRef:363
Topic:TheOtherIndians
Skill:Conceptual

49) WhichofthefollowingdoesNOTrepresentacharacteristicofmostIndiansocietiesotherthan
theAndeanandMesoamericancivilizations?
A) Stronglykin-basedsocieties
B) Wealthasthebasisofsocialranking
C) Communalactionandownershipofresources
D) Importantsocialandpoliticalrolesforwomen
Answer: B
Diff:2
PageRef:363
Topic:TheOtherIndians
Skill:Conceptual

267

EssayQuestions
1) InwhatwaysdidtheAztecscontinuethecultureoftheclassicalMesoamericancivilization
andtheToltecs?
Answer: Toltecsconsideredgiversofcivilization;sharedsamelanguage;useofhumansacrifice;
establishmentofempirecenteredoncentralMexico;militarismofsociety;conceptof
nobilitytiedtoTolteclineageinitially;useofcity-stateorganization;templecomplexes
associatedwithstate;manydeitiesofpantheonofgods(Tlaloc,Quetzalcoatl);tribute
basedonsedentaryagriculturalsystem;cyclicalviewofhistoryandcalendarsystem.
Diff:2
PageRef:346-348
Topic:PostclassicMesoamerica,1000- 1500
Skill:Conceptual

2) DescribeAztecsocialorganization.
Answer: Attopwasemperorwhowasheldtobesemi-divine;nobilityorpipiltindevelopedafter
earlyconquest,separatedthemselvesfromclangroups(calpulli),associatedwith
priesthoodandmilitary;largemassofcommonersgroupsin calpulli,landdistributedby
clanheads,providedtribute,labortotemples;classofserfsassociatedwithlandsof
nobility;scribes,artisans,healers;long-distancemerchants(pochteca).
Diff:2
PageRef:351-353
Topic:AztecSocietyinTransition
Skill:Conceptual

3) CompareandcontrasttheIncaandAztecempiresintermsofpoliticaladministration.
Answer: Similarities:eachhademperorsupportedbynobilitythatservedaspersonnelofstate;
bothbasedontributesystemwithimperialredistributionofgoods;bothwere
militaristic;eachrecognizedindigenousrulersinreturnforrecognitionofimperial
sovereignty.Differences:IncaEmpiremoreintegrated;AztecEmpirebasedmoreon
conceptofcity-states;AztecEmpiremoreopentotrade;IncaEmpirealmostentirely
reliedonstateredistributionofgoods;Aztecuseofhumansacrificeasweaponof
politicalterror.
Diff:2
PageRef:353-363
Topic:Twantinsuyu:WorldoftheIncas
Skill:Conceptual

4) HowdidtheIndianculturesoutsidetheAndeanandMesoamericancivilizationzonescontrast
inpoliticalandsocialorganizationwiththeAztecandIncaempires?
Answer: Lackofstateformation;existenceofalllevelsofsocialcomplexityfromlargechiefdoms
tohuntingandgatheringgroups;greaterrelianceonstrictlykin -basedsocial
organization;tendencytocommunalownershipofresources;wealthnotabasisfor
socialstatus;womenheldpositionsofgreaterpoliticalandsocialimportance;less
dependenceonsedentaryformsofagriculture;vastlylessdemographicdensity;lackof
monumentalarchitecture(withsomeexceptions).
Diff:2
PageRef:362-364
Topic:TheOtherIndians
Skill:Conceptual

268

5) Howhavehistoriansexplainedtheexistenceandprevalenceofhumansacrificeamongthe
Aztecs?
Answer: ItwasgreatlyexaggeratedbytheSpanishasameansofvalidatingEuropeanconquest
andculturalsuperiority;itwasareligiousactessentialtothegrantofrain,sun,and
otherblessingsofthegods;itwasanintentionaluseofawidespreadpracticeto
terrorizetheirneighborsandtokeepthelowerclassessubordinate;itwasaformof
populationcontroltolowerpopulationdensity;itwasaresponsetoalackofprotein
andtheabsenceoflargemammalsassociatedwithanimalsacrifice.
Diff:2
PageRef:360-361
Topic:PostclassicMesoamerica,1000- 1500
Skill:Conceptual

269

Chapter17 ReunificationandRenaissanceinChinese
Civilization:TheEraoftheTangand
SongDynasties
Multiple-ChoiceQuestions
1) WhatwasthecapitaloftheSongrulers?
A) Peking
B) Hangzhou
C) Jurchen
D) Xian
Answer: B
Diff:2
PageRef:366
Topic:Introduction
Skill:Factual

2) HowdidWendiwinwidespreadsupport?
A) Byincreasingtaxes
B) Byloweringtaxesandestablishinggranariesthroughouthisdomain
C) BysupportingtheChoudynasty
D) BymovingthecapitaltoBeijing
Answer: B
Diff:2
PageRef:369
Topic:Introduction
Skill:Conceptual

3) Thedynastythatemergedattheendofthe6thcenturywasthe
A) Tang.
B) Chou.
C) Song.
D) Sui.
Answer: D
Diff:1
PageRef:367
Topic:RebuildingtheImperialEdificeintheSui-TangEra
Skill:Factual

270

4) WhokilledWendi?
A) Hisson,Yangdi
B) TheChou
C) ABuddhistmonk
D) NomadicpeopleswhoconquerednorthChina
Answer: A
Diff:2
PageRef:369
Topic:Introduction
Skill:Conceptual

5) WhatmadepossibletherapidrevivalofempireundertheTang?
A) TheabandonmentofConfucianisminfavorofthemorewidelypracticedBuddhism
B) Thebrevityoftheperiodofpoliticaldislocation
C) ThewillingnessoftheTangtoabandontraditionalapproachestogovernment
D) ThepreservationinthemanykingdomsoftheConfuciantraditionsthathadbeencentral
toChinesecivilization
Answer: D
Diff:2
PageRef:370
Topic:Introduction
Skill:Conceptual

6) ThemanresponsibleforthecreationoftheSuidynastywas
A) Xuanzong.
B) YangGuifei.
C) Wendi.
D) LiYuan.
Answer: C
Diff:2
PageRef:367
Topic:RebuildingtheImperialEdificeintheSui-TangEra
Skill:Factual

7) WhatmadethereunificationofChinaunderthefirstSuiemperorpossible?
A) Thesupportofthenomadicwarriorelite
B) ThesupportoftheethnicChinesearistocracy
C) ThesupportoftheBuddhistmonasteries
D) ThesupportoftheConfucianscholar-gentry
Answer: A
Diff:2
PageRef:368
Topic:RebuildingtheImperialEdificeintheSui-TangEra
Skill:Conceptual

271

8) WhatwastheprimaryreformenactedduringthereignofthefirstSuiemperor?
A) ThereconstructionoftheConfucianscholar-gentry
B) PersecutionoftheBuddhists
C) Thecreationofgranariestorelievethethreatoffamine
D) TheconstructionoftheGrandCanal
Answer: C
Diff:2
PageRef:369
Topic:RebuildingtheImperialEdificeintheSui-TangEra
Skill:Factual

9) Unlikehisfather,thesecondSuiemperorfavored
A) theConfucianscholar-gentry.
B) thegreatChinesearistocraticfamilies.
C) theBuddhists.
D) nomadicelementsinnorthernChina.
Answer: A
Diff:2
PageRef:369
Topic:RebuildingtheImperialEdificeintheSui-TangEra
Skill:Conceptual

10) WhatledtothedownfalloftheSuidynasty?
A) Nomadicinvasions
B) Excessiveexpensesassociatedwithgrandiosebuildingprojectsandmilitarycampaigns
C) WidespreadBuddhistrebellion
D) ThedissatisfactionoftheConfucianscholar-gentry
Answer: B
Diff:2
PageRef:369
Topic:RebuildingtheImperialEdificeintheSui-TangEra
Skill:Conceptual

11) WhowasthefounderoftheTangdynasty?
A) Yangdi
B) LiBo
C) LiYuan
D) ZhaoKuangyin
Answer: C
Diff:2
PageRef:369
Topic:RebuildingtheImperialEdificeintheSui-TangEra
Skill:Factual

272

12) WhichofthefollowingstatementsconcerningtheextentoftheTangEmpireismostaccurate?
A) TheTangextendedtheempireinalldirectionsexceptwestward,wheretheTurks
remainedentirelyindependentoftheChineseemperor.
B) TheTangEmpirewasunabletorecovertheterritorialextentoftheHan,butdidrecover
northernareasfromthenomads.
C) TheTangEmpireincorporatedIndiaandsoutheastAsia,aswellastheareasnorthofthe
YellowRiverplain.
D) TheTangbuiltanempirethatwasfarlargerthanthatoftheearlyHan,anempirewhose
boundariesinmanydirectionsextendedbeyondthebordersofmodernChina.
Answer: D
Diff:2
PageRef:370
Topic:RebuildingtheImperialEdificeintheSui-TangEra
Skill:Conceptual

13) WhatwastheattitudeoftheTangemperorstowardtheConfucianscholar -gentry?


A) TheTangcontinuedtosupportandpatronizethegrowthofBuddhisminChinaatthe
expenseoftheConfucianscholar-gentry.
B) TheTangsupportedtheresuscitationoftheConfucianscholar-gentry,oftenatthe
expenseofthearistocracy.
C) TheTangfearedthedevelopmentofthescholar-gentryandcontinuedtosupportthe
nomadicaristocracyofChina.
D) ConfucianismcontinuedtowaneduringtheTangdynastyandwasonlyresuscitated
undertheSong.
Answer: B
Diff:2
PageRef:370
Topic:RebuildingtheImperialEdificeintheSui-TangEra
Skill:Conceptual

14) Whatministryofthecentralimperialgovernmentwasresponsiblefortheadministrationof
theexaminationsystem?
A) Rites
B) Publicworks
C) Justice
D) War
Answer: A
Diff:2
PageRef:370
Topic:RebuildingtheImperialEdificeintheSui-TangEra
Skill:Factual

273

15) Thetitleofjinshiwasreservedforthosewho
A) weremembersoftheChinesearistocracy.
B) servedintheChinesemilitary.
C) weremembersoftheimperialfamily.
D) passedthemostdifficultexamsonallofChineseliterature.
Answer: D
Diff:2
PageRef:371
Topic:RebuildingtheImperialEdificeintheSui-TangEra
Skill:Factual

16) WhichofthefollowingstatementsconcerningentryintotheChinesebureaucracyismost
accurate?
A) Althoughahigherpercentageofcandidatesreceivedofficethroughtheexamination
systemthanduringtheHandynasty,birthcontinuedtobeimportantinsecuringhigh
office.
B) UndertheTangfamilyconnectionsceasedtobeofsignificance,asallcandidates
receivedofficebasedontheirscoreintheexaminationsystem.
C) Althoughtheexaminationsystemcontinuedtobemonitored,almostallofficialreceived
positionsasaresultoffamilyconnections.
D) TheexaminationsystemwaseliminatedduringtheTangdynasty,andonlymembersof
theimperialfamilyservedinthebureaucracy.
Answer: A
Diff:2
PageRef:371
Topic:RebuildingtheImperialEdificeintheSui-TangEra
Skill:Conceptual

17) MembersoftheeliteclassofChinaweremoreattractedtowhatvariantofBuddhism?
A) Legalist
B) PureLand
C) Chan
D) Daoist
Answer: C
Diff:2
PageRef:372
Topic:RebuildingtheImperialEdificeintheSui-TangEra
Skill:Factual

274

18) WhichofthefollowingdescriptionsofPureLandBuddhismisNOTaccurate?
A) ItwasastrainofMahayanaBuddhism
B) Itstressedmeditationandtheappreciationofnaturalandartisticbeauty
C) ItappealedtomembersofthemassofChinesecommoners
D) ItwasasalvationistvariantofBuddhism
Answer: B
Diff:3
PageRef:372
Topic:RebuildingtheImperialEdificeintheSui-TangEra
Skill:Conceptual

19) WhatTangruleractuallyattemptedtohaveBuddhismrecognizedasastatereligion?
A) Yangdi
B) Wendi
C) Gaozu
D) EmpressWu
Answer: D
Diff:3
PageRef:372
Topic:RebuildingtheImperialEdificeintheSui-TangEra
Skill:Factual

20) WhatprovedtobethemostdamagingattackonBuddhismspopularitywiththepeople
duringtheearlyTangdynasty?
A) TheBuddhistsinsistenceonrebellionagainsttheemperor
B) TheConfucianssuccessfulcampaigntoconvincetheemperorthattheBuddhist
monasticestablishmentrepresentedaneconomicthreat
C) ThearistocracysconcernthatthegrowingBuddhistmonasticestablishmentwas
monopolizinglandthatotherwisewouldbelongtothem
D) TheentryofnomadicinvaderswhowereIslamicduringthe9thcentury
Answer: B
Diff:2
PageRef:373
Topic:RebuildingtheImperialEdificeintheSui-TangEra
Skill:Conceptual

275

21) InwhatdecadedidopenpersecutionofBuddhismwithintheChineseEmpirebegin?
A) 720s
B) 790s
C) 840s
D) 910s
Answer: C
Diff:2
PageRef:373
Topic:RebuildingtheImperialEdificeintheSui-TangEra
Skill:Factual

22) WhatwastheresultoftheimperialattempttosuppressBuddhismwithintheChineseempire?
A) DespiteofficialattemptstosuppressBuddhism,monasticorderscontinuedtoexert
politicalinfluenceandcontrollandedwealthastheydidinthefirstdecadesofTangrule.
B) Buddhismsurvivedthebriefimperialpersecutionandwasrestoredastheprimary
religionofthestateduringtheSongdynasty.
C) Buddhismsurvivedtherepression,butinareducedstatewithoutthepoliticalinfluence
oftheearlyTangyears.
D) BuddhismwaseradicatedinChinabytheendoftheTangeraandreplacedbythe
traditionalDaoistandConfucianbeliefs.
Answer: C
Diff:2
PageRef:373
Topic:RebuildingtheImperialEdificeintheSui-TangEra
Skill:Conceptual

23) WhatwastheimpactonConfucianismoftheTangrepressionoftheBuddhists?
A) ConfucianismemergedasthecentralideologyofChinesecivilizationuntilthe20th
century.
B) Confucianismemergedinastrengthenedcondition,butstillremainedbehindBuddhism
inthesenseofprovidingabasisfortheintellectualrationaleofChinesecivilization.
C) BecauseConfucianscholar-gentrywereassociatedwiththepersecution,Confucianism
failedtogeneratemuchpopularsupportoutsidetheimperialgovernment.
D) Confucianism,likeBuddhism,declinedinpopularity,anditsplacewastakenby
Daoism.
Answer: A
Diff:2
PageRef:373
Topic:RebuildingtheImperialEdificeintheSui-TangEra
Skill:Conceptual

276

24) WhichofthefollowingwasNOTawomanwhogainedinfluenceandpowerduringthelater
yearsoftheTangdynasty?
A) Wu
B) Wei
C) YangGuifei
D) Xuanzong
Answer: D
Diff:2
PageRef:374
Topic:TangDeclineandtheRiseoftheSong
Skill:Factual

25) WhichofthefollowingwasNOTaproblemduringthelastyearsoftheTangdynasty?
A) TherevoltofAnLushan
B) TheentryofnomadsintolargeareasofthenorthernChinaplain
C) TheHuang-chaorebellion
D) ABuddhistrebellioninsouthernChina
Answer: D
Diff:2
PageRef:374
Topic:TangDeclineandtheRiseoftheSong
Skill:Conceptual

26) WhowasthefounderoftheSongdynasty?
A) ZhaoKuangyin
B) Xuanzong
C) LiBo,DukeofSong
D) Xi-Xia
Answer: A
Diff:2
PageRef:375
Topic:TangDeclineandtheRiseoftheSong
Skill:Factual

27) ThefirstSongemperorrestoredtheunityofChinaEXCEPTforwhatnortherndynasty?
A) Jurchen
B) Silla
C) Liao
D) Hangzhou
Answer: C
Diff:2
PageRef:375
Topic:TangDeclineandtheRiseoftheSong
Skill:Factual

277

28) HowdidtheSongEmpirecomparetotheTang?
A) TheSongEmpirewasgreaterinterritorialextentthantheTangEmpire.
B) TheSongEmpireandtheTangEmpirewerevirtuallyidenticalinterritorialextent.
C) TheSongEmpirewassmallerinterritorialextentthantheTangEmpire.
D) Althoughapproximatelythesamesize,theSongEmpireextendedfarthernorththanthe
Tang.
Answer: C
Diff:1
PageRef:375-376
Topic:TangDeclineandtheRiseoftheSong
Skill:Conceptual

29) WhataccountsfortherelativeweaknessoftheSongEmpire?
A) ItneversucceededinachievingthedegreeofcentralizationthathadtypifiedtheTang
Empire.
B) Thescholar-gentryquicklylostinfluenceundertheSong,andthebureaucracyceasedto
functioneffectively.
C) LackofagriculturalproductivityproducedageneralfailureoftheChineseeconomy
duringtheSongdynasty.
D) Themilitarywassubordinatedtothecivilianadministratorsofthescholar-gentry,
leavingthedynastyvulnerabletonomadicdynastiesonthefrontier.
Answer: D
Diff:2
PageRef:375-376
Topic:TangDeclineandtheRiseoftheSong
Skill:Conceptual

30) Whichofthefollowingstatementsmostaccuratelydescribesthepositionofthescholar-gentry
undertheSongdynasty?
A) TheConfucianscholar-gentrycontinuedtoexist,buttheexaminationsystemweakened
noticeablycausingtheimperialbureaucracytodependmoreandmoreonthe
aristocracy.
B) Theascendancyofthescholar-gentryoveritsaristocraticandBuddhistrivalswasfully
securedintheSongera.
C) TheSongemperorscametorelyincreasinglyonthemilitaryasdefenseoftheborders
placedlessemphasisontheroleofintellectualideologyandmoreonwarfare.
D) TheSongcontinuedtorelyontheConfucianscholar-gentry,butmorerigorous
examinationsystemsresultedinfewermenenteringthecivilserviceasaresultof
education.
Answer: B
Diff:2
PageRef:375-376
Topic:TangDeclineandtheRiseoftheSong
Skill:Conceptual

278

31) WhowasthemostprominentoftheNeo-ConfuciansduringtheSongera?
A) LiBo
B) YangGuifei
C) Hangzhou
D) ZhuXi
Answer: D
Diff:2
PageRef:376
Topic:TangDeclineandtheRiseoftheSong
Skill:Factual

32) WhichofthefollowingwasNOTaneventualimpactofthedevelopmentofNeo-Confucian
schoolsofthought?
A) HostilitytoforeignphilosophicalsystemslikeBuddhism
B) Thestiflingofinnovationandcriticalthinkinginfavoroftraditionalism
C) DestructionoftheMandarins
D) Reinforcementofclass,age,andgenderdistinctions
Answer: C
Diff:2
PageRef:376
Topic:TangDeclineandtheRiseoftheSong
Skill:Conceptual

33) WangAnshi,aprimeministerinthe1070sand1080s,attemptedtoreformSonggovernment
onthebasisofwhatConfucianschoolofthought?
A) Daoism
B) Shiism
C) Constructionism
D) Legalism
Answer: D
Diff:2
PageRef:376
Topic:TangDeclineandtheRiseoftheSong
Skill:Factual

279

34) WhichofthefollowingwasNOToneofthereformsintroducedbyWangAnshininthefaceof
conservativeopposition?
A) Theintroductionofgovernment-assistedirrigationprojectstoencourageagricultural
expansion
B) Taxationofthelandlordandscholarlyclasses
C) Theestablishmentofwell-trainedmercenaryforces
D) Theendoftributepaymentstonomadicdynastiesonthenorthernborder
Answer: D
Diff:2
PageRef:376-377
Topic:TangDeclineandtheRiseoftheSong
Skill:Conceptual

35) WhatcausedtheflightoftheSongdynastyfromtheircapitalinnorthernChina?
A) TheinvasionsoftheJurchenswhohadformedtheQinkingdom
B) TheHuang-chaorebellion
C) Arebellionledbyanomadicgeneral,AnLushan
D) ExtensivefloodingintheYellowRiverbasin
Answer: A
Diff:2
PageRef:377
Topic:TangDeclineandtheRiseoftheSong
Skill:Conceptual

36) WhywastheconstructionoftheGrandCanalnecessary?
A) MajorriversystemsinChinaranfromnorthtosouth,andthecanalwasnecessaryto
connectthecoastalregionswiththewesternfrontier.
B) Chinesepopulationwasincreasinglyconcentratedalongthenorthernplainsalongthe
YangziRiver.
C) TheYangziRivervalleywasbecomingthemajorfood-producingregionofChinabythe
lateTangera.
D) ThecanalconnectedtheTangcapitalsofsouthernChina, ChanganandLoyang,withthe
newlyacquiredregionsinthenorth.
Answer: C
Diff:2
PageRef:378
Topic:TangandSongProsperity:theBasisofaGoldenAge
Skill:Conceptual

280

37) WhichofthefollowingwasNOTaneconomicdevelopmentduringtheperiodofcommercial
expansionduringtheTangandSongdynasties?
A) TradeincreasinglycarriedbyChineseshipsandsailors
B) Enlargedmarketquartersfoundinallcitiesandmajortowns
C) Growingsophisticationincommercialorganizationandformsofcredit
D) Militarydominationoftheseas
Answer: D
Diff:2
PageRef:378
Topic:TangandSongProsperity:theBasisofaGoldenAge
Skill:Conceptual

38) WhichofthefollowingstatementsconcerningurbanizationinChinaduringtheTang -Song


eraismostaccurate?
A) ChineseurbanizationwaspronouncedinnorthernChina,butlaggedinsouthernChina.
B) Theimperialcapitalswereenormous,buttherewerefewotherChinesecitiesofsize.
C) ThedegreeofurbanizationreachedduringtheHandynastywasneverachievedduring
theTang-Songera,althoughsomerecoveryofcitiesaccompaniedthecommercial
revival.
D) ChineseurbanizationmushroomedduringtheTang-Songerawithahigherproportion
ofthepopulationlivingincitiesthanthatfoundinanyothercontemporarycivilization.
Answer: D
Diff:2
PageRef:379
Topic:TangandSongProsperity:theBasisofaGoldenAge
Skill:Conceptual

39) WhatportionofthepopulationlivedinlargecitiesduringtheSongdynasty?
A) 10percent
B) 20percent
C) 30percent
D) 40percent
Answer: A
Diff:1
PageRef:379
Topic:TangandSongProsperity:theBasisofaGoldenAge
Skill:Factual

281

40) WhichofthefollowingwasNOTaresultoftheagriculturalpoliciesoftheSuiandTang
emperors?
A) Numbersoffreepeasantryincreased
B) Thescholar-gentrywereremovedaslandlords
C) Fortunesoftheoldaristocraticfamiliesdeclined
D) Landsweredistributedmoreequitablytothefreepeasanthouseholdsoftheempire
Answer: B
Diff:3
PageRef:379
Topic:TangandSongProsperity:theBasisofaGoldenAge
Skill:Conceptual

41) WhichofthefollowingstatementsaboutChineselandscapepaintingismostaccurate?
A) LandscapepaintingwasvirtuallyunknownuntilaftertheMongolinvasions.
B) AsintheWest,landscapepaintingwasthespecialskillofthemilitaryelite.
C) MembersoftherulingpoliticaleliteinChinaproducedmanyofthepaintingsinthe
Songera.
D) Songsubsidiestoforeignpaintersresultedintheproductionofmagnificentlandscapes.
Answer: C
Diff:3
PageRef:383
Topic:TangandSongProsperity:theBasisofaGoldenAge
Skill:Conceptual

42) InwhatwaydidfootbindingservetodiminishtheindependenceofChinesewomenbythe
endoftheSongera?
A) Becausefootbindingcouldonlybeaffordedbytheelite,poorerwomenwereassignedto
alowersocialstatus.
B) Footbindingsufficientlycrippledwomentoeffectivelyconfinetheirmobilitytotheir
household.
C) Asfootbindingwasrequiredinordertopracticecertainprofessions,Chinesewomen
foundthatoccupationalalternativeswerediminished.
D) Footbinding,althoughconsideredsociallyattractive,wascondemnedby
Neo-ConfucianswhousedthepracticeasameansofrelegatingChinesewomento
subordinateroles.
Answer: B
Diff:2
PageRef:382
Topic:TangandSongProsperity:theBasisofaGoldenAge
Skill:Conceptual

282

43) WhichofthefollowingwasNOTatechnologicalinnovationoftheTang -Songera?


A) Gunpowder
B) Abacus
C) Paper
D) Complexbridges
Answer: C
Diff:2
PageRef:384
Topic:TangandSongProsperity:theBasisofaGoldenAge
Skill:Factual

44) Whichofthefollowingintellectualschoolswasresponsiblefortheproductionofmostliterary
andartisticworksduringtheTang-Songera?
A) Confucian
B) Daoist
C) PureLandBuddhist
D) ChanBuddhist
Answer: A
Diff:2
PageRef:384
Topic:TangandSongProsperity:theBasisofaGoldenAge
Skill:Conceptual

45) WhatgroupwasresponsibleforthefallofthesouthernSongdynastyin1279?
A) Jurchens
B) Tanguts
C) Liao
D) Mongols
Answer: D
Diff:1
PageRef:386
Topic:Conclusion:TheEndoftheSong,theLegacyofTwoGreatDynasties
Skill:Factual

283

46) Whatwastheprimarydifferencebetweenmarriagesoftheupperandlowerclassesin
Tang-SongChina?
A) Intheupperclasses,malesfrequentlymarriedfemalesofayoungergeneration.
B) Amongmembersofthelowerclasses,marriagestendedtobeconsummatedatanearlier
agethanamongmembersoftheelite.
C) Membersoftheeliteoftenmarriedbeforetheageofpuberty.
D) Householdsofthelowerclassestendednottobepatriarchal.
Answer: B
Diff:2
PageRef:381
Topic:TangandSongProsperity:theBasisofaGoldenAge
Skill:Conceptual

47) WhichofthefollowingstatementsconcerningthestatusofwomenintheTang -Songerais


mostaccurate?
A) Tang-Songlawrecognizednowomensrights.
B) TheconditionofwomeningeneralimprovedduringtheTang-Songera.
C) Theassertionofmaledominancewithinthefamilyandbeyondwasespecially
pronouncedinthethinkingoftheNeo-Confucianphilosophers.
D) ThespreadofBuddhismprovidedwomenwithgreateropportunitiesandcareer
alternativeswithinthemonasticmovement.
Answer: C
Diff:2
PageRef:381
Topic:TangandSongProsperity:theBasisofaGoldenAge
Skill:Conceptual

48) Whatartisanwasresponsibleforthedevelopmentofmovabletype?
A) LiBo
B) BiSheng
C) Gaozu
D) ZhuXi
Answer: B
Diff:2
PageRef:384
Topic:TangandSongProsperity:theBasisofaGoldenAge
Skill:Factual

284

49) TheoutstandingpoetoftheTangerawas
A) LiBo.
B) BiSheng.
C) Gaozu.
D) ZhuXi.
Answer: A
Diff:2
PageRef:385
Topic:TangandSongProsperity:theBasisofaGoldenAge
Skill:Factual

50) DuringtheSongdynastytheinterestofConfucianintellectualsinnaturewasmostapparentin
theirproductionof
A) riddles.
B) devotionalobjects.
C) religioushomilies.
D) landscapepaintings.
Answer: D
Diff:2
PageRef:385
Topic:TangandSongProsperity:theBasisofaGoldenAge
Skill:Factual

EssayQuestions
1) ContrasttheEraofDivisionwiththeSui-Tangera.
Answer: EraofDivision:dominatedbypoliticaldivisionamongmanysmallwarringstatesoften
ruledbynomadicinvaders;periodofBuddhistdominance;growthofmonastic
movement;lossofimperialcentralization;lossofdominanceofscholar-gentryinfavor
ofmilitarizedaristocracy.Sui-Tang:returntocentralizedadministration,unified
empire;reconstructionofbureaucracy;reconstructionofConfucianscholar-gentryat
expenseofbothBuddhistsandaristocracy;restorationofConfucianismascentral
ideologyofstate.
Diff:2
PageRef:367-371
Topic:RebuildingtheImperialEdificeintheSui-TangEra
Skill:Conceptual

2) ExplainthedeclineofBuddhisminthelaterTangandSongdynasties.
Answer: Restorationofimperialgovernmentimpliedstrengtheningoftraditionalschoolsof
Confucianismandresuscitationofscholar-gentry;ConfuciansattackedBuddhismasa
foreigninnovationinChina;convincedemperorsthatmonasticcontrolofland
representedaneconomicthreat;persecutionofBuddhistsintroducedin840s.
Diff:2
PageRef:372-379
Topic:RebuildingtheImperialEdificeintheSui-TangEra
Skill:Conceptual

285

3) CompareandcontrasttheempireundertheTangandtheSongdynasties.
Answer: Similarities:continuedintellectualandpoliticaldominanceofConfucian
scholar-gentry;growthofbureaucracyessentialtoimperialadministration.Differences:
smallerinsize;unabletocontrolnomadicdynastiesofthenorth;paymentoftributeto
nomadicstates;militarydeclinewithsubjectionofaristocracytoscholar -gentry;failure
ofWangAnshisreformsledtomilitarydefeat.
Diff:2
PageRef:367-379
Topic:TangDeclineandtheRiseoftheSong
Skill:Conceptual

4) WhatweretheelementsofTang-Songeconomicprosperity?
Answer: ThefullincorporationofsouthernChinaintotheeconomyasamajorfood-producing
region,centeroftrade;commercialexpansionwithWest,southernAsia,southeastAsia;
establishmentofChinesemerchantmarine;developmentofnewcommercial
organizationandcredittechniques;improvedagriculturalproductivitywithexpansion
ofacreage,greaterproductionperacre;expandedurbanizationthroughoutChina.
Diff:2
PageRef:377-378
Topic:TangandSongProsperity:theBasisofaGoldenAge
Skill:Conceptual

5) InwhatwaysdidtheChineseEmpireduringtheTang -Songeradepartfromprevious
developmentsinChinesecivilization?
Answer: FullincorporationofsouthernChinaintoeconomy;dominanceofsouthas
food-producingregion,centerofpopulation,politicalcapitalofsouthernSong;decline
ofinfluenceofBuddhism;increasingtrendtowardintellectualandtechnological
isolation;extraordinarylevelofurbanization,upto10percentofpopulation;levelof
technology.
Diff:2
PageRef:386
Topic:Conclusion:TheEndoftheSong:theLegacyofTwoGreatDynasties
Skill:Conceptual

286

Chapter18 TheSpreadofChineseCivilization:Japan,
Korea,andVietnam
Multiple-ChoiceQuestions
1) WhatregionsofAsiaweremostdrawntoChineseculturalandpoliticalmodels?
A) Theagrariansocieties
B) Thenomadicsocietiesonthenorth
C) IndianizedpeoplesofsoutheastAsia
D) TheislandsocietiesofthePacificrim
Answer: A
Diff:2
PageRef:389
Topic:Introduction
Skill:Factual

2) WhichofthefollowingwasLEASTinfluencedbyChinesecivilization?
A) Korea
B) Vietnam
C) India
D) Japan
Answer: C
Diff:1
PageRef:389
Topic:Introduction
Skill:Conceptual

3) WhenwasChinaspeakinfluenceonJapan?
A) 3rdcenturyB.C.E.
B) 1stcenturyB.C.E.
C) 2ndand3rdcenturiesC.E.
D) 7thand8thcenturiesC.E.
Answer: D
Diff:2
PageRef:390
Topic:Introduction
Skill:Factual

287

4) WhatisthenameofthenaturespiritsofJapan?
A) Hapu
B) Kami
C) Hara-kiri
D) Genji
Answer: B
Diff:2
PageRef:390
Topic:Introduction
Skill:Factual

5) WhatreligionplayedakeyroleinthetransmissionofChinesecivilizationtoJapan?
A) Buddhism
B) Daoism
C) Islam
D) Christianity
Answer: A
Diff:1
PageRef:390
Topic:Introduction
Skill:Factual

6) Whatwerethereformsenactedin646thatintendedtothoroughlyincorporateChineseculture
andpoliticalstructureintoJapanesesociety?
A) Onin
B) Gempei
C) Taika
D) Yoritomo
Answer: C
Diff:2
PageRef:390
Topic:Japan:TheImperialAge
Skill:Factual

7) Whatwasthecentralpurposeofthereformsof646inJapan?
A) ToremaketheJapanesemonarchintoanabsolutistChinese-styleemperor
B) TodestroytheConfucianscholar-gentryinfavorofamilitaryaristocracy
C) ToincreasethepoweroftheBuddhistmonasticstructure
D) Thedestructionofthetraditionalpeasant-conscriptarmy
Answer: A
Diff:2
PageRef:390
Topic:Japan:TheImperialAge
Skill:Conceptual

288

8) WhatgroupsothreatenedthesecurityoftheJapaneseimperialcourtinthe8thcenturythat
theimperialfamilymovedtoHeian?
A) Thearistocracy
B) Buddhistmonks
C) Thepeasantry
D) NomadicinvadersfromManchuria
Answer: B
Diff:2
PageRef:391
Topic:Japan:TheImperialAge
Skill:Factual

9) WhatwastheimmediateimpactoftheimperialmovetoHeian?
A) BuddhismceasedtoplayamajorroleinJapanesesociety.
B) Thescholar-gentrywasabletoassertitselfthroughthestatesacceptanceofaformal
examinationsystemmodeledontheChinesebureaucracy.
C) ThearistocracywasrestoredtocounterbalancethepoweroftheBuddhistmonasteries
andtookovermostofthepositionsinthecentralgovernment.
D) ShintoismwasformallysuppressedfollowingthemarriageofEmpressKokentoa
formermonk.
Answer: C
Diff:2
PageRef:391
Topic:Japan:TheImperialAge
Skill:Conceptual

10) WhatwasthemilitaryorganizationoftheHeiangovernmentinJapan?
A) Theemperorformalizedthecreationofapeasant-conscriptarmyonthemodelofthe
Chinese.
B) TheemperoracceptedtheintroductionofChinesetroopsasthebasisfortheJapanese
army.
C) Formalmilitaryorganizationwasabandonedbecauseofthesecuritytheemperor
perceivedfollowingthedefeatofaChinesenavy.
D) Localmembersofthearistocracywereorderedtoorganizemilitiaforcesthateventually
playedacriticalroleerodingthecontroloftheimperialgovernment.
Answer: D
Diff:2
PageRef:394
Topic:Japan:TheImperialAge
Skill:Conceptual

289

11) LifeintheimperialcourtatHeianwasdescribedinwhatJapanesenovel?
A) AshikageAnalects
B) TaleofGenji
C) MirrorofLadyMurasaki
D) RomanceoftheRose
Answer: B
Diff:2
PageRef:392
Topic:Japan:TheImperialAge
Skill:Factual

12) WhatappearstohavebeentheprimaryconcernsattheimperialJapanesecourtatHeian?
A) Militaryexercise
B) Scienceandtechnology
C) Socialconventionsandoriginalverse
D) Commerce
Answer: C
Diff:2
PageRef:392
Topic:Japan:TheImperialAge
Skill:Conceptual

13) Bythemiddleofthe9thcentury,whataristocraticfamilyexertedexceptionalinfluenceover
imperialaffairsattheJapanesecourtatHeian?
A) Minamoto
B) Fujiwara
C) Taira
D) Yoritomo
Answer: B
Diff:2
PageRef:393
Topic:Japan:TheImperialAge
Skill:Factual

14) Warriorleadersinthe10thcenturyinJapanwhocontrolledprovincialareasandruledfrom
smallfortressesinthecountrysidewerecalled
A) bushi.
B) samurai.
C) kata.
D) seppuku.
Answer: A
Diff:2
PageRef:394
Topic:Japan:TheImperialAge
Skill:Factual

290

15) Whichofthefollowingstatementsconcerningtheriseofaprovincialmilitaryeliteduringthe
10thcenturyinJapanisNOTcorrect?
A) Provincialelitefamiliesoftenarosefromlocallandowners,estatemanagers,orlocalstate
officials.
B) Theprovincialelitecametocontrollandandlaborlocallyandtodenytheseresourcesto
theimperialcourt.
C) Theriseoftheprovincialelitecorrespondedtotherecoveryoftheimperialgovernment
anditsoverthrowofthearistocracyofthecourt.
D) Withintheirlittlekingdoms,warriorleadersadministeredlaw,supervisedpublicworks
projects,andcollectedrevenue.
Answer: C
Diff:2
PageRef:394
Topic:Japan:TheImperialAge
Skill:Conceptual

16) Mountedtroopsowingloyaltytothemilitaryelitewerecalled
A) bushi.
B) samurai.
C) kata.
D) yoritomo.
Answer: B
Diff:1
PageRef:394
Topic:Japan:TheImperialAge
Skill:Factual

17) Whichofthefollowingstatementsconcerningtherelationshipoftheimperialcourttothe
provincialmilitaryeliteismostaccurate?
A) Theprovincialmilitaryelitewaskeptincheckbythecontinueddevelopmentofthe
imperialpeasant-conscriptarmy.
B) Theprovincialmilitaryelitewasrapidlysubjectedtothearistocraticarmiesofthe
imperialcourt.
C) Intheabsenceofanimperialmilitaryforce,lawandorderbrokedownleadingboththe
emperorandhighofficialstohireprovinciallordsandtheirmilitaryretainers.
D) TheemperorcreatedanalliancewiththeChinesethatpermittedhimtocrushthe
regionalmilitarylords.
Answer: C
Diff:2
PageRef:394
Topic:Japan:TheImperialAge
Skill:Conceptual

291

18) Whichofthefollowingstatementsconcerningthenatureofwarfareamongthe bushiismost


accurate?
A) Thebushidependedoninfantrytactics,equippingthesamurai initiallywithlongspears.
B) Theintroductionofgunpowderinthe11thcenturyallowedthebushitorelyoncannons
androcketsastheirprimarymeansofassault.
C) BattlesdependedontheJapanesephalanxofmountedsamurai andmassedassaults
predicatedonthewillingnessoftheretainerstosacrificethemselvesfortheirleaders.
D) Battleshingedonman-to-manduelsofgreatchampionstypicaloftheheroicstageof
warfare.
Answer: D
Diff:2
PageRef:394
Topic:Japan:TheImperialAge
Skill:Conceptual

19) Whatwastheimpactoftheriseofthesamurai onthepeasantryinJapan?


A) Japanesepeasantswerereducedtothestatusofserfsboundtothelandtheyworked.
B) Thesamuraiwerecriticaltothedevelopmentofafreepeasantryonwhichthewarriors
dependedforsuppliesoffoodandarms.
C) Thecreationofthesamurai createdaperiodofgreatsocialmobilityinJapanduring
whichpeoplerapidlymovedoutofthepeasantryandintotheclassofwarriors.
D) Althoughseparatedfromthewarriorsbyrigidclassbarriers,thepeasantryachieved
greaterlevelsofpersonalfreedomandeconomicprosperity.
Answer: A
Diff:2
PageRef:395
Topic:Japan:TheImperialAge
Skill:Conceptual

20) Bythe11thand12thcenturies,whatwasthestatusoftheJapanesecourtaristocracy?
A) Theycontinuedtodominatetheimperialgovernmentandtodependontheiralliance
withtheBuddhistmonasteries.
B) Aristocraticfamiliesatthecourtdependedonallianceswiththeprovincialwarriorelite
inordertoexerciseanypower.
C) Theresuscitationoftheemperorledtothedestructionofthecourtaristocracyandthe
suppressionoftheregionalwarriorelite.
D) Thecourtaristocracysucceededbythe11thcenturyinsettingasidetheemperorinfavor
ofregionalizedgovernment.
Answer: B
Diff:3
PageRef:395
Topic:TheEraofWarriorDominance
Skill:Conceptual

292

21) Between1180and1185thestrugglebetweenthetwomajorprovincialfamilies ,theTairaand


theMinamoto,weredecidedinwhatwars?
A) Onin
B) Koguryo
C) Gempei
D) Murasaki
Answer: C
Diff:2
PageRef:395
Topic:TheEraofWarriorDominance
Skill:Factual

22) ThevictoryoftheMinamotoin1185ledtothecreationof
A) thebakufu,ormilitarygovernmentatKamakura.
B) theAshikagaShogunate.
C) theTokugawaShogunate.
D) theimperialgovernmentatHeian(Kyoto).
Answer: A
Diff:2
PageRef:395
Topic:TheEraofWarriorDominance
Skill:Factual

23) ThevictoryoftheMinamotomarksthebeginningofwhatperiodinJapanesehistory?
A) ThecentralizedConfucianbureaucracy
B) Thefeudalage
C) TheOninwars
D) TheTokugawaShogunate
Answer: B
Diff:2
PageRef:395
Topic:TheEraofWarriorDominance
Skill:Conceptual

293

24) WhichofthefollowingwasNOTaresultofthegrowthofpoweroftheprovincialwarrior
elite?
A) TherelevanceofChineseprecedentsandinstitutionstotheJapanesediminished.
B) TheConfucianpreceptthatwarriorsshoulddominatethesocialandpoliticalworldwas
strengthened.
C) Pretensionstoaheavenlymandateandcentralizedpowerbecameludicrous.
D) Theemergenceofascholar-gentrywasstifledbythereassertionofaristocraticpower
andprerogatives.
Answer: B
Diff:2
PageRef:396
Topic:TheEraofWarriorDominance
Skill:Conceptual

25) WhichofthefollowingstatementsaboutChineseinfluenceinJapanismostaccurate?
A) Chineseinfluenceincreasedfromthe16thcenturyonward
B) Chineseinfluenceincreasedfromthe12thcenturyonward
C) Chineseinfluencedeclinedfromthe12thcenturyonward
D) Chineseinfluencedeclinedfromthe2ndcenturyonward
Answer: C
Diff:3
PageRef:395
Topic:TheEraofWarriorDominance
Skill:Factual

26) Whattitlewasgiventothemilitaryleaderofthe bakufu atKamakura?


A) Bakshi
B) Shogun
C) Sensei
D) Kata
Answer: B
Diff:1
PageRef:396
Topic:TheEraofWarriorDominance
Skill:Factual

294

27) FollowingthedeathofYoritomo,whatfamilydominatedthemilitarygovernmentofthe
BakufuatKamakura?
A) Yoshitsune
B) Fujiwara
C) Minamoto
D) Hojo
Answer: D
Diff:2
PageRef:396
Topic:TheEraofWarriorDominance
Skill:Factual

28) WhichofthefollowingstatementsmostaccuratelydescribesthenatureoftheJapanese
governmentfollowingthedeathofYoritomo?
A) RealpowerrestedintheHojofamily,whomanipulatedtheMinamotoshoguns,whoin
turnclaimedtoruleinthenameoftheemperoratKyoto.
B) Therewasatwo-tieredsystemofgovernmentwithpowerrestingwiththerestored
emperoratHeianwhocommandedabodyofaristocraticwarriorsassociatedwiththe
BakufuatKyoto.
C) Powerwasrapidlydissipatedaftertheemperorwasofficiallydeposedwithouta
successor.
D) ThesuccessorsofYoritomoseizedtheimperialthroneintheirownnameandsetoffa
civilwarbetweentheoldimperialfamilyatNaraandthenewoneinKamakura.
Answer: A
Diff:3
PageRef:396
Topic:TheEraofWarriorDominance
Skill:Conceptual

29) WhatgovernmentreplacedtheKamakuraregimeintheearly14thcenturyinJapan?
A) TokugawaShogunate
B) AshikagaShogunate
C) BushidoShogunate
D) OninShogunate
Answer: B
Diff:2
PageRef:396
Topic:TheEraofWarriorDominance
Skill:Factual

295

30) WhatwastherelationshipbetweentheAshikagaShogunateandtheemperor?
A) TheAshikagashogunsunseatedtheemperoratHeianandtooktheimperialtitlefor
themselves.
B) TheAshikagashogunssworefealtytotheemperoratKyotoandrestoredimperial
authority.
C) TheAshikagashogunsfoughttheemperoratKyotowhorefusedtorecognizetheir
authorityanddrovehimtothemountaintownofYoshinowhilesettingupapuppet
emperorintheoldcapitalofKyoto.
D) TheAshikagashogunsbroughttheimperialfamilytoKamakurainordertomoreclosely
controltheiractivities.
Answer: C
Diff:3
PageRef:396
Topic:TheEraofWarriorDominance
Skill:Conceptual

31) WhatJapaneserulingfactionwasdestroyedbythefull-scalecivilwarthatragedfrom1467to
1477?
A) TheFujiwara
B) TheMinamoto
C) TheKamakura
D) TheAshikagaShogunate
Answer: D
Diff:1
PageRef:397
Topic:TheEraofWarriorDominance
Skill:Factual

32) WhatwasthepoliticalresultofthewarsthatdestroyedtheAshikagaShogunate?
A) Restoredauthorityfortheemperor
B) Restoredauthorityforthecourtaristocracy
C) DivisionofJapaninto300littlestatesunderthedaimyos
D) Theriseoffourlargearistocraticstatesunderpowerfulbushi
Answer: C
Diff:2
PageRef:397
Topic:TheEraofWarriorDominance
Skill:Conceptual

296

33) Howdidtheprinciplesofwarfarechangeunderthe daimyos?


A) Heroiccombatbetweenchampionsremainedtherule,buttheweaponofchoicechanged
fromthebowtothecurvedsword.
B) Peasantforceswerereducedinsignificance,astheywerereplacedbyprofessional
soldiers.
C) Scientificwarfarebasedonspying,timelyassaults,wisecommandandorganizationof
massivearmiesreplacedheroiccombat.
D) Theriseofgunpowderandcannonmadethefortressesandcastlesofthewarriorelite
obsolete.
Answer: C
Diff:2
PageRef:398
Topic:TheEraofWarriorDominance
Skill:Conceptual

34) Whichofthefollowingstatementsconcerningthesocietyandeconomyofthewarlorderain
Japanesehistoryismostaccurate?
A) Duetotheincessantwarfare,theJapaneseeconomywasreducedtobarbarism.
B) Thepeasantrywerefurtherreducedinstatusastheagriculturaleconomyexperienced
rapidreductionsintheacreageunderproduction.
C) Despitepoliticalchaos,improvementsinagriculturaltechniquesandincentivesledto
occupationofpreviouslyuncultivatedareas.
D) Thefrequentcombatofthewarlorderasuppressedthedevelopmentofartisanor
merchantclassesinJapan.
Answer: C
Diff:3
PageRef:398
Topic:TheEraofWarriorDominance
Skill:Conceptual

35) Whichofthefollowingstatementsconcerningthestatusofwomenduringtheeraofthe
daimyosismostaccurate?
A) Womensstatusamongtheelitecertainlyimprovedduringtheeraofthewarlords,as
theygainedrightstoinheritance.
B) Womenoftenparticipatedinmartialtrainingduringtheeraofthewarlords,thus
improvingtheirsocialstatus.
C) Whiledaughtersofthebushi oftenreceivedsomemartialtraining,underthedaimyos
womenwereexcludedfrominheritanceandtreatedasdefenselessappendagesoftheir
warriorfathersorhusbands.
D) Whilethestatusofwomenamongtheartisanandmerchantclassesdeclined,thestatus
ofwomenamongthewarrioreliteprobablyimprovedmarginally.
Answer: C
Diff:2
PageRef:399
Topic:TheEraofWarriorDominance
Skill:Conceptual

297

36) Duringtheperiodofthewarringdaimyos,whatwasthefactorthatledtoarevivalofChinese
influenceontheculturallevel?
A) Confucianism
B) Thestrengthofthemerchantclass
C) Chinesemilitarytechnology
D) ZenBuddhism
Answer: D
Diff:2
PageRef:399
Topic:TheEraofWarriorDominance
Skill:Conceptual

37) WhichofthefollowingwasNOTadevelopmentduringtheeraofthedaimyosthatlaidthe
foundationsforlastingpoliticalunificationinJapan?
A) Economicandculturalgrowth
B) RestorationoftheimperialgovernmentatKyoto
C) Strengthofthecommercialandartisanclasses
D) Improvedadministrationwithinthedaimyo domains
Answer: B
Diff:2
PageRef:398-399
Topic:TheEraofWarriorDominance
Skill:Conceptual

38) InwhatyeardidtheHanemperorsconquerthefirstKoreankingdomofChoson?
A) 109B.C.E.
B) 220C.E.
C) 476C.E.
D) 643C.E.
Answer: A
Diff:2
PageRef:400
Topic:Korea:BetweenChinaandJapan
Skill:Factual

298

39) Whatkingdomwassuccessfulinestablishingaunifiedandindependentgovernmentin
Korea?
A) Koguryo
B) Silla
C) Paekche
D) Kyoto
Answer: B
Diff:1
PageRef:401
Topic:Korea:BetweenChinaandJapan
Skill:Factual

40) WhichofthefollowinglistsgivesthecorrectchronologicalsequencefortheKoreandynasties?
A) Silla,Mongol,Koryo,Yi
B) Silla,Yi,Mongol,Koryo
C) Silla,Yi,Koryo,Mongol
D) Silla,Koryo,Mongol,Yi
Answer: D
Diff:2
PageRef:401-403
Topic:Korea:BetweenChinaandJapan
Skill:Factual

41) WhichofthefollowingstatementsconcerningtheKoreanbureaucracyundertheinfluenceof
Chinaismostaccurate?
A) WiththeestablishmentoftheConfucianexaminationsystem,advancementwithinthe
civilservicewasdeterminedalmostexclusivelybytestscoresratherthanbybirth.
B) AlthoughKoreaemulatedtheChinesebureaucraticstructure,thesatellitenevercreated
aConfucianexaminationsystemasameansofqualification.
C) KoreaestablishedaConfucianexaminationsystemontheChinesemodel,butadmission
tothebureaucracywasdeterminedalmostexclusivelybybirthratherthantestscores.
D) Koreanabsolutismdependedmoreonawarriorelitethanadministrativesophistication,
soaformalbureaucracywasneverestablished.
Answer: C
Diff:3
PageRef:401-402
Topic:Korea:BetweenChinaandJapan
Skill:Conceptual

299

42) WhatwasthereligiouspreferenceoftheKoreanelite?
A) Confucianism
B) Christianity
C) Buddhism
D) Islam
Answer: C
Diff:1
PageRef:402
Topic:Korea:BetweenChinaandJapan
Skill:Factual

43) WhichofthefollowingstatementsconcerningthenatureoftheKoreansocietyismost
accurate?
A) Koreansocietyistypifiedbyequalitybetweenallclasses.
B) ChinahadnoinfluenceonKorea.
C) ThearistocratsweretheonlypeoplewhoreallycountedforanythinginKoreansociety.
D) KoreansocietyfollowedtheChinesemodelofastrongscholar-gentry,aweak
aristocracy,andahighlyregardedthoughpoliticallyimpotentpeasantry.
Answer: C
Diff:3
PageRef:403
Topic:Korea:BetweenChinaandJapan
Skill:Conceptual

44) WhichofthefollowingwasNOTadifferencebetweentheVietandChineseculturespriorto
theirconquestbytheHanemperors?
A) Language
B) VillageautonomyamongtheViets
C) HighersocialstatusforVietwomen
D) Viettendencytoextendedfamiliesandclangroups
Answer: D
Diff:2
PageRef:404
Topic:BetweenChinaandSoutheastAsia:theMakingofVietnam
Skill:Conceptual

300

45) WhichofthefollowingChineseculturaltraitswasNOTintroducedintoVietnamfollowing
theHanconquestof111B.C.E?
A) Chineseexaminationsystemandbureaucracy
B) Chineseagriculturalcroppingtechniquesandirrigationtechnology
C) Chineseinsistenceonthenuclearfamily
D) Chinesemilitaryorganization
Answer: C
Diff:2
PageRef:404
Topic:BetweenChinaandSoutheastAsia:theMakingofVietnam
Skill:Conceptual

46) WhichofthefollowingwasacriticalfactorinthefailureoftheChinesetoholdorassimilate
theVietnamese?
A) ThefragilityofthelinksthatboundtheVietnamesetotheChinese
B) ThefailureoftheChinesetointroduceBuddhism
C) ThecontinuedculturalimpactofIndiancultureinsoutheastAsia,particularlyamongthe
Vietnamese
D) ThewidespreaddestructionoftheVietnameseelite
Answer: A
Diff:2
PageRef:405
Topic:BetweenChinaandSoutheastAsia:theMakingofVietnam
Skill:Conceptual

47) FollowingtheassertionofVietnameseindependencefromChinainthe10thcentury,what
aspectsofChineseculturewereretainedbytheVietnameserulers?
A) Chinesefamilyorganization
B) TheChineseadministrativesystembasedontheConfucianexaminationsystem
C) Ahighlycentralizedadministrativesystemmannedbyapowerfulscholar-gentry
D) Thesuccessfulsuppressionofthepeasantry
Answer: B
Diff:2
PageRef:406-407
Topic:BetweenChinaandSoutheastAsia:theMakingofVietnam
Skill:Conceptual

301

48) WhatwasthepoliticalresultoftheVietnamesedrivetoconquerregionssouthoftheRed
Riverbasin?
A) ThecreationofahighlycentralizedkingdomwithitscapitalatHanoi
B) ThedefeatoftheVietnameseandthefragmentationofthekingdominto300small
kingdomsruledbyawarriorelite
C) ThereconquestoftheRedRivervalleybytheChineseduringthesouthernSongera
D) ThedivisionoftheVietnameseintotwokingdomswithcapitalsatHueandHanoi
Answer: D
Diff:2
PageRef:409
Topic:BetweenChinaandSoutheastAsia:theMakingofVietnam
Skill:Conceptual

EssayQuestions
1) HowdidtheJapaneseimportationofChineseculturedifferfromthatofVietnamandKorea?
Answer: JapanwasneverconqueredanddirectlyruledbyChineseaswereVietnamandKorea;
abletoselectivelyimportChineseculture;afterfailureofTaikareforms,influenceof
Chineseculturewaned;nogrowthofscholar-gentry;lackofcentraladministration;
basicallyreceivedonlyChinesescript,art,andBuddhism.
Diff:2
PageRef:390-408
Topic:Japan:TheImperialAge
Skill:Conceptual

2) DiscussthegovernmentofJapanbetweentheGempeiwarsandtheTokugawaShogunate.
Answer: Gempeiwarsmarkeddominanceofprovincialmilitaryaristocracyoverimperialcourt;
Minamotofamilyestablishedfirstdominancewithmilitarygovernmentor Bakufuat
Kamakura;declineofcentraladministrationandscholar-gentry;Hojofamily
dominatedBakufu;finallyKamakuragovernmentoverthrownbyAshikagaShogunate;
allcentralauthoritydissipatedduringOninWarfrom1467 -1477;countrydividedup
into300smallkingdomsruledbydaimyos.
Diff:2
PageRef:395-400
Topic:TheEraofWarriorDominance
Skill:Conceptual

3) HowdidthesocialdevelopmentofpostclassicalJapandifferfromthedevelopmentofKorea
duringtheperiodofChineseinfluence?
Answer: UnderdaimyosdeclineofcourtaristocracyinJapan;completedecentralizationof
politicalauthority;daimyossoughttofostertradeandmerchantclass;growthofboth
merchantandartisanclassescontrarytoConfuciansocialprecepts.InKorea,court
aristocracymonopolizedallChineseimports;tookovercentraladministration;
forestalleddevelopmentofmerchantorartisanclass;Koreafailedtodevelopmuch
beyondaristocracyandstatus-poorpeasantry.
Diff:2
PageRef:395-403
Topic:Korea:BetweenChinaandJapan
Skill:Conceptual

302

4) WhywasChinaunabletoassimilatetheVietnamesedespitedirectruleforalmosta
millennium?
Answer: Vietnameseculturallydifferentfromtheoutset:differentlanguage,traditionoflocal
authorityinherentinvillageleaders,emphasisonnuclearfamilyratherthantypically
Chineseextendedfamilies,higherstatusaccordedtowomen;Chineseabletoexert
someinfluence:introductionofcentraladministrationbasedonConfucianexam
system,someintroductionofextendedfamilyandancestorworship,useofChinese
militaryorganization;ultimatefailurebasedoninabilitytoimpactVietnamese
peasantrywhoremainedsignificantonlocallevel;onlyBuddhismimpactedpeasantry.
Diff:2
PageRef:403-406
Topic:BetweenChinaandSoutheastAsia:theMakingofVietnam
Skill:Conceptual

5) HowdidtheextensionofChineseculturetoitssatellitecivilizationsdifferfromotherglobal
civilizations?
Answer: Chinesecultureextendedonlywithinsemi-closedeastAsianculturalsystem;unlike
IslamthatspreadfromtheMiddleEasttoAfricaandtosouthandsoutheastAsia;
unlikecommonculturalexchangesbetweenIslamandpostclassicalWest;eastAsian
culturalexchangeoccurredinsemi-isolationfromotherglobalcultures.
Diff:2
PageRef:410
Topic:Conclusion:DivergentPathsinEastAsianDevelopment
Skill:Conceptual

303

Chapter19 TheLastGreatNomadicChallenges:From
ChinggisKhantoTimur
Multiple-ChoiceQuestions
1) WhatwastheattitudeoftheMongolstothereligionstheyencountered?
A) TheydespisedIslambutconvertedtoChristianity.
B) Theybannedallreligions.
C) Theytoleratedmostreligions.
D) TheymergedHinduismandIslamtocreateanewstatereligion.
Answer: C
Diff:2
PageRef:415
Topic:Introduction
Skill:Factual

2) Inmostways,theMongolsepitomizedwhattypeofsocietyandculture?
A) Nomadic
B) Sedentaryagricultural
C) Huntingandgathering
D) Urban
Answer: A
Diff:1
PageRef:415
Topic:TheMongolEmpireofChinggisKhan
Skill:Conceptual

3) TheMongolherdsconsistedprimarilyof
A) sheepandgoats.
B) camels.
C) cattle.
D) horses.
Answer: A
Diff:2
PageRef:415
Topic:TheMongolEmpireofChinggisKhan
Skill:Factual

304

4) WhatwasthebasicunitofMongolsociety?
A) Thenuclearfamily
B) Thetribe
C) Theguild
D) Thecity-state
Answer: B
Diff:1
PageRef:415
Topic:TheMongolEmpireofChinggisKhan
Skill:Conceptual

5) WhichofthefollowingstatementsconcerningleadershipinMongolsocietyismostcorrect?
A) TheMongolsrecognizedtworoyalfamilies,andallleadersweretheoldestmalesofthe
lineages.
B) Mongolsoftenrecognizedtheleadershipoffemaleswhotracedtheirdescentfrom
femaledeities.
C) MongolleadershipwasbasedonpatrilinealdescentfromtheKuriltaitribe.
D) Leaderswereelectedbyfreemales.
Answer: D
Diff:2
PageRef:416
Topic:TheMongolEmpireofChinggisKhan
Skill:Conceptual

6) Intheearly12thcentury,theQinkingdomofnorthernChinawasdefeatedbyaMongol
confederationundertheleadershipofChinggisKhansgreatgrandfather,
A) Ogedei.
B) KabulKhan.
C) MuhammadIIShah.
D) Batu.
Answer: B
Diff:2
PageRef:416
Topic:TheMongolEmpireofChinggisKhan
Skill:Factual

305

7) WhichofthefollowingstatementsconcerningChinggisKhansearlylifeismostaccurate?
A) HesucceededtothekingshipoftheMongolsatanearlyageastheonlyheirofhis
grandfather.
B) Followingthedeathofhisfather,heimmediatelyenjoyedmilitarysuccessoverrival
clans.
C) Followinghisfathersdeath,hewasabandonedbymanyfollowersandcapturedbya
rivaltribe.
D) HefledfollowinghisfathersdeathtotheChinese,whoprovidedhimwithmilitary
support.
Answer: C
Diff:2
PageRef:416
Topic:TheMongolEmpireofChinggisKhan
Skill:Conceptual

8) ChinggisKhanwaselectedkhagan(supremeruler)oftheMongoltribesinwhatyear?
A) 1154
B) 1189
C) 1206
D) 1312
Answer: C
Diff:2
PageRef:416
Topic:TheMongolEmpireofChinggisKhan
Skill:Factual

9) TheMongolmilitaryforceswere
A) composedofamixtureofinfantryandcavalry.
B) entirelycavalry.
C) entirelyinfantry.
D) technologicallyinferiortomostoftheiropponents.
Answer: B
Diff:2
PageRef:416
Topic:TheMongolEmpireofChinggisKhan
Skill:Factual

306

10) Mongolarmiesweredividedintounitscalled
A) kuriltai.
B) tobruk.
C) karakorum.
D) tumens.
Answer: D
Diff:1
PageRef:416
Topic:TheMongolEmpireofChinggisKhan
Skill:Factual

11) WhichofthefollowingweaponswasNOTpartofthearsenaloftheMongolarmyunder
ChinggisKhan?
A) Bowsandarrows
B) Chainmail
C) Bronzecannon
D) Ironmaces
Answer: B
Diff:2
PageRef:416
Topic:TheMongolEmpireofChinggisKhan
Skill:Factual

12) ChinggisKhansfirstcampaignsweredirectedagainst
A) Russia.
B) BaghdadandtheIslamicheartland.
C) India.
D) theTangutkingdomofXi-XiaandtheQinkingdomofnorthernChina.
Answer: D
Diff:2
PageRef:418
Topic:TheMongolEmpireofChinggisKhan
Skill:Factual

13) In1219ChinggisKhandirectedhistroopagainsttheIslamicterritoryof
A) Khwarazm.
B) Kubilai.
C) Karakorum.
D) Samarkand.
Answer: A
Diff:2
PageRef:418
Topic:TheMongolEmpireofChinggisKhan
Skill:Factual

307

14) WhattacticonthefieldofbattlewasemployedmostfrequentlybyChinggisKhansforces?
A) Frontalassaultbymassedcavalry
B) Massedartillerybarragefollowedbyinfantryattacksontheflanks
C) Trenchwarfare
D) Pretendedflighttodrawtheenemyoutfollowedbyheavycavalryattacksontheflanks
Answer: D
Diff:2
PageRef:418
Topic:TheMongolEmpireofChinggisKhan
Skill:Conceptual

15) WherewasthecapitaloftheMongolempireunderChinggisKhan?
A) Samarkand
B) Karakorum
C) Tatu
D) Khwarazm
Answer: B
Diff:2
PageRef:418
Topic:TheMongolEmpireofChinggisKhan
Skill:Factual

16) WhendidChinggisKhandie?
A) 1027
B) 1127
C) 1227
D) 1327
Answer: C
Diff:2
PageRef:420
Topic:TheMongolEmpireofChinggisKhan
Skill:Conceptual

17) WhichofthefollowingwasNOToneofthepositiveaspectsofChinggisKhansimperialrule?
A) HepromulgatedalegalcodetoenddivisionsandquarrelsamongtheMongolclans.
B) HebroughtpeacetomuchofAsia.
C) Hepromotedthegrowthoftradeandcommerce.
D) HeorderedthecreationofhugepasturesinnorthernChinafortheuseoftheMongol
clans.
Answer: D
Diff:2
PageRef:418-419
Topic:TheMongolEmpireofChinggisKhan
Skill:Conceptual

308

18) FollowingthedeathofChinggisKhan,whosucceededhimaskhaganoftheMongols?
A) Kubilai
B) Batu
C) Berke
D) Ogedei
Answer: D
Diff:2
PageRef:420
Topic:TheMongolDrivetotheWest
Skill:Factual

19) FollowingChinggisKhansdeath,whatwastheprovisionfortheadministrationofthe
empire?
A) Itwasdividedintofourregionalkingdoms,orkhanates,ruledbyhissonsand
grandsons.
B) ItwascentralizedwithaMongolbureaucracylocatedattheChinesecapitalofTatu.
C) Theempireimmediatelyfragmentedintoitsconstituenttribesandclans.
D) ItpassedasasinglegovernmentwithitscapitalatKarakorumtoChinggisKhansoldest
son.
Answer: A
Diff:2
PageRef:420
Topic:TheMongolDrivetotheWest
Skill:Conceptual

20) WhichofthekhanatesundertookthetaskofconqueringRussia?
A) TheIlkhankhanate
B) TheKarakorumkhanate
C) TheChinesekhanate
D) TheGoldenHorde
Answer: D
Diff:1
PageRef:420
Topic:TheMongolDrivetotheWest
Skill:Factual

309

21) WhichofthekhanatesundertooktheinvasionoftheMuslimheartland?
A) TheIlkhankhanate
B) TheKarakorumkhanate
C) TheChinesekhanate
D) TheGoldenHorde
Answer: A
Diff:1
PageRef:423
Topic:TheMongolDrivetotheWest
Skill:Factual

22) WhatwasthepoliticalorganizationofRussiaatthetimeoftheMongolinvasioninthefirst
halfofthe13thcentury?
A) RussiawasunitedundertheruleofKiev.
B) RussiawasunitedundertheruleoftheprinceofNovgorod.
C) RussiawasunitedundertheruleoftheprinceofMoscovy.
D) Russiawasdividedintonumerouspettykingdomscenteredontradingcities.
Answer: D
Diff:2
PageRef:420
Topic:TheMongolDrivetotheWest
Skill:Conceptual

23) WhatwasthenatureoftheRussianresistancetotheMongolinvasion?
A) TheRussiancitiesalliedrapidlytoconfronttheMongolswithaunitedforce.
B) AstheleadingcommercialcentersofRussia,KievandNovgorodalliedtocreatea
substantialmilitaryforcethatsavedthosecitiesfromdestruction.
C) TheRussiansappealedtotheByzantineEmpiretoprovidethemwithmilitaryassistance
againsttheMongols.
D) TheprincesofRussiarefusedtocooperateandwereroutedindividually.
Answer: D
Diff:2
PageRef:420
Topic:TheMongolDrivetotheWest
Skill:Conceptual

310

24) WhatwasthesocialimpactoftheMongolconquestonRussia?
A) TheRussiannobilitywasexterminatedgivingrisetoasocietylargelycomposedoffree
peasants.
B) DuetothecrushingburdenoftributepaidtoMongolsandprinces,theRussian
peasantrywasreducedtoserfdom.
C) ThecessationoftradedestroyedthecommercialandartisanclassesofRussia.
D) Russianwomenwereelevatedtonewlevelsofsocialprominence.
Answer: B
Diff:2
PageRef:421
Topic:TheMongolDrivetotheWest
Skill:Conceptual

25) WhatRussiancityprofitedmostfromtheMongolinvasion?
A) Novgorod
B) Kiev
C) Moscow
D) Petrograd
Answer: C
Diff:1
PageRef:421
Topic:TheMongolDrivetotheWest
Skill:Factual

26) WhataccountsforthegrowingpoliticaldominanceofMoscowunderMongolrule?
A) MoscowwastheonlyRussiancitythatwasnotdestroyedduringtheinvasion.
B) AsthecitylocatedfarthestsouthinRussia,Moscowwasbetterabletobenefitfrom
renewedtradeundertheMongols.
C) MoscowbecamethecapitaloftheGoldenHorde.
D) TheprincesofMoscowcollectedtributefortheMongolkhansandbecametheseatofthe
MetropolitanoftheOrthodoxChurch.
Answer: D
Diff:2
PageRef:421
Topic:TheMongolDrivetotheWest
Skill:Conceptual

311

27) TheprinceofMuscovyinalliancewithotherRussianprinces,defeatedtheGoldenHordein
1380atthebattleof
A) Bratislava.
B) Cracow.
C) Kulikova.
D) AinJalut.
Answer: C
Diff:2
PageRef:422
Topic:TheMongolDrivetotheWest
Skill:Factual

28) WhatwasthemostsignificantimpactoftheperiodoftheMongolruleonRussia?
A) TheperiodofMongolrulereinforcedtheisolationofRussiafromwesternEuropeand
thedevelopmentsoftheRenaissanceandReformation.
B) TheMongolsaidedtheRussiansingainingpoliticaldominanceoverthepeoplesofthe
Asiaticsteppes.
C) TheperiodofMongolruleintroducedmanyIslamicpeopleintotheregionofRussia.
D) TheMongoldominationresultedinthedestructionofEasternOrthodoxyandtheriseof
NestorianChristianity.
Answer: A
Diff:2
PageRef:423
Topic:TheMongolDrivetotheWest
Skill:Conceptual

29) WhydidtheMongolsnotpursuetheconquestofwesternEurope?
A) TheyweredefeatedatthebattleofBratislavabyKingBelaofHungaryandalliedknights
ofeasternEurope.
B) AttacksbyTimur-iLangonthesouthernlimitsoftheterritoriesbelongingtotheGolden
HordeinterruptedtheMongolassault.
C) ThedeathofKhaganOgedeiinKarakorumprecipitatedastruggleforthesuccession
involvingthekhanoftheGoldenHorde.
D) TheMongolshadnointerestinwesternEurope.
Answer: C
Diff:2
PageRef:423
Topic:TheMongolDrivetotheWest
Skill:Conceptual

312

30) TheruleroftheIkhanhordewas
A) Berke.
B) Kubilai.
C) Batu.
D) Hulegu.
Answer: D
Diff:1
PageRef:423
Topic:TheMongolDrivetotheWest
Skill:Factual

31) InadditiontothedestructionoftheAbbasidpoliticalcapitalatBaghdadandtheweakeningof
theMuslimmilitarystrength,whatsignificantimpactdidtheMongolconquesthaveonthe
Islamicheartland?
A) ThedestructionofcitiesfromcentralAsiatotheshoresoftheMediterraneandevastated
thefocalpointsofIslamiccivilization.
B) ThesuccessfulassaultontheeastAfricancity-statesweakenedtheinternationaltrading
systemofIslam.
C) ShiismwaseliminatedasamajorfactorwithinIslam.
D) MuchofthepopulationoftheIslamicheartlandwasconvertedtotheanimismcommon
amongtheMongoliannomads.
Answer: A
Diff:2
PageRef:423
Topic:TheMongolDrivetotheWest
Skill:Conceptual

32) TheMongolsweredefeatedin1260atAinJalutbythe
A) crusaderstatesandtheirAssassinallies.
B) MamlukdynastyofEgypt.
C) OttomanTurks.
D) forcesoftheByzantineEmpire.
Answer: B
Diff:1
PageRef:424
Topic:TheMongolDrivetotheWest
Skill:Factual

313

33) InadditiontotheMamlukpowerinEgypt,whydidthekhanoftheIlkhanhordenotfollow
upontheconquestoftheIslamicheartland?
A) TheforcesoftheChristiancrusaderstateswerelikelytobringthepowerfularmiesof
westernEuropeintothestruggleagainsttheMongols.
B) Thekhandied,leavingaminorastheruleroftheIlkhanhorde.
C) ThesuccessorofBatuasruleroftheGoldenHorde,Berke,hadconvertedtoIslamand
wasapotentialallyoftheMameluks.
D) TheIlkhanhordedissipatedintorivalclangroupsfollowingtheirdefeatatthehandsof
theMameluks.
Answer: C
Diff:2
PageRef:425
Topic:TheMongolDrivetotheWest
Skill:Conceptual

34) ManyinwesternEuropebelievedthattheMongolkhanwas
A) PresterBob.
B) PresterJohn.
C) JesusChrist.
D) Muhammad.
Answer: B
Diff:2
PageRef:423
Topic:TheMongolDrivetotheWest
Skill:Conceptual

35) TheMongolcommanderinthe13th-centurycampaignsagainsttheSongwasChinggisKhans
grandson,
A) Ogedei.
B) KubilaiKhan.
C) Batu.
D) Berke.
Answer: B
Diff:1
PageRef:425
Topic:TheMongolInterludeinChineseHistory
Skill:Factual

314

36) WhichofthefollowingstatementsconcerningtheMongolconquestofSongChinaismost
accurate?
A) TheMongolswereunabletoestablishpoliticalcontrolovertheSongdynasty.
B) SongChina,becauseofitsrelativeweakness,capitulatedwithintwoyearsafterthe
initiationoftheMongolconquest.
C) ThecampaignsagainsttheSongwereinterruptedbypowerstrugglesamongthe
Mongols,buttookslightlymorethanfiveyears.
D) SongChinaprovedoneofthetoughestareasfortheMongolstoconquer,takingfrom
1235to1279tosubdue.
Answer: D
Diff:2
PageRef:425
Topic:TheMongolInterludeinChineseHistory
Skill:Conceptual

37) In1271,KubilaiKhanchangedthenameoftheMongoldynastyinChinato
A) Qin.
B) Chin.
C) Yuan.
D) Ming.
Answer: C
Diff:1
PageRef:425
Topic:TheMongolInterludeinChineseHistory
Skill:Factual

38) WhichofthefollowingstatementsconcerningtheYuansocialorderismostaccurate?
A) BeneaththeMongolsintheYuansocialsystemweretheethnicChinese.
B) BelowtheMongolswereJapaneseartisansintheYuansocialscheme.
C) MuslimsandcentralAsianalliesrankedrightbelowtheMongolsbutabovetheChinese.
D) TheMongolsrankedallotherethnicgroupsinasinglecohortofsubordinates.
Answer: C
Diff:3
PageRef:425
Topic:TheMongolInterludeinChineseHistory
Skill:Conceptual

315

39) WhichofthefollowingwasNOTamethodemployedbyKubilaiKhantomaintainthe
separationofMongolsandChinese?
A) ChinesewereforbiddentoserveatanyleveloftheMongoladministration.
B) ChinesescholarswereforbiddentolearntheMongolscript.
C) MongolswereforbiddentomarryethnicChinese.
D) Onlywomenfromnomadicfamilieswereselectedfortheimperialharem.
Answer: A
Diff:2
PageRef:425
Topic:TheMongolInterludeinChineseHistory
Skill:Conceptual

40) WhichofthefollowingwasamajorchangeintheadministrationofChinaundertheMongols?
A) ThecentralbureaucracywasdismissedandtheMongoldynastyruledwithamilitary
elite.
B) TheMongolsdiscontinuedtheuseoftheexaminationsystemtokeepthescholar -gentry
fromgainingtoomuchpower.
C) TheMongolsdividedallofChinaintofourgreatkhanatesunderseparateand
independentrulers.
D) ConfucianismwassuppressedandDaoismbecamethestatereligionofChina.
Answer: B
Diff:2
PageRef:426
Topic:TheMongolInterludeinChineseHistory
Skill:Conceptual

41) WhatwasthestatusofMongolianwomenduringtheYuandynastyofChina?
A) MongolianwomenloststatusastheyfellunderthesocialapparatusoftheConfucian
ideology.
B) MongolianwomensufferedsocialandpoliticalisolationasMongolmenadoptedthe
preferenceforwomenwhohadundergonefootbinding.
C) Mongolwomenwereincreasinglypreventedfromparticipatinginhuntingandmartial
activities,althoughtheyretainedsomeinfluenceinthehousehold.
D) Mongolwomenremainedrelativelyindependent,refusedtoadoptthepracticeof
footbinding,andretainedtheirrightsinproperty.
Answer: D
Diff:2
PageRef:426
Topic:TheMongolInterludeinChineseHistory
Skill:Conceptual

316

42) WhatwastheYuanpolicywithrespecttoreligionduringtheMongoloccupationofChina?
A) LiketheChinesedynasties,overtimetheYuanadoptedConfucianismastheprimary
ideologyofthestate.
B) BecauseitwasclosertotheanimismpracticedbytheMongolsonthesteppes,theYuan
adoptedDaoismasthestatereligion.
C) Despitetheirdesiretoremainseparate,theYuanemperorswereconvertedtoBuddhism
undertheinfluenceofChabi,KubilaiKhanswife.
D) TheYuan,liketheirancestors,insistedonreligioustoleration.
Answer: D
Diff:2
PageRef:427
Topic:TheMongolInterludeinChineseHistory
Skill:Conceptual

43) WhattwosocialgroupsthatsufferedlowstatusundertheConfuciansocialsystembenefited
mostfromtheYuanadministration?
A) Peasantsandaristocracy
B) Scholar-gentryandaristocracy
C) Merchantsandartisans
D) Militaryeliteandscholar-gentry
Answer: C
Diff:2
PageRef:429
Topic:TheMongolInterludeinChineseHistory
Skill:Conceptual

44) WhatcausedadeclineinthemilitaryreputationoftheMongolYuandynastyinChina?
A) ThefailureofexpeditionsagainsttheJapanese
B) ThedemolitionoftheGreatWall
C) ThedefeatoftheYuanatthehandsoftheGoldenHorde
D) TheinvasionofnorthernChinabytheKoreanKoryodynasty
Answer: A
Diff:2
PageRef:430
Topic:TheMongolInterludeinChineseHistory
Skill:Conceptual

317

45) ThefounderoftheMingdynasty,JuYuanzhang,camefromwhatsocialclassinChina?
A) Aristocracy
B) Scholar-gentry
C) Merchants
D) Peasantry
Answer: D
Diff:1
PageRef:430
Topic:TheMongolInterludeinChineseHistory
Skill:Factual

46) WhowastheTurkicnomadicleaderwhobeganaperiodofconquestbeginninginthe1360s?
A) MuhammanShah
B) IbnPasha
C) Timur-iLang
D) IbnKhaldun
Answer: C
Diff:1
PageRef:430
Topic:Conclusion:TheMongolLegacyandanAftershock
Skill:Factual

47) WhichofthefollowingareaswasnotexposedtotheTurkicexpansionofthe1360s?
A) Persia
B) China
C) India
D) SouthernRussia
Answer: B
Diff:2
PageRef:431
Topic:Conclusion:TheMongolLegacyandanAftershock
Skill:Factual

48) WhatwasthecommonnamefortheplaguethattheMongolarmiesunknowinglyspread?
A) TheBlackDeath
B) TheWhiteDeath
C) TheNomadicDeath
D) TheMongolDeath
Answer: A
Diff:2
PageRef:432
Topic:Conclusion:TheMongolLegacyandanAftershock
Skill:Conceptual

318

EssayQuestions
1) Whydidsedentarycivilizationsprogressivelydominatenomadicsocietiesafterthe15th
century?
Answer: BlackDeathreducednomadicpopulations;populationrecoveryneverasrapidoras
completeasthesedentarycivilizations;sedentarycivilizationsafter1400foundwaysto
centralizepower,mobilizeresourcesforwarfare;industrializationofwarfareincluding
technologicalinnovationswithmilitaryapplicationsprovidedinsuperableadvantages
tosedentarycivilizations.
Diff:2
PageRef:431-432
Topic:Conclusion:TheMongolLegacyandanAftershock
Skill:Conceptual

2) DefinetheterritorialextentoftheMongolEmpireatitslargest.Howdidthisaffect
inter-culturalexchange?
Answer: MongolempireextendedfromRussiaandeasternEuropeinwesttoMesopotamiaas
farasEgyptinthesouthacrosstheCaspianSearegionandtheAsiaticsteppesto
includeallofChina.MongolEmpirelinkedgreatglobalcivilizationsofEastern
Hemisphere,westernandeasternEurope,Islam,China;permittedfreeexchangeof
goodsandideasbetweenglobalculturesalongtraditionalroutesoftrade.
Diff:2
PageRef:415-430
Topic:TheMongolEmpireofChinggisKhan
Skill:Conceptual

3) InwhatwaywasthepoliticalimpactoftheMongolconquestsofRussiaandtheIslamic
heartlandsimilar?
Answer: Inbothcasesthetraditionalpoliticalstructurewasremovedandthepathwassmoothed
fornewpoliticalorganizationtotakeplace.InRussia,Kievansuperioritywasforever
destroyedandMoscowwasabletoachievepoliticaldominanceamongthepetty
kingdomsthroughitscontroloftributeandbybecomingtheseatofRussianOrthodoxy.
InIslam,theAbbasiddynastywasendedandtheSeljukTurkswhohadruledthrough
itsappurtenancesweredevastated,openingthewayfortheriseoftheMameluksin
EgyptandtheOttomanTurksinAsiaMinor.
Diff:2
PageRef:420-425
Topic:TheMongolDrivetotheWest
Skill:Conceptual

4) InwhatwaysdidtheMongoldynastyofChina(theYuan)attempttoalterthetraditional
Chinesesocialstructure?
Answer: ByrefusingtoreinstatetheConfucianexaminationsystem,theYuanattemptedto
destroythesocialandpoliticaldominanceofthescholar-gentry;thisattemptwas
secondedbydividingtheChinesesocialstructureethnicallyMongolsandIslamicallies
ontop,northernChinesesecond,ethnicChineseandminoritiesatbottom;inaddition,
Mongolspromotedsocialadvanceofartisansandmerchants,whohadbeen
discriminatedagainstintraditionalChinesesociety.
Diff:2
PageRef:425-430
Topic:TheMongolInterludeinChineseHistory
Skill:Conceptual

319

5) WhatcanbesaidtohavebeenthepositiveaspectsoftheMongolconquests?
Answer: Madepossibleandpromotedcommercialandculturalexchangesbetweenglobal
civilizationsofEasternHemisphere;broughtstablegovernmentbasedonprecedentsin
IslamicandChineseadministrationandreligioustolerationtomuchofAsia;provided
lengthyperiodofpeacepredicatedonestablishmentofunifiedlawcodeatleastuntil
1260.
Diff:2
PageRef:415-430
Topic:Conclusion:TheMongolLegacyandanAftershock
Skill:Conceptual

320

Chapter20 TheWestandtheChangingWorldBalance
Multiple-ChoiceQuestions
1) WhichofthefollowingwasNOTasymptomofdeclineintheArabiccaliphateby1300?
A) Thenarrowingofintellectuallifesymbolizedbythetriumphofreligionoverliterature,
philosophy,andscience
B) Landlordsseizedpoweroverpeasants
C) ThedeclineoftheSufis
D) Declineoftaxrevenuesforthestate
Answer: C
Diff:2
PageRef:436-437
Topic:TheDeclineoftheOldOrder
Skill:Conceptual

2) BywhatdatehadtheMiddleEasternpeasantsincreasinglylosttheirfreedom,becomingserfs
onlargeestates?
A) 900
B) 1000
C) 1100
D) 1350
Answer: C
Diff:2
PageRef:437
Topic:TheDeclineoftheOldOrder
Skill:Factual

3) WhichofthefollowingstatementsconcerningArabictradeafter1100ismostaccurate?
A) Arabiccontroloftheseaswasstrengthenedfollowing1100.
B) AlthoughArabicdynamismintradewasreduced,Muslimsremainedactiveinworld
markets.
C) ThetotalcollapseoftheIslamicworldinthe12thcenturycanbestbecomparedtothe
falloftheRomanEmpire.
D) TheArabtradingcomplexwasreducedafter1100totheMiddleEast.
Answer: B
Diff:2
PageRef:437
Topic:TheDeclineoftheOldOrder
Skill:Conceptual

321

4) WhichofthefollowingstatementsconcerningthepoliticalfragmentationoftheArabicworld
ismostaccurate?
A) AfterthefalloftheAbbasidcaliphate,theemergingOttomanEmpiresoonmastered
mostofthelandsoftheoldcaliphateplustheByzantinecorner.
B) ThepoliticalfragmentationcausedbythefallofBaghdadlastedforseveralcenturies
underthedecentralizedadministrationoftheSeljukTurks.
C) TheMongolconquestseliminatedanyformofcentralizedgovernmentintheMiddle
Eastuntilthe17thcentury.
D) FollowingthefalloftheAbbasidcaliphate,theMiddleEastbecamepartofthecolonial
empireoftheemergingfeudalstatesofwesternEurope.
Answer: A
Diff:2
PageRef:437
Topic:TheDeclineoftheOldOrder
Skill:Conceptual

5) WhichofthefollowingstatementsconcerningtheOttomanEmpireismostaccurate?
A) TheriseoftheOttomanEmpirerestoredthefullinternationalvigorthattheIslamic
caliphatehadpossessed.
B) TurkishrulerspromotedtrademoreactivelythandidtheirArabpredecessors.
C) TheexpansionistpoweroftheOttomanEmpirewasveryreal,butthefocusonconquest
andadministrationovershadowedwidercommercialambitions.
D) TheOttomanEmpirehadnoexpansionistinterestsorcapabilities.
Answer: C
Diff:2
PageRef:437
Topic:TheDeclineoftheOldOrder
Skill:Conceptual

6) WhatarearepresentedanewconquestfortheOttomanEmpire?
A) AsiaMinor
B) NorthAfrica
C) SoutheasternEurope
D) Mesopotamia
Answer: C
Diff:2
PageRef:437
Topic:TheDeclineoftheOldOrder
Skill:Conceptual

322

7) WhichofthefollowingstatementsconcerningtheOttomanEmpireismostaccurate?
A) TurkishrulersdidnotpromotemaritimetradeasvigorouslyashadtheArabs.
B) Scientificandphilosophicalinvestigationsreachedthelevelofinnovationthattheyhad
enjoyedundertheAbbasids.
C) TheTurksrefusedtopatronizethetraditionalPersianartistsandcraftsmenwhohad
dominatedthelaterAbbasidcourt.
D) TheOttomansweremoreinterestedinculturalpatronagethaninmilitaryorganization.
Answer: A
Diff:2
PageRef:437
Topic:TheDeclineoftheOldOrder
Skill:Conceptual

8) Whichofthefollowingcivilizationsfirstattemptedtofillthecommercialvacuumcreatedby
thedeclineinArabictrade?
A) Russia
B) Japan
C) China
D) India
Answer: C
Diff:2
PageRef:437
Topic:TheDeclineoftheOldOrder
Skill:Factual

9) WhatChinesedynastysucceededtheMongolYuandynastyinChina?
A) Chou
B) Ming
C) Han
D) Tang
Answer: B
Diff:2
PageRef:437
Topic:TheDeclineoftheOldOrder
Skill:Factual

323

10) WhatwastheinnovationlaunchedbytheMingdynasty?
A) ReceivingtributepaymentsfromKorea
B) ExtendingtheirpoliticalcontroloverVietnam
C) Useofacentralizedbureaucracy
D) Mountinghuge,state-sponsoredtradingexpeditionsthroughoutAsiaandbeyond
Answer: D
Diff:2
PageRef:437
Topic:TheDeclineoftheOldOrder
Skill:Conceptual

11) InwhatyeardidtheMingdynastyhaltstate-sponsoredcommercialvoyages?
A) 1358
B) 1405
C) 1433
D) 1487
Answer: C
Diff:2
PageRef:437
Topic:TheDeclineoftheOldOrder
Skill:Factual

12) WhichofthefollowingwasNOTareasonusedbytheMingdynastytohaltthetrading
expeditions?
A) Theoppositionofthescholar-gentryandbureaucracy
B) ThetechnologicalinferiorityofChineseshipsandnavigation
C) ThegrowingmilitaryexpensesofthecampaignsagainsttheMongols
D) ThetraditionalpreferenceoftheChineseforAsianproducts
Answer: B
Diff:2
PageRef:437
Topic:TheDeclineoftheOldOrder
Skill:Conceptual

324

13) Whichofthefollowingstatementsconcerningthecessationofstate-sponsoredtradebythe
Mingdynastyismostaccurate?
A) ThecessationoftradeseverelydamagedtheinternaleconomyofChinaandproduced
theinevitablepeasantrevolutionsthatoverthrewtheMingdynasty.
B) TheendofinternationaltradesignaledageneraldecentralizationofgovernmentinMing
China.
C) BecauseoftheChinesedependenceonimportsfromabroad,thedecisiontoendthe
state-sponsoredexpeditionswasparticularlycriticalininitiatingculturaldecline.
D) InChineseterms,itwasthebrieftradingflurrythatwasunusual,notitscessation.
Answer: D
Diff:2
PageRef:437-438
Topic:TheDeclineoftheOldOrder
Skill:Conceptual

14) WhatadmiralcommandedChinasgreatoverseasexpeditionsbetween1405and1433?
A) Zhenghe
B) JungTziLung
C) XunXi
D) YanXuanshang
Answer: A
Diff:1
PageRef:437
Topic:TheRiseoftheWest
Skill:Conceptual

15) WhichofthefollowingwasNOTadrawbacktotheWestsemergenceasaglobalpower?
A) WesternnationslackedthepoliticalcoherenceandorganizingabilityofimperialChina.
B) TheWestdidnotbegintoestablishkeymaritimeandcommerciallinksuntilafter1600.
C) Thechurch,longoneoftheorganizinginstitutionsofWesterncivilization,wasunder
attack.
D) ThelivesandeconomicactivitiesofordinaryEuropeans,theartisansandpeasants,were
inseriousdisarray.
Answer: B
Diff:2
PageRef:438
Topic:TheRiseoftheWest
Skill:Conceptual

325

16) WhichofthefollowingwasNOTacontributingfactortotheeconomiccrisesofthe14th
century?
A) Withdrawalfromtheglobaltradingnetwork
B) Bubonicplague
C) Lackoftechnologicaladvanceinagriculture
D) Recurrentfamine
Answer: A
Diff:2
PageRef:438-439
Topic:TheRiseoftheWest
Skill:Conceptual

17) WhatproportionoftheEuropeanpopulationdiedasaresultofthe14th -centuryplague?


A) onetenth
B) onefourth
C) onethird
D) onehalf
Answer: C
Diff:2
PageRef:438
Topic:TheRiseoftheWest
Skill:Factual

18) WhichofthefollowingwasNOTasourceofWesterndynamisminthe14thand15th
centuries?
A) Thestrengtheningoffeudalmonarchy
B) Thegrowthofcitiesandurbaneconomies
C) Advancesinmetallurgy
D) Twocenturiesofpeaceamongthemajornations
Answer: D
Diff:2
PageRef:438-439
Topic:TheRiseoftheWest
Skill:Conceptual

19) Strongregionalmonarchiestookholdinthisperiodin
A) FranceandEngland
B) theOttomanEmpire
C) SpainandPortugal
D) GermanyandAustria
Answer: C
Diff:2
PageRef:439
Topic:TheRiseoftheWest
Skill:Conceptual

326

20) FromwhatsourcedidmanytechnologicalinnovationsarriveintheWestduringthe13thand
14thcenturies?
A) Asia
B) Africa
C) TheAmericas
D) TheIslamicregionsoftheMiddleEast
Answer: A
Diff:1
PageRef:439
Topic:TheRiseoftheWest
Skill:Factual

21) InwhatregionofEuropedidtheRenaissancebegin?
A) Germany
B) Italy
C) France
D) England
Answer: B
Diff:1
PageRef:440
Topic:TheRiseoftheWest
Skill:Factual

22) Incomparisontomedievalculture,Renaissanceculturewas
A) moreconcernedwithAristotelianphilosophy.
B) moreconcernedwiththingsofthisworld.
C) disinterestedinclassicalmodels.
D) basedlessonurbanvitalityandexpandingcommerce.
Answer: B
Diff:2
PageRef:442
Topic:TheRiseoftheWest
Skill:Conceptual

327

23) WhichofthefollowingwasNOToneofthereasonsthatItalyemergedasthecenterofthe
earlyRenaissance?
A) TheemergenceofcentralizedstatesinItalyallowedformoreextensivepatronageofthe
arts.
B) ItalyretainedmorecontactwithRomantraditionsthandidtherestofEurope.
C) ItalyledtheWestbythe14thcenturyinbankingandtrade.
D) Italyhadclosercontactswithforeignscholars,particularlythoseinlateByzantium.
Answer: A
Diff:2
PageRef:442
Topic:TheRiseoftheWest
Skill:Conceptual

24) WhatFlorentinepainterledthewayinthemovementtowardnatureandpeopleasthe
primarysubjectmatterofRenaissanceart?
A) Giotto
B) Petrarch
C) Masaccio
D) FraAngelico
Answer: A
Diff:2
PageRef:442
Topic:TheRiseoftheWest
Skill:Factual

25) WhatItaliancity-statewasbestplacedtoengageinthenew,Western -orientedcommercial


venturesofthe15thcentury?
A) Venice
B) Florence
C) Genoa
D) Pisa
Answer: C
Diff:2
PageRef:442
Topic:TheRiseoftheWest
Skill:Factual

328

26) AlongwithItaly,akeycenterforpoliticalchangeinthe14thand15thcenturieswas
A) France.
B) Germany.
C) England.
D) theIberianpeninsula.
Answer: D
Diff:2
PageRef:442
Topic:TheRiseoftheWest
Skill:Factual

27) WhatwasuniqueaboutthedevelopmentofstatesintheIberianpeninsula?
A) Thesegovernmentswerebasedoncity-statesratherthannation-states.
B) BasedonCastileandAragon,theIberianstateswereuniqueintheiradoptionofIslam.
C) SpainandPortugaldevelopedeffectivenewgovernmentswithaspecialsenseof
religiousmissionandreligioussupport.
D) ThestatesofSpainandPortugalwereabletodevelopwithoutemphasisonthemilitary.
Answer: C
Diff:2
PageRef:442
Topic:WesternExpansion:TheExperimentalPhase
Skill:Conceptual

28) WhichofthefollowingwasNOTaninternationaleconomicdifficultyexperiencedbytheWest
by1400?
A) WesterneliteswereaccustomedtoincreasingconsumptionofAsianluxuryproducts.
B) TradewithAsiahadceasedasaresultoftheMongolconquests.
C) Westerntradeforluxurieshadtobepaidforingold,establishinganegativebalanceof
trade.
D) TheriseoftheOttomanEmpireonceagainclosedofftheeasternMediterraneanfor
Westernmerchantsandshipping.
Answer: B
Diff:2
PageRef:440
Topic:WesternExpansion:TheExperimentalPhase
Skill:Conceptual

329

29) WhatwastheWesternresponsetotheproblemsofinternationaltradethattheyexperiencedin
1400?
A) WesternnationshaltedtradewithAsiaandtheEastandbecamemoredependenton
European-producedcommodities.
B) ManynationssoughttoestablishallianceswiththeOttomanEmpireinordertorestore
thetraderoutes.
C) OverlandtraderoutesthroughnorthernRussiawereestablishedtotheEast.
D) WesternnationsbeganexplorationsofalternativeroutestoAsiathatwouldbypassthe
MiddleEastandMuslimrealms.
Answer: D
Diff:2
PageRef:442-443
Topic:WesternExpansion:TheExperimentalPhase
Skill:Conceptual

30) ThekeythemeofPolynesianculturefromthe7thcenturyto1400was
A) theadoptionofJapanesecivilizationintheislandsocieties.
B) thedevelopmentofauniformwrittenscript.
C) contractionasaresultoftheworld-wideepidemicofthe14thcentury.
D) spurtsofmigrationandconquestthatimplantedPolynesianculturebeyondtheinitial
baseintheSocietyIslands.
Answer: D
Diff:2
PageRef:445-446
Topic:OutsidetheWorldNetwork
Skill:Conceptual

31) WhichofthefollowingwasNOTcharacteristicofHawaiianculture?
A) Urbanization
B) Animalhusbandryfeaturingswine
C) Warlikeregionalkingdoms
D) Highlystratifiedsocialstructure
Answer: A
Diff:2
PageRef:446-448
Topic:OutsidetheWorldNetwork
Skill:Conceptual

330

32) WhichofthefollowingrepresentsasignificantdifferencebetweenNewZealandandHawaii?
A) Artbasedoncarvedwood
B) Acoldandharshclimate
C) Ahighlystratifiedsociety
D) Asocietybasedonwarfare
Answer: B
Diff:1
PageRef:447
Topic:OutsidetheWorldNetwork
Skill:Conceptual

33) Thepracticeofjudgingotherpeoplesbythestandardsandpracticesofonesowncultureof
ethnicgroupis
A) ethnocentrism.
B) genocide.
C) anthropomorphism.
D) aversionism.
Answer: A
Diff:2
PageRef:446
Topic:WesternExpansion:TheExperimentalPhase
Skill:Conceptual

34) WhichofthefollowingwasNOTaresultoftheEuropeancontactwithsub-SaharanAfrica
after1500?
A) TradepatternsinwestAfricashiftedfromtheMediterraneantotheAtlantic
B) TradeshiftedinwestAfricafromMuslimtoEuropeanhands
C) SeizureofslavesforEuropeanuseaffectedmanyregionsdeeply
D) RegionalkingdomslostallinfluenceinwestAfricaandwerereplacedbyEuropean
governments
Answer: D
Diff:2
PageRef:447
Topic:OutsidetheWorldNetwork
Skill:Conceptual

EssayQuestions
1) AccountforthedeclineoftheMiddleEast.
Answer: DeclineofintellectualvigoraccompanieddisintegrationofAbbasidEmpire;emphasis
shiftedtoreligionandawayfromphilosophyandscience;riseofSufis;landlordsseized
controlofland,reducedpeasantrytoserfdom;declineinstaterevenuesfromtaxation;
declineofinterestininternationaltrade.
Diff:2
PageRef:436-438
Topic:TheDeclineoftheOldOrder
Skill:Conceptual

331

2) EvaluatethepositionofChinaintheinternationalbalanceofpowerin1450.
Answer: ChinahadrestoredpoliticalunityunderMingdynasty;temporaryinterestinexpansion
ofinternationaltrade;endedin1433withwithdrawaltointernaleconomy;remained
economicallyprosperous;requiredlittlefromothercivilizations,butshutofffrom
technologicalandscientificdynamismofpost-1500West.
Diff:2
PageRef:438-439
Topic:TheDeclineoftheOldOrder
Skill:Conceptual

3) WhatwerethesourcesofdynamismthatfueledtheWesternadvancesrelativetoother
civilizations?
Answer: Populationexpandedatmorerapidratethananyothercontinent;feudalmonarchy
providedbeginningsofmorecentralizedpoliticalunits;growthofcitiesandurban
economiesspurredcommercialdevelopment;technology,particularlyironworking,
continuedtoadvance;senseofweaknessincomparisontoothercivilizationsledto
searchfornewtraderoutesleadingtoAtlanticexploration;Renaissanceopenedupnew
culturalavenuesinWest.
Diff:2
PageRef:438-442
Topic:TheRiseoftheWest
Skill:Conceptual

4) DescribeearlyWesternexplorationandcolonizationpatterns.
Answer: NeedfornewroutestoavoidOttomandominanceofeasternMediterranean,establish
morefavorablebalanceoftrade;explorationsbeguninAtlantic,reachedislandgroups ,
theCanaries,Madeira,Azores;someexplorationofAfricancoastline;earlysettlementof
islandgroupsestablishedpatternsforlatercolonizationofAmericas;establishmentof
largeagriculturalestatestoproducemoneycrops(sugar,cotton,tobacco);extensiveuse
ofslavelabor.
Diff:2
PageRef:442-445
Topic:WesternExpansion:TheExperimentalPhase
Skill:Conceptual

5) Inwhatsensewerethecivilizationsoutsidetheworldnetworkofculturalexchange(the
Americas,Africa,andPolynesia)vulnerabletointeractionwithothercivilizations?
Answer: Americas:empires(AztecandInca)disintegratingduetointernalstresses;lacked
technology;unexposedtodiseasescommoninEurope.Africanpoliticalunits
experiencedvarioussuccess;declineofZimbabwe,Mali,riseofSonghay;Africabetter
equippedtodealwithinteraction;hadmetallurgy;exposedtoEuropeandiseases;
alreadyinvolvedinlong-distancetradesystems;lessisolatedthanothercivilizations.
Polynesia,likeAmericas,totallyisolatedfromworldtradenetwork;individualisland
groupsbecomingmoreisolated;unexposedtoEuropeandiseases;lackedmetallurgy.
Diff:2
PageRef:445-448
Topic:OutsidetheWorldNetwork
Skill:Conceptual

332

Chapter21 TheWorldEconomy
Multiple-ChoiceQuestions
1) WhichcountrieswerethelargestrecipientsofNewWorldsilver?
A) RussiaandMongolia
B) AustraliaandNewZealand
C) ChinaandIndia
D) BritainandFrance
Answer: C
Diff:2
PageRef:458
Topic:Introduction
Skill:Conceptual

2) Whichofthefollowingstatementsistrue?
A) From1100Europeanleadersandmerchantsisolatedthemselvesfromtherestofthe
world.
B) From1100EuropeanleadersandmerchantshadbeguntocolonizetheAmericas.
C) From1100EuropeanleadersandmerchantshadforgedtradinglinkswiththeAmericas.
D) From1100Europeanleadersandmerchantshadbecomeincreasinglyawareofthelarger
world.
Answer: D
Diff:1
PageRef:460
Topic:Introduction
Skill:Factual

3) WhohadcrossedtheAtlanticinthe10thcenturyandnamedNorthAmericaVinland?
A) China
B) TheVikings
C) Africans
D) MiddleEasterners
Answer: B
Diff:1
PageRef:492
Topic:Introduction
Skill:Factual

333

4) WhichofthefollowingwasNOTatechnologicalimprovementintroducedduringthe15th
centuryintheWest?
A) Useofthecompassfornavigation
B) Deep-draft,round-hulledships
C) Lateensails
D) Gunpowderadaptedtogunnery
Answer: C
Diff:2
PageRef:460-461
Topic:TheWestsFirstOutreach
Skill:Factual

5) TheinitiativeforWesternexplorationandconquestcamefromthekingdomof
A) Portugal.
B) Spain.
C) France.
D) Sicily.
Answer: A
Diff:2
PageRef:461
Topic:TheWestsFirstOutreach
Skill:Factual

6) WhatnoblewasresponsibleforthedirectionofaseriesofexpeditionsalongtheAfricancoast
andoutwardtotheAzoresinthe15thcentury?
A) PrinceHenrytheNavigator
B) CardinalMazarin,regentofFrance
C) HughDupuy,CountMarechal
D) PrinceHenryofEngland
Answer: A
Diff:2
PageRef:461
Topic:TheWestsFirstOutreach
Skill:Factual

7) ThefirstPortuguesefleetroundedtheCapeofGoodHopein
A) 1291.
B) 1354.
C) 1488.
D) 1519.
Answer: C
Diff:2
PageRef:461
Topic:TheWestsFirstOutreach
Skill:Factual

334

8) ThecaptainofthefirstPortugueseshiptoreachIndiawas
A) AmerigoVespucci.
B) VascodaGama.
C) ChristopherColumbus.
D) FerdinandMagellan.
Answer: B
Diff:2
PageRef:461
Topic:TheWestsFirstOutreach
Skill:Factual

9) WhatregionintheAmericaswasclaimedbyPortugal?
A) Mexico
B) Peru
C) Panama
D) Brazil
Answer: D
Diff:1
PageRef:461
Topic:TheWestsFirstOutreach
Skill:Factual

10) WhatlandwasclaimedforSpainastheresultofFerdinandMagellanscircumnavigationof
theglobebeginningin1519?
A) Philippines
B) NewZealand
C) Chile
D) Australia
Answer: A
Diff:2
PageRef:463
Topic:TheWestsFirstOutreach
Skill:Factual

11) WhatItaliancaptainsailingforthemonarchsofSpainreachedtheAmericasin1492?
A) FerdinandMagellan
B) AmerigoVespucci
C) DanteAlighieri
D) ChristopherColumbus
Answer: D
Diff:1
PageRef:461
Topic:TheWestsFirstOutreach
Skill:Factual

335

12) WhydidtheinitiativeinearlyconquestandexplorationpasstonorthernEuropeannationsin
thelater16thcentury?
A) SpainandPortugalweredefeatedinacriticalwarwiththeOttomanEmpire.
B) TheDutchandtheBritishimprovedthedesignofoceanicvessels,producingfasterships
thantheirCatholicrivals.
C) FamineanddiseasedisastrouslyreducedthepopulationoftheIberianpeninsulaafter
1588.
D) TheSpanishdefeatoftheEnglishArmadacutEnglandofffromfurtheradvancein
EuropeandforcedEnglishattentiontoforeignconquest.
Answer: B
Diff:2
PageRef:463
Topic:TheWestsFirstOutreach
Skill:Conceptual

13) WhatwasthepurposeoftheearlyEnglishvoyagestoNorthAmerica?
A) Toestablishcolonies
B) Tocreateafortifiedportandtradingregion
C) TodiscoveranorthwestpassagetoIndia
D) TodrivetheSpanishfromtheAmericas
Answer: C
Diff:2
PageRef:464
Topic:TheWestsFirstOutreach
Skill:Conceptual

14) InwhatwayweretheearlyDutchandBritishexplorationandtradeprojectsdifferentfrom
thoseoftheIberiannations?
A) TheexpeditionsofSpainandPortugaldidnotenjoygovernmentsupport.
B) DutchandBritishexplorationowedmuchtoprivateinitiativeofmerchantgroupsand
theformationofcharteredtradingcompanies.
C) DutchandBritishexploratoryexpeditionswereindependentoftheirrespective
governments.
D) TheDutchandBritishoperatedjointexplorationsinthenamesofbothgovernments
whilePortugalandSpaincompetedinthecompetitionforconquest.
Answer: B
Diff:2
PageRef:465
Topic:TheWestsFirstOutreach
Skill:Conceptual

336

15) WhatBritishcommercialinstitutionruledIndiaformuchofthe18thcentury?
A) TheBritishTeaandSpiceCompany
B) TheBritishEastIndiaCompany
C) TheRajTradingCompany
D) TheCalcuttaandMadrasLimited
Answer: B
Diff:1
PageRef:465
Topic:TheWestsFirstOutreach
Skill:Factual

16) WhichofthefollowingwasNOTaresultofEuropesmaritimedominance?
A) Itcreatedanewinternationalpoolforbasicexchangesoffoods,diseases,andafew
manufacturedproducts.
B) Itcreatedanewworldeconomy,involvingthefullerinclusionofAfricaandthe
Americasininternationaltrade.
C) AllregionsofAsiawereforthefirsttimesubjecttothemonopolizationoftradeby
Westernnations.
D) ItcreatedtheconditionsfordirectWesternpenetrationofsomepartsoftheworld
throughtheformationofcolonies.
Answer: C
Diff:2
PageRef:465
Topic:TowardaWorldEconomy
Skill:Conceptual

17) AspartoftheColombianExchange,whichofthefollowingwasaEuropeancontributionto
theAmericas.
A) Bullion
B) Rawmaterials
C) Disease
D) Corn
Answer: C
Diff:2
PageRef:465
Topic:TowardaWorldEconomy
Skill:Conceptual

337

18) WhatisthemostacceptedfigureforthepercentageofthepopulationofAmericanIndians
whodiedfollowingtheEuropeancolonization?
A) Over15percent
B) Over25percent
C) Over33percent
D) Over50percent
Answer: D
Diff:2
PageRef:465
Topic:TowardaWorldEconomy
Skill:Factual

19) WhatwastheimpactoftheintroductionofAmericancropsintoEurope?
A) TheintroductionofthepotatoledtomajorpopulationgrowthinEurope.
B) FungiintroducedtoEurope,alongwithAmericancrops,ledtoaseveredecreasein
agriculturalproductivity.
C) AlthoughAmericancropswereintroducedaroundtheworldbyEuropeantraders,they
werenotadoptedinEuropeitself.
D) PlantationagriculturefueledbyslavelaborbecamethenorminEuropeanagricultural
systems.
Answer: A
Diff:2
PageRef:466
Topic:TowardaWorldEconomy
Skill:Conceptual

20) WhichofthefollowingareasoftradewasNOTdominatedbytheWestaftertheestablishment
ofaglobaltradingnetworkinthe17thcentury?
A) TheAtlantic
B) ThePacific
C) TheIndianOcean
D) TheMediterranean
Answer: C
Diff:2
PageRef:466
Topic:TowardaWorldEconomy
Skill:Conceptual

338

21) ASpanishfleetdefeatedtheOttomanEmpirein1571atthebattleof
A) theSargassoSea.
B) Lepanto.
C) Civitate.
D) Nicaea.
Answer: B
Diff:2
PageRef:466
Topic:TowardaWorldEconomy
Skill:Factual

22) DespiteJapansofficialpolicyofisolation,theDutchwereabletogainspecialaccesstothe
portof
A) Osaka.
B) Kyoto.
C) Nissei.
D) Nagasaki.
Answer: D
Diff:2
PageRef:468
Topic:TowardaWorldEconomy
Skill:Factual

23) WhichofthefollowingstatementsaccountsfortheSpanishfailuretoholdapositionof
dominanceinworldtrade?
A) TheSpanishwithdrewvoluntarilyfromtheraceforworldtradedominanceand
establishedapolicyofinternationalisolation.
B) TheCatholicchurchthatdominatedSpanishsocietyarguedagainsttheestablishmentof
acommercialmentalityinSpain.
C) SpainsinterestswereincreasinglydirectedtowardthedestructionoftheOttoman
Empire.
D) Spainsinternaleconomyandbankingsystemwerenotsufficienttoaccommodatethe
bullionfromthenewworld.
Answer: D
Diff:2
PageRef:466
Topic:TowardaWorldEconomy
Skill:Conceptual

339

24) Whateconomicpolicysupplementedthedevelopmentofcolonies,particularlyinnorthern
Europe?
A) Mercantilism
B) Freetrade
C) Socialism
D) Communism
Answer: A
Diff:1
PageRef:466
Topic:TowardaWorldEconomy
Skill:Factual

25) WhatimportfromtheAmericashelpedSpaintoinitiallydominateworldtrade?
A) cows
B) silver
C) horses
D) copper
Answer: B
Diff:2
PageRef:466
Topic:TowardaWorldEconomy
Skill:Conceptual

26) WhichofthefollowingregionswasNOTpartofthecorezoneoftheglobaltradenetwork?
A) Holland
B) France
C) England
D) Sub-SaharanAfrica
Answer: D
Diff:2
PageRef:466
Topic:TowardaWorldEconomy
Skill:Conceptual

27) Dependenceintheworldeconomyandtheconsequentneedtoproduceunprocessedgoods
cheaplyledtothedevelopmentof__________laborsystems.
A) proletariat
B) independent
C) coercive
D) socialist
Answer: C
Diff:1
PageRef:467
Topic:TowardaWorldEconomy
Skill:Conceptual

340

28) Whichofthefollowingstatementsmostaccuratelydescribestheimpactofthedevelopmentof
core-dependenteconomiczonesonstateformation?
A) ForcedlaborandEuropeaninfluencetendedtogenerateweakgovernmentsin
dependentregions,whileincreasedtraderevenuestendedtogenerateincreasing
governmentstrengthincorestates.
B) Whiletheprofitsofglobaltradetendedtostrengthenthegovernmentsofcoreregions,
thecreationofcoloniesextendedpowerfulgovernmentstodependentzonesaswell.
C) Theeffortsofinternationaltradetendedtoenrichprivatecommercialinterestsbut
weakencoregovernmentswhowereunabletotapthewealth.Dependentzone
governments,basedoncompanyorganization,tendedtobestrong.
D) Nostategovernmentbenefittedfromthecreationofcore-dependentzones.Neithercore
regionsnorcolonieswereabletodevelopstrong,centralizedgovernments.
Answer: A
Diff:2
PageRef:466-467
Topic:TowardaWorldEconomy
Skill:Conceptual

29) WhichofthefollowingstatementsconcerningtherelationshipbetweenAsiancivilizationsand
theworldcommercialnetworkofthe16thand17thcenturiesisNOTaccurate?
A) Asiancivilizationshadamplepoliticalstrengthandeconomicsophisticationtoavoid
dependentstatus.
B) EastAsiaconstitutedthecivilizationthatremainedmostfullyandconsciouslyexternal
totheworldeconomy.
C) ChinadependedonextensivegovernmentregulationtokeepEuropeanactivitiesin
check.
D) Chinawasable,thankstotheexistenceofitscoastalnavy,topreventtheestablishment
ofEuropeanports.
Answer: D
Diff:2
PageRef:468
Topic:TowardaWorldEconomy
Skill:Conceptual

30) WhichofthefollowingWesterntradegoodswasofmostinteresttotheJapanese?
A) Gunnery
B) Porcelain
C) Woolencloth
D) Cottoncloth
Answer: A
Diff:2
PageRef:468
Topic:TowardaWorldEconomy
Skill:Factual

341

31) Whichofthefollowingcivilizationswasfullypartoftheglobaltradingnetworkinthe16th
century?
A) Russia
B) OttomanEmpire
C) Indonesia
D) MughalEmpire
Answer: C
Diff:2
PageRef:468
Topic:TowardaWorldEconomy
Skill:Conceptual

32) WhichofthefollowingstatementsconcerningtheBritishpoliciestowardIndiainthe17th
centuryismostaccurate?
A) Britainattemptedtofosterthedevelopmentofthecottonmanufacturingindustryin
India.
B) BritainappliedtariffstodestroythecottonindustryinIndiaasameansofprotectingthe
Britishclothindustry.
C) BritainwasdisinterestedinexpandingthetradewithIndia.
D) TheBritishbalanceoftradewithIndiainthe17thcenturyremainednegative,asBritain
wasforcedtoexchangebullionforIndianproducts.
Answer: B
Diff:2
PageRef:468
Topic:TowardaWorldEconomy
Skill:Conceptual

33) WhatwasthenatureofthetradebetweeneasternandwesternEuropeinthe17thcentury?
A) WesternEuropeimportedgraininincreasingamountsfromeasternEuropeinreturnfor
artobjectsandmanufacturedgoods.
B) EasternandwesternEuroperemainedeconomicallyisolated,asRussia,Poland,and
PrussiaextendedtheirtraderelationshipswiththeOttomanEmpire.
C) TheinabilityofeasternEuropetoproduceanythingofvaluetotheWestfrustrated
attemptstoestablishtradingconnectionsbetweenthetwohalvesofEurope.
D) EasternEuropeemergedasapowerfulchallengertoWesterndominationoftheglobal
commercialnetwork.
Answer: A
Diff:2
PageRef:469
Topic:TowardaWorldEconomy
Skill:Conceptual

342

34) InwhatyeardidSpanishsettlementoftheAmericanmainlandbegin?
A) 1492
B) 1509
C) 1588
D) 1607
Answer: B
Diff:2
PageRef:469
Topic:ColonialExpansion
Skill:Factual

35) WherewasthefirstSpanishcolonyontheAmericanmainland?
A) Peru
B) Panama
C) Mexico
D) Florida
Answer: B
Diff:2
PageRef:469
Topic:ColonialExpansion
Skill:Factual

36) WhichofthefollowingregionswasNOTpartoftheSpanishcolonialempire?
A) TheAztecempire
B) TheIncaempire
C) Cuba,Jamaica,andPuertoRico
D) Brazil
Answer: D
Diff:1
PageRef:469-470
Topic:ColonialExpansion
Skill:Factual

37) WhileitwasmostcommonfortheEuropeanstoestablishcoastalfortressesinAfricawithout
attemptingtoclaimlargeterritoriesoftheirown,Portugalcreatedamoredisruptivecolonyin
__________insearchofslaves.
A) CapeColony
B) Angola
C) theSudan
D) Zimbabwe
Answer: B
Diff:2
PageRef:474
Topic:ColonialExpansion
Skill:Factual

343

38) ThemostimportantDutchcolonyinAfricawaslocatedon
A) Mozambique.
B) Madagascar.
C) theCapeofGoodHope.
D) theislandofCeylon.
Answer: C
Diff:1
PageRef:474
Topic:ColonialExpansion
Skill:Factual

39) TheBritishEastIndiaCompanythroughnegotiationwithlocalMughalprincesgaineda
stationat
A) Goa.
B) Calcutta.
C) Constantinople.
D) Delhi.
Answer: B
Diff:1
PageRef:474
Topic:ColonialExpansion
Skill:Factual

40) TheBritishwereabletoousttheFrenchfromIndiafollowingthe
A) Taipeirebellion.
B) Sepoyrebellion.
C) battleofManzikert.
D) SevenYearsWar.
Answer: D
Diff:2
PageRef:474
Topic:ColonialExpansion
Skill:Factual

41) WhendidtheDutchestablishtheCapeColony?
A) 1707
B) 1744
C) 1652
D) 1803
Answer: C
Diff:2
PageRef:474
Topic:ColonialExpansion
Skill:Factual

344

42) WhatwasaBoer?
A) Dutchfarmer
B) Indianprince
C) Britishofficer
D) Frenchcitizen
Answer: A
Diff:2
PageRef:474
Topic:ColonialExpansion
Skill:Conceptual

43) InwhichofthefollowingregionswasEuropeansettlementasignificantfactorinthe
establishmentofcolonies?
A) WestIndies
B) Indonesia
C) DutchSouthAfrica
D) China
Answer: C
Diff:2
PageRef:474
Topic:ColonialExpansion
Skill:Conceptual

44) Whichofthefollowingrepresentsanimpactofthedevelopmentofcoloniesonwestern
Europe?
A) ColonialdevelopmentresultedinagreatersenseofcooperationamongEuropean
nations.
B) Thedevelopmentofcolonieshastenedthegrowthofcentralizedgovernmentsand
destroyedthegrowthofthemerchantclassinwesternEurope.
C) TheuseofcoloniallyproducedsugarspreadwidelyinEurope,particularlyincountries
withrelativelycoldclimates.
D) TheintroductionofcolonialisminplaceslikeIndiaresultedinbringingthese
civilizationswithintheframeworkofWesterncivilization.
Answer: C
Diff:2
PageRef:475-476
Topic:ColonialExpansion
Skill:Conceptual

345

45) InwhichofthefollowingcoloniesdidafullerdevelopmentofWesterninstitutionsandvalues
takeplace?
A) SpanishLatinAmerica
B) FrenchandBritishcoloniesofNorthAmerica
C) Brazil
D) India
Answer: B
Diff:1
PageRef:472
Topic:ColonialExpansion
Skill:Conceptual

46) WhydidthesoutherncoloniesoftheAtlanticseaboardwinimportancebeforethosefarther
north?
A) GoldwasdiscoveredwithinthesoutherncoloniesoftheAtlanticseaboard.
B) Theharshclimateofthenortherncoloniesleftthoseregionsvirtuallyunsettled.
C) OnlythesoutherncolonieswereabletoeliminatethenativeIndianpopulation.
D) Patternsofplantationproductionofcashcropsproducedbycoercivelaboremerged
there.
Answer: D
Diff:2
PageRef:472
Topic:ColonialExpansion
Skill:Conceptual

47) WhichofthefollowingstatementsconcerningtheinteractionofNorthAmericancolonists
withtheIndiansismostaccurate?
A) ColonistsinteractedwithIndians,learnedfromthem,andmisusedthem,butdidnot
forgeanewculturalamalgamasoccurredinmuchofLatinAmerica.
B) ConstantwarfarebetweenthenumeroussedentaryagriculturaltribesandtheEuropean
colonistsresultedinlimitedimmigrationfromEurope.
C) RapidintermarriagebetweenEuropeansandtheNativeAmericansresultedinthe
creationofanewclassofpeople,themestizos,whocontinuedtoplayasignificantrolein
NorthAmericancolonialdevelopment.
D) TheoccurrenceofdiseasethatrapidlydecimatedtheIndianpopulationsofLatin
AmericadidnottakeplaceinNorthAmerica,thusIndianpopulationsremainedlarge
andintermingledwiththeEuropeanimmigrants.
Answer: A
Diff:2
PageRef:473
Topic:ColonialExpansion
Skill:Conceptual

346

48) AsaresultofwhattreatydidtheFrenchlosetheircolonyinNorthAmericatotheBritish?
A) TreatyofWestphalia,1648
B) TreatyofParis,1763
C) TreatyofUtrecht,1714
D) TreatyofRyswick,1705
Answer: B
Diff:2
PageRef:472
Topic:ColonialExpansion
Skill:Factual

EssayQuestions
1) DescribetheearlyEuropeanexplorationoftheworld.Characterizethenatureofearly
settlement.
Answer: InitialexplorationsinthehandsofSpanishandPortuguese;developmentofAfrican
coast,Caribbeanislands,Brazil;PortuguesevoyagestoIndia;Magellansvoyage
openedupPacifictoexplorationandconquest;DutchopenedupIndonesia,established
colonyonsoutherntipofAfrica;BritishandFrenchbeganexplorationofNorth
America.WithexceptionofDutchcolonyinAfrica,mostofearlycolonizationlimitedto
establishmentoffortressesandtradingpostsoncoastsofexploredregions.
Diff:2
PageRef:460-465
Topic:TheWestsFirstOutreach
Skill:Conceptual

2) WhatwasthenatureoftheColombianExchange?
Answer: TheAmericasreceiveddiseasesfromEurope,Africanslaves,domesticatedanimals
(horses,cattle,swine);Westgotbullion,rawmaterials,Americancrops(corn,tobacco,
sugar,coffee,potato).BenefitsoftheexchangewerealmostentirelytotheEuropeans.
Asidefromtheintroductionoflargedomesticatedanimals,theAmericasbenefited
minimally.DiseasesravagedIndianpopulations.ImportationofAmericancropshelped
tofuelEuropeanpopulationgrowth,aidinreducingfoodshortages.
Diff:2
PageRef:465-468
Topic:TowardaWorldEconomy
Skill:Conceptual

3) TheEuropeanexpansionofthe16thand17thcenturiescreatedanimbalanceinworldtrade.
Discusstheemergenceofdominantcoreareasandperipheraldependentzones.
Answer: Coreareaswerethoseareasoftheworldeconomytypifiedbyproductionof
manufacturedgoods,controlofshipping,monopolyofbanking,andcommercial
services.CoreareaswerelocatedprimarilyinnorthwesternEurope:Britain,France,
andHolland.Dependentzoneswereregionstypifiedbyproductionofrawmaterials,
supplyofbullion,plantationagricultureofcashcropsproducedbycoercivelabor
systems.DependentzonessurroundedtheEuropeancoreincludingsouthernand
easternEurope,Asia,andthecolonialdiscoveriesoftheEuropeanexplorers.
Diff:2
PageRef:465-476
Topic:TowardaWorldEconomy
Skill:Conceptual

347

4) Priorto1600,whatareasoftheworldremainedoutsidetheglobaltradingnetwork?What
areaswerethenincorporatedintothenetworkafter1600?
Answer: EastAsia,particularlyChinaandJapanremainedoutsideofglobaltradenetwork;
MughalIndiaonlyminimallyinvolved;OttomanEmpirerestrictedtradetoEuropean
enclavesincities;Russiaalsoremainedoutsidesystem;outsideofslaveregions,Africa
notinvolved.After1600,IndiaincreasinglydominatedbyFranceandEngland;Eastern
EuropebroughtintosystemassupplierofgraintoWest.
Diff:2
PageRef:467-471
Topic:TowardaWorldEconomy
Skill:Conceptual

5) Ofallthecolonialsystems,theAtlanticcoloniesofNorthAmericamostfullyincorporated
Westerncivilization.InwhatwaysweretheAtlanticcoloniessimilartotheWest?Inwhat
waysdidtheydiffer?
Answer: Similarities:marriagepatternswithemphasisonnuclearfamily;emphasisoncareof
children;developedrepresentativeinstitutionssimilartothoseofEurope;adopted
muchofEnlightenmentpoliticalphilosophy.Differences:lackedelaborateartand
urbanization;greatersenseofindividualfreedom,despiteexistenceofslaveryin
southerncolonies.economicconditionsmoreequalthaninEurope;lackofformal
aristocracy.
Diff:2
PageRef:471-474
Topic:ColonialExpansion
Skill:Conceptual

348

Chapter22 TheTransformationoftheWest,1450-1750
Multiple-ChoiceQuestions
1) WheredidtheRenaissancefirstdevelop?
A) Italy
B) Holland
C) England
D) Spain
Answer: A
Diff:2
PageRef:480
Topic:Introduction
Skill:Conceptual

2) WhichofthefollowingsequencesliststhemajordevelopmentsofWesterncivilizationin
propersequence?
A) ProtestantReformation,Renaissance,absolutemonarchy,Enlightenment
B) Absolutemonarchy,Renaissance,Enlightenment,ProtestantReformation
C) Renaissance,ProtestantReformation,absolutemonarchy,Enlightenment
D) Renaissance,ProtestantReformation,Enlightenment,absolutemonarchy
Answer: C
Diff:2
PageRef:480-492
Topic:Introduction
Skill:Factual

3) WhichofthefollowingstatementsabouttheRenaissanceisNOTaccurate?
A) TheRenaissancechallengedmedievalintellectualvaluesandstyles.
B) TheRenaissancesketchedbrasherspiritthatmayhavehelpedcreateanewWestern
interestinexploring.
C) TheRenaissancefailedtodevelopanynewideasconcerningpoliticalorganization.
D) TheRenaissancewasbuiltonamorecommercializedeconomy.
Answer: C
Diff:2
PageRef:481
Topic:TheFirstBigChanges:CultureandCommerce
Skill:Conceptual

349

4) WhichofthefollowingwasNOTaparticipantinthe15thcenturyItalianRenaissance?
A) Michelangelo
B) LeonardodaVinci
C) NiccoloMachiavelli
D) Giotto
Answer: D
Diff:2
PageRef:481
Topic:TheFirstBigChanges:CultureandCommerce
Skill:Factual

5) WhichofthefollowingstatementsconcerningItalianhumanismismostaccurate?
A) Humanistsfocusedonhumankindasthecenterofintellectualandartisticendeavor.
B) HumanistsattackedChristianityasrifewithsuperstitionandwitchcraft.
C) Humanistscarvedoutnewliterarystyleswithoutreferencetoclassicalormedieval
models.
D) Humanistsemphasizedthecorporateandcommunalaspectsofhumansociety.
Answer: A
Diff:2
PageRef:481
Topic:TheFirstBigChanges:CultureandCommerce
Skill:Conceptual

6) WhichofthefollowingwasNOTanimpactoftheRenaissanceonpoliticsandcommerce?
A) Renaissancemerchantsimprovedbankingtechniquesandbecamemorecapitalist.
B) Underhumanistinfluence,warsamongItaliancity-statesbecamelessfrequentand
violent.
C) City-stateleadersexperimentedwithnewpoliticalformsandfunctionsandjustified
theirauthorityonthebasisofwhattheycoulddotoadvancethegeneralwell -being.
D) Thecity-statesintroducedtheregularexchangeofambassadorsandtheexerciseof
diplomacy.
Answer: B
Diff:2
PageRef:481
Topic:TheFirstBigChanges:CultureandCommerce
Skill:Conceptual

350

7) WhichofthefollowingaccountsinpartforthedeclineoftheItalianRenaissanceca.1500?
A) RoutesthroughRussiatotheEastundercuttheItalianmonopolyoftrade.
B) Thecreationofasinglenation-stateinnorthernItalysappedthevitalityofartistic
patronage.
C) FrenchandSpanishmonarchsinvadedthepeninsula,cuttingdownonpolitical
independence.
D) MuchofItalywasconqueredbytheOttomanTurks.
Answer: C
Diff:2
PageRef:481
Topic:TheFirstBigChanges:CultureandCommerce
Skill:Conceptual

8) WhatwasoneoftheprimarydifferencesbetweentheNorthernandItalianRenaissances?
A) TheNorthernRenaissanceoccurredacenturyearlierthantheItalianRenaissance.
B) NorthernhumanistsfocusedmoreonreligionthantheirItaliancounterparts.
C) TherewerenomajorliteraryfiguresintheNorthernRenaissance.
D) TheNorthernRenaissancedidnotmakeuseoftheclassicallanguagestypicalofthe
ItalianRenaissance.
Answer: B
Diff:2
PageRef:481
Topic:TheFirstBigChanges:CultureandCommerce
Skill:Conceptual

9) JohannesGutenbergwasresponsiblefor
A) theunificationoftheHolyRomanEmpirein1537.
B) theconstructionofWittenbergcathedralduringthe15thcentury.
C) thedefeatoftheCatholicforcesduringtheThirtyYearsWar.
D) theinventionofmovabletypeintheWest.
Answer: D
Diff:1
PageRef:481
Topic:TheFirstBigChanges:CultureandCommerce
Skill:Factual

351

10) WhatwastheEuropean-stylefamilypatternthatemergedinthe15thcentury?
A) Extendedfamilies,earlymarriageages
B) Nuclearfamilies,earlymarriageages
C) Extendedfamilies,latemarriageages
D) Nuclearfamilies,latemarriageages
Answer: D
Diff:2
PageRef:482
Topic:TheFirstBigChanges:CultureandCommerce
Skill:Conceptual

11) WhatdeterminedtheageofmarriageformanypeopleinEurope?
A) Theoccupationofthehusband
B) Accesstorealproperty
C) Theapprovalofthechurch
D) Securinglicensetomarryfromthegovernment
Answer: B
Diff:2
PageRef:482
Topic:TheFirstBigChanges:CultureandCommerce
Skill:Conceptual

12) Bythe16thcentury,atwhatagedidmostEuropeansmarry?
A) Earlyteens
B) Early20s
C) Mid20s
D) Late20s
Answer: D
Diff:2
PageRef:482
Topic:TheFirstBigChanges:CultureandCommerce
Skill:Factual

13) WhoisgenerallycreditedwithinitiatingtheProtestantReformationin1517?
A) JeanCalvin
B) HenryVIII
C) IgnatiusLoyola
D) MartinLuther
Answer: D
Diff:1
PageRef:482
Topic:TheFirstBigChanges:CultureandCommerce
Skill:Factual

352

14) WhichofthefollowingwasNOTareligiouspropositionadvancedbyMartinLuther?
A) Saleofindulgence,orgrantsofsalvation,formoneywaswrong
B) Onlyfaithcouldgainsalvation
C) Monasticismwaswrong
D) Priestsshouldpracticecelibacy
Answer: D
Diff:2
PageRef:482
Topic:TheFirstBigChanges:CultureandCommerce
Skill:Conceptual

15) WhichofthefollowingstatementsmostaccuratelydescribesthereasonwhyLutherpickedup
widespreadsupportamongtheGermanelite?
A) LutherproposedmovingthepapacyfromRometoGermany.
B) LutherssupportforamorecentralizedGermangovernmentunderthecontrolofthe
HolyRomanEmperorstruckaresponsivechordinGermannationalism.
C) GermanprinceswhoturnedProtestantcouldincreasetheirindependencefromthe
emperor,seizechurchlands,andcontrolthechurchintheirterritories.
D) LutherproposedthatindulgencesshouldbecollectedbytheHolyRomanEmperor
insteadofthepope.
Answer: C
Diff:2
PageRef:482
Topic:TheFirstBigChanges:CultureandCommerce
Skill:Conceptual

16) WhichofthefollowingreasonssuggestswhycommonpeoplesupportedtheLutheran
Reformation?
A) LutheradvocatedtheoverthrowoftheauthorityoftheGermanprinces.
B) Lutheranismsanctionedmoney-makingandotherearthlypursuitsmorewholeheartedly
thandidtraditionalCatholicism.
C) Luthersreformsmeantthatindulgencesandotherecclesiasticalmeansofsalvation
wouldbecomelessexpensiveandmorereadilyavailabletothepoor.
D) LutheradvocatedredistributionoflandandpropertythroughoutGermany.
Answer: B
Diff:2
PageRef:482
Topic:TheFirstBigChanges:CultureandCommerce
Skill:Conceptual

353

17) WhatwasthechurchestablishedbyHenryVIIIinEngland?
A) Lutheran
B) Calvinism
C) Jesuit
D) Anglican
Answer: D
Diff:1
PageRef:482
Topic:TheFirstBigChanges:CultureandCommerce
Skill:Factual

18) ThetheologicalfoundationofJeanCalvinsProtestantismwas
A) thedoctrineofpenance.
B) iconodulism.
C) predestination.
D) solipsism.
Answer: C
Diff:2
PageRef:482
Topic:TheFirstBigChanges:CultureandCommerce
Skill:Factual

19) WhatwasthepoliticalimpactofCalvinism?
A) DuetothelocationofthecenterofCalvinisminSwitzerland,mostgovernmentsthat
acceptedthenewreligionwerecity-states.
B) BecauseoftheinsistenceofCalvinismoftheacceptanceofasingleecclesiastical
authority,Calvinismspreadrapidlyamongtheabsolutemonarchies.
C) Calvinismwasregardedassopotentiallyrevolutionarythatitfailedtofindafoothold
outsideofGermany.
D) Calvinistssoughttheparticipationofallbelieversinchurchadministration,whichhad
thepoliticalimplicationsofencouragingtheideaofawideraccesstogovernment.
Answer: D
Diff:2
PageRef:482
Topic:TheFirstBigChanges:CultureandCommerce
Skill:Conceptual

354

20) WhichofthefollowingareaswasNOTsuccessfullydefendedbytheCatholicReformation?
A) TheNetherlands
B) Poland
C) SouthernEurope
D) Hungary
Answer: A
Diff:2
PageRef:482
Topic:TheFirstBigChanges:CultureandCommerce
Skill:Factual

21) WhatnewreligiousorderwasassociatedwiththeCatholicReformation?
A) Benedictines
B) Jesuits
C) Calvinists
D) Cistercians
Answer: B
Diff:1
PageRef:482
Topic:TheFirstBigChanges:CultureandCommerce
Skill:Factual

22) TheEdictofNantes,issuedinFrancein1598,
A) grantedtolerancetoProtestantsandhelpedendtheFrenchcivilwarsofreligion.
B) establishedCalvinismasthestatereligionofFrance.
C) decreedtheabolitionofProtestantisminFrance.
D) declaredwaragainsttheLutheranprincesofGermany.
Answer: A
Diff:2
PageRef:483
Topic:TheFirstBigChanges:CultureandCommerce
Skill:Factual

23) WhichofthefollowingwasNOTaresultoftheThirtyYearsWar?
A) ItreducedGermanprosperityandpowerforafullcentury
B) ThetreatythatendedthewarestablishedSpainastheprincipalpowerofwestern
Europe
C) ThetreatythatendedthewargrantedpoliticalindependencetotheProtestant
Netherlands
D) SomeprincelystatesinGermanychoseonereligion,someanother
Answer: B
Diff:2
PageRef:483
Topic:TheFirstBigChanges:CultureandCommerce
Skill:Conceptual

355

24) ThereligiouswarsthatfollowedtheProtestantReformationledgenerallyto
A) therestorationofCatholicunity.
B) theestablishmentofProtestantdominance.
C) alimitedacceptanceoftheideaofreligiouspluralism.
D) theendoftheinvolvementofthestateinreligion.
Answer: C
Diff:2
PageRef:483
Topic:TheFirstBigChanges:CultureandCommerce
Skill:Conceptual

25) WhichofthefollowingstatesfellbackfromEuropeanascendancyfollowingthereligious
wars?
A) TheNetherlands
B) Spain
C) Britain
D) France
Answer: B
Diff:2
PageRef:484
Topic:TheFirstBigChanges:CultureandCommerce
Skill:Conceptual

26) Whichofthefollowingstatementsmostaccuratelydescribesachangeinpopularmentalityas
aresultoftheProtestantReformation?
A) Protestantsweremorelikelytocreditmiraclesordivineinterruptionsinnaturescourse.
B) Protestantchurches,asphysicalstructures,weremorecloselyconnectedtomarket
activitiesinthecities,encouragingtheideathatreligionanddailylifewererelated.
C) ProtestantsandCatholicsconsideredthefamilyinmorepositivetermsnotsimplyasan
institutionnecessarybecauseofhumanlust.
D) Religiouschangetendedtodiscouragethegrowthofliteracyintheerafollowingthe
ProtestantReformation.
Answer: C
Diff:2
PageRef:484
Topic:TheFirstBigChanges:CultureandCommerce
Skill:Conceptual

356

27) Whichofthefollowingwasnottypicalofthecommercialrevolutionofthe16thcentury?
A) SubstantialimportsofAmericanbullion
B) Formationofgreattradingcompanies
C) Stimulationofmanufacturing
D) Significantreductioninpricesencouragingconsumerspending
Answer: D
Diff:2
PageRef:485
Topic:TheFirstBigChanges:CultureandCommerce
Skill:Conceptual

28) Theaveragewesternpeasantorartisanownedabouthowmanytimesmorethingsthanhis
orhercounterpartinsoutheasternEurope?
A) 10
B) five
C) eight
D) three
Answer: B
Diff:2
PageRef:485
Topic:TheFirstBigChanges:CultureandCommerce
Skill:Factual

29) InflationandcommercializationintheWestproducedagroupofpeoplewithoutaccessto
producingpropertycalledthe
A) bourgeoisie.
B) sansculottes.
C) proletariat.
D) Provenales.
Answer: C
Diff:1
PageRef:485
Topic:TheFirstBigChanges:CultureandCommerce
Skill:Factual

357

30) WhichofthefollowingwasNOTaWesternresponsetothecommercialrevolutionofthe16th
century?
A) Amorecaringattitudetowardtheproblemsofthepoor
B) Amoreelaboratefamilylifeincludinggreatermaterialwealth
C) Awaveofpopularprotestresultinginrisingscausedbymassivedislocation
D) Greaterbeliefinpersonalachievementandthedemystificationofnature
Answer: A
Diff:3
PageRef:485
Topic:TheFirstBigChanges:CultureandCommerce
Skill:Conceptual

31) WhousedastronomicalobservationandmathematicalcalculationtodisprovetheHellenistic
beliefthattheEarthwasthecenteroftheuniverse?
A) Galileo
B) Copernicus
C) Vesalius
D) FrancisBacon
Answer: B
Diff:1
PageRef:486
Topic:ScienceandPolitics:TheNextPhaseofChange
Skill:Factual

32) WilliamHarveywasresponsibleforwhatdiscoveryduringtheScientificRevolution?
A) Oxygen
B) Themathematicalformulaeforgravity
C) Themathematicalcalculus
D) Thecircularmovementofbloodinanimals
Answer: D
Diff:2
PageRef:487
Topic:ScienceandPolitics:TheNextPhaseofChange
Skill:Factual

358

33) WhatwasIsaacNewtonsworkpublishedin1687thatdrewvarioustheoriestogetherintoa
frameworkofnaturallaws?
A) Analects
B) Principia
C) AnalogicaPedagogica
D) ArsMystica
Answer: B
Diff:1
PageRef:487
Topic:ScienceandPolitics:TheNextPhaseofChange
Skill:Factual

34) WhichofthefollowingstatementsmostaccuratelysummarizestheviewofDeists?
A) Whiletheremaybeadivinity,itsroleisonlytosetnaturallawsinmotion.
B) Godcanbefoundinallelementsofcreation,whetherplant,animal,ormineral.
C) Theauthorityofthechurchisparamount,andallpoliticalpowerisderivedfromdivine
sanction.
D) TheinstitutionalchurchhasfailedWesternsociety,anditisnecessaryforthe
reestablishmentofthechurchthroughnewinstitutionsfoundedbythestate.
Answer: A
Diff:2
PageRef:487
Topic:ScienceandPolitics:TheNextPhaseofChange
Skill:Conceptual

35) HowdidtheWesternviewofsciencecomparewiththatofothercivilizations?
A) TheWestwastheonlycivilizationtodevelopscientificandtechnologicalexpertise.
B) InChina,sciencewasbasedonpractical,empiricaladvances.
C) TheWestwasnotaloneindevelopingcrucialscientificdata,butitsthinkerswerethe
onlyonestoseescienceinbroaderphilosophicaltermsascentraltointellectuallife.
D) IslamremainedvastlyaheadoftheWestintermsofscientificknowledge,despitethe
clearadvancesmadeduringthescientificrevolutionofthe16thand17thcenturies.
Answer: C
Diff:3
PageRef:487
Topic:ScienceandPolitics:TheNextPhaseofChange
Skill:Conceptual

359

36) Allofthefollowingweretypicalof16thcenturyabsolutemonarchyEXCEPT
A) aprofessionalizedarmy.
B) thecessationofparliamentarygovernment.
C) agrowingbureaucracy.
D) thedestructionofprovincialcouncils.
Answer: D
Diff:3
PageRef:489
Topic:ScienceandPolitics:TheNextPhaseofChange
Skill:Conceptual

37) Themonarchmostassociatedwithabsolutemonarchywas
A) CharlesIofEngland.
B) FrederickWilliamofPrussia.
C) WilliamofOrangeoftheNetherlands.
D) LouisXIVofFrance.
Answer: D
Diff:1
PageRef:489
Topic:ScienceandPolitics:TheNextPhaseofChange
Skill:Factual

38) WhichcountrydidLouisXIVrule?
A) Spain
B) France
C) England
D) Belgium
Answer: B
Diff:2
PageRef:489
Topic:ScienceandPolitics:TheNextPhaseofChange
Skill:Conceptual

39) WherewasthebasicstructureofabsolutemonarchydevelopedoutsideofFrance?
A) TheNetherlands
B) Britain
C) Italy
D) Prussia
Answer: D
Diff:2
PageRef:489
Topic:ScienceandPolitics:TheNextPhaseofChange
Skill:Factual

360

40) Whichofthefollowingstatesstoodapartfromthetrendtowardabsolutemonarchyinthe17th
centuryandretainedaparliamentaryregime?
A) France
B) Britain
C) Spain
D) Austria-Hungary
Answer: B
Diff:2
PageRef:489
Topic:ScienceandPolitics:TheNextPhaseofChange
Skill:Factual

41) WhatmonarchisassociatedwiththeestablishmentofenlighteneddespotisminPrussiainthe
middleofthe18thcentury?
A) JosephII
B) MariaTeresa
C) WilliamIII
D) FredericktheGreat
Answer: D
Diff:1
PageRef:492
Topic:TheWestby1750
Skill:Factual

42) TheaftermathoftheScientificRevolutionspilledoverintoanewintellectualmovementinthe
18thcenturycalledthe
A) Renaissance.
B) Enlightenment.
C) GreatAwakening.
D) Risorgimento.
Answer: B
Diff:1
PageRef:492
Topic:TheWestby1750
Skill:Factual

361

43) AdamSmithseconomictheoryadvocated
A) governmentinterventioninordertocontroltheflowofbullionthroughextensivetariff
systems.
B) theuseofacontrolledmoneysupplyasameansoflimitinginflation.
C) thatgovernmentsavoidregulationinfavoroftheoperationofindividualinitiativeand
marketforces.
D) theinstitutionofstate-controlledguildstofixstandardsofproductionandwages.
Answer: C
Diff:2
PageRef:492
Topic:TheWestby1750
Skill:Conceptual

44) WhichofthefollowingwasNOTabasicprincipleoftheEnlightenment?
A) Societysgoalsshouldcenteronimprovementsinmaterialandsociallife.
B) Religionsthatreliedonfaithorrefusedtotoleratediversitywerewrong.
C) Ifpeoplewerenotcontrolled,generalsocialdeclinewasinevitable.
D) Humanbeingsarenaturallygoodandcanbeeducatedtobebetter.
Answer: C
Diff:2
PageRef:492
Topic:TheWestby1750
Skill:Conceptual

45) Whichofthefollowingchangesassociatedwithtreatmentofchildrenwasassociatedwiththe
Enlightenment?
A) Parentsbecamemoreinterestedinfreermovementandgreaterinteractionforyoung
children.
B) Physicaldisciplineofchildrentoencouragetheirdevelopmentbecamemorecommon.
C) Swaddlingofinfantscontinuedasameansofprotectinginfantsfrominjury.
D) Childhoodwasnolongerperceivedasastageforlearningandgrowth.
Answer: A
Diff:2
PageRef:494
Topic:TheWestby1750
Skill:Conceptual

362

46) Howdidagriculturechangeinthelate17thcentury?
A) WesternEuropecontinuedtorelylargelyonthemethodsandtechniquescharacteristic
oftheMiddleAges.
B) TomatoeswereintroducedfromtheAmericasandrapidlybecameamajorfoodsourcein
westernEurope.
C) Thepracticeoffallowingwasintroducedtorestorefertilityoffields.
D) Newtechnologyandbetterstock-breedingmethodsresultedinhigherproductivity.
Answer: D
Diff:2
PageRef:494
Topic:TheWestby1750
Skill:Conceptual

47) Whichofthefollowingstatementsmostaccuratelydescribesthenatureofmanufacturingin
thelater18thcentury?
A) Thefactorysystemwaswellestablishedsincethecommercialrevolutionofthe16th
centuryandcontinuedtodevelopduringthisperiod.
B) Bythe18thcenturytheeconomicgrowthtypicalofthe16thcenturyhadhalted,and
manufacturingsufferedfromthewithdrawalofcapital.
C) The18thcenturywitnessedarapidspreadofhouseholdproductionoftextilesandmetal
products,mostlybyruralworkerswhoalternatedmanufacturingwithsomeagriculture.
D) Thelackofnewtechnologycausedabottleneckinthemanufacturingprocessesandled
tostagnationinEuropeanproductivity.
Answer: C
Diff:2
PageRef:494
Topic:TheWestby1750
Skill:Conceptual

48) Whichofthefollowingstatementsconcerningmid-18thcenturyWesternsocietyismost
accurate?
A) Agriculturalchanges,commercialism,andmanufacturinghadcombinedtoproducea
rapidlygrowingpopulationintheWest.
B) Radicalchangesinthenatureofgovernmentresultedinthecreationofessentiallynew
politicalforms.
C) AsaresultoftheEnlightenment,establishedchurchesnolongerwereforcestobe
reckonedwithinWesternsociety.
D) Thespreadofdomesticmanufacturingdestroyedthetraditionalhabitsandfamily
patternsofearlierEurope.
Answer: A
Diff:3
PageRef:494
Topic:TheWestby1750
Skill:Conceptual

363

49) By1750,thestrandsofcommercial,cultural,andpoliticalchangehadbeencombinedtocreate
A) anunstablepoliticalenvironmentthatwouldeventuallyregressbackwards.
B) proofoftheinnatesuperiorityofWesterncivilization.
C) rapidadaptationnotfoundinothercivilizations.
D) anunusualversionofanagriculturalcivilization.
Answer: D
Diff:2
PageRef:495
Topic:Conclusion:InnovationandInstability
Skill:Conceptual

50) In1733,JamesKayofEnglandintroduced
A) double-entrybookkeepingtoWesternEurope.
B) theflyingshuttletoautomateweaving.
C) thesteamengine.
D) thepotatotoEuropeanagriculture.
Answer: B
Diff:1
PageRef:495
Topic:TheWestby1750
Skill:Factual

EssayQuestions
1) InwhatwayswasWesternEuropein1750differentfromthemedievalWest?
Answer: Economymorecommercialized,developmentofnewsocialgroups(proletariat)
associatedwithprocess;revolutioninWesterncultureassociatedwiththeScientific
RevolutionandEnlightenment;movementfromfeudalmonarchytomorecentralized
nation-state,eitherabsolutemonarchyorparliamentarymonarchy;Westmostdynamic
world-widetraderandcolonizer.
Diff:2
PageRef:478-496
Topic:Introduction
Skill:Conceptual

2) CompareandcontrasttheRenaissanceandtheScientificRevolution.
Answer: Renaissanceemphasizedliteraryandartisticclassicism;basedonhumanism,greater
secularisminItaly;NorthernRenaissancemoreorientedtowardChristianity;Scientific
Revolutionbasedondevelopmentofnaturallaws;greaterbeliefinhumanperfectibility;
sciencebrokewithclassicalperceptions;destructionofAristotelianandPtolemaic
astronomy.
Diff:2
PageRef:480-489
Topic:TheFirstBigChanges:CultureandCommerce
Skill:Conceptual

364

3) Inwhatwaysdidthecommercialrevolutionofthe16thcenturychangethesocialstructureof
theWest?
Answer: Producedproletariat, peoplewithoutaccesstorealproperty;workedindomestic
manufacturing,asagriculturallabor,orasunemployed,urbanpoor;createdgreater
divisionsbetweenestatessignifiedbypopularrisingsof16thand17thcenturies;
distrustofpoorreflectedinwitchcrafthysteria.
Diff:2
PageRef:485-489
Topic:TheFirstBigChanges:CultureandCommerce
Skill:Conceptual

4) Howwastheabsolutemonarchyofthe17thcenturydifferentfromthepoliticalformsofthe
MiddleAges?
Answer: Balancebetweenmonarchyandnoblesshiftedinfavorofthemonarchy;lossoffeudal
independence;monarchsgainednewpowers;moreambitiousmilitaryorganization
markedbyprofessionalizedarmies,improvedmethodsoftaxcollection;appointmentof
bureaucraciesmorecommon;declineofparliamentarygovernment;useofmercantilism
asstate-controlledeconomicsystem.
Diff:2
PageRef:489-492
Topic:ScienceandPolitics:TheNextPhaseofChange
Skill:Conceptual

5) HowdidtheEnlightenmenteffectchangesinpopularoutlook?
Answer: Beliefinhumanperfectibility,applicationofscientificdiscoveriestoimprovementof
humancondition;reasonwaskeytotruth,whilereligionwasafflictedwith
superstition;changesinupbringingofchildren,reductionofphysicaldiscipline,more
education,greaterbondsoffamilialaffection;changesineconomyreflectedinmass
consumerism;greatertechnologyappliedtoagriculture,nitrogen-fixingcrops,land
drainage,improvedstock-breeding,newtoolssuchasseeddrill,introductionofpotato
asmajorfoodcrop;growthofreadingclubs,coffeehouses,andpopularentertainment.
Diff:2
PageRef:492-494
Topic:TheWestby1750
Skill:Conceptual

365

Chapter23 TheRiseofRussia
Multiple-ChoiceQuestions
1) MuchoftheterritoryaddedtotheRussianempirewas
A) Balkan.
B) African.
C) Asian.
D) Polynesian.
Answer: C
Diff:2
PageRef:500
Topic:Introduction
Skill:Factual

2) WhatpoliticalcenterservedasthefocalpointfortheRussianliberationfromtheMongols?
A) Kiev
B) DuchyofMoscow
C) St.Petersburg
D) PrincipateofNovgorod
Answer: B
Diff:1
PageRef:500
Topic:RussiasExpansionistPoliciesUndertheTsars
Skill:Factual

3) UnderwhatRussianrulerwasalargepartofRussiafreedfromMongolcontrolinthe15th
century?
A) PeterIII
B) IvanIII
C) IvanIV
D) AlexisRomanov
Answer: B
Diff:2
PageRef:500
Topic:RussiasExpansionistPoliciesUndertheTsars
Skill:Factual

366

4) WhatwasthestateoftheRussianeconomyimmediatelyaftertheexpulsionoftheMongolsin
the15thcentury?
A) Russiawasalreadyadependentregionwithintheglobaleconomydominatedbythe
West.
B) FueledbytheestablishmentoftheTartartraderouteswiththeEast,Russiahad
developedasignificantexporttradeandmerchantclass.
C) Russiahadbecomeamorepurelyagriculturaleconomy,dependentonpeasantlabor.
D) RussiaseconomictieswerealmostexclusivelywiththeOttomanEmpireandhencewith
Africa.
Answer: C
Diff:2
PageRef:500
Topic:RussiasExpansionistPoliciesUndertheTsars
Skill:Conceptual

5) WhatgovernmentdidIvantheGreatclaimtohavesucceededasthethirdRome?
A) TheByzantineEmpire
B) TheAbbasidEmpire
C) TheOttomanEmpire
D) TheUmayyadEmpire
Answer: A
Diff:1
PageRef:500
Topic:RussiasExpansionistPoliciesUndertheTsars
Skill:Factual

6) IvanIV,calledIvantheTerrible,
A) wishedtoconfirmtsaristautocracybyattackingtheauthorityofboyars.
B) abandonedtheprinciplesofterritorialexpansioninfavorofcentralizingpowerathome.
C) alliedhimselfwiththeRussianaristocracyinapolicyofpoliticaldecentralization.
D) wasresponsiblefortheincorporationofPolandintotheRussianEmpire.
Answer: A
Diff:2
PageRef:500
Topic:RussiasExpansionistPoliciesUndertheTsars
Skill:Conceptual

367

7) WhydidtheRussianexpansionpolicyfocusparticularlyoncentralAsia?
A) TheRussianswishedtoseizecontrolofthetraderouteswithChina.
B) MostoftheRussianpopulationremainedethnicallyMongolwithclearculturaltiesto
centralAsia.
C) Therewerenaturalbarrierstowestwardexpansion.
D) TheRussiansweremotivatedbyadesiretopushtheformerMongoloverlordsfarther
backtopreventrenewedinvasion.
Answer: D
Diff:2
PageRef:500
Topic:RussiasExpansionistPoliciesUndertheTsars
Skill:Conceptual

8) PeasantsrecruitedtomigratetonewlyseizedlandsintheRussianEmpirewerecalled
A) emigrati.
B) boyars.
C) Cossacks.
D) OldBelievers.
Answer: C
Diff:1
PageRef:501
Topic:RussiasExpansionistPoliciesUndertheTsars
Skill:Factual

9) WhatwastheimpactofearlyRussianexpansiononcentralAsia?
A) None
B) IndependentcentralAsia,thesourceofnomadicculturesandinvasionforces,was
eliminated
C) ChinesetradewasrefocusedthroughRussiaandeasternEurope
D) Russiansocietybecamemoreculturallyandethnicallyhomogeneous
Answer: B
Diff:2
PageRef:501
Topic:RussiasExpansionistPoliciesUndertheTsars
Skill:Conceptual

368

10) WhichofthefollowingwasNOTaformofcontactwiththeWestduringthereignsofthe
Ivans?
A) DiplomaticmissionsweresenttoleadingWesternstates
B) WesternmerchantsestablishedoutpostsinMoscowandothercenters
C) Italianartistsandcraftsmenwereimportedtodesignchurchbuildingsandtheroyal
palaceintheKremlin
D) MilitaryalliancesweresignedwithSpainandPortugal
Answer: D
Diff:2
PageRef:501-502
Topic:RussiasExpansionistPoliciesUndertheTsars
Skill:Conceptual

11) FollowingthedeathofIvanIV,Russianboyarsattemptedtolimittsaristautocracyandgain
governingrightsforthemselvesduringthe
A) Russiancivilwar.
B) BoyarWar.
C) TimeofTroubles.
D) GreatSchism.
Answer: C
Diff:1
PageRef:502
Topic:RussiasExpansionistPoliciesUndertheTsars
Skill:Factual

12) Whatfamilywasselectedin1613toestablishanewrulingdynastyinRussia?
A) Romanov
B) Radishev
C) Molotov
D) Habsburg
Answer: A
Diff:1
PageRef:502
Topic:RussiasExpansionistPoliciesUndertheTsars
Skill:Factual

369

13) WhichofthefollowingwasNOTanaccomplishmentofTsarMichael?
A) Theestablishmentofinternalorder
B) Drivingoutforeigninvaders
C) Restorationofautocracy
D) InclusionofpartoftheUkraine,includingKiev,intheRussianempire
Answer: C
Diff:2
PageRef:502
Topic:RussiasExpansionistPoliciesUndertheTsars
Skill:Conceptual

14) Whattsarwasresponsiblefortheabolitionoftheassembliesofthenoblesandreformofthe
Orthodoxchurch?
A) IvanIII
B) IvanIV
C) PeterIII
D) Alexis
Answer: D
Diff:2
PageRef:502
Topic:RussiasExpansionistPoliciesUndertheTsars
Skill:Factual

15) RussianswhorefusedtoaccepttsaristreformsoftheOrthodoxchurchandwhowereexiledto
Siberiafortheirconservatismwerecalled
A) Cossacks.
B) boyars.
C) Bogomils.
D) OldBelievers.
Answer: D
Diff:1
PageRef:502
Topic:RussiasExpansionistPoliciesUndertheTsars
Skill:Factual

16) Politically,whataspectsofWesternculturedidPetertheGreatemulateinRussia?
A) Parliamentarygovernment
B) Aristocraticcontrolofthebureaucracy
C) Streamlinedbureaucracyandreorganizedmilitary
D) Republicanism
Answer: C
Diff:2
PageRef:503
Topic:RussiasFirstWesternization
Skill:Conceptual

370

17) WhatcountrydidPetertheGreatgotowarwithandasaresultgainterritoryontheeastern
coastoftheBalticSea?
A) China
B) England
C) TheOttomanEmpire
D) Sweden
Answer: D
Diff:2
PageRef:504
Topic:RussiasFirstWesternization
Skill:Conceptual

18) WhatseabecamecriticalinthedevelopmentofRussianpowerduringthereignofPeterthe
Great?
A) Caspian
B) Mediterranean
C) Baltic
D) Azov
Answer: C
Diff:2
PageRef:504
Topic:RussiasFirstWesternization
Skill:Factual

19) WhichofthefollowingwasNOToneofPetertheGreatspoliticalreforms?
A) Improvedmilitaryweaponry
B) Eliminationofnoblecouncils
C) Reductionoftaxesonthepeasantry
D) Systemizationoflawcodes
Answer: C
Diff:2
PageRef:504
Topic:RussiasFirstWesternization
Skill:Conceptual

20) PetertheGreatestablishedanewcapitalforRussiaat
A) Kiev.
B) Moscow.
C) Novgorod.
D) St.Petersburg.
Answer: D
Diff:1
PageRef:504
Topic:RussiasFirstWesternization
Skill:Factual

371

21) WherewasPetertheGreatsprogramofeconomicdevelopmentconcentrated?
A) Clothproduction
B) Miningandmetallurgicalindustries
C) Urbanization
D) Potteryproduction
Answer: B
Diff:2
PageRef:504
Topic:RussiasFirstWesternization
Skill:Factual

22) WhatwasthepurposeofPetertheGreatspolicyofeconomicdevelopment?
A) Tocreateafreeworkingclassdevotedtoindustryratherthanagriculture
B) Tocreateawealthymerchantclasscapableoffinancingfurtherexpansion
C) ToraisethestandardoflivingofallRussians
D) Toavoidtheneedofimportingmilitaryarmaments
Answer: D
Diff:2
PageRef:504
Topic:RussiasFirstWesternization
Skill:Conceptual

23) WhatwasthelimitationofPetertheGreatspoliciesofculturalWesternization?
A) HemadenoattempttointroduceWesterneducation,particularlyintechnological
subjects.
B) Westernizationwaslimitedtotheelite.
C) Petermadenoattempttoenforceculturalreforms.
D) Despitetsaristproclamations,WesternizationfailedtohaveanyimpactonRussian
society.
Answer: B
Diff:2
PageRef:505
Topic:RussiasFirstWesternization
Skill:Conceptual

24) FollowingthedeathofPetertheGreat,thenextpowerfulrulerofRussiawas
A) CatherinetheGreat.
B) PeterIII.
C) IvanIV.
D) AlexanderIII.
Answer: A
Diff:1
PageRef:505
Topic:RussiasFirstWesternization
Skill:Factual

372

25) WhatwasCatherinetheGreatsattitudetowardtheprogramofWesternization?
A) CatherineflirtedvigorouslywiththeideasoftheFrenchEnlightenment,butfailedto
takestepstoabolishserfdom.
B) CatherinerejectedtheconceptsofWesternizationinfavorofadistinctiveRussian
culture.
C) CatherineearnedthetitleofEnlightenedMonarchbyfullyembracingtheideasofthe
FrenchEnlightenment,includingtheabolitionoftheserfs.
D) CatherinewaseagertocontinuethepolicyofWesternization,butwasunabletoattract
WesternphilosopherstobackwardRussia.
Answer: A
Diff:2
PageRef:506
Topic:RussiasFirstWesternization
Skill:Conceptual

26) WhatwastherelationshipbetweenCatherinetheGreatsgovernmentandlocal
administration?
A) ThegovernmentestablishedbyCatherinetheGreatexercisedexclusivecontroloverboth
centralandlocalgovernment.
B) Thetsaristgovernmentcombinedarealmonopolyofformalpoliticsbyacentral
administration,butovernoble-ownedestatesthepowerofthegovernmentwas
nonexistent.
C) Thepowersofthegovernmentwereexercisedalmostentirelybythenobilityandlittle
formalauthoritywasexercisedbythecentraladministration.
D) UnderCatherinetheGreat,therewaslittlecentralauthority,butvillagegovernments
underthejurisdictionofpeasantheadmencontinuedtofunction.
Answer: B
Diff:2
PageRef:506
Topic:RussiasFirstWesternization
Skill:Conceptual

27) TheRussianradicalandnoblemanwho,inspiredbythephilosophiesoftheWest,urgedthe
abolitionofserfdomduringthereignofCatherinetheGreatwas
A) Turgenev.
B) Pugachev.
C) Ulanov.
D) Radishev.
Answer: D
Diff:1
PageRef:506
Topic:RussiasFirstWesternization
Skill:Factual

373

28) WhichofthefollowingareaswasNOTcolonizedorclaimedbytheRussianEmpireduringthe
reignofCatherinetheGreat?
A) Alaska
B) PartsofSiberia
C) Bengal
D) ThePacificcoastofNorthAmericaasfarsouthasmodernCalifornia
Answer: C
Diff:1
PageRef:508
Topic:RussiasFirstWesternization
Skill:Factual

29) AllofthefollowingcountriesparticipatedinthesuccessivepartitionsofPolandEXCEPT
A) Prussia.
B) Hungary.
C) Austria.
D) Russia.
Answer: B
Diff:1
PageRef:506
Topic:RussiasFirstWesternization
Skill:Factual

30) WhichofthefollowingstatementsconcerningthepoliticalvoiceoftheRussiannobilityismost
accurate?
A) ThepoliticsoftheRussiannobilitywereexpressedthroughserviceinthetsariststateand
thepreeminentpowertheywieldedoverthepeasantserfs.
B) TheRussiannobilityexercisedenormousinfluencethroughthepowerfulassembliesthat
continuedtolegislatethroughthe18thcentury.
C) TheexclusionoftheRussiannobilityunderCatherinetheGreatfromanyroleinthe
centralgovernmentorthemilitaryrelegatedthemtostrictlylocalauthority.
D) UnderCatherinetheGreattheRussiannobilitywasvirtuallyexterminated.
Answer: A
Diff:2
PageRef:508
Topic:ThemesinEarlyModernRussianHistory
Skill:Conceptual

374

31) WhatwasoneoftheprimarydifferencesbetweenthesocialorganizationoftheWestand
Russiainthe17thand18thcenturies?
A) RussiasmerchantclasswasmorefullydevelopedthanthatoftheWest.
B) TheWesthadnoformalaristocracybythe18thcentury,butinRussiathenobility
retainedtheirpoliticalandsocialfunction.
C) RussiasawaprogressiveintensificationofserfdomwhiletheWestwasrelaxingthis
institutioninfavorofotherlaborsystems.
D) TheagriculturallaboroftheWestwassubjecttoamorerestrictiveformofserfdomthan
thatofRussia.
Answer: C
Diff:2
PageRef:510
Topic:ThemesinEarlyModernRussianHistory
Skill:Conceptual

32) WhendidRussianserfdombecomehereditary?
A) 1482
B) 1558
C) 1618
D) 1649
Answer: D
Diff:3
PageRef:509
Topic:ThemesinEarlyModernRussianHistory
Skill:Factual

33) ThedevelopmentofcoercivelaborsystemsineasternEuropewasindicativeof
A) RussiaseconomicdominanceoverAsiantrade.
B) easternEuropesgrowingeconomicsubordinationtotheWest.
C) thecreationofacoreeconomiczoneinRussia.
D) thedevelopmentofindustrialstrengthintheregion.
Answer: B
Diff:2
PageRef:509
Topic:ThemesinEarlyModernRussianHistory
Skill:Conceptual

375

34) WhatportionoftheRussianpeasantrywereserfsin1800?
A) Half
B) Quarter
C) 10percent
D) Fivepercent
Answer: A
Diff:2
PageRef:509
Topic:ThemesinEarlyModernRussianHistory
Skill:Factual

35) WhatpercentageoftheRussianpopulationremainedruralinthe18thcentury?
A) 95percent
B) 75percent
C) 60percent
D) 50percent
Answer: A
Diff:2
PageRef:510
Topic:ThemesinEarlyModernRussianHistory
Skill:Factual

36) WholedtheRussianpeasantrebellionofthe1770s?
A) Radishev
B) Constantinov
C) Pugachev
D) Kosygin
Answer: C
Diff:2
PageRef:510
Topic:ThemesinEarlyModernRussianHistory
Skill:Factual

37) AsubstantialmerchantclassinRussiaduringthe18thcentury
A) wasrestrictedtothecitiesofMoscow,Novgorod,andSt.Petersburg.
B) sprangfrompeasantorigins.
C) emergedamongthelessernobilitywithintheRussiancities.
D) failedtodevelop.
Answer: D
Diff:2
PageRef:510
Topic:ThemesinEarlyModernRussianHistory
Skill:Conceptual

376

38) ComparedtoWesterngovernments,howgreataroledidtheRussiangovernmentplayin
economicdevelopment?
A) Becauseoftheabsenceofamerchantclass,theRussiangovernmentplayedagreaterrole
thanwascommoninWesternstates.
B) LiketheWest,thegovernmentsroleintheeconomywaslargelyrestrictedto
establishingtariffsandprotectivemeasuresfordomesticindustries.
C) BecauseofthedependentnatureoftheRussianeconomy,thegovernmentplayeda
relativelylesssignificantroleineconomicdevelopment.
D) TheRussiangovernmenthadnoroleineconomicdevelopment,atasktheyabandoned
toforeigninvestors.
Answer: A
Diff:2
PageRef:510
Topic:ThemesinEarlyModernRussianHistory
Skill:Conceptual

39) WhichofthefollowingisNOTindicativeofRussiasgrowingeconomicdependenceonthe
Westduringthe18thcentury?
A) Exportationofrawmaterials, furs,grain,andtimber
B) Importationofluxuriesandsomemanufacturedgoods
C) Acoercivelaborsystem
D) Self-sufficiencyinmetalsandweapons
Answer: D
Diff:2
PageRef:510
Topic:ThemesinEarlyModernRussianHistory
Skill:Conceptual

40) WholedapeasantrebellioninRussiainthe1770s?
A) PetertheIII
B) CatherinetheGreat
C) PetertheGreat
D) Pugachev
Answer: D
Diff:1
PageRef:510
Topic:ThemesinEarlyModernRussianHistory
Skill:Conceptual

377

41) IvantheGreatdeclaredthattheRussianEmpirewasthesuccessortowhichgreatempire?
A) HanChina
B) Rome
C) GuptaIndia
D) TheOttomanEmpire
Answer: B
Diff:2
PageRef:500
Topic:RussiasExpansionistPoliciesUndertheTsars
Skill:Factual

42) EasternEuropesharedwithRussiaallofthefollowingcharacteristicsEXCEPT
A) thedominanceofthelandedaristocracy.
B) rigidserfdom.
C) thelackofanativecommercialclassandasignificanturbanculture.
D) thedevelopmentofempire.
Answer: D
Diff:2
PageRef:511
Topic:Conclusion:RussiaandEasternEurope
Skill:Conceptual

43) WhateasternEuropeannationdeclinedmostdramaticallyafter1500?
A) Poland
B) Hungary
C) Austria
D) Prussia
Answer: A
Diff:1
PageRef:511
Topic:Conclusion:RussiaandEasternEurope
Skill:Conceptual

44) HowdidthePolishgovernmentdifferfromtheRussianmodelafter1600?
A) Polandwasmoreurbanized
B) Thecentralgovernmentwaspowerless
C) Therewasanabsenceofamerchantclass
D) Polandlackedalandedaristocracy
Answer: B
Diff:2
PageRef:511
Topic:Conclusion:RussiaandEasternEurope
Skill:Conceptual

378

EssayQuestions
1) WhatwasthenatureofearlyRussianexpansionundertheIvans?
Answer: EarlyexpansionfocusedoncentralAsia;needtodriveMongolsfartherfromRussia;
extendedtoUralMountainsinWestandCaspianSeatothesouth;recruitedpeasantsto
migratetonewlyconqueredterritories;Cossacksservedbothagriculturalandmilitary
purposes;newterritoriessimilartoWesterncolonialism,becameeconomically
dependentonRussia;eliminatedindependentcentralAsiaassourceofnomadic
invasions;incorporatedmanyethnicandreligiousgroupsintoempire.
Diff:2
PageRef:500-502
Topic:RussiasExpansionistPoliciesUndertheTsars
Skill:Conceptual

2) InwhatwaydidPetertheGreatreformtheeconomyandgovernmentofRussiathrough
Westernization?
Answer: Economy:builtupmetallurgicalandminingindustries;primarypurposewastomake
militarytechnology,remainindependentofWest;suppliedindustrieswithcheaplabor
sourcefromserfs.Politics:seizedonabsolutistformsofgovernmentinWest;organized
militaryalongWesternlines;builtnavy;completeddestructionofnoblecouncils;
provincialgovernorsappointedfromcenter;rationalizedlawcodes;newtaxsystem
installed.
Diff:2
PageRef:502-505
Topic:RussiasFirstWesternization
Skill:Conceptual

3) InwhatwayswerethepoliciesofWesternizationundertakenbyPetertheGreatandCatherine
theGreatmoreappearancethansubstance?
Answer: PetertheGreat:culturalchangesonlyaffectedelite;noattempttoextendreformsto
masses;economicreformssignificant,butnoattempttoestablishexportingindustries,
onlyinterestedinmilitarytechnology.CatherinetheGreat:interestedinFrench
Enlightenment,butintroducednomeaningfulreformalongEnlightenmentlines;no
attempttocureproblemsofcoercivelaborsystem;legalsystemactuallymadeharsher;
nobilitygivenalmostabsolutecontrolovermasses;governmentlostcontactwithserfs
inreturnforaristocraticgovernmentservice.
Diff:2
PageRef:502-508
Topic:RussiasFirstWesternization
Skill:Conceptual

4) Whatweretheprimarydifferencesbythe18thcenturybetweenRussiaandtheWest?
Answer: DominanceofthenobilitygreaterthaninWest;dependentonincreasinglycoercive
systemofserfdomasWestwasdevelopingdifferentlaborpatterns;by1649serfdom
wasahereditarycaste;failureofurbanization;lackofsubstantialmerchantclassleft
stateincontrolofcapitalizingindustrialdevelopment;remainedalmostentirely
agricultural;couldnotavoiddependenceonWest.
Diff:2
PageRef:508-510
Topic:ThemesinEarlyModernRussianHistory
Skill:Conceptual

379

Chapter24 EarlyLatinAmerica
Multiple-ChoiceQuestions
1) WhichofthefollowingwasNOTcharacteristicofIberiansociety?
A) Heavyurbanization
B) Absenceofslaveholdingtraditions
C) Emphasisonnobility
D) Emphasisonpatriarchalideals
Answer: B
Diff:2
PageRef:517
Topic:SpaniardsandPortuguese:FromReconquesttoConquest
Skill:Conceptual

2) HowwasthecommercialexperienceofthePortugueseextendedtotheAmericas?
A) ThePortuguesewerefamiliarwiththeroutestotheFarEastthatresultedinthe
circumnavigationoftheglobe.
B) ThePortuguesewereresponsiblefortheuseofgalleysintheAtlanticpassage.
C) ThePortugueseexperienceinAfricaandtheirinvolvementinslavetradingwas
extendedtotheAmericas.
D) ThePortugueseintroductionoftobaccoestateagricultureintotheCaribbeanwasa
significantfactorinAmericancolonization.
Answer: C
Diff:2
PageRef:517
Topic:SpaniardsandPortuguese:FromReconquesttoConquest
Skill:Conceptual

3) WhereintheAmericasdidtheSpanishcreatethemodelsthatwereappliedthroughouttheir
possessionsintheNewWorld?
A) Mexico
B) Peru
C) Brazil
D) TheCaribbean
Answer: D
Diff:1
PageRef:518
Topic:SpaniardsandPortuguese:FromReconquesttoConquest
Skill:Factual

380

4) ThefirstSpanishcolonyintheNewWorldwasestablishedon
A) Hispaniola.
B) Cuba.
C) PuertoRico.
D) theAntilles.
Answer: A
Diff:2
PageRef:518
Topic:SpaniardsandPortuguese:FromReconquesttoConquest
Skill:Factual

5) HowdidSpanishAmericacitiesdifferfromthoseofEurope?
A) Americancitieswerelaidoutinagridplan
B) Americancitieslackedchurches
C) TherewasanabsenceofcommerceinAmericancities
D) TherewerenoCaribbeancities
Answer: A
Diff:2
PageRef:519
Topic:SpaniardsandPortuguese:FromReconquesttoConquest
Skill:Conceptual

6) ThegrantsofIndianstoindividualSpaniardsasalaborsystemwerecalled
A) obrajes.
B) consulados.
C) encomiendas.
D) audiencias.
Answer: C
Diff:1
PageRef:518
Topic:SpaniardsandPortuguese:FromReconquesttoConquest
Skill:Factual

7) WhatgroupofIndianssuppliedagriculturallaborfortheSpaniardsintheCaribbean?
A) Aztec
B) Lakota
C) Unami
D) Taino
Answer: D
Diff:2
PageRef:518
Topic:SpaniardsandPortuguese:FromReconquesttoConquest
Skill:Factual

381

8) TheDominicanfriarBartolome deLasCasas,aconquistadorturnedpriest
A) wasresponsibleforthebrutallawsoppressingtheIndians.
B) becameanardentsupporterofconversionofIndiansandanadvocateofIndianrights.
C) wasresponsibleforthebloodyannihilationoftheIndianpopulationofTenochtitlanin
1520.
D) wasnamedheadoftheCounciloftheIndiesin1518.
Answer: B
Diff:2
PageRef:520
Topic:SpaniardsandPortuguese:FromReconquesttoConquest
Skill:Factual

9) ThemanresponsiblefortheconquestoftheAztecEmpireinMexicowas
A) FranciscoPizarro.
B) PedrodeValdivia.
C) HernanCortes.
D) FranciscoVazquezdeCoronado.
Answer: C
Diff:2
PageRef:522
Topic:SpaniardsandPortuguese:FromReconquesttoConquest
Skill:Factual

10) WhichofthefollowingstatementsconcerningthemenwhoconqueredmuchofLatinAmerica
forSpainisNOTaccurate?
A) Fewoftheconquerorswereprofessionalsoldiers
B) Leadershipwasbasedonreputationandpastachievement
C) AnagreementwasdrawnupbetweentheleaderandtheSpanishcrownthatgranted
authorityfortheexpeditioninreturnforapromisetopayone-fifthofalltreasuretothe
crown
D) Theconquerors,manyofhumbleorigins,cametoseethemselvesasanewnobility
entitledtodominionoveranewpeasantrytheIndians
Answer: B
Diff:2
PageRef:523
Topic:SpaniardsandPortuguese:FromReconquesttoConquest
Skill:Conceptual

382

11) AllofthefollowingwereadvantagestheSpanishenjoyedovertheIndiansEXCEPT
A) epidemicdiseasethatweakenedtheIndiansandreducedtheirnumbers.
B) horses,firearms,steelweapons.
C) internaldivisionsandinternalrivalriesamongtheIndians.
D) thefailureofnomadictribestomountsignificantresistancetoconquest.
Answer: D
Diff:2
PageRef:523
Topic:SpaniardsandPortuguese:FromReconquesttoConquest
Skill:Conceptual

12) WhataccountedforthemajorityofthepopulationlosssufferedbyNativeAmericansafterthe
Europeanarrival?
A) Lossesinwarfare
B) Enslavement
C) Epidemicdiseases
D) FailureofmarriagepatternsamongtheIndians
Answer: C
Diff:1
PageRef:523
Topic:TheDestructionandTransformationofIndianSocieties
Skill:Conceptual

13) ThetremendousdeclineoftheIndianpopulationwasmatchedbytherapidincreasein
A) technologicaldevelopment.
B) Europeanlivestock.
C) Spanishwomen.
D) importsofcottoncloth.
Answer: B
Diff:2
PageRef:524
Topic:TheDestructionandTransformationofIndianSocieties
Skill:Conceptual

14) WhichofthefollowingIndianinstitutionswasretainedbytheSpanishtoserveEuropean
administrativepurposes?
A) NativeAmericanreligion
B) Thepriestlyclass
C) TheIndiannobility
D) TheAztecemperor
Answer: C
Diff:2
PageRef:524
Topic:TheDestructionandTransformationofIndianSocieties
Skill:Factual

383

15) Whyweretheencomiendasdiscontinuedbythe1620s?
A) TheSpanishcrownwasunwillingtoseethegrowthofanewnobilityintheNewWorld.
B) TheIndiansrefusedtocontinuetoserveundertheimperialconditionsestablishedinthe
1500sanddemandedanewarrangementwiththeSpanishcrown.
C) Despitethecontinuedeconomicprosperityoftheencomienda system,theSpanishcrown
discontinuedtheminordertoestablishafreelaborsystemintheAmericas.
D) TheviceroysoftheAmericancoloniesorderedtheirabolitioninfavorofenslavementof
theIndianpopulation.
Answer: A
Diff:2
PageRef:524
Topic:TheDestructionandTransformationofIndianSocieties
Skill:Conceptual

16) Thecolonialgovernmentsreplacedthelaboroftheencomienda withIndianlaborextracted


throughlocalofficials.Suchforcedlaborwascalledthe
A) consulado.
B) mita.
C) obrajes.
D) peninsulares.
Answer: B
Diff:2
PageRef:524
Topic:TheDestructionandTransformationofIndianSocieties
Skill:Factual

17) Whichofthefollowingstatementsmostaccuratelydescribesthenatureoftheeconomyin
SpanishAmerica?
A) ThemajorityofpeopleinLatinAmericawererapidlyorganizedintoalightindustrial
economyintendedtoproducegoodsforAmericansociety.
B) Althoughthemajorityofpeoplewereengagedinagriculture,thewholeSpanish
commercialsystemwasorganizedaroundtheminingeconomy.
C) LatinAmericareceivedalmostitsentirefoodsupplyfromEurope,becausetheSpanish
colonieswereentirelygearedtotheproductionofsugaronestateagriculturalsystems.
D) TheagriculturaleconomyofLatinAmericaabsorbedvirtuallyitsentirepopulation
becauseoftheabsenceoflargedomesticatedanimalspriorto1800.
Answer: B
Diff:2
PageRef:526-527
Topic:ColonialEconomiesandGovernments
Skill:Conceptual

384

18) Thegreatestsilverminewaslocatedat
A) Huancavelica.
B) MexicoCity.
C) Santiago.
D) Potosi.
Answer: D
Diff:1
PageRef:527
Topic:ColonialEconomiesandGovernments
Skill:Factual

19) WhywasthediscoveryofmercuryinPerucriticaltothecolonialeconomy?
A) Mercurywasevenmorevaluablethansilverasanexportcommodity.
B) MercurywasacriticalfoodcomponentforthegrowinglivestockinventoriesofNew
Worldranches.
C) Mercurywasindispensabletotheextractionofsilverfromore-bearingrock.
D) MercurybecameamediumofexchangeinplaceofmoneyintheAmericaneconomy.
Answer: C
Diff:2
PageRef:528
Topic:ColonialEconomiesandGovernments
Skill:Conceptual

20) Becauseoftheswitchfromindigenousmethodsofminingtotheextractionofsilverthrougha
processofamalgamationwithmercury,silverproductionintheAmericasexpandedrapidly
after
A) 1600.
B) 1610.
C) 1630.
D) 1580.
Answer: D
Diff:2
PageRef:528
Topic:ColonialEconomiesandGovernments
Skill:Factual

385

21) WhichofthefollowingstatementsconcerningtheagriculturalsystemofSpanishAmericais
NOTaccurate?
A) ColonistsfacedwithdecliningIndianpopulationsfoundlandownershipmoreattractive.
B) Inplaceswherelargesedentarypopulationsexisted,Indiancommunalagricultureof
traditionalcropscontinued.
C) PlantationcropslikesugarandlatercacaowereexportedtoEuropeinsufficient
quantitiestoexceedthevalueofbullionexports.
D) SpanishAmericaremainedpredominantlyanagrarianeconomy.
Answer: C
Diff:2
PageRef:528
Topic:ColonialEconomiesandGovernments
Skill:Conceptual

22) WhichofthefollowingstatementsconcerningtheSpanishcommercialsystemismost
accurate?
A) ThemerchantguildinSevillehadvirtualmonopolyrightsovergoodsshippedto
Americaandhandledmuchofthesilverreceivedinreturnuntilthe18thcentury.
B) AlltradefromSpainafterthemid-16thcenturywasfunnelledthroughthecityof
Madrid.
C) NearlyalltradewiththeSpanishcolonieswascarriedinshipsbuiltintheNewWorld
andcaptainedbycolonists.
D) Theintentoftheconsulado wastokeeppricesintheSpanishcolonieslow.
Answer: A
Diff:2
PageRef:529
Topic:ColonialEconomiesandGovernments
Skill:Conceptual

23) TheSpanishconvoysystemthatcontrolledtradewiththeAmericaswasmadepossiblebythe
developmentof
A) theastrolabe.
B) galleons.
C) oaredgalleys.
D) gunpowder.
Answer: B
Diff:1
PageRef:529
Topic:ColonialEconomiesandGovernments
Skill:Factual

386

24) InwhatwaydidtheimportationofAmericanbullionnegativelyaffecttheSpanisheconomy?
A) TheapparentdependenceofSpainonitsbullionsupplycausedEuropeanbankersto
avoidloaningmoneytotheSpanishgovernment.
B) Americanbullionmadeupnearly90percentofSpainsstaterevenue,sothatany
disruptioninthearrivalofsilverfromAmericaplungedthegovernmentintocrisis.
C) ThesupplyofbulliontoSpainwashighlyirregular,sothatthegovernmentcouldnot
accuratelygaugeitsincomeoranticipateitsexpenditures.
D) ThearrivalofAmericantreasurecontributedtoasharpriseinpricesandageneral
inflation.
Answer: D
Diff:2
PageRef:529
Topic:ColonialEconomiesandGovernments
Skill:Conceptual

25) TheTreatyofTordesillasof1494dividedtheworldintospheresofinfluencebelongingto
A) PortugalandCastile.
B) SpainandFrance.
C) Portugal,Spain,andEngland.
D) SpainandEngland.
Answer: A
Diff:1
PageRef:529
Topic:ColonialEconomiesandGovernments
Skill:Factual

26) WhatgroupofpeoplewasmostcriticaltothebureaucraticadministrationoftheSpanish
coloniesinAmerica?
A) Friarsandmonks
B) Thenobility
C) University-trainedlawyers
D) Militarycommanders
Answer: C
Diff:2
PageRef:529
Topic:ColonialEconomiesandGovernments
Skill:Factual

387

27) ThelawcodethatbecamethebasisoflawintheIndieswascalledthe
A) CasadelaContratacion.
B) Recopilacion.
C) CorpusJurisCivilis.
D) LegaliaIndiana.
Answer: B
Diff:2
PageRef:529
Topic:ColonialEconomiesandGovernments
Skill:Factual

28) TheviceroyaltiesofNewSpainandPeruweredividedinto10judicialdivisionscontrolledby
superiorcourts,or
A) consulados.
B) audiencias.
C) obrajes.
D) encomiendas.
Answer: B
Diff:1
PageRef:529
Topic:ColonialEconomiesandGovernments
Skill:Factual

29) TheCatholicchurchintroducedallofthefollowingtoAmericanlifeEXCEPT
A) universities.
B) theconstructionofbaroquechurches.
C) thesenseofindependencefromthestate.
D) theprintingpress.
Answer: C
Diff:2
PageRef:530-531
Topic:ColonialEconomiesandGovernments
Skill:Conceptual

30) Whatcolonybecamethefirstmajorplantationzone,organizedtoproduceatropicalcropin
demandinEurope?
A) Mexico
B) Brazil
C) Chile
D) Peru
Answer: B
Diff:1
PageRef:531
Topic:Brazil:TheFirstPlantationEconomy
Skill:Factual

388

31) ThefirstlandfallinthecolonyofBraziltookplacein1500undertheleadershipof
A) HernanCortes.
B) FranciscoPizarro.
C) PedrodeValdivia.
D) PedroAlvaresCabral.
Answer: D
Diff:1
PageRef:531
Topic:Brazil:TheFirstPlantationEconomy
Skill:Factual

32) Between1532and1549,colonizationinBrazilwasinthehandsofminorPortuguesenobles
grantedstripsoflandcalled
A) consulados.
B) captaincies.
C) duchies.
D) proprietarycolonies.
Answer: B
Diff:2
PageRef:531
Topic:Brazil:TheFirstPlantationEconomy
Skill:Factual

33) By1700,slavescomprisedapproximatelywhatproportionoftheBrazilianpopulation?
A) onequarter
B) onethird
C) onehalf
D) twothirds
Answer: C
Diff:2
PageRef:531
Topic:Brazil:TheFirstPlantationEconomy
Skill:Factual

34) TheBrazilianmodelofaplantationcolonywaslaterfollowedinthe18thcenturybyother
Europeannationsin
A) Mexico.
B) Canada.
C) Panama.
D) theCaribbean.
Answer: D
Diff:2
PageRef:531
Topic:Brazil:TheFirstPlantationEconomy
Skill:Factual

389

35) WhatwastheprimarydifferencebetweentheSpanishandPortugueseempires?
A) PortuguesecoloniesdidnothavetheheavyinfluenceoftheCatholicchurchfoundin
Spanishcolonies.
B) PortuguesecolonieslackedthebureaucraticstructurethatcharacterizedtheSpanish
coloniesafterthemiddleofthe16thcentury.
C) UnliketheSpanishEmpirethatwasalmostexclusivelyAmerican,thePortuguese
EmpireincludedcoloniesandoutpostsinAsiaandAfricaaswellasBrazil.
D) ThePortuguesecolonyofBrazilwasmoreintellectuallyindependentofthemother
countrythanweretheSpanishcoloniesinLatinAmerica.
Answer: C
Diff:2
PageRef:532
Topic:Brazil:TheFirstPlantationEconomy
Skill:Conceptual

36) WhatconditionsundercutthepositionoftheBraziliansugarplantationeconomy?
A) AdemographicdisasteramongtheIndiansofBrazilresultedinashortageoflaborfor
thesugarplantationsshortlyafter1700.
B) CompetitionfromEnglish,French,andDutchplantationcoloniesintheCaribbeanledto
risingpricesforslavesandfallingpricesforsugar.
C) TheEuropeanmarketwasfloodedwithsugarsuppliedfromAsiancolonies.
D) Aseriesofunusuallywetwintersfloodedthetraditionalsugarregionsandcaused
Brazilianplanterstoseeknewlandfortheproductionofsugar.
Answer: B
Diff:2
PageRef:532
Topic:Brazil:TheFirstPlantationEconomy
Skill:Conceptual

37) InwhatregionwasgolddiscoveredinBrazil?
A) Salvador
B) CasasOrientes
C) MinasGerais
D) SigloDOro
Answer: C
Diff:1
PageRef:532
Topic:Brazil:TheFirstPlantationEconomy
Skill:Factual

390

38) WhatportwasassociatedwiththediscoveryofgoldinBrazilandsubsequentlybecamethe
capitalofthecolony?
A) BuenosAires
B) Salvador
C) Santiago
D) RiodeJaneiro
Answer: D
Diff:1
PageRef:533
Topic:Brazil:TheFirstPlantationEconomy
Skill:Factual

39) WhatwasthenegativeimpactofthediscoveryofgoldonPortugal?
A) Portugalfailedtodevelopinternalindustriesbecausethesupplyofgoldallowedthe
PortuguesetopurchasemanufacturedgoodsfromotherEuropeancountries.
B) PortugalwasforcedtohandBrazilovertothemorepowerfulDutchnavy.
C) Portugalsagriculturaleconomywasdevastatedbytheflowofcapitalfromdomestic
producetoBrazilianimports.
D) PortugalbecameincreasinglydependentonFrance.
Answer: A
Diff:2
PageRef:533
Topic:Brazil:TheFirstPlantationEconomy
Skill:Conceptual

40) WhatwasthebasisforthesocialhierarchythatdevelopedintheAmericas?
A) Wealth
B) Theprestigeassociatedwithonesoccupation
C) Race
D) Gender
Answer: C
Diff:1
PageRef:533
Topic:MultiracialSocieties
Skill:Conceptual

391

41) WhatSpanishdynastywasresponsiblefortheseriesofeighteenthcenturyreformsthatrecast
thecolonialadministrationoftheAmericas?
A) Bourbon
B) Habsburg
C) Aragon
D) Carlist
Answer: A
Diff:1
PageRef:536
Topic:TheEighteenth-CenturyReforms
Skill:Factual

42) TheministeroftheIndiesresponsibleforthe18thcenturyreformswithintheSpanishempire
was
A) GarciaFloridablanca.
B) theMarquisofPombal.
C) ElijioMartinez.
D) JosedeGa lvez.
Answer: D
Diff:2
PageRef:537
Topic:TheEighteenth-CenturyReforms
Skill:Factual

43) Whichofthefollowingstatementsconcerningthe18th-centurySpanishreformsinAmericais
mostaccurate?
A) TheEnglishsystemofjusticesofthepeacewasintroducedtoreplacetheaudiencias,
causingthedeclineininfluenceoftheeducatedlawyersinAmerica.
B) TheSpanishcolonieswerelargelydemilitarized,asSpainbecameincreasingly
dependentonthenavyofFrancetoprotectitsinterestsinAmerica.
C) TheFrenchIntendancysystemwasintroduced.Governmentimproved,butthe
traditionalpatternsofinfluenceandpoweramongtheCreolebureaucratswasdisrupted.
D) TheSpanishreformsdidlittletoalterthepatternsoflocaladministrationandthe
fundamentalstructureofpowerandauthorityintheLatinAmericancolonies.
Answer: C
Diff:3
PageRef:537
Topic:TheEighteenth-CenturyReforms
Skill:Conceptual

392

44) WhichofthefollowingwasNOTanimpactoftheBourbonreformsoncolonialeconomy?
A) Theestablishmentofstatemonopoliesonitemsliketobaccoandgunpowder
B) TheextensionoftheplantationagriculturalsystemtoCuba
C) ThegrowthofBuenosAiresasaregionaltradecenter
D) ThefrontiersofSpanishAmericabecamefixed
Answer: D
Diff:2
PageRef:537-538
Topic:TheEighteenth-CenturyReforms
Skill:Conceptual

45) Theministerresponsibleforthe18thcenturyreformsinPortugalandBrazilwas
A) GarciaFloridablanca.
B) theMarquisofPombal.
C) ElijioMartinez.
D) JosedeGa lvez.
Answer: B
Diff:2
PageRef:538
Topic:TheEighteenth-CenturyReforms
Skill:Factual

46) Whatwastheimpactofthe18thcenturyreformsonslaveryinBrazil?
A) Slaverywasabolished
B) TheslavetradewithAfricawasabolished
C) Slaveimportswererestrictedtoencouragetheeliminationoftheplantationeconomy
D) Brazilwasjustasprofoundlybasedonslaveryinthelate18thcenturyasithadeverbeen
Answer: D
Diff:2
PageRef:538
Topic:TheEighteenth-CenturyReforms
Skill:Conceptual

393

47) WhichofthefollowingstatementsmostaccuratelydescribesthestateofthePortugueseand
SpanishAmericancoloniesbythemiddleofthe18thcentury?
A) ThepopulationoftheAmericancoloniesneverrecoveredfromtheinitiallossofIndian
population.
B) WhilethepopulationoftheAmericancolonieswasgrowing,largelyduetothe
importationofAfricanslaves,theeconomywaslargelystagnant.
C) TheAmericancoloniesofSpainandPortugalwereexperiencingconsiderablegrowthin
populationandproductivecapacity.
D) Followingthedisruptionoftheplantationeconomies,thePortugueseandSpanish
Americancoloniesexperiencedpopulationlossandeconomicdepression.
Answer: C
Diff:2
PageRef:538
Topic:TheEighteenth-CenturyReforms
Skill:Conceptual

48) WhowastheleaderoftheIndianrevolutioninPeruin1781?
A) GarciaFloridablanca
B) NezPerce
C) TupacAmaru
D) ChichenItza
Answer: C
Diff:2
PageRef:539
Topic:TheEighteenth-CenturyReforms
Skill:Factual

49) Whataccountsforthegeneralfailureof18th-centurycolonialrevolutionsagainstSpanishand
Portugueserule?
A) Thevariousracialandsocialgroups,fearfulofunsettlingthesocialhierarchy,failedto
acttogethertounseatthecolonialgovernments.
B) TheSpanishreformsweregenerallysosuccessfulthattherewerefewdissatisfied
elementsinthecolonies.
C) Rebelarmiesenjoyednosuccessagainstthemilitaryforcesofthecolonialgovernment.
D) IndiansweresolittleincorporatedintoAmericansocietythattheywerenotinterestedin
changingthepoliticalorganizationoftheirmasters.
Answer: A
Diff:2
PageRef:539
Topic:TheEighteenth-CenturyReforms
Skill:Conceptual

394

EssayQuestions
1) InwhatsensecanitbesaidthatthenatureoftheLatinAmericancoloniesweresimplya
reflectionofIberiansociety?
Answer: HeavilyurbanizedsocietyinIberianpeninsulareproducedinAmericancolonies;useof
plannedcities;tendencytopopulationagglomerationratherthanevendistribution;
emphasisonnobilitycarriedovertoAmerica;beliefofconquerorsofrighttoIndian
laborasnewformofserfdom;patriarchalsocietyreplicatedinhouseholdsofAmerica,
alsoencomiendasystem;traditionofslaveholdingindigenoustoIberia;already
establishedsystemofslavetradingfromAfricancolonies;systemofplantation
agriculturealsoapparentinAfrica.
Diff:2
PageRef:517-523
Topic:Introduction
Skill:Conceptual

2) InwhatwaysdidtheSpanishadministrationoftheirAmericancolonieschangefromtheir
initialestablishmentunderColumbusto1700?
Answer: ColumbusattemptedtoemploywestAfricanmodelofcolonialadministration,trade
forts,privateinvestmentunderroyalcontract,tradeandgoldandslaves;modifiedafter
Columbussfamilyremovedasprimaryadministrators;establishmentofroyal
administrationconsistingatfirstofroyalgovernor,treasury,royalcourtofappealsor
audiencia;laterregularizedintotwoviceroyalties(laterexpandedtofour),eachdivided
into10audiencias;localgovernmentcarriedoutbylocalcorregidores.
Diff:2
PageRef:517-531
Topic:ColonialEconomiesandGovernments
Skill:Conceptual

3) CompareandcontrastthecolonialeconomiesofSpanishAmericaandBrazil.
Answer: SpanishAmerica:althoughmajorityofpeopleinvolvedinagriculture,heartofeconomy
wasmining,productionofsilver;basedonIndianlabor,someslaves;Brazil:fromoutset
basedonplantationestatesystemproducingsugar;basedonslavelaboralmost
exclusively;in17thcenturydiscoveryofgoldinMinasGeraisalteredeconomytomore
emphasisonmining;shiftaidedbyfailureofBraziltomaintainmonopolyofsugar
production;greaterdevelopmentofsmallindustriesinSpanishAmerica.
Diff:2
PageRef:517-536
Topic:Brazil:TheFirstPlantationEconomy
Skill:Conceptual

4) HowwasthesocietyoftheLatinAmericancoloniesorganized?Whatdeterminedthesocial
hierarchy?
Answer: MiscegenationandracialmixturemadeAmericansocietyfundamentallydifferentthan
Westernsociety;socialhierarchybasedonraceandplaceofbirth;whitesattopwith
thoseborninEuropegivenprecedenceoverCreoles;mixedraces(blacks/Indians,
Indians/whites,blacks/whites)inintermediarypositions;Africanslaves,Indiansat
bottomofsocialorder.
Diff:2
PageRef:533-536
Topic:MultiracialSocieties
Skill:Conceptual

395

5) WhatwastheintentoftheSpanishreformsofthe18thcentury?Towhatextentwerethey
successful?
Answer: SpanishpossessionswerethreatenedmilitarilyandeconomicallybyFrance,England,
Holland;needtorestorecolonialeconomies,improveefficiencyofcolonialgovernment,
improvemilitarysecurity;positiveresults:introductionofcommerciolibreimproved
trade,expandedeconomyofCaribbean,BuenosAires;introducedFrenchIntendancy,
improvedadministrationofcolonies,eliminatedCreolesfromupperbureaucracy;
regulartroopssenttoAmerica,ledtoexpansionoffrontiers;negativeresults:alienated
Creoles,createdsenseofself-identity,preludetorevolts.
Diff:2
PageRef:536-540
Topic:TheEighteenth-CenturyReforms
Skill:Conceptual

396

Chapter25 AfricaandtheAfricansintheAgeofthe
AtlanticSlaveTrade
Multiple-ChoiceQuestions
1) WhatwastheAfricancontributiontotheColombianExchange?
A) Largemammals
B) Tapioca
C) Slaves
D) Manioc
Answer: C
Diff:1
PageRef:545
Topic:Introduction
Skill:Factual

2) WhichofthefollowingwasNOTconsideredafactorinthedevelopmentoflargekingdomsin
Africa?
A) Apopulationexpansionthatfollowedthediffusionofirontoolsandimproved
agriculture
B) ThecollapseoftheChristiankingdominEthiopiainthefaceofMuslimadvance
C) Europeandemandforslaves
D) Theuseoffirearms
Answer: B
Diff:2
PageRef:545
Topic:Introduction
Skill:Conceptual

3) WhatEuropeannationfirstestablisheddirectcontactwithblackAfrica?
A) Spain
B) England
C) Italy
D) Portugal
Answer: D
Diff:1
PageRef:545
Topic:TheAtlanticSlaveTrade
Skill:Factual

397

4) OftheearlyPortuguesetradeforts,wherewasthemostimportantlocated?
A) ElMina
B) Zambesi
C) Zimbabwe
D) Sokoto
Answer: A
Diff:2
PageRef:545
Topic:TheAtlanticSlaveTrade
Skill:Factual

5) WhichofthefollowingstatementsconcerningtheearlyPortuguesetradefortsismost
accurate?
A) ThePortuguesetradefortspermittedthepoliticalcontrolofmuchoftheAfricaninterior.
B) WherePortuguesetradefortswereestablished,largeEuropeancoloniesrapidly
developed.
C) Mostofthefortswereestablishedwiththeagreementorlicenseoflocalrulers.
D) ThePortuguesetradefortswerethenodalpointsforcolonialadministrationonthe
modeloftheAmericancolonies.
Answer: C
Diff:2
PageRef:545
Topic:TheAtlanticSlaveTrade
Skill:Conceptual

6) WhichoftheAfricankingdomswasmostsuccessfullyconvertedtoChristianitybyPortuguese
missionaries?
A) Benin
B) Zimbabwe
C) Sokoto
D) Kongo
Answer: D
Diff:1
PageRef:545
Topic:TheAtlanticSlaveTrade
Skill:Factual

398

7) SouthoftheirtradefortsalongtheGoldCoast,thePortugueseestablishedLuanda,which
becamethebasisforthePortuguesecolonyof
A) Angola.
B) Kongo.
C) Lesotho.
D) Matabele.
Answer: A
Diff:2
PageRef:546
Topic:TheAtlanticSlaveTrade
Skill:Factual

8) WhichofthefollowingstatementsconcerningthePortuguesepresenceineastAfricaismost
accurate?
A) Afterthe1570s,thePortuguesegainedcompletecontrolovertheeastAfricantradeinthe
IndianOceanandtheRedSea.
B) UnlikethePortuguesesettlementsonthecoastofwesternAfrica,Mozambiquebecame
thebasisforalargeEuropeancolony.
C) PortuguesebasesonMozambique,Mombassa,Sofala,andKilwagavethemaccesstothe
goldtradeofeastAfrica,buttheywereunabletocompletelydisrupttheMuslimtrade
withtheregion.
D) ThePortuguesepresenceineastAfricaprecipitatedthepopulationmovementamongthe
BantuthatbecametheMfecane.
Answer: C
Diff:2
PageRef:546
Topic:TheAtlanticSlaveTrade
Skill:Conceptual

9) FollowingthemodelestablishedbythePortuguese,whichofthefollowingwasNOToneof
theprincipalpatternsofEuropeancontactwithAfrica?
A) Asystemoffortifiedtradingstations
B) Thecombinationofforceanddiplomacyincludingallianceswithlocalleaders
C) Thepredominanceofcommercialrelations
D) WidespreadEuropeancolonization
Answer: D
Diff:2
PageRef:546
Topic:TheAtlanticSlaveTrade
Skill:Conceptual

399

10) WhichofthefollowingstatementsaboutslaveryinEuropebefore1450wasmostaccurate?
A) Europehadneverhadatraditionofslavery.
B) Slaveryhadbeenextensiveintheancientworld,buthaddiedoutduringtheMiddle
AgesinallregionsintheWest.
C) SlaveryhaddiedoutintheMiddleAgesinmostofEuropeexceptalongthemilitary
frontierbetweenChristiansandMuslimsintheMediterranean.
D) SlaverywascommontoEuropeinalleraspriorto1450,althoughittendedtobecome
moredominantduringtheMiddleAges.
Answer: C
Diff:2
PageRef:546
Topic:TheAtlanticSlaveTrade
Skill:Conceptual

11) ThefirstslavesbroughtdirectlytoPortugalfromAfricaarrivedin
A) 1441.
B) 1492.
C) 1509.
D) 1557.
Answer: A
Diff:2
PageRef:546
Topic:TheAtlanticSlaveTrade
Skill:Factual

12) InwhatmannerdidthePortugueseseizemostoftheslavesthatweretransportedfrom
Africa?
A) TheycapturedtheminraidsintotheAfricaninterior.
B) TheytradedforthemwithAfricanrulers.
C) AsaresultofthedefeatofmostoftheAfricankingdoms,thePortugueseobtaineda
readysupplyofslaves.
D) TheypurchasedthemfromtheMuslimslavetradersoftheeastAfricantradingcities.
Answer: B
Diff:2
PageRef:546
Topic:TheAtlanticSlaveTrade
Skill:Conceptual

400

13) WhatdevelopmentledtoanintensificationofthePortugueseinvolvementintheAfricanslave
trade?
A) TheextensionoftheagriculturalsysteminEurope
B) TheBlackDeath,whichdecreasedtheavailablesupplyoflaborinEurope
C) ThedevelopmentofsugarplantationsontheAtlanticislandsofMadeiraandthe
Canaries
D) ThegrowthoftheOttomanEmpire
Answer: C
Diff:2
PageRef:546
Topic:TheAtlanticSlaveTrade
Skill:Conceptual

14) WhatisthebestestimateofthenumberofAfricansshippedacrosstheAtlanticbetween1450
and1850?
A) Eightmillion
B) 10million
C) 12million
D) 14million
Answer: C
Diff:2
PageRef:547
Topic:TheStructureoftheAtlanticSlaveTrade
Skill:Factual

15) InwhatcenturydidtheAtlanticslavetradereachitszenithintermsofnumbersofAfricans
exported?
A) 15th
B) 16th
C) 17th
D) 18th
Answer: D
Diff:2
PageRef:547
Topic:TheStructureoftheAtlanticSlaveTrade
Skill:Factual

401

16) WhichofthefollowingstatementsconcerningthevolumeoftheslavetradetotheAmerican
coloniesismostaccurate?
A) Thevolumeoftheslavetradedwindledrapidlyafterthe17thcenturybecausethe
plantationeconomiesoftheAmericascollapsed.
B) Thehighvolumeoftheslavetradewasanecessitybecauseslavemortalitywashighand
fertilitywaslow,leadingtoalossofslavepopulation.
C) Theshiftfromplantationeconomiestominingeconomiesinwhichslaveswerenot
utilizedrapidlydiminishedtheAmericandemandforslaves.
D) Demandforslavescontinuedtoremainhighduetothecompetingdemandforslavesin
Europe.
Answer: B
Diff:2
PageRef:547
Topic:TheStructureoftheAtlanticSlaveTrade
Skill:Conceptual

17) WhatregionintheAmericasreceivedmoreslavesthananyotherbetween1550and1850?
A) Brazil
B) ThesouthernBritishcoloniesofNorthAmerica
C) Mexico
D) TheCaribbean
Answer: A
Diff:2
PageRef:547
Topic:TheStructureoftheAtlanticSlaveTrade
Skill:Factual

18) WhichofthefollowingstatementsconcerningtheshapeofthecommerceinAfricanslavesis
mostaccurate?
A) TheemergenceoftheAtlanticslavetradecausedtheimmediateendoftheolder
trans-SaharanslavetradeinthehandsoftheMuslims.
B) TheAtlantictradedrewitsslavesalmostexclusivelyfromsouthernAfricauntilthe19th
century.
C) TheAfricanstatesoftheinterioractivelyresistedtheslavetrade.
D) TheAtlanticslavetradedrewslavesfromacrosstheAfricancontinentandits
concentrationshiftedfromSenegambiatocentralandwesternAfricaovertime.
Answer: D
Diff:2
PageRef:548
Topic:TheStructureoftheAtlanticSlaveTrade
Skill:Conceptual

402

19) Inwhatwaydidthetrans-SaharanslavetradedifferfromthatoftheAtlanticslavetrade?
A) TheAtlanticslavetradewascarriedoutalmostexclusivelybyMuslims.
B) Thetrans-SaharanslavetradewascarriedoutinmuchgreatervolumethantheAtlantic
slavetrade.
C) Thetrans-Saharanslavetradeconcentratedonwomen,buttheAtlanticslavetrade
concentratedonyoungmen.
D) TheAfricanpreferenceforretainingyoungmaleslavestoextendkinshiplinesimplied
thatprimarilywomenwereavailabletotheAtlantictrade,whilemenconvertedtoIslam
weremorelikelytradeobjectsforthetrans-Saharantrade.
Answer: C
Diff:2
PageRef:548-549
Topic:TheStructureoftheAtlanticSlaveTrade
Skill:Conceptual

20) WhatwasthedemographicimpactoftheslavetradeonAfrica?
A) Theslavetradeexportedmillions,butthelosswasmadeupasaresultofthenatural
prolifacyoftheAfricans.
B) PopulationinAfricaseemsactuallytohavegrown,despitethenumberofmen,women,
andchildrenexportedtotheAmericas.
C) TheslavetradehadtheimpactofskewingthepopulationofcentralAfricainfavorofa
disproportionalnumberofwomen.
D) Asaresultoftheslavetrade,thepopulationofAfricawasonlyone -thirdofwhatit
wouldhavebeenwithouttheexportofmenandwomen.
Answer: C
Diff:2
PageRef:549
Topic:TheStructureoftheAtlanticSlaveTrade
Skill:Conceptual

21) HowdidtheBritishorganizetheshipmentofslavestotheAmericas?
A) InBritain,unlikeelsewhere,theslavetradewascarriedoutbyuncontrolledprivate
venture.
B) InBritain,thecharteredRoyalAfricanCompanywasgrantedamonopolyoverthe
shipmentofslavestocoloniesintheAmericas.
C) TheBritishrefusedtoparticipateintheslavetradeandattemptedtointerceptshipments
ofslavestotheAmericasbeginninginthe1660s.
D) TheBritishgovernmentdirectlyparticipatedintheslavetradethroughuseoftheRoyal
Navy.
Answer: B
Diff:2
PageRef:550
Topic:TheStructureoftheAtlanticSlaveTrade
Skill:Conceptual

403

22) TheSpanishtermforahealthyadultmaleslavewas
A) Indiespiece.
B) calderon.
C) servusdei.
D) casadelacontratacio n.
Answer: A
Diff:1
PageRef:550
Topic:TheStructureoftheAtlanticSlaveTrade
Skill:Factual

23) WhatwastheaverageprofitabilityoftheEnglishslavetradeinthelate18thcentury?
A) 5-10percent
B) 15-20percent
C) 25-30percent
D) Over50percent
Answer: A
Diff:2
PageRef:550
Topic:TheStructureoftheAtlanticSlaveTrade
Skill:Factual

24) Howdidtheprofitabilityoftheslavetradecomparetothatofothercontemporarybusiness
ventures?
A) Theslavetradewaslessprofitableonthewholethanotherbusinessventuresbecauseof
thehighcostsandrisksinvolved.
B) Theslavetradebecameincreasinglydangerousandrisky,sothatbythe18thcenturyits
returnswereminimalandmostparticipantswerebankrupted.
C) Profitsfromtheslavetradeinthe18thcenturyweresolucrativethatcapitalderived
fromthetradeinhumanbeingswasusedtocapitalizetheEuropeanIndustrial
Revolution.
D) Theslavetradewaslittlemoreprofitablethanmostbusinessactivitiesoftheageandwas
notamajorsourcefortheindustrialrevolutionofEurope.
Answer: D
Diff:2
PageRef:550
Topic:TheStructureoftheAtlanticSlaveTrade
Skill:Conceptual

404

25) WhatwasthetermutilizedforthecommercialarrangementbywhichAfricanslaveswere
shippedtotheAmericas,sugarandtobaccowerecarriedtoEurope,andEuropean
manufacturedgoodsweretransportedtoAfrica?
A) TheColombianExchange
B) Mercantilism
C) Triangulartrade
D) TheAtlanticExpress
Answer: C
Diff:2
PageRef:550
Topic:TheStructureoftheAtlanticSlaveTrade
Skill:Factual

26) WhichofthefollowingstatementsconcerningslaveryonthecontinentofAfricabeforethe
arrivaloftheEuropeansismostaccurate?
A) MostAfricansocietieswereegalitarian, thatis,mostpeopleenjoyedthesamesocial
status,andslaverywasunknown.
B) SlaveryinAfricawasrestrictedtothoseareaswhereIslamicinfluencehadintroduced
theconceptintheperiodafterthe7thcentury.
C) WhilecommonelsewhereinAfrica,slaverywasnotcommonintheforeststatesofwest
AfricapriortothecomingoftheEuropeans.
D) SinceinmanyAfricansocietiesalllandwasownedbythestateoftheruler,thecontrol
ofslaveswasoneofthefewwaysinwhichindividualsorlineagescouldincreasetheir
wealthandstatus.
Answer: D
Diff:3
PageRef:551
Topic:AfricanSocieties,SlaveryandtheSlaveTrade
Skill:Conceptual

27) WhatwasthepoliticalimpactofthepresenceofEuropeansontheAfricancoast?
A) StatesweremorelikelytoforminthesavannaregionsofAfrica.
B) Strongcentralizedstatesbegantoformonthecoastlineincloseproximitytothe
Europeantradeforts.
C) WestandcentralAfricankingdomsjustinlandfromthefortsbegantoredirecttheir
tradeandexpandtheirinfluence.
D) StateformationinAfricatookplaceontheIndianOceancoastawayfromthetrade
routesestablishedbytheEuropeans.
Answer: C
Diff:2
PageRef:551
Topic:AfricanSocieties,SlaveryandtheSlaveTrade
Skill:Conceptual

405

28) InwhatwaydidtheEuropeanslavetradeenablecentralizingstatestoexpandmorerapidly?
A) Slavesweretradedforfirearmsthatallowedexpandingstatestooverpowertheir
neighbors,resultinginmoreslaves.
B) TheEuropeansrapidlycreatedmilitaryalliancesandaddedtheirarmiestothoseoftheir
slave-tradingallies.
C) TheEuropeanslavetradeweakenedthestatesofcentralandwesternAfrica,allowing
thecentralizingstatesofeasternAfricatoexpandwithoutcompetition.
D) Theslavetradewasrestrictedtothecoasts,leavingthepoliticalunitsofinteriorAfrica
freeofEuropeaninterference.
Answer: A
Diff:2
PageRef:551
Topic:AfricanSocieties,SlaveryandtheSlaveTrade
Skill:Conceptual

29) WhichofthefollowingwasalargeAfricanstatethatdevelopedinwesternAfricaduringthe
periodoftheAtlanticslavetrade?
A) Ghana
B) Mali
C) Swazi
D) Asante
Answer: D
Diff:1
PageRef:552
Topic:AfricanSocieties,SlaveryandtheSlaveTrade
Skill:Factual

30) UnderwhoserulewasunityachievedamongthenumerousAkanclansofAsante?
A) UsumanDanFodio
B) OseiTutu
C) Shaka
D) Agaja
Answer: B
Diff:2
PageRef:552
Topic:AfricanSocieties,SlaveryandtheSlaveTrade
Skill:Factual

406

31) ThetitlegiventothesupremecivilandreligiousruleroftheAsantekingdomwas
A) bunyoro.
B) swazi.
C) asantehene.
D) kowazi.
Answer: C
Diff:2
PageRef:552
Topic:AfricanSocieties,SlaveryandtheSlaveTrade
Skill:Factual

32) OntheeastcoastofAfrica,theSwahilitradingcities
A) weredecimatedfollowingEuropeannavalattacks.
B) fellentirelywithintheorbitofthePortugueseglobaltradenetwork.
C) continuedtheircommerceintheIndianOceanwithboththePortugueseandthe
OttomanTurks.
D) wereuniqueinAfricabecauseoftheirrefusaltoparticipateintheslavetrade.
Answer: C
Diff:2
PageRef:553
Topic:AfricanSocieties,SlaveryandtheSlaveTrade
Skill:Conceptual

33) OneoftheuniquefeaturesoftheeastAfricancoastthatdifferedfromwestAfricawas
A) theexistenceoftheslavetrade.
B) theestablishmentofplantationsproducingclovesusingAfricanslavelabor.
C) theexistenceoflargeEuropeancolonies.
D) thewidespreadappearanceofepidemicdisease.
Answer: B
Diff:2
PageRef:554
Topic:AfricanSocieties,SlaveryandtheSlaveTrade
Skill:Conceptual

34) AnexampleoftheacceptanceofNiloticpeoplesasrulingdynastiesamongtheBantuwasthe
Luoat
A) Benin.
B) ElMina.
C) Zanzibar.
D) Bunyoro.
Answer: D
Diff:2
PageRef:555
Topic:AfricanSocieties,SlaveryandtheSlaveTrade
Skill:Factual

407

35) TheMuslimreformmovementofthe1770sthatsweptthroughthetradenetworksin
SenegambiaandthewesternSudanwaswhatvariantofIslam?
A) Ismaili
B) Fatimid
C) Sufi
D) Nizari
Answer: C
Diff:2
PageRef:556
Topic:AfricanSocieties,SlaveryandtheSlaveTrade
Skill:Factual

36) UsumanDanFodiobeganarebellioninwhatAfricankingdomin1804?
A) Hausa
B) Benin
C) Kongo
D) Luanda
Answer: A
Diff:2
PageRef:556
Topic:AfricanSocieties,SlaveryandtheSlaveTrade
Skill:Factual

37) UsumanDanFodiosrebellioneventuallyledtotheformationofwhatcaliphalstate?
A) Lesotho
B) Benin
C) Sokoto
D) Kongo
Answer: C
Diff:2
PageRef:556
Topic:AfricanSocieties,SlaveryandtheSlaveTrade
Skill:Factual

38) WhatareaofAfricawasleastaffectedbytheslavetrade?
A) ThesavannaandtheSudan
B) SouthernAfrica
C) CentralandwestAfrica
D) EastAfrica
Answer: B
Diff:2
PageRef:556
Topic:WhiteSettlersandAfricansinSouthernAfrica
Skill:Factual

408

39) In1652whatgroupestablishedacolonyattheCapeofGoodHope?
A) TheBritishEastIndiaCompany
B) TheRoyalAfricanCompany
C) TheDutchEastIndiaCompany
D) TheOttomanEmpire
Answer: C
Diff:2
PageRef:557
Topic:WhiteSettlersandAfricansinSouthernAfrica
Skill:Factual

40) In1818whoassumedleadershipintheZuluchiefdomoftheNgunipeopleofsouthernAfrica?
A) OseiTutu
B) Agaja
C) UsumanDanFodio
D) Shaka
Answer: D
Diff:1
PageRef:557
Topic:WhiteSettlersandAfricansinSouthernAfrica
Skill:Factual

41) WhatwastheimpactoftheMfecane?
A) ItdestroyedtheBantupeoplesofeasternAfrica
B) Asaresult,thePortuguesewereabletoabsorbmostoftheEastAfricanSwahilitrading
cities
C) TheBoerswereforcedoutofsouthernAfrica
D) TheresultingpoliticaldisruptionsentAfricangroupsfleeingbeforetheZulusintoboth
PortuguesecoastalregionsandtheBoerfarmsofsouthernAfrica
Answer: D
Diff:2
PageRef:557
Topic:WhiteSettlersandAfricansinSouthernAfrica
Skill:Conceptual

42) WhatwastheaveragemortalityforslavesshippedtotheAmericasintheAtlanticslavetrade?
A) 10-15percent
B) 18-20percent
C) 25-40percent
D) 55-65percent
Answer: B
Diff:3
PageRef:559
Topic:TheAfricanDiaspora
Skill:Factual

409

43) TheslavevoyagetotheAmericaswasreferredtoasthe
A) AtlanticMistral.
B) OceanExpress.
C) AmericanTragedy.
D) MiddlePassage.
Answer: D
Diff:2
PageRef:559
Topic:TheAfricanDiaspora
Skill:Factual

44) WhywereAfricanssoughtforplantationlaborintheAmericas?
A) TherewasnootherlaborsupplyavailableintheAmericas.
B) WestAfricanswerealreadyfamiliarwithmetallurgy,herding,andintensiveagriculture
whereasIndianswerenot.
C) SugarwasacropnativetoAfricaandexportedtotheAmericasfromthere.
D) AfricansrapidlyexpandedtheirpopulationintheLatinAmericancolonies.
Answer: B
Diff:2
PageRef:559
Topic:TheAfricanDiaspora
Skill:Conceptual

45) HowweretheBritishcoloniesofthesouthernAtlanticcoastofNorthAmericadifferentfrom
theLatinAmericancolonies?
A) Therewasnoslaverythere.
B) Althoughurbanslaverywascommon,therewasnoplantationagriculturalsystemonthe
NorthAmericanmainland.
C) ManumissionofslavestendedtobemorecommonintheBritishcolonies.
D) TheBritishcoloniesdependedlessonimportedAfricansbecauseofthepositiverateof
growthamongtheslaves.
Answer: D
Diff:2
PageRef:560
Topic:TheAfricanDiaspora
Skill:Conceptual

410

46) Whichofthefollowingstatementsconcerningslavefamiliesismostaccurate?
A) Despiteenormousdifficulties,slavescontinuedtoliveinfamilyunits.
B) Theonerousconditionsofslaveryandtheshortageofwomenresultedinthecreationof
artificialclansandextendedlineages.
C) Lackingwomen,mensubstitutedslavefraternitiesforthemoretraditional
organization.
D) TheconditionsofslaverydestroyedfamilyconceptsamongAfricansintheNewWorld.
Answer: A
Diff:2
PageRef:560
Topic:TheAfricanDiaspora
Skill:Conceptual

47) WhichofthefollowingwasNOTanAfricanreligiontransportedtotheAmericas?
A) Obeah
B) Vodun
C) Asantehene
D) Candomble
Answer: C
Diff:2
PageRef:560
Topic:TheAfricanDiaspora
Skill:Factual

48) Whichofthefollowingstatementsconcerningresistancetoslaveryismostaccurate?
A) Recalcitrance,runningaway,anddirectrebellionwerepresentwhereverslaveswere
employed.
B) Althoughtherewasoneortwoslaverebellionsinisolatedareas,ingeneraltherewasno
organizedresistancetoslavery.
C) Resistancetoslaverywasexperiencedonthemainland,buttheplantationeconomiesof
theCaribbeansufferednorebellions.
D) TheonlyconsistentlocationforslaveresistanceandrebellionwastheBritishcoloniesof
thesouthernAtlanticcoast.
Answer: A
Diff:2
PageRef:561
Topic:TheAfricanDiaspora
Skill:Conceptual

411

49) WhichofthefollowingwasaBritishopponentoftheslavetrade?
A) JohnC.Calhoun
B) JohnWilkes
C) WilliamWilberforce
D) WilliamHanover
Answer: C
Diff:1
PageRef:563
Topic:TheAfricanDiaspora
Skill:Factual

50) Whichofthefollowingstatementsconcerningtheendingoftheslavetradeismostaccurate?
A) Ingeneral,theEnlightenment,withitsadvocacyoffreetrade,acceptedifitdidnot
approveoftheslavetrade.
B) LeadershipinthedrivetoabolishtheslavetradewasassumedbyFrance.
C) WhileitistruethatlegitimateproductsbegantoreplaceslavesintheEuropean
commercewithAfrica,itisdifficulttodemonstratealinkbetweeneconomicself-interest
andtheabolitionoftheslavetrade.
D) TheendoftheAtlanticslavetradeinthe19thcenturyledtotheendofslaveryinAfrica
itself.
Answer: C
Diff:2
PageRef:563
Topic:TheAfricanDiaspora
Skill:Conceptual

EssayQuestions
1) WhatmodeldidthePortugueseestablishfortheexploitationofAfrica?
Answer: EstablishmentoftradefortsandfactoriesonthecoastsofAfricaratherthan
developmentoftheinteriororEuropeancolonization;constructionofallianceswith
Africanrulersasthebasisofcommerce;earlyinvolvementintheslavetradeasa
profitablecommodity;useofChristianmissionaries.
Diff:2
PageRef:545-550
Topic:TheAtlanticSlaveTrade
Skill:Conceptual

412

2) DefinethestructureoftheAtlanticslavetrade.Considerthevolumeof
slavesbycentury,theplacestowhichslaveswereshipped,andtheoriginsof
slaveswithinAfrica.
Answer: Between1450and1850over12millionslavesshippedfromAfrica;80percentshipped
during18thcentury;from1530to1650mostslavesshippedtoLatinAmerican
mainland;after1650CaribbeanplantationislandsandsouthernBritishAtlanticcolonies
alsobecameimportant;in16thcenturymostslavescamefrom
Sengambia,in17thcenturyfromwestandcentralAfrica;in18thcenturyfrom
interiorstatesofAsanteandDahomey.
Diff:2
PageRef:547-550
Topic:TheStructureoftheAtlanticSlaveTrade
Skill:Conceptual

3) WhydidEuropeansparticipateintheAtlanticslavetrade?
Answer: NeedtosupplyqualifiedlaborforAmericancoloniesinLatinAmerica,Caribbean,and
NorthAmerica;Africansalreadyskilledinmetallurgyandintensiveagricultureas
Indianswerenot(theyhadnometaltools);slavesgenerallyfailedtoreproduceinNew
WorldandrequiredconstantresupplyfromAfrica;slavetradewasprofitable,although
notmuchmoresothanotherbusinessventures;creationoftriangulartradeslavesto
NewWorld,plantationproductstoEurope,manufacturedgoodstoAfrica.
Diff:2
PageRef:547-550
Topic:TheStructureoftheAtlanticSlaveTrade
Skill:Conceptual

4) TowhatextentwasslaveryafeatureofAfricansocietypriortothecomingoftheEuropeans?
Howwasitintertwinedwiththedevelopmentofmorecentralizedstates?
Answer: AfricanshadpracticedslaverybeforeEuropeansasresultofcaptivestakeninwar;
emphasisonfemaleslaveryasmeansofextendinglineage,authority;constantwarfare
inwesternandcentralAfricasuppliedstreamofslaves;interruptionofEuropeansgave
externalfocustoslavetrade,primarilyinmales;exchangeoffirearmsforslavesenabled
expandingstatestoutilizeslavetradeasmeansofenlargingpoliticalpower.
Diff:2
PageRef:550-556
Topic:AfricanSocieties,SlaveryandtheSlaveTrade
Skill:Conceptual

5) EvaluatethecausesfortheendoftheAtlanticslavetrade.
Answer: Economic:Africanstatesbegantoswitchfromslavestolegitimateproducts,less
Europeandemandforslaves;noclearevidenceofeconomicmotivationasplantation
economiesremainedstrong.Enlightenment:generalsocialandphilosophicalreversion
forslaveryafter18thcentury.Religious:evangelicalreligiousgroupsbegantoadvocate
endofslavetrade,abolitionofslavery.ImpactofBritain:Britaininfluencedbyallof
above;tookroleofsuppressingslavetradeinAtlantic.
Diff:2
PageRef:562-564
Topic:TheAfricanDiaspora
Skill:Conceptual

413

Chapter26 TheMuslimEmpires
Multiple-ChoiceQuestions
1) WhichofthefollowingwasNOToneoftheearlymodernIslamicempires?
A) Ottoman
B) Safavid
C) Abbasid
D) Mughal
Answer: C
Diff:1
PageRef:568
Topic:Introduction
Skill:Factual

2) WhichofthefollowingdoesNOTrepresentasimilarityamongthethreeMuslimearly
modernempires?
A) AllwerederivedfromTurkicnomadicculturesofthecentralAsiansteppe.
B) Eachempirewasbasedonreligionanditssocietywasorientedtowarddominationof
themullahs.
C) Alldynastiesdependedoneffectiveuseoffirearmsonthebattlefieldandinsiege
warfare.
D) Supportofthebureaucraciesandmilitarywasdrawnfromtaxesleviedonthepeoplesof
theagrariansocietiesthateachconqueredandruled.
Answer: B
Diff:2
PageRef:568
Topic:Introduction
Skill:Conceptual

3) PriortotheMongolinvasionsoftheirempire,theAbbasiddynastywasdominatedby
A) theOzbegTurks.
B) Timur-i-Lang.
C) theSeljukTurks.
D) theWest.
Answer: C
Diff:2
PageRef:569
Topic:TheOttomans:FromFrontierWarriorstoEmpireBuilders
Skill:Factual

414

4) TheoriginalbaseoftheOttomanTurkswas
A) Mesopotamia.
B) Syria.
C) theBalkans.
D) Anatolia.
Answer: D
Diff:2
PageRef:570
Topic:TheOttomans:FromFrontierWarriorstoEmpireBuilders
Skill:Factual

5) TheOttomansconqueredConstantinopleandendedtheByzantineEmpirein
A) 1245.
B) 1337.
C) 1453.
D) 1519.
Answer: C
Diff:2
PageRef:570
Topic:TheOttomans:FromFrontierWarriorstoEmpireBuilders
Skill:Factual

6) WhichofthefollowingstatementsconcerningOttomannavalpowerismostaccurate?
A) TheOttomanEmpireneverdevelopedanavy,thuswasunabletocontrolthe
MediterraneanorchallengethegrowingnavalsupremacyoftheWest.
B) PowerfulOttomangallyfleetscapturedmajorislandbasesintheeasternMediterranean,
butwereunabletopreventwesternadvancesintheIndianOcean.
C) TheOttomanEmpirewassufficientlypowerfulatseathatuntilthe18thcenturyitwas
abletomaintainnotonlydominanceoftheeasternMediterraneanbutalsoto
monopolizetheIndianOcean.
D) TheOttomansretainedtheirnavaldominanceintheIndianOcean,butalmost
immediatelylostalloftheirMediterraneanpossessions.
Answer: B
Diff:2
PageRef:575
Topic:TheOttomans:FromFrontierWarriorstoEmpireBuilders
Skill:Conceptual

415

7) TheJanissarieswere
A) religiousleadersunderthecontroloftheOttomanstate.
B) powerfulmembersofthewarrioraristocracywithintheOttomanEmpirewho
dominatedregionaladministration.
C) slavetroopsoftheOttomanforciblyconscriptedasadolescentsfromconquered
territories.
D) eunuchsfromtheroyalfamilychargedwithcareoftheimperialharem.
Answer: C
Diff:1
PageRef:571
Topic:TheOttomans:FromFrontierWarriorstoEmpireBuilders
Skill:Factual

8) WhatpermittedtheJanissariestogainapositionofprominenceintheOttomanEmpire?
A) Theircontrolofartilleryandfirearmsgavethemprominenceoverthearistocratic
Turkishcavalry.
B) Theircontrolofthebureaucracymadethemindispensibletotheoperationoftheempire.
C) Asmembersoftheroyalfamily,theyhadaccesstothesultans.
D) TheyrapidlygainedcontrolofthemosquesoftheOttomanEmpireandwereableto
definereligiousorthodoxy.
Answer: A
Diff:2
PageRef:571
Topic:TheOttomans:FromFrontierWarriorstoEmpireBuilders
Skill:Conceptual

9) TheheadoftheOttomancentralbureaucracywasthe
A) vizier.
B) dhimmi.
C) eunuch.
D) Caliph.
Answer: A
Diff:1
PageRef:572
Topic:TheOttomans:FromFrontierWarriorstoEmpireBuilders
Skill:Factual

416

10) WhatwastheprincipleofsuccessionwithintheOttomanEmpire?
A) LikeearlierIslamicdynasties,theOttomanEmpirelackedaprincipleofsuccession, afact
thatledtoprotractedwarfareamongprospectivesuccessors.
B) SuccessionwithintheOttomanEmpirewasbasedonprimogeniture,thatis,theoldest
sonautomaticallysucceededtheprevioussultan.
C) LiketheearlyIslamicadministrationoftheorthodoxcaliphs,thesuccessionswithinthe
OttomanEmpirewaselective.
D) Ottomansultansselectedtheirsuccessorspriortotheirdeathandelevatedthemas
co-rulers.
Answer: A
Diff:2
PageRef:572
Topic:TheOttomans:FromFrontierWarriorstoEmpireBuilders
Skill:Conceptual

11) OneofthemostbeautifuloftheOttomanmosquesofConstantinoplewasthe
A) TajMahal.
B) KahilJibran.
C) Saffah.
D) Suleymaniye.
Answer: D
Diff:2
PageRef:572
Topic:TheOttomans:FromFrontierWarriorstoEmpireBuilders
Skill:Factual

12) WhatdidtheOttomansdotoConstantinoplefollowingitsfallin1453?
A) TheOttomansdestroyedthecityandmovedtheircapitaltoSophia.
B) Theoriginalcityremained,butinamuchreducedconditionthattheOttomansdidlittle
torestore.
C) Soonafteritsconquest,theOttomansultanundertooktherestorationandbeautification
ofConstantinople.
D) TheOttomansrapidlyabandonedConstantinopletotheleadersoftheOrthodoxChurch
whowereresponsibleforitsrestorationandtheconstructionofsignificantchurches.
Answer: C
Diff:2
PageRef:572
Topic:TheOttomans:FromFrontierWarriorstoEmpireBuilders
Skill:Conceptual

417

13) InwhatwayweretheartisansofConstantinoplesimilartotheircounterpartsintheWest?
A) Theyhadbeguntoformaproletariat
B) LiketheircounterpartsinmedievalEuropeantowns,theartisanswereorganizedinto
guilds
C) InthecapitalcityoftheOttomanEmpire,artisanswerefreeofgovernmentalsupervision
D) LiketheearlierWest,craftproductionwaslimitedandtherewerefewindependent
artisans
Answer: B
Diff:2
PageRef:573
Topic:TheOttomans:FromFrontierWarriorstoEmpireBuilders
Skill:Conceptual

14) WhatwasthechosenlanguageoftheOttomancourt?
A) Greek
B) Arabic
C) Persian
D) Turkish
Answer: D
Diff:2
PageRef:573
Topic:TheOttomans:FromFrontierWarriorstoEmpireBuilders
Skill:Factual

15) HowdidtheOttomandynastycomparetootherrulingfamilies?
A) TheOttomandynastylastedforover600years, afeatmatchedbynootherrulingfamily.
B) TheOttomandynastywasamongthemostlong-lastinginhistory,survivingforover
200years,exceededonlybytheChinesedynasties.
C) Duetothelackofaprincipleofsuccession,theOttomansenduredconstantdynastic
changewithindividualfamiliessurvivingonlyfordecades.
D) BecausetheOttomansuccessionwaselective,itismisleadingtothinkofthesultansasa
singledynasty.
Answer: A
Diff:2
PageRef:573
Topic:TheOttomans:FromFrontierWarriorstoEmpireBuilders
Skill:Conceptual

418

16) WhichofthefollowingwasNOTacauseforthedeclineoftheOttomanEmpire?
A) Localofficialsbegantoretainincreasingamountsofrevenuefortheirownpurposes.
B) Oppressivedemandsoflocalofficialscausedthepeasantrytoabandontheirholdings
andflee.
C) Theabilityofindividualsultanstoruledeclined.
D) TheadditionofEuropeanmilitarytechnologysuchaslightartillerymadetheJanissaries
sopowerfulthattheycouldchallengetheauthorityofthesultan.
Answer: D
Diff:2
PageRef:574-576
Topic:TheOttomans:FromFrontierWarriorstoEmpireBuilders
Skill:Conceptual

17) Onthesea,theOttomangalleyswereeclipsedbyWesternnavalpowerasearlyasthe
A) 14thcentury.
B) 15thcentury.
C) 16thcentury.
D) 17thcentury.
Answer: C
Diff:2
PageRef:575
Topic:TheOttomans:FromFrontierWarriorstoEmpireBuilders
Skill:Factual

18) WhatEuropeannationfirstthreatenedtheOttomanmonopolyoftradewitheastAfricaand
India?
A) Spain
B) Portugal
C) England
D) France
Answer: B
Diff:1
PageRef:575
Topic:TheOttomans:FromFrontierWarriorstoEmpireBuilders
Skill:Factual

419

19) WhichofthefollowingwasNOTaresultoftheOttomanlossofmonopolyovertheIndian
trade?
A) SpicescarriedbyEuropeansaroundAfricaenrichedtheOttomansChristianrivals.
B) DirectcarriageofeasterngoodstoportsintheWestimpliedlossofrevenuesintaxesin
Muslimtradingcenters.
C) MilitarysetbacksrevealedtheobsolescenceoftheMuslimfleets.
D) BullioncausedbyanegativebalanceoftradeflowedoutoftheOttomanempiretothe
West.
Answer: D
Diff:2
PageRef:576
Topic:TheOttomans:FromFrontierWarriorstoEmpireBuilders
Skill:Conceptual

20) WhichofthefollowinggroupsrepresentedsuchextremeconservatismwithintheOttoman
Empirethatreformwasfrustrated?
A) Janissaries
B) Artisans
C) Merchants
D) Sultans
Answer: A
Diff:1
PageRef:576
Topic:TheOttomans:FromFrontierWarriorstoEmpireBuilders
Skill:Factual

21) WhichofthefollowingrepresentsadifferencebetweentheoriginsoftheOttomansandthe
Safavids?
A) TheSafavidsoriginatedfromaTurkishnomadicgroup
B) TheSafavidsrepresentedahighlymilitantstrainofIslam
C) TheSafavidswereoriginallyfrontierwarriors
D) TheSafavidsrepresentedtheShiitestrainofIslam
Answer: D
Diff:2
PageRef:576
Topic:TheShiiteChallengeoftheSafavids
Skill:Conceptual

420

22) ThecenteroftheSafavidEmpirewasthemodern-daystateof
A) Syria.
B) Iraq.
C) Jordan.
D) Iran.
Answer: D
Diff:1
PageRef:576
Topic:TheShiiteChallengeoftheSafavids
Skill:Factual

23) TheSafaviddynastyhaditsoriginsinthe14thcenturyinafamilydevotedtowhatvariantof
Islam?
A) Sunnite
B) Ismaili
C) Sufi
D) Sikh
Answer: C
Diff:2
PageRef:576
Topic:TheShiiteChallengeoftheSafavids
Skill:Factual

24) InwhatyearwasthefirstSafaviddeclaredshah?
A) 1453
B) 1501
C) 1553
D) 1722
Answer: B
Diff:2
PageRef:576
Topic:TheShiiteChallengeoftheSafavids
Skill:Factual

25) FollowersoftheSafavidsfollowerswerecalled
A) RedHeads.
B) YellowTurbans.
C) theWhiteLotusSociety.
D) theRedHand.
Answer: A
Diff:2
PageRef:576
Topic:TheShiiteChallengeoftheSafavids
Skill:Factual

421

26) ThefirstSafavidShahwas
A) AbbastheGreat.
B) SuleymantheGreat.
C) Akbar.
D) Isma il.
Answer: D
Diff:2
PageRef:576
Topic:TheShiiteChallengeoftheSafavids
Skill:Factual

27) WhywasthebattleofChaldiranin1514soimportant?
A) ThebattleestablishedthemilitarysupremacyoftheSafavidsovertheOttomansand
markedtheendtoeasternexpansionoftheOttomanEmpire.
B) TheSafavidsweredealtadevastatingdefeatthatcheckedthewestwardadvanceof
ShiismanddecimatedtheranksoftheTurkicwarriorswhohadbuilttheSafavid
Empire.
C) ThecombinedarmiesoftheSafavidsandOttomansdefeatedtheMughalarmiesand
endedthepolicyofexpansionundertakenbytheMughalemperorsofIndia.
D) ThedefeatoftheSafavidsbyaWesternarmyreducedtheIslamicEmpiretoeconomic
dependencyontheWestandmilitaryinferioritytotheotherMuslimempires.
Answer: B
Diff:2
PageRef:577
Topic:TheShiiteChallengeoftheSafavids
Skill:Conceptual

28) TheSafavidEmpirereacheditsgreatestextentunderShah
A) SuleymantheGreat.
B) TahmaspI.
C) AbbastheGreat.
D) Ismail.
Answer: C
Diff:2
PageRef:578
Topic:TheShiiteChallengeoftheSafavids
Skill:Factual

422

29) WhatwasthestatusoftheTurkicchiefsundertheSafavidshahs?
A) UnliketheOttomans,theTurkishcavalrymenweredrivenfromtheSafavidEmpire.
B) TheformerTurkishwarriorswerereplacedentirelybytheindigenousPersiannobility
whoformedamercenarymilitaryforce.
C) LiketheOttomans,theTurkishchiefsweregraduallytransformedintoawarriornobility
withassignedvillagesandpeasantlabor.
D) TheformerTurkishnomadswerereducedtoslavery,butstillcomposedthefoundation
oftheSafavidmilitary.
Answer: C
Diff:2
PageRef:580
Topic:TheShiiteChallengeoftheSafavids
Skill:Conceptual

30) AfterChaldiran,theofficiallanguageoftheSafavidEmpirebecame
A) Turkish.
B) Persian.
C) Arabic.
D) Hindi.
Answer: B
Diff:1
PageRef:580
Topic:TheShiiteChallengeoftheSafavids
Skill:Factual

31) ThecapitaloftheSafavidEmpireunderAbbastheGreatwas
A) Tabriz.
B) Baghdad.
C) Mosul.
D) Isfahan.
Answer: D
Diff:1
PageRef:580
Topic:TheShiiteChallengeoftheSafavids
Skill:Factual

423

32) HowdidtheSavafideconomycomparetothatoftheOttomans?
A) OnlytheOttomansultansactivelysoughttoencouragehandicraftproductionandtrade
intheirempire.
B) TheOttomanEmpirebenefittedintheshortrunfromnon-Muslimtraders(Christians
andJews)whohadextensivecontactswithoverseasempiresthattheSafavidempire
lacked.
C) TheSafavidEmpiremadeamoreconcertedandsuccessfulattempttoenlargeitsmarket
economy,particularlybyattractingmerchantsfromtheWest.
D) TheSafavideconomy,becauseofthegeographicallocationoftheempire,wasoriented
exclusivelytowardtheEastandhadnocontactwithWesternmerchants.
Answer: B
Diff:2
PageRef:582
Topic:TheShiiteChallengeoftheSafavids
Skill:Conceptual

33) WhatledtotherapiddemiseoftheSafavidEmpire?
A) LiketheOttomanEmpire,thelackofaprincipleofsuccessionledAbbastheGreatto
eliminateallcapablerivals,leavingnocapablerulerfollowinghisdeath.
B) TheSafaviddefeatatthebattleofPanipatatthehandsofaRussianarmystrippedthe
empireofitsmilitaryforcesjustaspressurefromoutsideenemiesincreased.
C) ThecollapseoftheSafavideconomyinthe18thcenturydiminishedtherevenuesofthe
empiretothepointthatthecentralgovernmentcouldnolongerfunction.
D) ThesuccessfulconquestoftheOttomanEmpireoverextendedtheSafavidresources,so
thatthecentralgovernmentbecameincreasinglyinefficient.
Answer: A
Diff:2
PageRef:582
Topic:TheShiiteChallengeoftheSafavids
Skill:Conceptual

34) TheimmediatesuccessoroftheSafaviddynastyinPersiawas
A) BaburtheGreat.
B) AkbartheGreat.
C) SuleymantheGreat.
D) NadirKhanAfshar.
Answer: D
Diff:1
PageRef:582
Topic:TheShiiteChallengeoftheSafavids
Skill:Factual

424

35) ThefounderoftheMughaldynastywas
A) Timur.
B) Akbar.
C) Babur.
D) NadirKhanAfshar.
Answer: C
Diff:1
PageRef:582
Topic:TheMughalsandtheApexofMuslimCivilizationinIndia
Skill:Factual

36) WherewasBaburborn?
A) India
B) France
C) China
D) CentralAsia
Answer: D
Diff:1
PageRef:583
Topic:TheMughalsandtheApexofMuslimCivilizationinIndia
Skill:Factual

37) WhichofthefollowingdescriptionsoftheaccomplishmentsofBaburisNOTaccurate?
A) Hewasafinemilitarystrategistandfiercefighterwhowentintobattlealongsidehis
troops.
B) HewroteoneofthegreathistoriesofIndiaandwasafinemusician.
C) HereformedtheineffectiveLodibureaucracytocreateastreamlinedadministration.
D) HewasafinemusiciananddesignedgardensforhisnewcapitalatDelhi.
Answer: C
Diff:2
PageRef:583
Topic:TheMughalsandtheApexofMuslimCivilizationinIndia
Skill:Conceptual

425

38) WhichofthefollowingstatementsmostaccuratelycharacterizesthereignofHumayan?
A) Humayanwasabletobuildonthereformsofhisfatherandenjoyedoneofthemost
stableandprosperousreignsoftheMughaldynasty.
B) HumayancompletedthetaskofcrushingoppositiontotheMughaldynastyduringthe
first20yearsofhisreign,butsettledintodissolutebehaviorlaterinhislife.
C) DespitetheseemingstabilityofBabursreign,hissonHumayanwasexiledby1540and
forcedtofighttorestoretheMughalrule.
D) HumayanwasassassinatedshortlyafterhisaccessionandtheMughaldynastywas
brieflyreplacedbyaHindufamily.
Answer: C
Diff:2
PageRef:583
Topic:TheMughalsandtheApexofMuslimCivilizationinIndia
Skill:Conceptual

39) WhichofthefollowingstatementsconcerningthereignofAkbarisNOTaccurate?
A) Hepersonallyoversawthebuildingofthemilitaryandadministrativesystem
B) Hepatronizedthearts
C) HeextendedtheMughalconquestsincentralandnorthernIndia
D) HeattemptedtopurifyIslambyremovingHinduinfluences
Answer: D
Diff:2
PageRef:583-585
Topic:TheMughalsandtheApexofMuslimCivilizationinIndia
Skill:Conceptual

40) WhatwasthecriticalfeatureofAkbarsnewreligion,theDin-i-Ilahi?
A) Itattemptedtoblendelementsofthemanyfaithswithwhichhewasfamiliarasameans
ofreconcilingHindusandMuslims.
B) ItsoughttopurifytheIslamicfaithandremoveHinduinfluencesbyadoptingmanyof
theSufiteachingsalreadyextantinIndia.
C) LiketheSafavidstowhomearlymembersoftheMughaldynastyfledinexile,Akbar
adoptedmilitantShiism.
D) AlthoughitretainedtheformalappearanceofIslam,thepopularaspectsofthenew
religionwerealmostentirelyBuddhist.
Answer: A
Diff:2
PageRef:585
Topic:TheMughalsandtheApexofMuslimCivilizationinIndia
Skill:Conceptual

426

41) WhatwastheoutcomeoftheDin-i-Ilahi?
A) AcceptedbyHinduswhobenefittedfromthelenientpoliciesofAkbar,thenewreligion
wasrejectedbytheMuslims.
B) AcceptedbyMuslimswhosawitasameansofcementingtheauthorityoftheMughals
overthetraditionalHindurulers,thenewreligionwasrejectedbyHindus.
C) ThenewreligionwaswidelyacceptedduringAkbarslifetimeandbecamethestate
religionoftheMughalEmpire.
D) ByAkbarsdeathin1605,theDin -i-IlahiwasrejectedbybothMuslimsandHindus.
Answer: D
Diff:2
PageRef:585
Topic:TheMughalsandtheApexofMuslimCivilizationinIndia
Skill:Conceptual

42) WhichofthefollowingstatementsconcerningtheeconomyoftheMughalEmpireismost
accurate?
A) UnliketheotherMuslimempires,theMughalssuccessfullybannedEuropeanmerchants
fromtheirmarkets.
B) TheMughalEmpireproducednothingofvaluetotheWest,butservedasaconduitof
productsfromsoutheasternAsiamuchindemandamongtheEuropeans.
C) EuropeantradersbroughtproductsfromthroughoutAsiatoexchangeforthe
subcontinentsfamedcottontextiles.
D) TheIndianmarketsoftheMughalswerefloodedwithWesternproducts.
Answer: C
Diff:2
PageRef:585
Topic:TheMughalsandtheApexofMuslimCivilizationinIndia
Skill:Conceptual

43) WhatwasthestateoftheMughaldynastyattheoutsetofthereignofAurangzeb?
A) Althoughtheculturalleveloftheempirehaddeclined,theadministrativereformsof
ShahJahanrenewedthespiritoftheempire.
B) ThestatusofwomenintheMughalEmpirewashigherthanatthebeginningofthe
dynasty.
C) TheMughalbureaucracyremainedefficientandeffective,butthemilitaryhadlapsed
intotechnologicalconservatism.
D) TheMughalbureaucracywasbloatedandcorrupt,thearmywasbackwardinweaponry
andtactics,andthepeasantsandartisanshadseenlivingstandardsfall.
Answer: D
Diff:2
PageRef:586
Topic:TheMughalsandtheApexofMuslimCivilizationinIndia
Skill:Conceptual

427

44) WhatwerethegoalsofAurangzeb?
A) Hewishedtoretreatfrommilitantprogramsofhispredecessorsinordertoemphasize
theBuddhistprogramofpacifismandwithdrawalfromthethingsofthisworld.
B) HewasdeterminedtoextendMughalcontroloverthewholeoftheIndiansubcontinent
andbelieveditwashisdutytopurifyIndianIslam.
C) HewasinsistentonintroducingWesterncultureandWesterntechnologytothe
conservativeMughalempire.
D) AsadevoutHindu,AurangzebwishedtoridtheempireofMuslimtraditions.
Answer: B
Diff:2
PageRef:586
Topic:TheMughalsandtheApexofMuslimCivilizationinIndia
Skill:Conceptual

45) WhatwastheimpactofAurangzebsreligiouspolicies?
A) AurangzebsreligiouspoliciesgravelyweakenedtheinternalalliancesbetweenHindus
andMuslimsanddisruptedthesocialpeacethatAkbarhadestablished.
B) AurangzebsreligiouspoliciessucceededinremovingBuddhismfromIndia.
C) TheemperorsucceededinreestablishingthepoliticalandsocialdominanceoftheHindu
majorityinIndia.
D) AurangzebsintroductionofChristianitytoIndiabegantheprocessbywhichmillionsof
IndianswereconvertedtothenewreligionandprovidedanentryforWestern
commerce.
Answer: A
Diff:2
PageRef:586
Topic:TheMughalsandtheApexofMuslimCivilizationinIndia
Skill:Conceptual

46) WhatwasthestateoftheMughalEmpirefollowingAurangzebsdeathin1707?
A) TheMughalEmpirehadshrunkensomuchduringAurangzebsreignthatthedynasty
controlledonlyBengal.
B) Theempireincludedmoreterritorythaneverbeforeandtherewasgreaterreligious
homogeneitythanearlierinthereign.
C) Theempirewasfarlargerthanearlier,butcontrolandstaterevenuespassedincreasingly
toregionallordswhogavelittlemorethantributepaymentstotheemperors.
D) TheempirecollapsedinthefaceofaSafavidinvasionfromtheIndusrivervalley.
Answer: C
Diff:2
PageRef:586
Topic:TheMughalsandtheApexofMuslimCivilizationinIndia
Skill:Conceptual

428

47) WhichofthefollowingstatementsconcerningtherelationshipsoftheMuslimempirestothe
WestisNOTaccurate?
A) NoneoftheIslamicemperorssystematicallymonitoredtechnologicaladvancesin
Europe.
B) KeytaxrevenuesandmerchantprofitsweredrainedoffbytheriseofEuropeantrading
empiresinAsia.
C) Theabsenceofminoritypeopleswithintheempiresleftthemwithoutcommercialand
culturalcontactswiththeWest.
D) MuslimscholarscontinuedtotakelittleinterestinEuropeanlearning.
Answer: C
Diff:2
PageRef:590
Topic:Conclusion:TheRiseofEuropeandtheEclipse
Skill:Conceptual

48) WhereistheTajMahal?
A) Egypt
B) Turkey
C) India
D) Iran
Answer: C
Diff:2
PageRef:587
Topic:TheOttomans:FromFrontierWarriorstoEmpireBuilders
Skill:Conceptual

EssayQuestions
1) CompareandcontrastthebasicstructureofthethreeIslamicempires.
Answer: Similarities:eachhadoriginsinTurkishnomadicculture;religiousfervoranddesirefor
conversioncommontoallthree;basedonmilitaryconquestandorientedtoward
militaryclasses;successdependentonuseoffirearms;ruledbyabsolutemonarchs;
supportofempiresdependentontaxationofagrarian,indigenouspopulations.
Differences:religiousmajorityofempiresdiffered;differedonSunni/Shiasplit.
Diff:2
PageRef:566-572
Topic:Introduction
Skill:Conceptual

429

2) DiscussthesocialandpoliticalorganizationoftheOttomanEmpire.
Answer: Turkishcavalrymentransformedintolandholdingaristocracy;grantedpossessionof
villageswithpeasantlabor;createdregionalpowerbases;supplementarymilitaryforce
createdfromslavetroopsdrawnfromconqueredterritories,Janissaries;controlled
firearmsandartilleryasbasisofpower,gainedpoliticalinfluence;sultanstechnically
absolute,butdependentonbureaucracy;someinfluenceofreligiousleaderswithin
bureaucracy.
Diff:2
PageRef:569-572
Topic:TheOttomans:FromFrontierWarriorstoEmpireBuilders
Skill:Conceptual

3) CompareandcontrastthecausesfordeclineandcollapseineachoftheIslamicearlymodern
empire.
Answer: Similarities:declineinqualityofcentraladministration,corruption;incaseofSafavids
andOttomansdeclineinqualityofrulersduetoproblemswithprincipleofsuccession;
growthofpowerofregionalaristocracywithincreasingcontroloverlocalrevenuesthat
weredivertedfromthestate;oftenaccompaniedbyabandonmentofland;failureto
successfullycountergrowingeconomicandmilitarypowerofWest.Differences:
Safavidsactuallyconqueredbyoutsideinvadersin1722;MughalsandOttomans
continuedtorule,althoughinreducedcircumstances;Mughalslostreligiousalliance.
Diff:2
PageRef:568-590
Topic:TheOttomans:FromFrontierWarriorstoEmpireBuilders
Skill:Conceptual

4) CompareandcontrastthesocialandpoliticalorganizationoftheSafavidstothatofthe
Ottomans.
Answer: Similarities:OriginalTurkishwarriorsconvertedtolandholdingaristocracy;creationof
slavemilitaryforce;absoluteruleoftheshah.Differences:introductionofPersiansas
membersofbureaucracy;greaterinfluenceofIslamicclergy;adoptedmilitantShiismin
contrasttoOttomanSunnism.
Diff:2
PageRef:569-582
Topic:TheShiiteChallengeoftheSafavids
Skill:Conceptual

5) HowdidthereligiouscomplexionofMughalIndiadifferfromthatoftheotherIslamic
empires?WhatimpactdidreligiousdiversityhaveontheMughals?
Answer: MughalEmpirehadmajorityofnon-Muslims,mostofwhichwereHindu;previous
failuretoamalgamatereligionscontinuedinMughalEmpire;Akbarattemptedtocreate
newreligionthatwouldcombinebothIslamandotherIndianreligions;Akbars
Din-i-Ilahifailed,rejectedbyMuslimsandHindus;inreignofAurangzebreligious
trucearrangedbyAkbarfailed;Hindusbegantoloseallegiancetodynasty;contributed
togrowthofregionalauthority,failureofcentralgovernment.
Diff:2
PageRef:562-590
Topic:TheMughalsandtheApexofMuslimCivilizationinIndia
Skill:Conceptual

430

Chapter27 AsianTransitionsinanAgeof
GlobalChange
Multiple-ChoiceQuestions
1) WhattypeofshipsdidVascodaGamasailininhisvoyagetoIndia?
A) Caravelships
B) Longships
C) Steamships
D) Bedships
Answer: A
Diff:1
PageRef:594
Topic:Introduction
Skill:Factual

2) AllofthefollowingweresourcesofdisappointmenttotheEuropeanswhoarrivedinAsiain
the16thand17thcenturiesEXCEPT
A) AsianswereuninterestedinEuropeantradegoods.
B) AsianswereuninterestedinconvertingtoChristianity.
C) Asiancivilizationseemedmateriallyimpoverished.
D) theEuropeansweretoofewinnumberstoexertforceonAsiankingdoms.
Answer: C
Diff:2
PageRef:594-595
Topic:Introduction
Skill:Conceptual

3) WhosevoyagesofexplorationopenedthewayfortheEuropeanstotheIndies?
A) ChristopherColumbus
B) VascodaGama
C) FrancisXavier
D) PoncedeLeon
Answer: B
Diff:1
PageRef:592
Topic:Introduction
Skill:Factual

431

4) Despitetheirarmaments,whatfactorconvincedtheEuropeansthattheycouldmakelittle
headwayagainstthekingdomsofAsia?
A) TheinferiorityofEuropeanships
B) ThedistancefromEuropeanports
C) Europeanlackofbullion
D) ThelargepopulationsandwellentrenchedpoliticalandeconomicsystemsofAsian
kingdoms
Answer: D
Diff:2
PageRef:594
Topic:Introduction
Skill:Conceptual

5) WhatwasthePortugueselessonlearnedatCalicut?
A) ThattheIndianmarketshadlittleofinteresttoWesternconsumers
B) ThattheIndiansrefusedtotradewithEuropeans
C) ThattheWesternproductsbroughtfortradewereoflittleornovalueinIndianmarkets
D) ThatWesternbullionwasofnouseintheEast
Answer: C
Diff:2
PageRef:595
Topic:TheAsianTradingWorldandtheComingoftheEuropeans
Skill:Conceptual

6) WhatwastheinitialPortugueseresponsetotheencounteratCalicut?
A) ToabandonhopesofenteringtheAsianmarkets
B) ToexportincreasingamountsofWesternproductstoAsia
C) TobypassIndiaandtradedirectlywithChina
D) ToapplymilitaryforcetoobtaindesiredAsianproducts
Answer: D
Diff:2
PageRef:595
Topic:TheAsianTradingWorldandtheComingoftheEuropeans
Skill:Conceptual

432

7) WhatpeopleshadprecededthePortugueseinenteringthemarketsofsouthandsoutheast
Asia?
A) Africans
B) Muslims
C) Dutch
D) English
Answer: B
Diff:1
PageRef:594
Topic:TheAsianTradingWorldandtheComingoftheEuropeans
Skill:Factual

8) WhichofthefollowingproductswasassociatedwiththeArabzoneoftheAsianseatrading
network?
A) Glass
B) Cottontextiles
C) Paper
D) Porcelain
Answer: A
Diff:2
PageRef:595
Topic:TheAsianTradingWorldandtheComingoftheEuropeans
Skill:Factual

9) WhichofthefollowingproductswasassociatedwiththeIndianzoneoftheAsianseatrading
network?
A) Tapestry
B) Cottontextiles
C) Carpets
D) Paper
Answer: B
Diff:2
PageRef:595
Topic:TheAsianTradingWorldandtheComingoftheEuropeans
Skill:Factual

433

10) WhichofthefollowingproductswasNOToneoftheproductsassociatedwiththeChinese
zoneoftheAsianseatradingnetwork?
A) Paper
B) Porcelain
C) Silktextiles
D) Cottontextiles
Answer: D
Diff:1
PageRef:595
Topic:TheAsianTradingWorldandtheComingoftheEuropeans
Skill:Factual

11) WhatwasthesignificanceofthemainlandkingdomsandislandstatesofsoutheastAsiathat
surroundedthethreegreatmanufacturingzonesoftheAsianseatradingnetwork?
A) Theseregionswerevirtuallythesoleconsumersofgoodsmadeinthemanufacturing
regions.
B) Theseregionsfedrawmaterials, preciousmetalsandforestproducts,intothetrading
network.
C) Theseregionsprovidedthemilitarydefenseforthemanufacturingregions.
D) TheseregionshadnorelationshiptothethreemainmanufacturingzonesoftheAsian
seatradingnetwork.
Answer: B
Diff:2
PageRef:595
Topic:TheAsianTradingWorldandtheComingoftheEuropeans
Skill:Conceptual

12) WhatrawmaterialsweremosthighlyvaluedintheAsianseatradingnetwork?
A) Iron
B) Lumber
C) Spices
D) Fish
Answer: C
Diff:1
PageRef:595
Topic:TheAsianTradingWorldandtheComingoftheEuropeans
Skill:Factual

434

13) Whichofthefollowingitemswasmorelikelytobeexchangedwithintheportsofeachofthe
maintradingzoneratherthanovergreaterdistancesbetweenzones?
A) Rice
B) Spices
C) Ivory
D) Silktextiles
Answer: A
Diff:2
PageRef:595
Topic:TheAsianTradingWorldandtheComingoftheEuropeans
Skill:Factual

14) WhatwasthenatureofthesearoutesintheAsiantradingnetwork?
A) Well-establishedroutesdirectlycrossingthemajoroceansweremaintainedfromancient
times.
B) Mostofthenavigationwasalongthecoastlines.
C) OnlytheChineseandArabspracticednavigationintheAsiantradingnetwork.
D) Theonlysea-goingroutescrossedtheIndianOceanfromtheSwahiliportsofeastAfrica
toIndia.
Answer: B
Diff:2
PageRef:595
Topic:TheAsianTradingWorldandtheComingoftheEuropeans
Skill:Conceptual

15) WhichofthefollowingwasoneofthecrucialpointsintheAsianseatradingnetworkwhere
tradeconverged?
A) ThemouthoftheGangesRiver
B) Canton
C) ThestraitsofMalacca
D) ThemouthoftheHuangheRiver
Answer: C
Diff:2
PageRef:595
Topic:TheAsianTradingWorldandtheComingoftheEuropeans
Skill:Factual

435

16) WhichofthefollowingstatementsmostaccuratelydescribesthenatureoftheAsiansea
tradingnetwork?
A) TheentirenetworkwasdominatedbyArabmerchantswhoworkedincommoncauseto
establishamonopolyoftrade.
B) TheChinese,asaresultoftheirnavalsuperiority,wereabletosecuremilitary
dominanceofthesystem.
C) Therewasnocentralcontrol,andforcewasusuallyabsentfromcommercialexchanges.
D) ThesystemfunctionedonlysolongasitwasadministeredfromtheOttomanEmpire
withthetacitsupportoftheMingdynasty.
Answer: C
Diff:2
PageRef:595
Topic:TheAsianTradingWorldandtheComingoftheEuropeans
Skill:Conceptual

17) WhywerethePortugueseunwillingtoexchangebullionforproductswithintheAsian
commercialsystem?
A) Thedoctrineofmercantilismequatedpossessionofbullionwithpowerandargued
againstnegativetradebalances.
B) Theyhadnone.
C) TherewaslittlemerchandisethatthePortugueseconsideredofvalueintheAsiantrade
network.
D) AllofPortuguesebullionwasexchangedforslavesintheAfricancommercialsystem.
Answer: A
Diff:2
PageRef:595
Topic:TheAsianTradingWorldandtheComingoftheEuropeans
Skill:Conceptual

18) WhydidthePortuguesebelievetheycouldsuccessfullyentertheAsianseatradingbyforce?
A) TheAsianempireslackednavies.
B) TherewaslittlemilitaryforceinanyoftheMuslimorAsianempires.
C) ThePortugueserealizedthattheirarmiesweremorenumerousthanthoseoftheAsian
opponents.
D) ThePortuguesehadfewerships,buttheyweremoremaneuverableandbetterarmed
thanthoseoftheirAsianopponents.
Answer: D
Diff:2
PageRef:596
Topic:TheAsianTradingWorldandtheComingoftheEuropeans
Skill:Conceptual

436

19) ThePortuguesewonamajorseabattleoveracombinedfleetofEgyptianandIndianvesselsin
1509at
A) Malacca.
B) Diu.
C) Samarkand.
D) Batavia.
Answer: B
Diff:2
PageRef:596
Topic:TheAsianTradingWorldandtheComingoftheEuropeans
Skill:Factual

20) WhichofthefollowingwasNOTafortifiedtradingportestablishedbythePortugueseinthe
early16thcentury?
A) Batavia
B) Ormuz
C) Malacca
D) Goa
Answer: A
Diff:2
PageRef:596
Topic:TheAsianTradingWorldandtheComingoftheEuropeans
Skill:Factual

21) WhattradedidthePortugueseintendtomonopolizewithintheAsiantradingnetwork?
A) Slaves
B) Ivory
C) Spices
D) Cottontextiles
Answer: C
Diff:2
PageRef:596
Topic:TheAsianTradingWorldandtheComingoftheEuropeans
Skill:Factual

437

22) HowsuccessfulwasthePortuguesemonopolyonAsianproducts?
A) ForsomedecadestheywereabletomaintainacompletemonopolyoverAsianproducts
shippedtoEurope.
B) Thoughtheymanagedtomonopolizesomespicesgrowninlimitedlocales,the
Portugueselackedthemanpowerandshipstosustainamonopoly.
C) ThePortuguesewereunabletoachievecontroloveranyAsianproductsdueto
competitionfromtheChinesecommercialnavy.
D) ThePortuguesemonopolywasrigidlyenforcedoverAsianproductsforalmosttwo
centuries.
Answer: B
Diff:2
PageRef:597
Topic:TheAsianTradingWorldandtheComingoftheEuropeans
Skill:Conceptual

23) WhosucceededthePortugueseasthemostsuccessfulEuropeanentrantintotheAsiansea
tradingnetwork?
A) England
B) Italy
C) Spain
D) Holland
Answer: D
Diff:1
PageRef:597
Topic:TheAsianTradingWorldandtheComingoftheEuropeans
Skill:Factual

24) WherewasthechiefDutchtradingfortressandportinsoutheastAsia?
A) Ormuz
B) Goa
C) Batavia
D) Darsan
Answer: C
Diff:1
PageRef:597
Topic:TheAsianTradingWorldandtheComingoftheEuropeans
Skill:Factual

438

25) HowdidtheDutchcommercialstrategywithintheAsiantradenetworkdifferfromthatofthe
Portuguese?
A) TheDutchlackedasubstantialnavy,andcouldnotusewarshipstomaintaintheir
commercialadvantage.
B) TheDutchdidnotmakeuseoffortifiedtownsandfactories.
C) TheDutchweremoresystematicintheirmonopolycontrolofalimitednumberof
specificspices.
D) TheDutchweremorehumaneintheirtreatmentofislandpeopleswhocultivatedthe
spices.
Answer: C
Diff:2
PageRef:597
Topic:TheAsianTradingWorldandtheComingoftheEuropeans
Skill:Conceptual

26) InwhatwaydidtheDutchandEnglishparticipationwithintheAsianseatradingnetwork
changebythemiddledecadesofthe17thcentury?
A) ForboththeDutchandtheEnglish,peacefulcommercecametobemoreprofitablethan
forciblecontrolandmonopolieswereaimedatEuropeanratherthanAsianrivals.
B) BothnorthernEuropeannationsabandonedthecommerceinspicesinfavorofcotton
andsilktextiles.
C) UnlikethePortugueseandSpanish,thenorthernEuropeannationsundertookwholesale
conversiontoProtestantismoftheinhabitantsoftheSpiceIslands.
D) Asallies,theDutchandEnglishwereabletoestablishanavalsupremacyinAsia
sufficienttomonopolizealltradewithintheAsianseatradingnetwork.
Answer: A
Diff:2
PageRef:597
Topic:TheAsianTradingWorldandtheComingoftheEuropeans
Skill:Conceptual

27) WhatareaofthePhilippinesweretheSpanishNOTabletoconquer?
A) Luzon
B) Suhong
C) Java
D) Mindanao
Answer: D
Diff:2
PageRef:599
Topic:TheAsianTradingWorldandtheComingoftheEuropeans
Skill:Factual

439

28) AmongwhichofthefollowinggroupsdidRomanCatholicmissionariesenjoysomesuccess?
A) TheanimistsofthesouthernPhilippines
B) Hindubrahmans
C) TheChinese
D) OutcastegroupsinIndiancoastalregions
Answer: D
Diff:2
PageRef:599
Topic:TheAsianTradingWorldandtheComingoftheEuropeans
Skill:Factual

29) WhatJesuitmissionarywasresponsibleforcreatingthestrategyofconvertingHinduelitesas
ameansofachievingmassconversions?
A) FrancisXavier
B) RobertdiNobili
C) MatteoRicci
D) AdamSchall
Answer: B
Diff:2
PageRef:600
Topic:TheAsianTradingWorldandtheComingoftheEuropeans
Skill:Factual

30) InwhatsensewastheSpanishconversionoftheFilipinossimilartotheirexperienceinthe
Americas?
A) AllChristiantenetsweretaughtinthelanguageoftheindigenouspeoples.
B) FilipinoconversiontoChristianitywaspredicatedonpoliticalequalitywiththeSpanish
conquerors.
C) LiketheAmerindians,theFilipinosbrandofChristianityrepresentedacreativeblendof
earlierbeliefsandpracticeswithChristianity.
D) FewFilipinoswereconvertedtoChristianity.
Answer: C
Diff:2
PageRef:600
Topic:TheAsianTradingWorldandtheComingoftheEuropeans
Skill:Conceptual

440

31) WhichofthefollowingwasNOTaEuropeancontributiontotheAsianseatradingnetwork?
A) Theadditionofnewroutes,includingthelinktoEuropearoundtheCapeofGoodHope
B) TheintroductionofseawarfareintotheAsiantradenetwork
C) TheestablishmentofnewtradingcenterssuchasGoa,Calcutta,andBatavia
D) TheestablishmentofanexchangeofnewcropsanddiseasessimilartotheColumbian
ExchangewiththeAmericas
Answer: D
Diff:2
PageRef:600
Topic:TheAsianTradingWorldandtheComingoftheEuropeans
Skill:Conceptual

32) ThefirstMingemperorofChinawas
A) Yunglo.
B) Hongwu.
C) Kangxi.
D) Zhenghe.
Answer: B
Diff:1
PageRef:601
Topic:MingChina:AGlobalMissionRefused
Skill:Factual

33) WhichofthefollowingreformswasNOTintroducedbythefirstMingemperor?
A) Thepositionofthescholar-gentrywithinthebureaucracywasrestored.
B) Statesubsidiesforimperialacademiesandregionalcollegeswerereinstituted.
C) FamilyinfluenceintheselectionofmentotheChinesebureaucracywaseliminated.
D) Thecivilserviceexaminationsystemwasreinstated.
Answer: C
Diff:2
PageRef:601
Topic:MingChina:AGlobalMissionRefused
Skill:Conceptual

441

34) WhichofthefollowingwasareforminstitutedbythefirstMingemperortoreducecourt
factionalismandthepowerofthescholar-gentry?
A) Achiefministerwasappointedfromtheroyalfamilytooverseeallworkoftheimperial
bureaucracy.
B) Corruptorincompetentmembersofthebureaucracywerepunishedbybeingbeatenon
thebarebuttocks.
C) Imperialwivescouldonlycomefromspecifiednoblefamiliesofgoodrepute.
D) Eunuchswereexpelledfromtheroyalhousehold.
Answer: B
Diff:2
PageRef:602
Topic:MingChina:AGlobalMissionRefused
Skill:Conceptual

35) WhichofthefollowingstatementsconcerningMingreformsinfavorofthepeasantryismost
accurate?
A) TheearlyMingemperorswerecompletelyuninterestedintheplightofthepeasantry.
B) ThefirstMingemperorattemptedtoincreasetheforcedlabordemandsonthepeasantry
inordertorestoretheChineseeconomyfollowingtheexpulsionoftheMongols.
C) Despitesomeattemptstoimproveeconomicconditionsforthepeasantry,thegrowing
poweroftherurallandlordsledtoincreasedtenancyandlandlesslaborers.
D) TheMingreformsresultedinareductionintheauthorityofthelocallandlordsandthe
establishmentofsmallfarmingoperationsthroughoutChina.
Answer: C
Diff:2
PageRef:603
Topic:MingChina:AGlobalMissionRefused
Skill:Conceptual

36) WhichofthefollowingstatementsconcerningMingsocialorganizationismostaccurate?
A) TheadoptionofmoreBuddhistbeliefsbegantobreakdownthestrictpatternsof
deferencethathadbeencustomaryinHanandSongChina.
B) Occupationalalternativesforwomenofallsociallevelsdramaticallyexpandedduring
theMingera.
C) Amongthegroupsgrantedalmosttotalfreedomfromthebondsofsocialstatuswerethe
studentsseekingentryintothescholar-gentry.
D) Underthecontinuedinfluenceofneo-Confucianideology,Mingsocietyremained
rigidlystratifiedwithemphasisondeferenceofyouthtoeldersandwomentomen.
Answer: D
Diff:2
PageRef:603
Topic:MingChina:AGlobalMissionRefused
Skill:Conceptual

442

37) WhichofthefollowingreasonsisatleastinpartresponsibleforthepeoplingoftheYangzi
regioninthesouthernpartofChinaduringtheMingera?
A) TheintroductionofcropsfromtheAmericasthatcouldbecultivatedoninferiorsoils
thatdidnotrequireirrigation
B) TheenforcedmigrationofremainingMongolswithintheChinesepopulationtothe
regionoftheYangze
C) Theabandonmentofriceandmilletcultivationintheregioninfavorofwheat
introducedbyPortuguesemerchants
D) TheopeningoftheregiontosettlementfromSiamandVietnam
Answer: A
Diff:2
PageRef:604
Topic:MingChina:AGlobalMissionRefused
Skill:Conceptual

38) WherewereforeignerspermittedtodobusinessinChinaduringtheMingera?
A) Atanyport
B) OnlyattheMingcapitalatBeijing
C) AtMacaoandCanton
D) Nowhere
Answer: C
Diff:2
PageRef:604
Topic:MingChina:AGlobalMissionRefused
Skill:Factual

39) WhichofthefollowingstatementsconcerningtheMingeconomyismostaccurate?
A) MerchantsfailedtorealizeprofitsfromtheMingcommercialboom
B) Almostallcommercialprofitswerereinvestedintrade
C) TheChinesegovernmentdidnottaxtrade,thusdidnotprofitfromcommercialgrowth
D) Muchmerchantwealthwasinvestedinlandasameansofsocialadvancement
Answer: D
Diff:2
PageRef:604
Topic:MingChina:AGlobalMissionRefused
Skill:Conceptual

443

40) Intermsofliterature,whatwasthechiefaccomplishmentoftheMingera?
A) Poetry
B) Narrativehistory
C) Thenovel
D) Haiku
Answer: C
Diff:1
PageRef:605
Topic:MingChina:AGlobalMissionRefused
Skill:Factual

41) DuringthereignofwhatMingemperordidtheChineselaunchcommercialexpeditionsto
southeastAsia,Persia,andAfrica?
A) Hongwu
B) Yunglo
C) Chongzhen
D) Kangxi
Answer: B
Diff:2
PageRef:605
Topic:MingChina:AGlobalMissionRefused
Skill:Factual

42) WhydidtheChineseabandonthecommercialvoyagesoftheZengheexpeditions?
A) Manyoftheshipswerelostasaresultofpoorshipdesignandinadequatesailing
technology.
B) Thesizeofthefleetswassolimitedthattheycouldnotcompetewiththegreatercapacity
oftheEuropeanvoyages.
C) TherewaslittleofvaluefortheChinesetoimportintrade,andthevoyageswere
expensivetocarryout.
D) ThetradewithforeignregionsproducedanegativebalanceoftradeforChinathat
drainedbullionfromimperialcoffers.
Answer: C
Diff:2
PageRef:606-607
Topic:MingChina:AGlobalMissionRefused
Skill:Conceptual

444

43) InwhatwaydidtheJesuitmissionariesmaintaintheirpositionsatthecourtoftheMing
emperors?
A) ByconvertingYunglotoChristianity
B) BymaintainingasmallbutpowerfulEuropeanarmyintheChinesecapital
C) Byacceptingthesupportofthescholar-gentry
D) Bydemonstratingknowledgeofscientificandtechnologicalskills
Answer: D
Diff:2
PageRef:608
Topic:MingChina:AGlobalMissionRefused
Skill:Factual

44) WhichofthefollowingwasNOToneofthethreemilitarycentralizersofJapan?
A) Nobunaga
B) HiataAshikaga
C) ToyotomiHideyoshi
D) TokugawaIeyasu
Answer: B
Diff:2
PageRef:609-610
Topic:FendingOfftheWest:JapansReunificationandtheFirstChallenge
Skill:Factual

45) InwhatyearwastheTokugawaShogunate founded,markingthereestablishmentofcentral


governmentinJapan?
A) 1593
B) 1603
C) 1633
D) 1653
Answer: B
Diff:2
PageRef:610
Topic:FendingOfftheWest:JapansReunificationandtheFirstChallenge
Skill:Factual

445

46) WhydidtheearliestoftheJapanesemilitarycentralizersacceptChristianmissionaries?
A) HiswifewasaChristianwhowasabletoexertherinfluencethroughouthishousehold.
B) ThePortuguesesuppliedalargearmytorulerswhoofferedtoacceptChristianity.
C) ChristianitywasseenasacounterforcetotheBuddhistordersthatopposedthe
impositionofcentralrule.
D) Priortohisfirstmilitaryvictory,Nobunagasawacrossinthesky.
Answer: C
Diff:2
PageRef:610
Topic:FendingOfftheWest:JapansReunificationandtheFirstChallenge
Skill:Conceptual

47) WhichofthefollowingwasNOTapolicyimposedasaresultofJapaneseisolationinthe17th
century?
A) ChristianitywasbannedandChristianswerepersecuted.
B) ForeigntraderswereconfinedtotheislandofDeshimainNagasakiBay.
C) Neo-Confucianphilosophygavewaytotheinfluenceofthinkerswhochampionedthe
schoolofNationalLearning.
D) TheJapaneseeliteabandonedallcontactwithWesternlearningandtechnological
advance.
Answer: D
Diff:2
PageRef:611-612
Topic:FendingOfftheWest:JapansReunificationandtheFirstChallenge
Skill:Conceptual

EssayQuestions
1) DefinethecharacteristicsoftheAsianseatradingnetwork.InwhatwaysdidtheEuropean
incursionchangetheAsiansystem?
Answer: Threemajormanufacturingzones:Arabproducingcarpets,tapestry,glass;Indian
producingcottontextiles;Chinaproducingporcelain,paper,silks.Nocentralcontrolof
system,nouseofmilitaryforce.Portuguesebroughtuseofmilitaryforceintosystem;
addednewroutesincludingroutearoundCapeofGoodHopetoEurope;additionof
newtradingcenterssuchasthoseatGoa,Ormuz,Batavia;introductionofconceptof
seapowerandmilitaryforce;introductionofChristianity,tributekingdoms.
Diff:2
PageRef:592-599
Topic:TheAsianTradingWorldandtheComingoftheEuropeans
Skill:Conceptual

446

2) WhatwasthenatureoftheMingrestorationoftraditionalChinesevalues?Whatinnovations
weremade?
Answer: Mingreturnedtouseofneo-Confucianphilosophyasbasisofculture;restoredposition
ofscholar-gentry;reinstitutedexaminationsystemasbasisofcivilservice.Early
emperorsattemptedtocurtailpowerofscholar-gentry;abolishedpositionofchief
minister;restrictedimperialmarriagetocommonerfamiliestoreduceopportunityfor
courtintrigue;numberofeunuchslimited;potentialrivalstosuccessionexiledto
provinces;greatesteconomicreformwasZenghevoyagestodistantmarkets.
Diff:2
PageRef:600-609
Topic:MingChina:AGlobalMissionRefused
Skill:Conceptual

3) CompareandcontrastChineseandEuropeanmeansandmotivesforcommercialexpansion.
Answer: Means:EuropeantechnologicaladvancesgavethemadvantagesoverChineseinterms
ofanimalandmachinepowertheycouldgenerate,allowedthemtooffsetChinese
advantagesinmanpower;bothcivilizationspossessedshipbuildingandnavigational
skillsandtechnology.Motives:WidespreadsupportforexpansioninEuropeamong
merchants,nationalleaders;lookedforopportunitiesforexpansionofChristianity;
lookedforexpansionofplantationeconomy;Chineseexpansionwasprojectofone
emperor,notmerchantsorscholar-gentry;alreadypossessedsufficientmarkets;
wantedmoneyfordefense.
Diff:2
PageRef:605-608
Topic:MingChina:AGlobalMissionRefused
Skill:Conceptual

4) FollowingtheestablishmentoftheTokugawaShogunate,whatwasthenatureofJapanese
isolation?
Answer: FirststeptakenwaspersecutionofChristians,thenbanningofChristianityin1614;after
1616foreignmerchantslimitedtofewports;by1640s,onlyDutchandChinese
admittedatDeshima;in18thcenturyneo-Confucianphilosophyabandonedinfavorof
schoolofNationalLearningbasedonindigenousJapaneseculture;differedfrom
ChineseinmaintainingoversightofEuropeantechnologicaldevelopments.
Diff:2
PageRef:610-612
Topic:FendingOfftheWest:JapansReunificationandtheFirstChallenge
Skill:Conceptual

5) EvaluatetheimpactoftheEuropeanentryintotheAsianseatradingnetwork.
Answer: LimitedimpactofChristianmissionaries expelledfromJapan,controlledbyChinese;
onlytruemissionarysuccessonPhilippines;EuropeantradeinChinaandJapanlimited
tofewportcities;moredefiniteimpactonSpiceIslandsonperipheryofAsianempires ,
creationoftributekingdoms;establishmentofseaportsandfortresses.
Diff:2
PageRef:612-613
Topic:Conclusion:AsiaandtheFirstPhaseofEuropesGlobalExpansion
Skill:Conceptual

447

Chapter28 TheEmergenceofIndustrialSocietyinthe
West,1750-1914
Multiple-ChoiceQuestions
1) WhichofthefollowingstatementsconcerningthepoliticalphilosophyoftheEnlightenmentis
mostaccurate?
A) Enlightenmentphilosopheswerecreaturesofthemonarchswhoweretheirpatronsand
supportedthepowersofthekings.
B) AlthoughtheEnlightenmentphilosophersweregenerallyopposedtotheauthorityofthe
church,theyarguedthatonlymonarchycouldinsurestabilityforthemasses.
C) Enlightenmentthinkerschallengedregimesthatdidnotgrantfullreligiousfreedomor
thatinsistedonaristocraticprivilege.
D) EnlightenmentintellectualswerethefirmestsupportersofthechurchleftinEuropean
culture.
Answer: C
Diff:2
PageRef:624
Topic:ForcesofChange
Skill:Conceptual

2) Whichofthefollowingstatementsismostaccurate?
A) WesternEuropeexperiencedahugepopulationjumpafterabout1730.
B) WesternEuropespopulationwasdevastatedbyaseriesofwarsandepidemicsatthe
beginningofthe18thcentury.
C) PovertyandpoornutritionledtoastagnationoftheEuropeanpopulationuntil1840.
D) Themostsignificantfactorinthemovementofpopulationinthe18thcenturywasthe
increasedmortalityamongchildrenleadingtofewersurvivors.
Answer: A
Diff:2
PageRef:625
Topic:Introduction
Skill:Conceptual

3) Aboveall,populationpressureinthe18thcentury
A) ledtowholesalereplacementofofficeholdersamongtheelite.
B) drovemanypeopleintotheworking-classproletariat.
C) ledthebusinessclassestoreducethenumberofchildrenintheirfamilies.
D) causedthemiddleclassestoreducerisk-taking.
Answer: B
Diff:2
PageRef:625
Topic:ForcesofChange
Skill:Conceptual

448

4) Proto-industrializationrefersto
A) thestrictlyagriculturaleconomythatprecededtheIndustrialRevolution.
B) thedevelopmentoftheruralfactorysystem.
C) theemploymentoflaborerswhoworkedathomebutinacapitalistsystemdependenton
urbanmerchants.
D) thedevelopmentofsystemsoftransportationandcommunicationnecessaryforfull
industrialization.
Answer: C
Diff:2
PageRef:625
Topic:ForcesofChange
Skill:Conceptual

5) Populationupheavalandthespreadofapropertylessclassworkingformoneyledtowhichof
thefollowingdevelopments?
A) Anincreaseintheauthorityofthemaleheadsofhouseholds
B) Ageneralacceptanceofauthority,whetherdomesticorpolitical
C) Adeclineinthepercentageofillegitimatebirths
D) Theadoptionofmoreurbanstylesofdress
Answer: D
Diff:2
PageRef:625
Topic:ForcesofChange
Skill:Conceptual

6) WhichofthefollowingwasNOTacauseoftheAmericanRevolution?
A) Britainsattemptstoimposenewtaxesandtradecontrolsafter1763
B) Adesiretooverthrowtheoldercolonialleadership
C) TheBritishinvasionoftheMississippiRiverValley
D) Restrictiononfreemovementintothefrontierareas
Answer: C
Diff:2
PageRef:625
Topic:ForcesofChange
Skill:Conceptual

449

7) InwhatyeardidtheAmericancoloniessetupanewconstitutionalstructurebasedon
Enlightenmentprinciples?
A) 1776
B) 1781
C) 1783
D) 1789
Answer: D
Diff:1
PageRef:625
Topic:ForcesofChange
Skill:Factual

8) WhichofthefollowingwasNOTacauseoftheFrenchRevolution?
A) EnlightenmentthinkersurgingtheneedtolimitthepowersoftheCatholicchurchand
thearistocracy
B) Thechurchseekinggreaterpowerovertheroyalgovernment
C) Themiddleclassdemandforgreaterpoliticalrepresentation
D) Thepeasantdesireforfreedomfrommanorialism
Answer: B
Diff:2
PageRef:626
Topic:ForcesofChange
Skill:Conceptual

9) InwhatyeardidtheFrenchRevolutionbegin?
A) 1789
B) 1791
C) 1795
D) 1798
Answer: A
Diff:1
PageRef:626
Topic:ForcesofChange
Skill:Factual

450

10) OnJuly14inthefirstyearoftheFrenchRevolution,thestormingofwhatpoliticalprison
providedarevolutionarysymbol?
A) Tuileries
B) PlacedesVosges
C) Montparnasse
D) Bastille
Answer: D
Diff:1
PageRef:626
Topic:ForcesofChange
Skill:Factual

11) Whichofthefollowingwasalastingreformpassedduringtheinitial,moderatephaseofthe
FrenchRevolution?
A) Universalmalesuffrage
B) TheintroductionofProtestantism
C) Peasantswerefreedfromalltracesofmanorialism
D) Universalmilitaryconscription
Answer: C
Diff:2
PageRef:626
Topic:ForcesofChange
Skill:Conceptual

12) TheradicalphaseoftheFrenchRevolutionledtoallofthefollowingEXCEPTthe
A) executionoftheking.
B) introductionofthemetricsystemofweightsandmeasures.
C) extensionoftherevolutiontowarfareintheLowCountries,Italy,andGermany.
D) full-scaleattackonprivateproperty.
Answer: D
Diff:2
PageRef:626-627
Topic:ForcesofChange
Skill:Conceptual

13) ThefinalphaseoftheFrenchRevolutionwasusheredinbythevictoryof
A) MaximilienRobespierre.
B) NapoleonBonaparte.
C) AugusteComte.
D) PartieGirondin.
Answer: B
Diff:1
PageRef:627
Topic:ForcesofChange
Skill:Factual

451

14) InwhatyeardidaEuropeanalliancefinallycrushtheFrenchEmpire?
A) 1809
B) 1811
C) 1815
D) 1822
Answer: C
Diff:2
PageRef:628
Topic:ForcesofChange
Skill:Factual

15) WhichofthefollowingstatementsconcerningtheimpactoftheFrenchRevolutionontherest
ofEuropeisNOTaccurate?
A) TheFrenchRevolutionspreadkeyrevolutionarylegislationthroughoutmuchofwestern
Europe.
B) TherevolutionencouragedpopularnationalismoutsideofFrance.
C) TheFrenchRevolutionanditssubsequentempirecreatedageneralconsensusafterthe
defeatofFranceforamoreliberalEurope.
D) Theideaofequalityunderthelawandattacksonprivilegewhetheraristocratic,guild,
orecclesiasticalspreadthroughoutEurope.
Answer: C
Diff:2
PageRef:628
Topic:ForcesofChange
Skill:Conceptual

16) WhichofthefollowingstatementsconcerningthepeaceconferenceatViennafollowingthe
fallofFranceismostaccurate?
A) ThealliesintendedtopunishFranceseverelyinordertomakecertainthatnofurther
revolutionwaspossible.
B) TerritorialadjustmentsreachedatViennakeptEuropefairlystableforalmosthalfa
century.
C) Successfulrestorationofconservativemonarchiesandpromotionofinternalpeacewas
achievedforacentury.
D) Polandemergedasoneofthewinnersintheterritorialrealignmentsthatfollowedthe
wars.
Answer: B
Diff:2
PageRef:629
Topic:ForcesofChange
Skill:Conceptual

452

17) WhichpoliticalgrouplistedbelowwouldespousethefollowingstatementThepoliticalgoals
ofgreatestsignificancearetheestablishmentofconstitutionalruleandtheextensionofthe
parliamentaryfranchisetopropertiedmenofthemiddleclass?
A) Radicals
B) Socialists
C) Liberals
D) Conservatives
Answer: C
Diff:2
PageRef:629
Topic:ForcesofChange
Skill:Conceptual

18) Whichpoliticalgrouplistedbelowwouldespousethefollowingstatement?Aslongas
propertyiscontrolledbyprivateindividuals,inequalitywillexist.Itistheroleofthestateto
managepropertyforthebenefitofallcitizens?
A) Radicals
B) Socialists
C) Liberals
D) Conservatives
Answer: B
Diff:2
PageRef:629
Topic:ForcesofChange
Skill:Conceptual

19) In1820,Greecefoughtforitsindependencefrom
A) Austria-Hungary.
B) Poland.
C) Italy.
D) theOttomanEmpire.
Answer: D
Diff:1
PageRef:629
Topic:ForcesofChange
Skill:Factual

453

20) Whichofthefollowingnationsbecameindependentforthefirsttimeasaresultofa
revolutionin1830?
A) Germany
B) France
C) Italy
D) Belgium
Answer: D
Diff:2
PageRef:629
Topic:ForcesofChange
Skill:Factual

21) WhatpieceoflegislationgavethevotetomanymembersofthemiddleclassinGreatBritain?
A) TheReformBillof1832
B) TheGreatCharterof1848
C) TheFranchiseActof1811
D) TheCornLaws
Answer: A
Diff:2
PageRef:629
Topic:ForcesofChange
Skill:Factual

22) Afterindustrializationademographictransitionoccurredtoanewsystemthatpromoted
stablepopulationlevelsthrough
A) government-sponsoredfamilyplanning.
B) legalsanctionsagainstillegitimatebirths.
C) lowbirthanddeathrates
D) increasedmortalityduetoindustrialaccidentsandenvironmentalpollution.
Answer: C
Diff:2
PageRef:631
Topic:TheConsolidationoftheIndustrialOrder
Skill:Conceptual

454

23) By1900whatproportionoftheWesternpopulationenjoyedconditionsabovethesubsistence
level?
A) Onefifth
B) Onethird
C) Onehalf
D) Twothirds
Answer: D
Diff:2
PageRef:631
Topic:TheConsolidationoftheIndustrialOrder
Skill:Factual

24) LouisPasteurwasresponsibleforthe
A) developmentofthetelegraph.
B) discoveryofgerms.
C) creationofthefirstFrenchdepartmentstore.
D) inventionofthebirthcontrolpill.
Answer: B
Diff:1
PageRef:633
Topic:TheConsolidationoftheIndustrialOrder
Skill:Factual

25) AsindustrializationspreadpeasantsinEuropeimprovedtheirconditionsbyallofthe
followingmeansEXCEPT
A) overthrowingcapitalism.
B) organizingcooperatives.
C) specializinginnewcashcrops
D) seekingeducationandnewtechnicalskills.
Answer: A
Diff:2
PageRef:633
Topic:TheConsolidationoftheIndustrialOrder
Skill:Conceptual

455

26) Whichofthefollowingstatementsconcerningconservativepoliticalstrategiesafter1850is
mostaccurate?
A) Withthedestructionoftheliberalmovement,conservativesfeltfreetorestorethe
aristocraticbiasofabsolutemonarchy.
B) Conservativesbegantoallymorecloselywithsocialistsinordertooffsetthepolitical
advantagesgainedbyliberalsduringtherevolutionsof1848.
C) ConservativesceasedtoplayanyroleinthepoliticalframeworkofEuropeafter1850.
D) Conservativesstrovetodevelopanewpoliticalconsensuswithliberalsthatwouldgrant
theappearanceofconstitutionalreformbutretainaristocraticprivilege.
Answer: D
Diff:2
PageRef:634
Topic:TheConsolidationoftheIndustrialOrder
Skill:Conceptual

27) WhatBritishconservativeprimeministerwasresponsibleforextendingthevoteto
working-classmalesin1867?
A) ViscountCastlereagh
B) GeorgeCanning
C) BenjaminDisraeli
D) WilliamE.Gladstone
Answer: C
Diff:2
PageRef:634
Topic:TheConsolidationoftheIndustrialOrder
Skill:Factual

28) WhowasresponsibleformanagingtheunificationofItaly?
A) BenitoMussolini
B) CountCamillodiCavour
C) GeorgioGaspari
D) GiovanniVillani
Answer: B
Diff:1
PageRef:634
Topic:TheConsolidationoftheIndustrialOrder
Skill:Factual

456

29) WhatGermanconservativewasresponsiblefortheunificationofGermanyin1871?
A) OttovonBismarck
B) TheElectorofHanover
C) KingJosephII
D) KlemensvonMetternich
Answer: A
Diff:1
PageRef:634
Topic:TheConsolidationoftheIndustrialOrder
Skill:Factual

30) WhichofthefollowingdidNOTrepresentanexpansionofgovernmentfunctionsintheWest
after1870?
A) Civilserviceexaminations
B) Extensionofregulatoryapparatus
C) Theestablishmentofmandatorynationalunionsforlaborers
D) Widerwelfaremeasures
Answer: C
Diff:2
PageRef:635
Topic:TheConsolidationoftheIndustrialOrder
Skill:Conceptual

31) WhichofthefollowingstatementscouldNOTbeattributedtothepoliticalphilosophyofKarl
Marx?
A) Earliersocialisttheoriesbasedonutopianschemesweresillyandunrealistic.
B) Intheaftermathofthevictoryoftheproletariat,thestatewouldemergepermanentlyas
apowerfuldictatorship.
C) Historywasshapedbytheavailablemeansofproductionandwhocontrolledthem.
D) Revolutionoftheproletariatagainstthebourgeoisiewasinevitable.
Answer: B
Diff:2
PageRef:636
Topic:TheConsolidationoftheIndustrialOrder
Skill:Conceptual

457

32) Inwhatnationdidsocialismproducethestrongestpoliticalparty?
A) Britain
B) TheUnitedStates
C) France
D) Germany
Answer: D
Diff:2
PageRef:636
Topic:TheConsolidationoftheIndustrialOrder
Skill:Factual

33) Socialistmovementsthatproposedthepossibilityofgradualandpeacefulchangeratherthan
revolutionwerecalled
A) Bolsheviks.
B) Revisionist.
C) Falangists.
D) Boulangists.
Answer: B
Diff:2
PageRef:636
Topic:TheConsolidationoftheIndustrialOrder
Skill:Factual

34) Whichofthefollowingstatementsmostaccuratelydescribestherelationshipbetweenscience
andtheartsinthelater19thcentury?
A) Scienceandartcontinuedtofollowthelinesofclassicalandrationaltraditionalism.
B) Scienceandartofthe19thcenturywerefreedfromthetraditionsofclassicalrationalism
andembarkedonaradicalshiftthatfavoredtheemotional.
C) SciencecontinuedtheWesterntrendoftraditionalrationalism,butartadoptedthemore
emotionalandimpressionistictheoriesofRomanticism.
D) Therewerefewscientificadvancesaftertheearlystagesofindustrializationandlittleif
anyinnovationinthefieldofart.
Answer: C
Diff:2
PageRef:638
Topic:CulturalTransformations
Skill:Conceptual

458

35) Whoproposedevolutionin1859asthebasisofbiologicaldevelopment?
A) CharlesDarwin
B) KarlLeBlanc
C) LouisPasteur
D) NilsBormann
Answer: A
Diff:1
PageRef:638
Topic:CulturalTransformations
Skill:Factual

36) AmericanexceptionalismsuggeststhattheUnitedStates
A) developedonitsowntermswithonlyincidentalcontactwithEurope.
B) dependedexclusivelyforitsculturaldevelopmentonEurope.
C) hadnoculturalrelationshipofanykindwithEurope.
D) achieveditsculturalfoundationsfromNativeAmericans.
Answer: A
Diff:2
PageRef:642-643
Topic:WesternSettlerSocieties
Skill:Conceptual

37) Westernexpansioninthe19thcenturymeantanextensionofWesternsocietyitself,andnot
primarilyanewculturalinterchange.WhichofthefollowingregionswasNOTanareaof
Westernexpansion?
A) Canada
B) Malaya
C) NewZealand
D) Australia
Answer: B
Diff:1
PageRef:641-642
Topic:WesternSettlerSocieties
Skill:Factual

38) BritainsAustraliancoloniesoriginatedin1788as
A) miningexpeditions.
B) plantationandranchingsettlements.
C) penalsettlements.
D) fortifiedtradingports.
Answer: C
Diff:1
PageRef:642
Topic:WesternSettlerSocieties
Skill:Factual

459

39) WhichofthefollowingcountrieswasNOTamemberoftheTripleEntente?
A) Russia
B) France
C) Britain
D) TheUnitedStates
Answer: D
Diff:2
PageRef:644
Topic:DiplomaticTensionsandWorldWarI
Skill:Factual

40) Balkannationalisminitiallyexacerbateddeterioratingrelationshipsbetweenwhattwopowers
withinterestsintheregion?
A) BritainandFrance
B) GermanyandItaly
C) RussiaandAustria-Hungary
D) FranceandRussia
Answer: C
Diff:2
PageRef:644
Topic:DiplomaticTensionsandWorldWarI
Skill:Factual

EssayQuestions
1) WhatwerethepermanentreformsoftheAmericanandFrenchrevolutions?
Answer: Bothcasesrelatedtotriumphofliberalconstitutionalism,Enlightenmentpolitical
philosophy;creationofparliamentaryinstitutions,assaultsonaristocraticand
ecclesiasticalprivilege;broadervotingrightsformiddleclasses;abolitionofserfdomin
France;establishmentofequalityunderthelaw.
Diff:2
PageRef:623-631
Topic:ForcesofChange
Skill:Conceptual

2) InwhatwaysdidtheNapoleonicperiod,despitethecreationofanauthoritarianregime,result
intheextensionofrevolutiontotherestofEurope?
Answer: SpreadconceptsofrevolutionoutsideboundariesofFrance, liberalconstitutionalism,
equalityunderthelaw,parliamentaryrestrictionsonabsolutism;destructionof
aristocraticprivilegecreateddemandsforgreaterpoliticalvoiceamongmiddleand
workingclasses;encouragedpopularnationalismonliberallines;ledtorevolutionsin
1820inGreeceandSpain,in1830inBelgium,France,andcentralEurope.
Diff:2
PageRef:627-629
Topic:ForcesofChange
Skill:Conceptual

460

3) After1850whatpoliticalcoalitionsledtosuccessfulnationalistmovements,particularlyin
centralEurope?
Answer: UnionofconservativessuchasCavourandBismarckwithliberalsandtoalesserextent
withsocialistsinordertopermitcentralization;inbothItalyandGermanyreforms
werepassed,universalmanhoodsuffrage,promotionofindustrialization,extensionof
welfare,education;interestsofliberalssubmergedinnationalpride,desireforunited
country.
Diff:2
PageRef:631-637
Topic:TheConsolidationoftheIndustrialOrder
Skill:Conceptual

4) HowdidtheemergenceofnewcentralEuropeannationsafter1870leadtoincreased
diplomatictensions?
Answer: Fewpartsofworldavailableforcolonialexpansion;latecomershadfewplacesavailable
tocolonize;createdtensionsandcompetitionamongnations;FranceandBritain
terrifiedbygrowingmilitary,naval,andeconomicpowerofGermany;Russiaand
FranceconcernedbydiplomaticisolationasGermanybegantocreatecentralEuropean
alliancesystem(TripleAllianceofGermany,Austria-Hungary,andItaly);ledto
creationofsecondalliancesystemamongoldernations(TripleEntente ofBritain,
France,andRussia);eachalliancesystemcontainedaweakernation Austria-Hungary
andRussia.
Diff:2
PageRef:644-646
Topic:DiplomaticTensionsandWorldWarI
Skill:Conceptual

461

Chapter29 IndustrializationandImperialism:The
MakingoftheEuropeanGlobalOrder
Multiple-ChoiceQuestions
1) WhodefeatedBritainatIsandhlwanainJanuary1879?
A) India
B) France
C) Germany
D) Zulus
Answer: D
Diff:2
PageRef:648
Topic:Introduction
Skill:Conceptual

2) Whichofthefollowingstatementsconcerningthemanagementofcolonialenterprisesbythe
DutchandBritishEastIndiacompaniesismostaccurate?
A) Thedirectorsofthecompanieswerelittleinterestedintheacquisitionofcolonial
territories.
B) Thedirectorsofthecompaniesmadepreciseplansfortheexpansionofcompany
administrativecontroloverthegovernmentsofindigenouspeoples.
C) Thecompaniesweregrantedmonopoliesbygovernmentswiththeclearexpectationthat
theywouldconquernewterritoriesfortheirrespectivenations.
D) ThecompaniesweremerefigureheadsfortheactiveanddirectinterventionofEuropean
nationsintheaffairsofAsianpeoples.
Answer: A
Diff:2
PageRef:651
Topic:TheShifttoLandEmpiresinAsia
Skill:Conceptual

3) Howwere18thcenturylandempiresinAsiaaccumulated?
A) Bydirectgovernmentintervention
B) BythepolicyofthedirectorsoftheDutchandBritishEastIndiacompanies
C) BytheinitiativeofoverseasagentsoftheDutchandBritishEastIndiacompaniesacting
intheabsenceofinstructionsfromthecompanydirectors
D) No18thcenturyterritorialacquisitionsweremade
Answer: C
Diff:2
PageRef:651
Topic:TheShifttoLandEmpiresinAsia
Skill:Conceptual

462

4) WhichofthefollowingwastheearliestexampleofaEuropeanempirebuiltasaresultof
independentinitiativeofcompanyagents?
A) Java
B) Siam
C) India
D) Libya
Answer: A
Diff:1
PageRef:652
Topic:TheShifttoLandEmpiresinAsia
Skill:Factual

5) WhichofthefollowingstatementsconcerningtheincursionoftheDutchEastIndiacompany
inJavaismostaccurate?
A) TheDutchwerecontentinthe1620stobethevassalsofthesultanofMataram.
B) TheDutchwonaseriesofmilitaryvictoriesinthe1620sthatestablishedtheirmilitary
dominanceinJava.
C) TheDutchreliedonshipsandmilitaryforcesfromHollandtoestablishtheirinitial
supremacyinJava.
D) TheDutchreplacedthelocalrulerswithacompanydirectoratetogovernthecolonyof
Javainthe1620s.
Answer: A
Diff:2
PageRef:652
Topic:TheShifttoLandEmpiresinAsia
Skill:Conceptual

6) WhattacticledtotheDutchcontroloftheentireislandofJava?
A) TheDutchwonaseriesofnavalbattlesoverforcesofthesultanofMataram.
B) TheDutchintroducedAfricanmercenariesintoJavatosecureamilitaryvictory.
C) TheDutch,usingmercenaryforcesrecruitedfromthepeopleofJava,intervenedin
successiondisputesinreturnforgrantsofland.
D) TheDutchusedtheprocessofconversionofmassesoftheJavanesepeopleaswellasthe
elitetogainapositionofsupremacyonJava.
Answer: C
Diff:2
PageRef:652
Topic:TheShifttoLandEmpiresinAsia
Skill:Conceptual

463

7) InwhatwaywastheBritishEastIndiaCompanysintrusionintoIndiasimilartotheDutch
entryintoJava?
A) TheconversionoftheIndianelitetoChristianity
B) Theuseofmercenariesrecruitedfromamongindigenouspeoples
C) TheBritishremovalofalllocalrulersinthe18thcentury
D) ThedirectinterventionoftheBritishgovernment
Answer: B
Diff:2
PageRef:653
Topic:TheShifttoLandEmpiresinAsia
Skill:Conceptual

8) InwhichofthefollowingwayswastheBritishexperienceinIndiadifferentfromthatofthe
DutchinJava?
A) ThelackofinvolvementoftheBritishEastIndiaCompany
B) TheabsenceofinterventioninlocalsquabblesandsuccessiondisputesinIndia
C) Thefailuretouseindigenouspeoplesinrecruitedarmies
D) TheexistenceofaglobalimperialrivalrywiththeFrench
Answer: D
Diff:2
PageRef:653
Topic:TheShifttoLandEmpiresinAsia
Skill:Conceptual

9) WhatwasthecriticalbattleinwhichtheBritishdefeatedacombinedforceofIndiansand
French?
A) Nawab
B) Siraj-ud-daula
C) Plassey
D) Calicut
Answer: C
Diff:1
PageRef:654
Topic:TheShifttoLandEmpiresinAsia
Skill:Factual

464

10) WhowastheBritishcommanderduringthemilitaryvictoriesovertheFrenchinIndiaduring
the18thcentury?
A) LordCornwallis
B) RobertClive
C) JohnChurchill,theDukeofMarlborough
D) RichardArkwright
Answer: B
Diff:1
PageRef:654
Topic:TheShifttoLandEmpiresinAsia
Skill:Factual

11) ThevictoryoftheforcesoftheBritishEastIndiaCompanyovertheFrenchgavethemdirect
controlof
A) Bengal.
B) Punjab.
C) theIndusRivervalley.
D) Mysore.
Answer: A
Diff:2
PageRef:654
Topic:TheShifttoLandEmpiresinAsia
Skill:Factual

12) TheterritoriescontrolledbytheBritishEastIndiaCompanyexpandedconcurrentlywiththe
collapseofthe
A) Safaviddominions.
B) MughalEmpire.
C) Mauryanterritories.
D) Raj.
Answer: B
Diff:2
PageRef:655
Topic:TheShifttoLandEmpiresinAsia
Skill:Factual

465

13) WhichofthefollowingcitieswasNOToneoftheadministrativecentersofthethree
presidenciesestablishedbytheBritishEastIndiaCompany?
A) Madras
B) Bombay
C) Calcutta
D) Delhi
Answer: D
Diff:2
PageRef:655
Topic:TheShifttoLandEmpiresinAsia
Skill:Factual

14) InwhatyeardidtheBritishwinthebattleofPlassey?
A) 1754
B) 1789
C) 1832
D) 1757
Answer: D
Diff:2
PageRef:654
Topic:TheShifttoLandEmpiresinAsia
Skill:Factual

15) WhichofthefollowingwasNOTahandicapfacedbytheIndianprincesindefendingtheir
kingdomsfromtheBritish?
A) ThelackofasenseofnationalidentityinIndia
B) ThewillingnessofIndianstoserveintheBritisharmies
C) ThecontinuedwarfareamongtheIndianprinces
D) ThesuccessoftheBritishinwinningmanyconvertstoChristianity
Answer: D
Diff:2
PageRef:655
Topic:TheShifttoLandEmpiresinAsia
Skill:Conceptual

466

16) AllofthefollowingwerereasonswhyIndiabecamethepivotofthegreatBritishempire
EXCEPT
A) thesizeoftheIndianlandarmy.
B) theutilityofIndianportsinmaintainingBritishseapower.
C) theresidenceofmorewhitesettlersthananyotherBritishcolony.
D) theexistenceofrawmaterialsusefultotheBritishindustries.
Answer: C
Diff:2
PageRef:655
Topic:TheShifttoLandEmpiresinAsia
Skill:Conceptual

17) WhichofthefollowingstatementsconcerningcolonialsocietyinIndiaandJavapriorto1850
ismostaccurate?
A) TheDutchandBritishwerecontenttoleavethesocialsystemsofJavaandIndiapretty
muchastheyfoundthem.
B) ThemassiveconversionoftheJavanesetoProtestantismcreatedasignificantchangein
socialmores,buttheBritishwereunabletocarryoutasimilarchangeinIndia.
C) ThearrivaloftheBritishandtheDutchcompletelydestroyedtheoriginalsocial
hierarchiesofIndiaandJava.
D) TheDutchandBritishincursionsresultedintheremovaloftheindigenousaristocracies
andthesubstitutionofdirectEuropeancontrolofthepeasants.
Answer: A
Diff:2
PageRef:655
Topic:TheShifttoLandEmpiresinAsia
Skill:Conceptual

18) WhichofthefollowingstatementsconcerningEuropeaninteractionwithindigenouspeoples
priorto1850ismostaccurate?
A) SocialtabooseffectivelypreventedanysocialinteractionbetweenEuropeansand
indigenouspeoples.
B) AsmostoftheEuropeansweremale,socialinteractionwaslimitedtosexualexploitation
offemalesinbrothelsorasslaves.
C) MixedmarriagesbetweenEuropeanmalesandindigenousfemalesbecamewidely
accepted,particularlyinJava.
D) BothEuropeanmalesandfemalesintermarriedwithindigenouspeoplesonacommon
basis.
Answer: C
Diff:2
PageRef:657
Topic:TheShifttoLandEmpiresinAsia
Skill:Conceptual

467

19) ThelaterrepresentativesoftheBritishEastIndiaCompanywhowentouttosecuresudden
wealth,oftenthroughcorruption,werecalled
A) nawabs.
B) nabobs.
C) nizards.
D) nygards.
Answer: B
Diff:2
PageRef:658
Topic:TheShifttoLandEmpiresinAsia
Skill:Factual

20) WhowasresponsibleforthesweepingreformsinIndiainthe1790s?
A) RobertClive
B) RichardArkwright
C) LordCornwallis
D) GeneralJohnBurgoyne
Answer: C
Diff:2
PageRef:658
Topic:TheShifttoLandEmpiresinAsia
Skill:Factual

21) WhatEnglishreligiousmovementwascriticaltothesocialreformmovementintheBritish
Empirebythebeginningofthe19thcentury?
A) Erastian
B) Calvinist
C) Evangelical
D) Separatist
Answer: C
Diff:1
PageRef:658
Topic:TheShifttoLandEmpiresinAsia
Skill:Factual

468

22) JeremyBenthamandJamesMillwereleadersofwhatphilosophicalmovementthatsupported
socialreforminthecolonies?
A) Sophism
B) ScientificPositivism
C) SocialDarwinism
D) Utilitarianism
Answer: D
Diff:2
PageRef:658
Topic:TheShifttoLandEmpiresinAsia
Skill:Factual

23) Whichofthefollowingcountriesgrantedcitizenshiptoeducatedinhabitantsoftheircolonies?
A) Britain
B) Germany
C) Holland
D) France
Answer: D
Diff:1
PageRef:656-657
Topic:TheShifttoLandEmpiresinAsia
Skill:Conceptual

24) DuringwhatdecadedidEuropeancountriesreducemuchofAfrica,Asia,andthePacificto
colonialpossessions?
A) 1840s
B) 1850s
C) 1860s
D) 1870s
Answer: D
Diff:2
PageRef:659
Topic:IndustrialRivalriesandthePartitionoftheWorld
Skill:Factual

469

25) Inthefirsthalfofthe19thcentury,whatEuropeannationdominatedoverseastradeand
empirebuilding?
A) Holland
B) Portugal
C) Britain
D) Germany
Answer: C
Diff:1
PageRef:659
Topic:IndustrialRivalriesandthePartitionoftheWorld
Skill:Factual

26) WhichofthefollowingnationsdidNOTenterthecompetitiveraceforcolonialempireand
industrialsupremacyafter1870?
A) Germany
B) Belgium
C) Spain
D) TheUnitedStates
Answer: C
Diff:2
PageRef:659
Topic:IndustrialRivalriesandthePartitionoftheWorld
Skill:Factual

27) Whichofthefollowingstatementsismostaccurate?
A) EuropeannationsweremilitarysuperiortoAfricannationsintheearly20thcentury.
B) EuropeannationscooperatedtodefeattheoutmannedarmiesofAfricannations.
C) Europeannationsrapidlycametoagreementsovertheterritorialdivisionofcolonial
holdings.
D) TheLeagueofNationssupervisedtheconstructionofEuropeancolonialempires.
Answer: A
Diff:2
PageRef:660
Topic:IndustrialRivalriesandthePartitionoftheWorld
Skill:Conceptual

470

28) WhichofthefollowingwasNOTamotiveforexpansioninthelate19thcentury?
A) Thepressureofpublicopinion
B) Theuseofcoloniesaspressurevalvestoreleasethepressuresofunemployedworkers
andsurplusgoods
C) TheabsenceofinfluenceofpoliticalleadersintheEuropeancountries
D) Theneedtoensureasupplyofrawmaterials
Answer: C
Diff:2
PageRef:659
Topic:IndustrialRivalriesandthePartitionoftheWorld
Skill:Conceptual

29) Thejingoisticpressandtheextensionofthevotetothelowermiddleandworkingclasses
A) madepublicopinionamajorfactorinforeignpolicy.
B) lefttheplanningofimperialexpansiontotheEuropeanaristocracy.
C) madeimperialexpansionimpossibletoachieve.
D) ledtodemandsformassiveprogramsofemigrationfromEuropeancountries.
Answer: A
Diff:2
PageRef:659
Topic:IndustrialRivalriesandthePartitionoftheWorld
Skill:Conceptual

30) Whichofthefollowingstatementsismostaccurate?
A) FacedwiththeadvancedmilitarytechnologyoftheEuropeans,indigenouspeople
ceasedresistingtheimperialadvance.
B) Despiteadvancesinmilitarytechnology,theEuropeansremainedunabletoovercome
theAsianadvantagesinpopulation.
C) AfricanandAsianpeoplesoftenfiercelyresistedcolonialrule,althoughwithoutrealistic
chancesofpermanentsuccess.
D) NoAfricanorAsianmilitaryforceswonsetpiecebattles.
Answer: C
Diff:2
PageRef:660
Topic:IndustrialRivalriesandthePartitionoftheWorld
Skill:Conceptual

471

31) Whichofthefollowingdescriptionsmostaccuratelydefinesthetermtropicaldependencies?
A) ImperialpossessionsinwhichthenumbersofEuropeansettlersandindigenouspeoples
wereapproximatelyequal
B) ColoniesinwhichsmallnumbersofEuropeansruledlargenumbersofnon -Western
peoples
C) Colonieswithsubstantialmajoritiesofwhite,Europeanimmigrants
D) ColoniesthatwerelargelyunpopulatedpriortothecomingoftheEuropeans
Answer: B
Diff:2
PageRef:662
Topic:PatternsofDominance:ContinuityandChange
Skill:Conceptual

32) WhichofthefollowingisanexampleofaWhiteDominion?
A) NewZealand
B) Hawaii
C) Australia
D) Kenya
Answer: C
Diff:2
PageRef:662
Topic:PatternsofDominance:ContinuityandChange
Skill:Factual

33) WhichofthefollowingstatementsmostaccuratelydescribestheEuropeanstrategywith
respecttotheircontroloftropicaldependencies?
A) TheEuropeansexploitedlongstandingethnicandculturaldivisionsbetweenindigenous
peoples.
B) EuropeancolonialismdependedonwholesaleconversionstoChristianity.
C) Inmanycases,theEuropeansutilizedtheirmilitarysuperioritytocarryoutgenocide
againstAfricanandAsianpeoples.
D) ColonialrulerstendedtofavorMuslimsattheexpenseofotherminoritiesinAfricanand
AsiancoloniesbecauseofthelevelofeducationofMuslimpeoples.
Answer: A
Diff:2
PageRef:662
Topic:PatternsofDominance:ContinuityandChange
Skill:Conceptual

472

34) HowdidtheestablishmentofeducationalsystemsinAfricadifferfromthoseofJavaand
India?
A) Theydependedmoreonstateassistancethanonreligiousmissionaries
B) Theydependedmoreonreligiousmissionariesthanonstatesupport
C) EducationalsystemsinAfricawererunalmostexclusivelybyindigenouspeoples
D) TherewasmorehighereducationpromotedinAfrica
Answer: B
Diff:2
PageRef:663-664
Topic:PatternsofDominance:ContinuityandChange
Skill:Conceptual

35) Whichofthefollowingincentivestogreaterproductionwascommonlyutilizedbycolonizers
inthelater19thcentury?
A) Higherwages
B) Betterlivingconditions
C) Theimpositionofheadorhuttaxespayableincommodities
D) TheconstructionofEuropean-styleworkerscommunities
Answer: C
Diff:2
PageRef:666
Topic:PatternsofDominance:ContinuityandChange
Skill:Conceptual

36) WhichofthefollowingeconomicsectorsdidNOTincreaseasaresultofEuropeaneconomic
exploitationoftheircolonies?
A) Mining
B) Cocoa
C) Palmoil
D) Foodcrops
Answer: D
Diff:2
PageRef:666
Topic:PatternsofDominance:ContinuityandChange
Skill:Factual

473

37) WhichofthefollowingstatementsconcerningtheinternaleconomiesoftheEuropeancolonies
ismostaccurate?
A) TheintroductionofEuropeantechnologysuchasrailwaysandtelegraphswereintended
toimprovetheinternaleconomiesofthecolonies.
B) Slowly,theindustrialsystemoftheWest,includingfactoriesandtheproductionof
manufacturedgoods,wasintroducedintoAfricaandAsia.
C) By1914,AsianandAfricancolonieshadwoneconomicindependencefromthe
Europeancolonizers.
D) ColonialeconomiesweresteadilyreducedtodependenceontheEuropean -dominated
globalmarket.
Answer: D
Diff:2
PageRef:666
Topic:PatternsofDominance:ContinuityandChange
Skill:Conceptual

38) TowhichareasdidmostofBritishforeigninvestmentflowby1913?
A) WhiteDominions
B) India
C) WestAfrica
D) SouthAfrica
Answer: A
Diff:2
PageRef:663
Topic:PatternsofDominance:ContinuityandChange
Skill:Conceptual

39) WhateventsettheBoercolonyinSouthAfricaonadifferentpaththantheWhiteDominions
ofCanadaandAustralia?
A) ThearrivaloftheBantuintothoseregionssettledbytheBoersinthe1790s
B) UprisingamongtheKhoikhoiin1802
C) TheannexationofthecolonybytheBritishin1815
D) TheGermaninvasionofsouthernAfricain1902
Answer: C
Diff:2
PageRef:667
Topic:PatternsofDominance:ContinuityandChange
Skill:Conceptual

474

40) WhichofthefollowingwasarepublicfoundedbytheBoersinthe1850s?
A) Oudenaarde
B) Ryswick
C) Transvaal
D) Senegal
Answer: C
Diff:1
PageRef:668
Topic:PatternsofDominance:ContinuityandChange
Skill:Factual

41) WhichofthefollowingwasNOTaresultofthefirstcontactbetweentheMaorisand
Europeansduringthe1790s?
A) Thespreadofalcoholismandprostitution
B) TheintroductionofEuropeanfirearmstoMaoriwarfare
C) TheintroductionofEuropeandiseasessuchassmallpox
D) PeacefulrelationsbetweentheMaorisandthewhitesettlers
Answer: D
Diff:1
PageRef:669
Topic:PatternsofDominance:ContinuityandChange
Skill:Conceptual

42) HawaiiwaseffectivelyopenedtotheWestthroughthevoyagesof
A) Magellan.
B) CaptainJamesCook.
C) HenryHudson.
D) AdmiralFarragut.
Answer: B
Diff:1
PageRef:669
Topic:PatternsofDominance:ContinuityandChange
Skill:Factual

43) WhatHawaiianprincecreatedaunitedkingdomin1810withtheaidofBritishweaponsand
advisors?
A) Makepani
B) Mahele
C) Haoalele
D) Kamehameha
Answer: D
Diff:1
PageRef:669
Topic:PatternsofDominance:ContinuityandChange
Skill:Factual

475

EssayQuestions
1) InwhatwayswastheBritishconquestofIndiasimilartotheDutchcolonizationofJava?In
whatwayswasitdifferent?
Answer: Similarities:ColonizationinitiallyinhandsofBritishEastIndiaCompany,notBritish
government;intervenedinwarsbetweenrivalprincestogainlandandauthorityas
MughalEmpiredisintegrated;recruitedarmyfromamongindigenouspeoples.
Differences:existenceofchallengefromFrench;warsagainstFrenchandIndianallies
ledtocontrolofBengal-Bihar.
Diff:2
PageRef:648-657
Topic:TheShifttoLandEmpiresinAsia
Skill:Conceptual

2) OneofthefirstelementsofEuropeanreformwithinthecolonieswastheintroductionof
educationalsystems.InwhatsensedidtheintroductionofWesterneducationplanttheseeds
ofdecolonization?
Answer: CreatedamiddleclassalienatedfrombothBritishandindigenouspopulation;received
lowerwagesthanEuropeans;servedasminoradministratorsbeneathEuropean
masters;lackedbothprestigeandauthority;moretechnologicallyorientedthan
indigenouspeasantry;morecomfortablewithindustrializedsociety;alienationledto
movementstorecastcoloniesinmoreacceptablemode;becameleadersofrevolutionary
movements.
Diff:2
PageRef:662-665
Topic:TheShifttoLandEmpiresinAsia
Skill:Conceptual

3) Inwhatwaysdidtheprocessofindustrializationandnationalcentralizationtypicalof19th
centuryEuropealtertheprocessofimperialism?
Answer: ApplicationofindustrializationtomilitarytechnologyallowedEuropeanstodominate
world;useoftelegraph,steamship,andrailwaysimprovedcommunications,permitted
statetoplaygreaterroleindeterminingpolicy;nationalcentralizationallowed
GermanytochallengeBritishsupremacy;alsoindustrializationofU.S.createdeconomic
challenge;ledtoarmsbuildups,alliances,competitionforempire.
Diff:2
PageRef:659-662
Topic:IndustrialRivalriesandthePartitionoftheWorld
Skill:Conceptual

476

4) Inwhatwaysdidthecolonizersofthelate19thcenturycontroltheindigenouspeoplesand
increaseeconomicexploitation?
Answer: Continuedtoexploitlongstandingethnicandculturaldivisions ,particularlyreligious;
playedanimistsorChristiansagainstMuslims;oftenutilizedminoritypeoplesto
administerterritoriesinreturnforpoliticalsupport;dividedpeopleintotribes;recruited
favoredgroupsforWesterneducation,rolesingovernment;createdeconomy
dependentonindustrializedWest;extractedrawmaterialsbyintroductionofincentives
orcoercion,taxes,productionquotas;increasedsectorsofeconomyusefultoWestern
industryatexpenseoflocaleconomy.
Diff:2
PageRef:655-666
Topic:PatternsofDominance:ContinuityandChange
Skill:Conceptual

5) Inwhatwaysdidthecolonialexperienceofcontestedsettlercoloniesdifferfromthatof
tropicaldependencies?
Answer: Intruecoloniessmallnumbersofwhitesgovernedlargepopulationsofindigenous
peoples;resultedinpermanentexploitationbyEuropeans;incontestedsettlercolonies,
strugglesbetweenwhitesettlersandindigenouspeoplesoftenresultedinbalance;
SouthAfricawastheearliestcontestedsettlercolony;strugglewithZulus,British
resolvedindecolonizationofBoers,supremacyoverSouthAfricanindigenouspeoples,
Bantus;NewZealandMaorissufferedfromentryofwhites,butlearneduseoflawsto
gainbalanceofpower,rightsoverlandandresources;similarresultsinHawaii.
Diff:2
PageRef:662-671
Topic:PatternsofDominance:ContinuityandChange
Skill:Conceptual

477

Chapter30 TheConsolidationofLatinAmerica,
1830-1920
Multiple-ChoiceQuestions
1) InwhatwaywasLatinAmericadifferentthantheotherregionsthatremainedoutsidethe
directcontrolofWesternimperialism?
A) LatinAmericahadnoeconomicrelationshipwiththeWest.
B) Duringthecenturyofimperialism,LatinAmericahadfoughtandwonindependence
fromtheSpanishandthePortuguese.
C) Unliketheothers,LatinAmericahadnopreviouspoliticalrelationshiptotheWest.
D) TheidealsoftheEnlightenmenthadlittleornoimpactonLatinAmerica.
Answer: B
Diff:2
PageRef:674
Topic:Introduction
Skill:Conceptual

2) InwhatcenturydidmostLatinAmericancountriesachievetheirindependence?
A) 17th
B) 18th
C) 19th
D) 20th
Answer: C
Diff:1
PageRef:674
Topic:Introduction
Skill:Factual

3) WhichofthefollowingwasNOToneofthebeliefsthatLatinAmericasharedwiththeWest
duringthe19thcentury?
A) Beliefinprogress
B) Beliefinreform
C) Beliefincolonialism
D) Beliefinprivateproperty
Answer: C
Diff:2
PageRef:674
Topic:Introduction
Skill:Conceptual

478

4) Whichofthefollowingeventswasrejectedbycreoleelitesasamodelofrevolutionbecauseof
excessiveradicalism?
A) TheAmericanRevolution
B) TheFrenchRevolution
C) TheindependencemovementinPortugal
D) TheNapoleonicWars
Answer: B
Diff:2
PageRef:675
Topic:FromColoniestoNations
Skill:Conceptual

5) TheleaderoftheslaveindependencemovementontheislandofSt.Dominguewas
A) BernardinoRivadavia.
B) FatherMigueldeHidalgo.
C) SimonBolivar.
D) ToussaintLOverture.
Answer: D
Diff:1
PageRef:675
Topic:FromColoniestoNations
Skill:Factual

6) ForwhomdidtheindependenceofHaitiserveasasymboloffreedomandhope?
A) Thecreoleelite
B) TheSpanish
C) Slaves
D) Mestizos
Answer: C
Diff:2
PageRef:675
Topic:FromColoniestoNations
Skill:Factual

7) WhatwastheimpactoftheHaitianindependencemovementonLatinAmerica?
A) ItservedasamodelfornationalindependencemovementsthroughoutSouthand
CentralAmerica
B) Itdemonstratedthepowerofcreoleelitesinleadingrevolutionarymovements
C) ItledimmediatelytoageneralabolitionofslaverythroughoutLatinAmerica
D) Creoleelitesvieweditwithhorrorasanexampleofgeneralsocialupheaval
Answer: D
Diff:2
PageRef:675
Topic:FromColoniestoNations
Skill:Conceptual

479

8) WhateventinEuropeprecipitatedthemovementsforindependenceinLatinAmerica?
A) TheforcedabdicationoftheroyalfamilyofSpainduringtheNapoleonicWars
B) TheconquestoftheMughalEmpirebythePortuguese
C) SpainslossofcolonialterritoriestotheBritishduringtheWarofJenkinsEar
D) TheSevenYearsWar
Answer: A
Diff:2
PageRef:675
Topic:FromColoniestoNations
Skill:Conceptual

9) WhoinvadedSpainin1808?
A) France
B) Britain
C) Holland
D) Denmark
Answer: A
Diff:2
PageRef:675
Topic:FromColoniestoNations
Skill:Factual

10) WhoinitiatedthemovementforindependenceinMexicobycallingonthemestizosand
Indianstorebelin1810?
A) ToussaintLOverture
B) FatherMigueldeHidalgo
C) SimonBolivar
D) ManueldeRosas
Answer: B
Diff:1
PageRef:675
Topic:FromColoniestoNations
Skill:Factual

11) WhowasproclaimedthefirstemperorofMexico?
A) ToussaintLOverture
B) BernardinoRivadavia
C) ManueldeRosas
D) AugustindeIturbide
Answer: D
Diff:1
PageRef:675
Topic:FromColoniestoNations
Skill:Factual

480

12) SimonBolivarwasresponsiblefortheindependencemovementin
A) Argentina.
B) Mexico.
C) northernSouthAmerica.
D) Guatemala.
Answer: C
Diff:1
PageRef:676
Topic:FromColoniestoNations
Skill:Factual

13) ThepersonresponsiblefortheindependencemovementsinArgentinaandChilewas
A) ManueldeRosas.
B) BernardinoRivadavia.
C) SimonBolivar.
D) JosedeSanMartin.
Answer: D
Diff:1
PageRef:676
Topic:FromColoniestoNations
Skill:Factual

14) BywhatdatehadallofSpanishSouthAmericagaineditsindependence?
A) 1808
B) 1814
C) 1816
D) 1825
Answer: D
Diff:2
PageRef:676
Topic:FromColoniestoNations
Skill:Factual

15) InwhatwaywastheexperienceoftheNapoleonicwarsdifferentforPortugalthanforSpain?
A) PortugalwasalliedwiththeFrenchemperor.
B) TheFrenchattemptedtoinvadePortugal,butfailed.
C) TheentireroyalfamilyfledfromtheFrenchtoBrazilandestablishedtheircapitalthere.
D) FollowingthedefeatofthePortuguese,theFrenchtookoverthecolonialadministration
ofBrazil.
Answer: C
Diff:2
PageRef:677
Topic:FromColoniestoNations
Skill:Conceptual

481

16) HowwasBrazilianindependenceachieved?
A) Througharebellionledbythecreoleelite
B) ThroughaslaverebelliononthemodelofHaiti
C) BecausetheFrenchfreedthecolonyunilaterally
D) Pedro,theprinceregentofBrazil,declaredindependence
Answer: D
Diff:2
PageRef:677
Topic:FromColoniestoNations
Skill:Conceptual

17) InwhatwaywasthegovernmentofindependentBrazildifferentfromthoseoftheother
newlycreatedLatinAmericannations?
A) Brazilwasamonarchyratherthanarepublic
B) Itwastheonlygovernmentthatimmediatelyabolishedslavery
C) Itwasruledbythepopularmajorityofformerslavesandpeopleofmixedrace
D) ItwastheonlyrepublicanformofgovernmentestablishedinLatinAmerica
Answer: A
Diff:2
PageRef:677
Topic:FromColoniestoNations
Skill:Conceptual

18) Whichofthefollowingstatementsismostaccurate?
A) WhilemostleaderssoughttomaintainCatholicismastheofficialreligionofthenew
states,somesoughttoendtheexclusionofotherfaiths.
B) RomanCatholicismhadbeentheonlystatereligionduringthecolonialperiod,andits
statusastheonlypermittedreligionremainedunchallengedafterindependence.
C) TheonsetofindependencewasaccompaniedbyageneraltrendawayfromCatholicism
towardProtestantreligions.
D) ThedefenseoftheRomanCatholicchurchbecamearallyingcryforLatinAmerican
liberals.
Answer: A
Diff:2
PageRef:677
Topic:NewPoliticalandEconomicRealities
Skill:Conceptual

482

19) InwhichofthefollowingareasdidslaveryNOTexistfollowing1854?
A) Cuba
B) PuertoRico
C) Mexico
D) Brazil
Answer: C
Diff:2
PageRef:677
Topic:NewPoliticalandEconomicRealities
Skill:Factual

20) WhichofthefollowingstateswasnotpartofGranColombia?
A) Paraguay
B) Ecuador
C) Venezuela
D) Colombia
Answer: A
Diff:2
PageRef:676
Topic:NewPoliticalandEconomicRealities
Skill:Factual

21) WhichofthefollowingstatementsconcerningthepoliticalorganizationofLatinAmericato
1850ismostaccurate?
A) MostofLatinAmericawasdividedupintoconsolidatedunitsthatmirroredthecolonial
viceroyalties.
B) Theexcellentcolonialroadsystemenabledthecreationoflargerstatesafter
independence.
C) PermanentconsolidationandunionwasmoretypicalofCentralAmericaandsouthern
SouthAmericathanelsewhere.
D) Mostattemptsatconsolidationandunionfailed.
Answer: D
Diff:2
PageRef:678
Topic:NewPoliticalandEconomicRealities
Skill:Conceptual

483

22) Independentleaderswhodominatedlocalareasbyforceindefianceofnationalpoliciesand
whosometimesseizedthenationalgovernmentwerecalled
A) Rurales.
B) caudillos.
C) guano.
D) fazendas.
Answer: B
Diff:1
PageRef:679
Topic:NewPoliticalandEconomicRealities
Skill:Factual

23) Whichofthefollowingcharacteristicsismostaccuratelyassociatedwiththecentralisttheory
ofgovernmentinLatinAmerica?
A) Fiscalandcommercialpoliciessetbyregionalgovernments
B) RecognitionofthepoliticalrightsofIndiansandmestizos
C) Democracy
D) Strongnationalgovernmentswithbroadpowers
Answer: D
Diff:1
PageRef:727
Topic:NewPoliticalandEconomicRealities
Skill:Conceptual

24) WhichofthefollowingpoliticalprincipleswouldNOTbeassociatedwithLatinAmerican
liberals?
A) Rightsofindividuals
B) SecularsocietymodeledontheUnitedStates
C) Theretentionofcorporategroups
D) Developmentofcommerce
Answer: C
Diff:2
PageRef:679
Topic:NewPoliticalandEconomicRealities
Skill:Conceptual

484

25) InwhatwaywerebothliberalandconservativepartiesinLatinAmericasimilar?
A) TheirdistrustforRomanCatholicism
B) Theiracceptanceoffederalistpoliticaltheory
C) Theirendorsementofcentralistpoliticaltheory
D) Thesocialoriginsoftheirleadership
Answer: D
Diff:2
PageRef:679
Topic:NewPoliticalandEconomicRealities
Skill:Conceptual

26) WhatEuropeannationwasthefirsttoofferrecognitionofandtradewiththenewly
independentnationsofLatinAmerica?
A) Germany
B) Britain
C) Spain
D) Portugal
Answer: B
Diff:2
PageRef:682
Topic:LatinAmericanEconomiesandWorldMarkets
Skill:Factual

27) TheUnitedStatesexpresseditsattitudetowardLatinAmericanindependencein1823withthe
issuanceofthe
A) ArticlesofConfederation.
B) SouthwestOrdinance.
C) CanningTreaty.
D) MonroeDoctrine.
Answer: D
Diff:1
PageRef:682
Topic:LatinAmericanEconomiesandWorldMarkets
Skill:Factual

28) WhatnationreplacedSpainpriorto1850asthedominanteconomicforceinLatinAmerica?
A) France
B) TheUnitedStates
C) Britain
D) Germany
Answer: C
Diff:2
PageRef:682
Topic:LatinAmericanEconomiesandWorldMarkets
Skill:Factual

485

29) WhichofthefollowingfactorsdidNOTaccountfortheeconomicstagnationinLatinAmerica
from1820to1850?
A) Theslowrecoveryoftheminingsectorafterwarsofindependence
B) Thelackofatransportationnetworkandportfacilities
C) Thelackofcapitalforinvestmentinindustry
D) Theabsenceofforeignmarketsforrawmaterials
Answer: D
Diff:2
PageRef:682-683
Topic:LatinAmericanEconomiesandWorldMarkets
Skill:Conceptual

30) PerusprimaryexporttoEuropebetween1850and1880wasguanoor
A) birddroppings.
B) mercury.
C) woolencloth.
D) turtleeggs.
Answer: A
Diff:1
PageRef:682
Topic:LatinAmericanEconomiesandWorldMarkets
Skill:Factual

31) ManyliberalpoliticiansinLatinAmericaadoptedAugusteComtesphilosophyof
A) utilitarianism.
B) existentialism.
C) positivism.
D) progressivism.
Answer: C
Diff:2
PageRef:683
Topic:LatinAmericanEconomiesandWorldMarkets
Skill:Factual

486

32) Whichofthefollowingdescriptionsofthepoliticianswholedliberalgovernmentsin
post-1860LatinAmericaisNOTaccurate?
A) Theyrepresentedanewgenerationofpoliticianswhohadmaturedafterindependence
B) TheyfavoredexpansionofthefranchisetoIndiansandmestizos whorepresentedthe
ancientaspectsofLatinAmericancivilization
C) TheirinspirationwasEngland,France,andtheUnitedStates
D) Theywerefirmbelieversinprogress,education,andfreecompetitionwithinasecular
society
Answer: B
Diff:2
PageRef:684
Topic:LatinAmericanEconomiesandWorldMarkets
Skill:Conceptual

33) Between1832and1854,whodominatedMexicanpolitics?
A) FatherMigueldeHidalgo
B) BenitoJua rez
C) AntonioLo pezdeSantaAnna
D) BernardinoRivadavia
Answer: C
Diff:1
PageRef:684
Topic:LatinAmericanEconomiesandWorldMarkets
Skill:Factual

34) WhichofthefollowingnationsdidNOTintervenemilitarilyinMexicanpoliticsbetween1829
and1865?
A) Spain
B) TheUnitedStates
C) France
D) Britain
Answer: D
Diff:2
PageRef:684-685
Topic:LatinAmericanEconomiesandWorldMarkets
Skill:Factual

487

35) ManifestDestinyreferredto
A) theinevitabilityofademocraticrepublicinMexico.
B) theeconomicdominanceofBritaininLatinAmerica.
C) theimpositionofFrenchgovernmentinMexicoduringthereignofNapoleonIII.
D) thebeliefthattheUnitedStateswasdestinedtoruletheNorthAmericancontinentfrom
coasttocoast.
Answer: D
Diff:1
PageRef:684
Topic:LatinAmericanEconomiesandWorldMarkets
Skill:Factual

36) TheagreementfollowingtheMexicanAmericanWarinwhichMexicocededaboutonehalfof
itsterritorytotheUnitedStateswastheTreatyof
A) SanAntonio.
B) Atchison-Jua rez.
C) Guadalupe-Hidalgo.
D) theRioGrande.
Answer: C
Diff:2
PageRef:684
Topic:LatinAmericanEconomiesandWorldMarkets
Skill:Factual

37) TheleaderoftheliberalrebellioninMexicocalledLaReforma was


A) BenitoJua rez.
B) BernardinoRivadavia.
C) SimonBolivar.
D) GeneralAntonioLo pezdeSantaAnna.
Answer: A
Diff:1
PageRef:684
Topic:LatinAmericanEconomiesandWorldMarkets
Skill:Factual

38) UnderJuanManueldeRosas,theUnitedProvincesoftheRiodelaPlata
A) adoptedthefederalistprogramofaweakcentralgovernmentandlocalautonomy.
B) overthrewthedominanceofBuenosAiresprovince.
C) undertookaprogramofeducationandeconomictrainingamongtheIndians.
D) introducedademocraticregimethatrecognizedpoliticaldiversitywithoutviolence.
Answer: A
Diff:2
PageRef:686
Topic:LatinAmericanEconomiesandWorldMarkets
Skill:Conceptual

488

39) WhatwasthenewexportcropinBrazilthatmadeupover40percentofBrazilsexportsby
1840?
A) Tea
B) Sugar
C) Cotton
D) Coffee
Answer: D
Diff:2
PageRef:688
Topic:LatinAmericanEconomiesandWorldMarkets
Skill:Factual

40) Followingthe1830s,theliteraryandartisticstylepreferredinLatinAmericawas
A) neo-classicism.
B) Romanticism.
C) existentialism.
D) Dadaism.
Answer: B
Diff:2
PageRef:689
Topic:SocietiesinSearchofThemselves
Skill:Factual

41) Whichofthefollowingstatementsconcerningthestatusofwomeninpost -independence


LatinAmericaismostaccurate?
A) Followingindependence,womengainedlittlegroundandtherewasvirtuallynochange
intheattitudestowardwomensproperroleinsociety.
B) Becauseoftheirparticipationintheindependencemovement,womenwererewardedin
post-revolutionaryLatinAmericawithvotingrightsandaccesstopoliticaloffice.
C) Becausewomeningeneralfailedtosupporttherevolutionarymovements,theywerenot
includedinthemassivereformsthatfollowedindependence.
D) Thestatusofwomenactuallydeclinedafterindependence,astheywerecutofffrom
areassuchaspubliceducationthathadbeenavailabletothemincolonialLatinAmerica.
Answer: A
Diff:2
PageRef:690
Topic:SocietiesinSearchofThemselves
Skill:Conceptual

489

42) By1888thestigmaofskincolorandformerslavestatus
A) wasnolongerafactorinLatinAmericansocialhierarchy.
B) wasnotafactorexceptintheformerplantationcoloniesoftheCaribbean.
C) wasnotafactorforIndians,butcontinuedtoplaguedescendantsofAfricanslaves.
D) continuedtocreatebarrierstosocialadvancementthroughoutLatinAmerica.
Answer: D
Diff:2
PageRef:691
Topic:SocietiesinSearchofThemselves
Skill:Conceptual

43) WhichofthefollowingbeliefsisNOTassociatedwiththeconceptofmodernizationor
Westernization?
A) Developmentwasamatterofincreasingpercapitaproductioninanysociety.
B) Themoreindustrializedandurbanizedanysocietybecame,themoresocialchangeand
improvementwerepossibleastraditionalpatternsandattitudeswereabandoned.
C) Changewouldtakeplacethroughradicalorrevolutionarytransitionsratherthan
gradually.
D) Astheprocessoccurred,therewouldbeanaturalmovementtowardmoredemocratic
formsofgovernmentandpopularparticipation.
Answer: C
Diff:2
PageRef:692
Topic:SocietiesinSearchofThemselves
Skill:Conceptual

44) Thedependencytheoryofeconomicdevelopmentisassociatedwithwhatpoliticalgroup?
A) Liberals
B) Marxists
C) Conservatives
D) Monarchists
Answer: B
Diff:2
PageRef:693
Topic:SocietiesinSearchofThemselves
Skill:Conceptual

490

45) WhichofthefollowingstatementsmostaccuratelydescribestheeconomyofLatinAmericain
theperiodfrom1880to1920?
A) Thecollapseofworldwidedemandforrawmaterialsledtoastagnationandthena
contractionofeconomiesthroughoutLatinAmerica.
B) LatinAmericaexperiencedaspurtofeconomicgrowthfueledbytheincreasingdemand
forrawmaterials,foodstuffs,andtropicalcrops.
C) Althoughthedemandinindustrializedcountriesforrawmaterialsfelloff,theslumpin
exportswasmorethanovercomebyadramaticsurgeinindustrialproductionand
export.
D) Theperiodwasmarkedbyincreasinggovernmentcontrolofallfacetsofproductionand
increasingtariffsplacedoncommercialexports.
Answer: B
Diff:2
PageRef:691
Topic:TheGreatBoom,1880-1920
Skill:Conceptual

46) FollowingJua rezsdeathin1876,whosucceededaspresidentandpoliticalleaderofMexico?


A) AntonioLo pezdeSantaAnna
B) PorfirioDiaz
C) BernardinoRivadavia
D) MaximilianvonHapsburg
Answer: B
Diff:1
PageRef:694
Topic:TheGreatBoom,1880-1920
Skill:Factual

47) WhichofthefollowingstatementsconcerningArgentinabetween1880and1920ismost
accurate?
A) Thecentralistgovernmentbecameincreasinglyrepressiveandactuallyreducedthe
numberofeligiblevoters.
B) TheimmigrationofEuropeanlaborersledtoanincreasinglyradicalworkforceandthe
developmentofaSocialistPartybythe1890s.
C) UnlikeotherregionsofLatinAmerica,Argentinaexperiencedaneconomicdepression
thatledtorapidturnoverwithinthegovernmentandpoliticalinstability.
D) RiodeJaneiroheldovertwomillioninhabitants,oraboutaquarterofArgentinastotal
population.
Answer: B
Diff:2
PageRef:694-696
Topic:TheGreatBoom,1880-1920
Skill:Conceptual

491

48) TheSpanishAmericanWarthatbrokeoutin1898centeredon
A) Mexico.
B) Brazil.
C) Cuba.
D) Haiti.
Answer: C
Diff:1
PageRef:697
Topic:TheGreatBoom,1880-1920
Skill:Factual

49) In1902whichofthefollowingUnitedStatespossessionswasgrantedindependence?
A) Philippines
B) PuertoRico
C) Hawaii
D) Cuba
Answer: D
Diff:2
PageRef:697
Topic:TheGreatBoom,1880-1920
Skill:Factual

50) WhichofthefollowingstatementsconcerningtheDiazgovernmentinMexicoismost
accurate?
A) UnderDiaz,reformswereundertakenthatfinallybegantoresolvetheinequityofland
distributioninMexico.
B) Laborunrestandpoliticalinstabilitydecreasedsignificantlybythebeginningofthe20th
century.
C) Diazsstronglycentralizedgovernmentactivelydiscouragedforeigninvestmentin
Mexicanminingandtransportation.
D) Undertheguiseofmodernization,theformsofliberalgovernmentweremaintainedbut
weresubvertedinordertokeepDiazinpower.
Answer: D
Diff:2
PageRef:694
Topic:TheGreatBoom,1880-1920
Skill:Conceptual

492

EssayQuestions
1) IntermsofmodelsofrevolutionavailabletoLatinAmericansintheearly19thcentury,which
exampleswereconsideredacceptable,whichunacceptable,andwhy?
Answer: AmericanRevolution:consideredacceptable,moderaterevolutionbycreoleelite
utilizingEnlightenmentpoliticalprinciples;lackofradicalrevisionofsocialhierarchy.
FrenchRevolution:consideredunacceptable;hadcorrectpoliticalphilosophy,but
includedregicide,removalofaristocraticprivilegeoverpeasantry,andassaulton
Catholicchurch.Haitianrevolution:consideredtotallyunacceptable;carriedoutby
slavesovercolonialmasters;incorporatedradicalrevisionofsocialhierarchy
considereddangerouselsewhereinLatinAmerica.
Diff:2
PageRef:674-677
Topic:FromColoniestoNations
Skill:Conceptual

2) InwhatwaysweretherevolutionarymovementsinMexico,northernSouthAmerica,and
southernSouthAmericasimilar?
Answer: Ledbycreoleelitesrepresentedbymilitaryleaders;exceptionwasearlyrevolutionin
MexicounderFatherMigueldeHidalgo;generallyexcludedparticipationbyIndians
andmestizos;resultedincreationofconservativerepublicsincorporatingmuchof
colonialsocialhierarchy;allrevolutionsaccomplishedbetween1810and1825.
Diff:2
PageRef:672-677
Topic:FromColoniestoNations
Skill:Conceptual

3) InwhatwaysweretheLatinAmericaneconomiesofthe19thcenturydependentonthe
industrializednationsofEuropeandNorthAmerica?
Answer: Dependentonindustrializedmarkets;surgeinLatinAmericaneconomiesinlater19th
centurycreatedbydemandforrawmaterials,agriculturalproductsinindustrialized
world;importationofmanufacturedgoodsfromindustrializedworldcausedslow
developmentofindigenousindustries;dependentoncapitalfromindustrializedworld
forinvestment;meantnewtransportationandcommunicationsystems,miningsectors
oftencontrolledbyforeigncorporations.
Diff:2
PageRef:682-688
Topic:LatinAmericanEconomiesandWorldMarkets
Skill:Conceptual

493

4) WhataretheexplanationsofferedfortherelativeunderdevelopmentofLatinAmerica?
Whichexplanationseemstobethemostsufficientexplanation?
Answer: Modernizationtheory:developmentbasedonabilitytofollowpatternofWestern
industrialization;socialchangeandimprovementnaturaloccurrencesthataccompany
processofindustrialization;LatinAmericafailedtoindustrialize,thusfailedtoemulate
developmentofWestsuccessfully.Dependencytheory:developmentofindustrialized
nationsaccomplishedatcostofLatinAmericanunderdevelopment;industrialnations
drewrawmaterials,labor,profitsfromdependenteconomies;industrialnations
accumulatedcapital;economyofdependentnationsdeterminedbyexternalforces.
Diff:2
PageRef:692-693
Topic:SocietiesinSearchofThemselves
Skill:Conceptual

5) DiscusstherelationshipoftheUnitedStateswithLatinAmericauntil1910.
Answer: FirstinterventioninMexicanAmericanWar(1846-1848);militarydefeatofMexicoled
toTreatyofGuadalupe-Hidalgo;Mexicolostnearlyhalfofitsterritorialpossessionsto
U.S.;following1870,U.S.begantocompetewithBritainforcapitalinvestmentinLatin
America,particularlyMexico;underDiaz,U.S.investmentwasactivelysought;second
militaryinterventioninSpanishAmericanWar(1898 -1902)allowedU.S.toannex
PuertoRico,PhilippinesfromSpain,gaindiplomaticcontroloverCuba;ledtodirect
U.S.interventioninCaribbean;PanamaCanalallowedentryintoCentralAmerica.
Diff:2
PageRef:696-698
Topic:TheGreatBoom,1880-1920
Skill:Conceptual

494

Chapter31 CivilizationsinCrisis:TheOttomanEmpire,
TheIslamicHeartlands,andQingChina
Multiple-ChoiceQuestions
1) WhichofthefollowingwasNOTaweaknessassociatedwiththeOttomanEmpireinthe18th
century?
A) Weakrulers
B) Competitionwithinfactionsoftheelite
C) ConversionofmuchofthepopulationtoChristianity
D) DeterioratingconditionsforartisansasaresultofcompetitionwiththeWest
Answer: C
Diff:2
PageRef:703
Topic:FromEmpiretoNation
Skill:Conceptual

2) WhichofthefollowingstatementsconcerningtheMuslimeconomyatthebeginningofthe
18thcenturyismostaccurate?
A) TheMuslimeconomyremaineddependentonArabmerchantswhotraded
predominantlywithAfrica.
B) ByholdingtheEuropeansatbay,theMuslimswereabletocapitalizeanindigenous
industrybasedontheproductionofcottontextiles.
C) TheprosperityoftheindustrialbaseoftheMuslimempiresledtoaclosealliance
betweentheartisansandthegovernment.
D) Merchantswithintheempire,especiallythosewhowereJewsorChristians,grewmore
dependentoncommercialdealingswithEuropeancounterparts,acceleratingtheinflux
ofWesterngoods.
Answer: D
Diff:2
PageRef:703
Topic:FromEmpiretoNation
Skill:Conceptual

3) WhichofthefollowingEuropeanpowersseizedterritoriesoftheOttomanEmpireintheearly
decadesofthe18thcentury?
A) Austria-Hungary
B) Britain
C) France
D) Italy
Answer: A
Diff:2
PageRef:703
Topic:FromEmpiretoNation
Skill:Factual

495

4) Inthelater1700swhatpowerbecamethemainthreattotheOttomanssurvival?
A) Britain
B) Austria-Hungary
C) France
D) Russia
Answer: D
Diff:2
PageRef:703
Topic:FromEmpiretoNation
Skill:Factual

5) ThefirstregiontosuccessfullyrebelandachieveindependencefromtheOttomanEmpirewas
A) theCrimea.
B) Greece.
C) Serbia.
D) Palestine.
Answer: B
Diff:2
PageRef:703
Topic:FromEmpiretoNation
Skill:Factual

6) Bythe1870s,theOttomanEmpire
A) hadrecoveredmostoftheirterritoriallossestoEuropeanpowers.
B) hadceasedtoruleanyportionofAsiaMinor.
C) hadbeendrivenfromvirtuallyalloftheBalkans.
D) haddriventheRussianarmiesbacktothesteppes.
Answer: C
Diff:2
PageRef:703
Topic:FromEmpiretoNation
Skill:Conceptual

7) WhatEuropeannationsupportedtheOttomanEmpireinordertopreventotherEuropean
powersfromgainingaccesstotheMediterranean?
A) Britain
B) France
C) Russia
D) Austria-Hungary
Answer: A
Diff:2
PageRef:703
Topic:FromEmpiretoNation
Skill:Factual

496

8) WhatwastheresultofthereformsofSultanSelimIII(1789-1807)?
A) Western-styleeducationwasintroducedthroughouttheempire
B) TheJanissarycorpswaseliminatedasapoliticalandmilitaryforce
C) ThesultanwastoppledfromthethronebyaJanissaryrevolt
D) Railwayswereconstructed,connectingtheempirewithEurope
Answer: C
Diff:2
PageRef:704
Topic:FromEmpiretoNation
Skill:Conceptual

9) WhatOttomansultansuccessfullyeliminatedtheJanissarycorpsasamilitaryandpolitical
influence?
A) SelimIII
B) MahmudII
C) AbdulHamid
D) SelimII
Answer: B
Diff:2
PageRef:704
Topic:FromEmpiretoNation
Skill:Factual

10) WhichofthefollowingstatementsconcerningthereformsofMahmudIIismostaccurate?
A) MahmudpatternedhisreformprogramonWesternprecedents,includingthecreationof
adiplomaticcorps.
B) Despitesubtlemilitaryandadministrativereforms,Mahmudwasunabletoshakeoffthe
influenceoftheJanissaries.
C) Mahmud,withtheconsentoftheulama andtheayan returnedtoatraditionalIslamic
formofgovernment.
D) Mahmudsprogramofreformwasactuallylessambitiousthanthatofhispredecessor,
SelimIII.
Answer: A
Diff:2
PageRef:704
Topic:FromEmpiretoNation
Skill:Conceptual

497

11) WhichofthefollowingwasNOTpartoftheTanzimatreformsintheOttomanEmpire?
A) TheintroductionofWestern-styleeducationintheuniversities
B) Theadditionofstate-runpostalandtelegraphsystems
C) ThecreationofaconstitutionbasedonEuropeanprototypes
D) Theeliminationofreligiousprotectionforminorityreligiousgroups
Answer: D
Diff:2
PageRef:704
Topic:FromEmpiretoNation
Skill:Conceptual

12) InwhatyearwastherevisedconstitutionintroducedaspartoftheTanzimatreforms?
A) 1839
B) 1848
C) 1876
D) 1898
Answer: C
Diff:2
PageRef:704
Topic:FromEmpiretoNation
Skill:Factual

13) WhatgroupwithintheOttomanEmpireactuallysufferedasaresultoftheTanzimatreforms?
A) Ayan
B) Ulama
C) Artisans
D) Merchants
Answer: C
Diff:2
PageRef:704
Topic:FromEmpiretoNation
Skill:Factual

498

14) Whichofthefollowingstatementsconcerningtheimprovementofwomensstatusasaresult
oftheTanzimatreformsismostaccurate?
A) TheinclusionofreformsforwomenwasnotevenconsideredinOttomansocietyduring
theperiodoftheTanzimatreforms.
B) Despitewidespreaddiscussionofthepracticesofseclusion,polygamy,andveiling,few
improvementsinwomenssocialstatuswerewoninthe19thcentury.
C) Whilenotallwomenbenefitted,elitewomenwerefreedfromtherestrictiveaspectsof
MuslimsocietyduringtheperiodoftheTanzimatreforms.
D) Muslimrestrictionsagainstthesocialequalityofwomenweresweptawayaspartofthe
Tanzimatreforms.
Answer: B
Diff:2
PageRef:705
Topic:FromEmpiretoNation
Skill:Conceptual

15) WhatOttomansultanattemptedtorollbacktheTanzimatreformsandreinstituteanabsolute
monarchyin1878?
A) SelimIII
B) MahmudII
C) YazidII
D) AbdulHamid
Answer: D
Diff:2
PageRef:705
Topic:FromEmpiretoNation
Skill:Factual

16) InwhichofthefollowingareasdidSultanAbdulHamidcontinuetopressforincreased
Westernization?
A) Freedomofthepress
B) Constitutionalreform
C) MilitaryreformandtheintroductionofWesterntechnology
D) Civilliberties
Answer: C
Diff:2
PageRef:705
Topic:FromEmpiretoNation
Skill:Conceptual

499

17) WhichofthefollowinggroupswasresponsiblefortheoverthrowoftheOttomansultanatein
1908?
A) BlackSeptember
B) TheOttomanSocietyforUnionandProgress
C) TheYoungArabs
D) TheMamluks
Answer: B
Diff:1
PageRef:705
Topic:FromEmpiretoNation
Skill:Factual

18) WhichofthefollowingreformsresultedfromthecoupintheOttomanEmpireof1908?
A) Thesultanatewasabolished
B) Theconstitutionof1876wasrestored
C) TheJanissarieswereremovedasapoliticalandmilitaryforce
D) RestrictionsagainstwomeninMuslimsocietywereremoved
Answer: B
Diff:2
PageRef:705
Topic:FromEmpiretoNation
Skill:Conceptual

19) WithwhatEuropeanpowerdidtheOttomanscontestthecontrolofLibyajustpriortoWorld
WarI?
A) Britain
B) Germany
C) Austria-Hungary
D) Italy
Answer: D
Diff:1
PageRef:706
Topic:FromEmpiretoNation
Skill:Factual

500

20) WhichofthefollowingstatementsconcerningtherelationshipsbetweentheYoungTurksand
theArabsoftheOttomanEmpireafterthe1908coupismostaccurate?
A) TheYoungTurksharboredresentmentagainsttheArabsoftheempireforfailingto
supportthe1908coup.
B) Arabsupportofthe1908coupwanedwhentheydiscoveredthattheYoungTurkshad
nointentionsofabandoningtheconceptofempire.
C) The1908coupresultedintheimmediateindependenceoftheArabportionsofthe
OttomanEmpire.
D) TheclosealliancebetweentheYoungTurksandtheArableadersoftheOttomanEmpire
continuedafterthe1908coup.
Answer: B
Diff:2
PageRef:706
Topic:FromEmpiretoNation
Skill:Conceptual

21) WhatnationsinvasionofEgyptin1798signalledthebeginningofEuropeanpenetrationof
theIslamicheartland?
A) France
B) Britain
C) Russia
D) Austria-Hungary
Answer: A
Diff:1
PageRef:708
Topic:WesternIntrusionsandtheCrisisintheArabIslamicHeartlands
Skill:Factual

22) WhatIslamicgroupruledEgyptin1798?
A) TheFatimids
B) TheUmayyads
C) TheMamluks
D) TheAlmoravids
Answer: C
Diff:2
PageRef:708
Topic:WesternIntrusionsandtheCrisisintheArabIslamicHeartlands
Skill:Factual

501

23) By1801whatrulerhadsucceededinestablishinghisdominanceoverEgypt?
A) Murad,commanderoftheMamluks
B) Napoleon,Frenchemperor
C) MuhammadAli,anAlbanianofficerintheOttomanarmy
D) Nurhaci,Almoravidmilitarycommander
Answer: C
Diff:1
PageRef:709
Topic:WesternIntrusionsandtheCrisisintheArabIslamicHeartlands
Skill:Factual

24) WhichofthefollowingreformswasNOTintroducedinEgyptafter1801?
A) TherestorationoftheMamlukarmies
B) TheintroductionofWestern-stylemilitaryconscription
C) ThehiringofFrenchmilitaryadvisors
D) TheimportationofWesternarms
Answer: A
Diff:2
PageRef:709
Topic:WesternIntrusionsandtheCrisisintheArabIslamicHeartlands
Skill:Conceptual

25) WhatpreventedMuhammadAlifromoverthrowingtheOttomanEmpire?
A) Hisfailuretodevelopamodernarmy
B) Thelackofanavy
C) HisdefeatbytheOttomansatOmdurman
D) TheoppositionofEuropeanpowers
Answer: D
Diff:2
PageRef:709
Topic:WesternIntrusionsandtheCrisisintheArabIslamicHeartlands
Skill:Conceptual

26) WhichofthefollowingreformsundertakenbyMuhammadAlifailed?
A) TheproductionofrawmaterialsindemandinEurope(cotton,hemp,indigo)
B) ImprovementsofEgyptianharborsandirrigationworksalongtheNile
C) Thebuild-upofanEgyptianindustrialsector
D) Modernizationofthearmy
Answer: C
Diff:2
PageRef:709
Topic:WesternIntrusionsandtheCrisisintheArabIslamicHeartlands
Skill:Conceptual

502

27) MuhammadAlissuccessorsasrulersofEgyptwerereferredtoas
A) sultans.
B) khedives.
C) caliphs.
D) kings.
Answer: B
Diff:1
PageRef:709
Topic:WesternIntrusionsandtheCrisisintheArabIslamicHeartlands
Skill:Factual

28) EuropeanfinancierslentmoneytotheprofligatesuccessorsofMuhammadAlibecausethey
desiredaccesstoEgyptscheapcottonand,bythe1850s,ashareinthe
A) OrientExpress.
B) CairoRailway.
C) SuezCanal.
D) PanamaCanal.
Answer: C
Diff:1
PageRef:709
Topic:WesternIntrusionsandtheCrisisintheArabIslamicHeartlands
Skill:Factual

29) WhichofthefollowingwasanIslamicmoderatein19th-century-Egyptwhourgedthe
adoptionofWesternscientificknowledgeandtechnology?
A) al-Afghani
B) IbnSina
C) IbnRochd
D) al-Mansur
Answer: A
Diff:3
PageRef:710
Topic:WesternIntrusionsandtheCrisisintheArabIslamicHeartlands
Skill:Factual

503

30) ThekhedivalgovernmentofEgyptwasthreatenedin1882byarebellionofEgyptianmilitary
officersunder
A) MuhammadAli.
B) AhmadOrabi.
C) MuhammadAchmad.
D) KhalifaAbdallahi.
Answer: B
Diff:2
PageRef:711
Topic:WesternIntrusionsandtheCrisisintheArabIslamicHeartlands
Skill:Factual

31) WhatwastheresultoftherebellionbyEgyptianarmyofficersin1882?
A) ThekhedivalgovernmentwasoverthrownbyanindigenousEgyptiangovernment.
B) AnewconstitutionwasinstitutedmodeledontheOttomanconstitutionof1876.
C) TherebellionwascrushedbytheTurkishelementswithintheEgyptianarmy.
D) ThekhedivecalledontheBritishtocrushtherebellionresultinginBritishoverlordship
ofEgypt.
Answer: D
Diff:2
PageRef:711
Topic:WesternIntrusionsandtheCrisisintheArabIslamicHeartlands
Skill:Conceptual

32) WhatwasthecenterofEgyptianadministrationintheSudan?
A) Omdurman
B) Cairo
C) Aboukir
D) Khartoum
Answer: D
Diff:1
PageRef:711
Topic:WesternIntrusionsandtheCrisisintheArabIslamicHeartlands
Skill:Factual

504

33) OnwhatbasisdidMuhammadAchmadclaimleadershipoftheSudanicresistanceto
Egyptianrule?
A) HeclaimeddirectdescentfromMuhammad
B) HeclaimedtobeadirectdescendantofthekingsofGhana
C) HewastheheadoftheSunniulama intheSudan
D) HeclaimedtobeadirectdescendantofMurad,thelastruleroftheMamluks
Answer: A
Diff:2
PageRef:711
Topic:WesternIntrusionsandtheCrisisintheArabIslamicHeartlands
Skill:Conceptual

34) ThesuccessortotheMahdi,KhalifaAbdallahi,
A) relievedtherestrictivesocialregulationsimposedbytheMahdi.
B) immediatelylostthemilitaryadvantagegainedbytheMahdi.
C) fellintheMahdistdefeatatthebattleofOmdurmanin1898.
D) overthrewtheOttomansultanandcapturedIstanbul.
Answer: C
Diff:2
PageRef:714
Topic:WesternIntrusionsandtheCrisisintheArabIslamicHeartlands
Skill:Conceptual

35) WhowasresponsiblefortheunificationoftheManchutribesmenpriortotheinvasionof
Chinainthe17thcentury?
A) LinZexu
B) Nurhaci
C) Cixi
D) HongLiaquan
Answer: B
Diff:1
PageRef:714
Topic:TheLastDynasty:TheRiseandFalloftheQingEmpireinChina
Skill:Factual

36) ThedynasticnametakenbytheManchudynastywas
A) Song.
B) Tang.
C) Qing.
D) Chou.
Answer: C
Diff:1
PageRef:714
Topic:TheLastDynasty:TheRiseandFalloftheQingEmpireinChina
Skill:Factual

505

37) WhichofthefollowingstatementsconcerningtheManchugovernmentismostaccurate?
A) Theydestroyedthescholar-gentryinordertoconsolidatetheirgriponthegovernment.
B) Thecivilserviceexaminationsystemwaseliminatedasameansofenteringthe
government.
C) ThoughManchusoccupiedadisproportionatenumberofthehighestpoliticalpositions,
therewerefewlimitsonChinesepromotionswithintheimperialbureaucracy.
D) Chineseofficialswereeliminatedatthelocaladministrativelevelsinordertopreventthe
extremeregionalizationthathadledtothedownfallofpreviousdynasties.
Answer: C
Diff:2
PageRef:714
Topic:TheLastDynasty:TheRiseandFalloftheQingEmpireinChina
Skill:Conceptual

38) InwhatareadidtheManchusattempttotakestrongmeasuresofreform?
A) Eliminationofthescholar-gentry
B) Removalofsocialrestrictionsonwomen
C) OverturningtheConfuciansocialhierarchyofageandsex
D) Alleviatingruraldistressandunrest
Answer: D
Diff:2
PageRef:715
Topic:TheLastDynasty:TheRiseandFalloftheQingEmpireinChina
Skill:Conceptual

39) WhataccountsforthegeneralfailureofManchuattemptsatreform?
A) Resistanceonthepartofthepeasantry
B) Enormouspopulationgrowthandthedisappearanceofopenlands
C) Buddhistresistance
D) LossofterritorytonomadsfromtheAsiansteppes
Answer: B
Diff:2
PageRef:716
Topic:TheLastDynasty:TheRiseandFalloftheQingEmpireinChina
Skill:Conceptual

506

40) ThenewgroupsofmerchantsthatdevelopedinChinaunderthemorerelaxedcommercial
systemoftheManchuswerecalled
A) waiqin.
B) pescadors.
C) compradors.
D) Boxers.
Answer: C
Diff:1
PageRef:715
Topic:TheLastDynasty:TheRiseandFalloftheQingEmpireinChina
Skill:Factual

41) AllofthefollowingsignsofdynasticdeclinewereapparentintheQingregimebythe
beginningofthe19thcenturyEXCEPT
A) corruptionoftheexaminationsystem.
B) diversionofrevenuefromstateprojectstoprivatefortunes.
C) failureofforeigncommerce.
D) foodshortages,massmigrations,andbanditry.
Answer: C
Diff:2
PageRef:716
Topic:TheLastDynasty:TheRiseandFalloftheQingEmpireinChina
Skill:Conceptual

42) UnhappyabouttheunfavorabletermsoftradeinChina,Britishmerchantshitonapossible
solutiontoreversetheflowofbullionintheformof
A) cottontextiles.
B) opiumfromIndia.
C) teas.
D) industrialmachinery.
Answer: B
Diff:1
PageRef:717
Topic:TheLastDynasty:TheRiseandFalloftheQingEmpireinChina
Skill:Factual

507

43) WhatwastheimpactoftheBritishopiumtradeonChina?
A) ItsusewasrestrictedtothepeasantryofnorthernChina,whereproductionoffood
rapidlydecreased.
B) Thegovernmentwasquicklyabletohalttheimportationofopium,sothatitdidnot
havethedisastrousimpactontheChinesepopulationthatwasexpected.
C) WithinyearsChinasfavorablebalanceoftradewasreversedandsilverbegantoflow
outofthecountry.
D) Duetotheaddictionoftheimperialcourt,theBritishwerewelcomedasavaluabletrade
partnerofChina.
Answer: C
Diff:2
PageRef:717
Topic:TheLastDynasty:TheRiseandFalloftheQingEmpireinChina
Skill:Conceptual

44) TheChineseofficialchargedwitheliminatingtheopiumtradeinthe1830swas
A) Cixi.
B) LinZexu.
C) HongLiuquan.
D) Kanxi.
Answer: B
Diff:2
PageRef:718
Topic:TheLastDynasty:TheRiseandFalloftheQingEmpireinChina
Skill:Factual

45) WhatwastheoutcomeoftheOpiumWar?
A) Despitetechnologicaladvantages,theBritishforceswereoverwhelmedbytheChinese
numericalsuperiorityandwereunabletopenetrateChinasisolation.
B) TheBritishsoonswepttheseasofopposition,butwerepreventedfromenteringChina
byoppositionfromotherEuropeanpowerswhofearedBritainsoverthrowofthe
Manchus.
C) TheBritishvictorywassooverwhelmingthattheManchudynastywasoverthrownby
1850andreplacedbyarepublic.
D) BritishvictoryintheOpiumWarallowedEuropeanpowerstoforceChinatoopentrade
anddiplomaticexchanges.
Answer: D
Diff:2
PageRef:718
Topic:TheLastDynasty:TheRiseandFalloftheQingEmpireinChina
Skill:Conceptual

508

46) Thesemi-ChristianrebellionthatbrokeoutinsouthernChinainthe1850sandearly1860s
wasthe
A) Boxerrebellion.
B) Kwangxirebellion.
C) Taipingrebellion.
D) Shandongrebellion.
Answer: C
Diff:1
PageRef:718
Topic:TheLastDynasty:TheRiseandFalloftheQingEmpireinChina
Skill:Factual

47) WhatwasthepoliticalandsocialpositionoftheManchurulersattheendofthe19thcentury?
A) TheManchurulersstubbornlyresistedthefar-reachingreformsthatweretheonlyhope
ofsavingtheregimeandChinesecivilization.
B) ThelastdecadesofthedynastyweredominatedbyCixi,awomanwhoproposedradical
reformsofthesocialorder.
C) TheChinesescholar-gentryandtheprovincialelitealliedwiththeemperorstointroduce
significantreformoflandholdingpracticesandregionaladministration.
D) ThedynastywholeheartedlyembracedtheongoingWesternizationoftheChinese
governmentandeconomy.
Answer: A
Diff:2
PageRef:719
Topic:TheLastDynasty:TheRiseandFalloftheQingEmpireinChina
Skill:Conceptual

48) InwhatyearwasthelastemperorofChinadeposedinfavorofarepublicanformof
government?
A) 1895
B) 1901
C) 1908
D) 1912
Answer: D
Diff:1
PageRef:720
Topic:TheLastDynasty:TheRiseandFalloftheQingEmpireinChina
Skill:Factual

509

EssayQuestions
1) HowdidtheemergenceoftheindustrializedWestaffectthepatternforthedeclineof
civilizations?
Answer: PriortoindustrializedWest,civilizationsdeclinedinforseeablepatterns(seeChapter
11);generallyinternaldeclineassociatedwithweakrulers,fragmentationofauthority,
economicdisorder,socialdisruption;externalthreattocivilizationsnormallymounted
bynomadicpeoples;civilizationshadlittleornoimpactonthedeclineofother
civilizations;afterriseofWest,EuropeandlaterNorthAmericabecamemajorfactorin
declineofcivilizations;advancedtechnologyledtodestructionofAztec,Inca,African,
Islamic,andChinesecivilizations.
Diff:2
PageRef:706-707
Topic:Introduction
Skill:Conceptual

2) HowdidtheexperienceoftheQingdynastyinChinaandtheOttomanEmpiredifferbetween
1700and1900?
Answer: Infirstcenturyandahalf,ManchusreformedChina,appearedtorestoretraditional
Chinesecivilization;OttomanssufferedterritoriallossestoEuropeans,continued
internalweaknessesduetoJanissaries,inabilitytocontrollocalelites,economic
competitionwithWest;by19thcentury,Qingdynastyinturmoilasreformsfailed,new
militarychallengesmountedbyWest,hugepopulationgrowthdestroyedabilityto
sustaintraditionalsocialorganization;OttomanEmpirerevivedafterseriesofreforms
patternedonWest.
Diff:2
PageRef:700-707714-722
Topic:Introduction
Skill:Conceptual

3) CompareandcontrasttheconditionsleadingtotheoverthrowoftheOttomanEmpirein1908
andtheQingdynastyin1912.
Answer: OttomanEmpire:AbdulHamidattemptedtoundoreforms,returntoabsolutism;
overthrownbyWesternizedandeducatedgroupofyoungermenseekingretentionof
reformscalledtheYoungTurks.Qingdynasty:alsooverthrownbyyoungermenwith
Westerncontacts(compradors)seekingreforms;emperorscontinued,asinOttoman
Empire,toberegressiveforce;factorofdirectforeigninterventioninChinanotpresent
inOttomanEmpire.
Diff:2
PageRef:700-707714-722
Topic:FromEmpiretoNation
Skill:Conceptual

510

4) CompareandcontrasttheBritishinterventioninEgyptwiththeirinterventioninChina.
Answer: Egypt:Britishinvestorshadinterestsinsupplyofrawmaterials,alsoinSuezCanal;
calledongovernmenttointervenetoprotectinvestmentsinKhedivalgovernment;
KhediveactuallycalledonBritishforcestodefendagainstrevoltofAchmadOrabi.
China:BritishinvestmentsinformofopiumtradefromIndia;merchantscalledon
governmenttoprotectlucrativetradedespiteattemptsofChinesegovernmenttohalt
drugtraffic;interventionresultedinforcibleentryofEnglishdiplomats,openingof
ChineseportstoEuropeantrade.
Diff:2
PageRef:706-714716-721
Topic:WesternIntrusionsandtheCrisisintheArabIslamicHeartlands
Skill:Conceptual

5) ContrastthereactionsoftheIslamicheartlandsandChinatothechallengeoftheWest.
Answer: Chinesecivilizationmorefundamentallythreatened,indeeddestroyedbyadventof
West;Muslimshadlong-termfamiliaritywithWest,militaryconfrontationsdating
backtoMiddleAges,similarJudaeo-Christianreligiousbackground,similar
backgroundinclassicalrationalismreceivedfromGreekphilosophy;Chinaremained
isolatedtotallyfromWesternthoughtandculture;Islamretainedreligiousfocusafter
politicaldefeat;ChineseConfucianismpassedwithpoliticaldefeat,lossofcultural
center;Muslimshadmanypoliticalcenters;ChinahadonlyQingdynasty.
Diff:2
PageRef:721-722
Topic:Conclusion:IslamicandChineseResponsestotheChallengeoftheWest
Skill:Conceptual

511

Chapter32 RussiaandJapan:IndustrializationOutside
theWest
Multiple-ChoiceQuestions
1) OfthefollowingregionsdefiedthecommonpatternofgrowingWesterndominationinthe
19thcentury?
A) RussiaandJapan
B) TheOttomanEmpire
C) TheCarribbean
D) WestAfrica
Answer: A
Diff:1
PageRef:726
Topic:Introduction
Skill:Conceptual

2) WhichofthefollowingstatementsconcerningthedevelopmentofRussiaandJapanto1900is
NOTaccurate?
A) NeitherRussianorJapanrivaledtheindustrialmightoftheWestby1900.
B) Bothnationsgainedsufficientpowertowieldimportantpoliticalandmilitaryinfluence
inthecolonialscramble.
C) RussiaandJapandidlaunchsignificantindustrializationby1914.
D) BothRussiaandJapanachievedbotheconomicautonomyandashareintheWestscore
position.
Answer: D
Diff:2
PageRef:726
Topic:Introduction
Skill:Conceptual

3) WhichofthefollowingrepresentsasignificantdifferencebetweenRussiaandJapan?
A) OnlyJapanhadasignificantaristocracypriortoindustrialization.
B) OnlyRussiaparticipatedinterritorialexpansionby1914.
C) RussiaengagedinselectiveborrowingfromWesternmodelsby1700.
D) OnlyJapanunderwentsignificantpoliticalrevolutionpriorto1914.
Answer: C
Diff:2
PageRef:724-742
Topic:Introduction
Skill:Conceptual

512

4) WhichofthefollowingreflectsasignificantsimilaritybetweenJapanandRussiaduringthe
periodofindustrializationpriorto1914?
A) Bothexperiencedsignificantpoliticalrevolutions.
B) BothJapanandRussiahadpriorexperienceofimitation:JapanfromChinaandRussia
fromByzantiumandtheWest.
C) Bothdemonstratedremarkablepoliticalflexibilityresultinginsweepingtransformations
ofpoliticalstructure.
D) BothengagedinterritorialacquisitionsintheOttomanEmpire.
Answer: B
Diff:2
PageRef:726
Topic:Introduction
Skill:Conceptual

5) RussiasfearaboutWesternizationinthefirstdecadesofthe19thcenturywasrootedin
A) concernaboutBritishinvasion.
B) theFrenchRevolution.
C) dislikeofWesterndress.
D) worryoverlossofPoland.
Answer: B
Diff:1
PageRef:727
Topic:RussiasReformsandIndustrialAdvance
Skill:Factual

6) WhatnationswerelinkedtogetherintheHolyAlliancethatgroupedconservativemonarchies
togetherindefenseofreligionandthestatusquoin1815?
A) Britain,France,Spain
B) Spain,Russia,Poland
C) Russia,Prussia,Austria
D) Austria,Japan,Russia
Answer: C
Diff:2
PageRef:727
Topic:RussiasReformsandIndustrialAdvance
Skill:Factual

513

7) WhatwasthenameoftheRussianrevoltinspiredbyWesternvaluesin1825?
A) TheDecembristring
B) TheNovemberrebellion
C) Pushkinsrevolt
D) Pugachevrebellion
Answer: A
Diff:1
PageRef:727
Topic:RussiasReformsandIndustrialAdvance
Skill:Factual

8) WhichofthefollowingpartlyaccountsforRussiaslackofsignificantrevolutionin1830and
1848?
A) Russialackedasubstantialhistoryofautocracy
B) TheabsenceofacoercivelaborsysteminRussialeftscantcauseforrebellion
C) Politicalrepression
D) LikeEngland,Russiashistoryofparticipatorygovernmentanditsnationalparliament
forestalledrevolution
Answer: C
Diff:2
PageRef:727
Topic:RussiasReformsandIndustrialAdvance
Skill:Conceptual

9) WhichofthefollowingstatementsconcerningRussianterritorialexpansionismostaccurate?
A) RussiaslossofPolandintherevoltof1830stimulatedotherattemptsatterritorial
expansion.
B) RussiaactivelyopposednationalistmovementsintheBalkansinkeepingwiththeir
conservativetradition.
C) WesternpowersactivelyaidedRussiaspursuitofterritoriesintheOttomanEmpire.
D) Nomassiveacquisitionsmarkedtheearly19thcentury,butRussiacontinuedtobean
aggressivecompetitorforterritorialexpansion.
Answer: D
Diff:2
PageRef:728
Topic:RussiasReformsandIndustrialAdvance
Skill:Conceptual

514

10) WhichofthefollowingstatementsbestdescribestheRussianeconomyatthebeginningofthe
19thcentury?
A) Russiahadachievedeconomicautonomyinthe18thcentury,althoughmostofeastern
Europeremainedlargelyagricultural.
B) TheRussianeconomywasgeographicallyorientedtotheOttomanEmpire ,afeatureof
theMongoldominationofRussiauntilthe15thcentury.
C) RussiaseconomicdynamismandinnovationrivaledtheWest.
D) Inreturnforlow-costgrainexports,RussiaandotherEastEuropeanareasimported
Westernluxurygoodsforthegreataristocratstodisplayasbadgesofrespectability.
Answer: D
Diff:2
PageRef:729
Topic:RussiasReformsandIndustrialAdvance
Skill:Conceptual

11) Whatwarinthemid-19thcenturydemonstratedRussiaswideninggapwiththeWest?
A) TheRusso-JapaneseWar
B) NapoleonsinvasionofRussia
C) TheCrimeanWar
D) TheSepoyrebellion
Answer: C
Diff:1
PageRef:729
Topic:RussiasReformsandIndustrialAdvance
Skill:Factual

12) WhataccountedfortheWestsvictoryoverRussiaintheCrimeanWar?
A) ThewarwasfoughtfarfromRussia,necessitatinglengthylinesofcommunicationand
supply.
B) Russiawasforcedtofightanoffensivewaragainstentrenchedpositions.
C) ThewarwasfoughtalmostentirelyatseawheretheRussianswereunabletobringtheir
numericalsuperioritytobear.
D) TheWesternnationswonnotbecauseofsuperiortacticsorinspiredprinciples,but
becauseofindustrialadvantages.
Answer: D
Diff:2
PageRef:729
Topic:RussiasReformsandIndustrialAdvance
Skill:Conceptual

515

13) Whattsarbegantheprocessofreforminthe19thcentury?
A) AlexanderI
B) AlexanderII
C) NicholasI
D) PetertheGreat
Answer: B
Diff:1
PageRef:729
Topic:RussiasReformsandIndustrialAdvance
Skill:Factual

14) WhichofthefollowingwasNOTaconsiderationinthereformofserfdom?
A) Itwasthepurposeofthetsartosweepawaythetightlyknitpeasantcommunitieson
whichserfdomdepended
B) Thedevelopmentofavigorousandmobilelaborforce
C) AdesiretomeetWesternhumanitarianstandards
D) Periodicpeasantuprisingsfocusedonlackoffreedom,undueobligations,andlackof
land
Answer: A
Diff:2
PageRef:729
Topic:RussiasReformsandIndustrialAdvance
Skill:Conceptual

15) InwhatyearwereRussianserfsemancipated?
A) 1831
B) 1854
C) 1861
D) 1868
Answer: C
Diff:2
PageRef:729
Topic:RussiasReformsandIndustrialAdvance
Skill:Factual

516

16) WhichofthefollowingstatementsconcerningtheemancipationoftheserfsinRussiaismost
accurate?
A) TheemancipationoftheserfsdestroyedtheRussianaristocracy.
B) Emancipationoftheserfsloosenedthegripofthetsariststate.
C) Inadditiontopersonalfreedom,theserfsweregrantedparcelsoflandsubjectto
redemptionpayments.
D) Followingemancipation,peasantswerefreetomoveaboutRussiaastheypleased
leadingtomassivemovementsofagriculturallabor.
Answer: C
Diff:2
PageRef:729
Topic:RussiasReformsandIndustrialAdvance
Skill:Conceptual

17) AllofthefollowingwerereformsintroducedinRussiainthe1860sand1870sEXCEPT
A) thecreationoftheDuma,anationalparliament.
B) thecreationoflocalpoliticalcouncils,thezemstvoes.
C) theissuanceofnewlawcodesthatcutbacktraditionalpunishments.
D) reorganizationofthemilitary.
Answer: A
Diff:2
PageRef:730
Topic:RussiasReformsandIndustrialAdvance
Skill:Conceptual

18) WhatwasthefirststeptowardindustrializationinRussia?
A) Theconstructionoffactories
B) Thedevelopmentoftheminingsector
C) Thecreationofanextensivesystemofrailways
D) TheendofthegraintradewiththeWest
Answer: C
Diff:2
PageRef:730
Topic:RussiasReformsandIndustrialAdvance
Skill:Conceptual

517

19) TheRussianministeroffinancefrom1892to1903responsibleformucheconomic
modernizationwas
A) Stolypin.
B) CountSergeiWitte.
C) KlemenzvonMetternich.
D) GregorMendel.
Answer: B
Diff:1
PageRef:732
Topic:RussiasReformsandIndustrialAdvance
Skill:Factual

20) WhichofthefollowingstatementsconcerningthecapitalizationofRussianindustryismost
accurate?
A) CapitalforRussianinvestmentwasalmostentirelyderivedfromliquidationof
agriculturalestatesinRussia.
B) By1900approximatelyhalfofRussianindustrywasforeign-ownedbyBritish,German,
andFrenchindustrialists.
C) ItwasthecontactwiththeJapanesethatledtoaninfluxofcapitalforRussian
industrialization.
D) Russianindustrywascapitalizedbyasubstantialmiddleclassthathadbuiltupwealth
inthegraintradewiththeWest.
Answer: B
Diff:2
PageRef:732
Topic:RussiasReformsandIndustrialAdvance
Skill:Conceptual

21) By1900,howsuccessfulwastheRussianindustrializationprogram?
A) Despitemassiveprogramsofforcedlaborandextensivegovernmentsubsidies,the
Russianprogramofindustrializationfailed.
B) Russianindustrializationprogressedslowlyandby1900hadreachedtenthintheworld
intermsofsteelproduction.
C) By1900,Russiahadsurgedtofourthrankintheworldinsteelproductionandwas
secondtotheUnitedStatesinthenewerareaofpetroleumproduction.
D) Withoutaccesstoplentifulrawmaterials,Russiawasdependentonconstantterritorial
acquisitionstofuelitslaggingindustrialprogram.
Answer: C
Diff:2
PageRef:732
Topic:RussiasReformsandIndustrialAdvance
Skill:Conceptual

518

22) WhichofthefollowingwaspresentduringtheRussianprogramofindustrialization?
A) AttitudinalchangeamongworkerssimilartotheWest
B) Alargemiddleclass
C) Richnaturalresources
D) Small,butefficient,factories
Answer: C
Diff:2
PageRef:732
Topic:RussiasReformsandIndustrialAdvance
Skill:Conceptual

23) AllofthefollowingwerepartoftherisingtideofunrestinRussiaduringthesecondhalfof
the19thcenturyEXCEPT
A) theOrthodoxchurch.
B) ethnicminorities.
C) peasants.
D) theintelligentsia.
Answer: A
Diff:2
PageRef:732-733
Topic:ProtestandRevolutioninRussia
Skill:Conceptual

24) WhatwasthegeneralgoaloftheRussianintelligentsia?
A) Politicalfreedom,socialreform,andretentionofRussianculture
B) Politicalfreedom,retentionofthesocialhierarchy,andincreasedWesternization
C) Restrictionofcivilliberties,honoranddeferencetotheemperor,retentionofRussian
culture
D) RadicalWesternizationaspartofaprogramofincreasedindustrialization
Answer: A
Diff:2
PageRef:732
Topic:ProtestandRevolutioninRussia
Skill:Conceptual

25) Russianradicalswhosoughttheabolitionofallformalgovernmentwerecalled
A) Decembrists.
B) Latitudinarians.
C) abolitionists.
D) anarchists.
Answer: D
Diff:1
PageRef:733
Topic:ProtestandRevolutioninRussia
Skill:Factual

519

26) Whatwasthechiefpoliticalmethodusedbytheanarchiststoachievereform?
A) Strikes
B) Terrorism
C) Politicalrallies
D) Voterregistration
Answer: B
Diff:1
PageRef:733
Topic:ProtestandRevolutioninRussia
Skill:Factual

27) WhichofthefollowingstatementsaboutRussianMarxismismostaccurate?
A) Marxistinsistenceoncarefulrevolutionaryorganizationandafocusontheworkingclass
wasrapidlyassimilatedbyanarchistsandpeasantgroups.
B) MarxistdoctrineswerenotimportedfromtheWest,butoriginatedamongtheRussian
intelligentsia.
C) LeninintroducedimportantinnovationsinMarxisttheory,includingtheideathata
proletarianrevolutioncouldtakeplacewithoutgoingthroughamiddle-classphase.
D) LeninwasdedicatedtothemasselectioneeringtypicalofWesternsocialistparties.
Answer: C
Diff:2
PageRef:733
Topic:ProtestandRevolutioninRussia
Skill:Conceptual

28) LeninsapproachwasadoptedbythegroupsofRussianMarxistsknownas
A) Mensheviks.
B) anarchists.
C) Decembrists.
D) Bolsheviks.
Answer: D
Diff:1
PageRef:733
Topic:ProtestandRevolutioninRussia
Skill:Factual

520

29) WhichofthefollowingdidNOTcontributetoworking-classradicalisminlate19th-century
Russia?
A) Theabsenceoflegalpoliticaloutlets
B) Severeconditionsofearlyindustrialization
C) Ruralunrestandadoptionofpeasantgrievances
D) Absenceofunions
Answer: D
Diff:2
PageRef:733
Topic:ProtestandRevolutioninRussia
Skill:Conceptual

30) FailureinwhatwarledtotheRussianrevolutionof1905?
A) Crimean
B) Sino-Japanese
C) Russo-Japanese
D) WorldWarI
Answer: C
Diff:1
PageRef:734
Topic:ProtestandRevolutioninRussia
Skill:Factual

31) Whatgroupdidtheimperialgovernmentappealtointhereformsfollowingtherevolutionof
1905?
A) Liberals
B) Workersorganizations
C) Marxists
D) Anarchists
Answer: A
Diff:2
PageRef:734
Topic:ProtestandRevolutioninRussia
Skill:Conceptual

521

32) Whatministerwasresponsibleforenactingreformsforthepeasantryfollowingtherevolution
of1905?
A) CountWitte
B) GrigoriRasputin
C) AlexiRomanov
D) Stolypin
Answer: D
Diff:1
PageRef:734
Topic:ProtestandRevolutioninRussia
Skill:Factual

33) TheDumawas
A) theconfrontationbetweenradialworkersandthetsaristarmyin1905.
B) asystemofcollectivefarmsforpeasantsintroducedafter1905.
C) anationalparliamentcreatedintheaftermathofthe1905revolution.
D) theimperialcouncilthattookovergovernmentaftertheabdicationofthetsarin1905.
Answer: C
Diff:1
PageRef:734
Topic:ProtestandRevolutioninRussia
Skill:Factual

34) Peasantswhorespondedtothereformsof1905byengaginginentrepreneurialactivity
includingincreasingproductionandbuyinguplandwerecalled
A) Duma.
B) kulaks.
C) anarchists.
D) Bakunin.
Answer: B
Diff:1
PageRef:734
Topic:ProtestandRevolutioninRussia
Skill:Factual

522

35) WhichofthefollowingRussiandevelopmentswasNOTadoptedinothereastEuropean
states?
A) Nationalparliaments
B) Emancipationofserfs
C) EconomicautonomyfromtheWest
D) Monarchicformsofgovernment
Answer: C
Diff:2
PageRef:735
Topic:ProtestandRevolutioninRussia
Skill:Conceptual

36) WhichofthefollowingwasNOTa19th-centuryRussiannovelist?
A) Turgenev
B) Pavlov
C) Tolstoy
D) Dostoevsky
Answer: B
Diff:1
PageRef:735
Topic:ProtestandRevolutioninRussia
Skill:Factual

37) WhichofthefollowingstatementsconcerningtheTokugawaShogunateinthe19thcenturyis
mostaccurate?
A) TheShogunatebureaucracyhadbeenopenedtotalentedcommoners ,areformthat
improvedthestandingofthegovernmentwiththemassesoftheJapanesepeople.
B) Bythe19thcentury,theTokugawawereabletodispensewiththefeudalorganizationof
earlierJapan.
C) IncreasinglytheShogunatedependedonitslong-standingallianceswithWestern
powerstomaintainitsdominance.
D) TheShogunatecontinuedtocombineacentralbureaucracywithsemi-feudalalliances
withregionaldaimyosandthesamurai.
Answer: D
Diff:2
PageRef:735
Topic:Japan:TransformationWithoutRevolution
Skill:Conceptual

523

38) WhichofthefollowingstatementsconcerningTokugawaintellectualandculturallifeismost
accurate?
A) JapaneseliteraturereacheditszenithduringthelastdecadesoftheTokugawa
Shogunate.
B) ConfucianismrapidlylostgroundtoBuddhismasthemajorreligiousandethicalbasis
forJapanesesociety.
C) JapancontinuedtobelargelyimitativeofconservativeChineseintellectualcurrents
ratherthandevelopingdynamicethicalandphilosophicalsystems.
D) LiteracyinJapanreachedlevelshigherthananywhereelseoutsidetheWest.
Answer: D
Diff:2
PageRef:735
Topic:Japan:TransformationWithoutRevolution
Skill:Conceptual

39) WhichofthefollowinggroupsinTokugawaJapanadvocatedinterestinWesternscientific
advance?
A) DutchStudiesgroup
B) Nationalstudiesgroup
C) Confucianscholars
D) Buddhistscholars
Answer: A
Diff:2
PageRef:736-737
Topic:Japan:TransformationWithoutRevolution
Skill:Factual

40) WhowasresponsiblefortheforcedopeningofJapanin1853?
A) CaptainJamesCook
B) CommodoreMatthewPerry
C) AdmiralHoratioNelson
D) CaptainWilliamFarragut
Answer: B
Diff:1
PageRef:736
Topic:Japan:TransformationWithoutRevolution
Skill:Factual

524

41) Inwhatyearwasanewemperor,MutsuhitobutcommonlycalledMeijiorEnlightened
One,proclaimed,signalingtheendofamajorpoliticalcrisis?
A) 1854
B) 1868
C) 1875
D) 1889
Answer: B
Diff:1
PageRef:736
Topic:Japan:TransformationWithoutRevolution
Skill:Factual

42) WhichofthefollowingwasNOTanadvantageofJapanoverChinainthecompetitionto
assumeleadershipandtoestablishindustrializationinAsia?
A) JapansleadershipwaslesssecularandbureaucraticthanthatofChina.
B) Japanalreadyknewthebenefitsofimitation,whichChinahadneveracknowledged.
C) Japanhadallowedamoreautonomousmerchanttradition.
D) Feudaltraditionslimitedtheheavyhandofgovernmentcontrolswhilestimulatinga
senseofcompetitiveness.
Answer: A
Diff:2
PageRef:737
Topic:Japan:TransformationWithoutRevolution
Skill:Conceptual

43) WhichofthefollowingstatementsconcerningJapanesepoliticalreformsintheperiodofthe
MeijistateisNOTaccurate?
A) Meijileadersestablishedanewconservativenobility,stockedwithformernoblesand
MeijileadersthatoperatedaBritish-styleHouseofPeers.
B) Samurai,destroyedbytheremovalofgovernmentstipends,werebannedfrom
participationintheMeijiDiet.
C) Theconstitutionissuedin1889assuredmajorprerogativesfortheemperoralongwith
limitedpowersforthelowerhouseoftheDiet.
D) Thebureaucracywasreorganized,insulatedfrompoliticalpressures,andopenedto
talentonthebasisofcivilserviceexaminations.
Answer: B
Diff:2
PageRef:738
Topic:Japan:TransformationWithoutRevolution
Skill:Conceptual

525

44) WhatwastheprimarydifferencebetweenthereformedJapanesegovernmentandreformed
Russianinstitutionsby1914?
A) Japanretainedanemperorattheheadofgovernment.
B) Japancreatedanationalparliament.
C) Japansgovernmentwaselectedbyabroadmajorityofthepopulation.
D) Japansgovernmenthadincorporatedbusinessleadersintoitsgoverningstructure.
Answer: D
Diff:2
PageRef:738
Topic:Japan:TransformationWithoutRevolution
Skill:Conceptual

45) OneofthemajorsimilaritiesbetweenJapaneseandRussianindustrializationwasthefactthat
A) bothlackednaturalresources.
B) scarcecapitalandunfamiliarityofnewtechnologycompelledstatedirection.
C) neitherwasabletocompleteconstructionofarailwaysystem.
D) neitherhadanyexperienceofculturalexchangewiththeWest.
Answer: B
Diff:2
PageRef:739
Topic:Japan:TransformationWithoutRevolution
Skill:Conceptual

46) HugeindustrialcombinesputtogetherinJapanbythe1890swerecalled
A) haiku.
B) terakoya.
C) zaibatsus.
D) khitan.
Answer: C
Diff:1
PageRef:739
Topic:Japan:TransformationWithoutRevolution
Skill:Factual

526

47) WhichofthefollowingstatementsconcerningJapaneseindustrializationpriortoWorldWarI
iscorrect?
A) Japansworkforcewasamongthehighestpaidintheworld.
B) AbundantnaturalresourcesmadeJapanvirtuallyself-sufficientasanindustrialized
nation.
C) By1914,JapanhadreachedthelevelofindustrializationfoundintheWest.
D) Japanneededexportstopayformachineandresourceimports.
Answer: D
Diff:2
PageRef:739
Topic:Japan:TransformationWithoutRevolution
Skill:Conceptual

48) WhichofthefollowingWesternculturalcharacteristicswasNOTadoptedbylargenumbersof
Japanese?
A) Hairstyles
B) Standardsofhygiene
C) Westerncalendar
D) Christianity
Answer: D
Diff:2
PageRef:740-741
Topic:Japan:TransformationWithoutRevolution
Skill:Conceptual

49) WhichofthefollowingreligionsgainednewadherentsinindustrializedJapan?
A) Shinto
B) Confucianism
C) Buddhism
D) Christianity
Answer: A
Diff:3
PageRef:741
Topic:Japan:TransformationWithoutRevolution
Skill:Factual

527

50) WhichofthefollowingwasNOTasignofsignificantsocialstressinindustrializedJapan?
A) DisputesbetweengenerationsoverWesternization
B) Theincreasingfreedomandpoliticalinfluenceofwomen
C) Growthofnationalism
D) Growthofurbanslums
Answer: B
Diff:2
PageRef:741-742
Topic:Japan:TransformationWithoutRevolution
Skill:Conceptual

EssayQuestions
1) CompareandcontrastJapaneseandRussiansocietypriortoindustrialization.
Answer: Russiangovernmentanempirewithlimitedparticipationbyaristocracy;Japanese
governmentTokugawaShogunatetypifiedbycentralbureaucracycomposedof
samuraiandregionalfeudalgovernmentsunderdaimyos;technicallyJapanretainedan
emperor,buthewasafigureheadpriorto1868;bothRussiaandJapanwereessentially
agriculturaleconomies,althoughJapanwasmuchmoreisolated;Russiaretained
programofgrainexportstoWest;bothcountrieshadpriortraditionofimitation,Russia
fromWest,JapanfromChina;Russiafollowedpolicyofterritorialexpansion,Japandid
not.
Diff:2
PageRef:724-727
Topic:Introduction
Skill:Conceptual

2) CompareandcontrastthepoliticalreformsundertakeninJapanandRussiapriortoand
duringindustrialization.
Answer: Russia:abolitionofserfdom,establishmentoflocalpoliticalunits(zenstvoes),military
reform,establishmentofDuma.Japan:abolitionoffeudalism,serfdom,militaryreform,
establishmentoflocalgovernmentunderregionalprefects,creationofDietwithupper
andlowerhouse.Despiteseemingsimilarityofreform,Japanesemoresweeping.
Russianreformwaslimitedintermsofshakingauthorityofaristocracy.
Diff:2
PageRef:727-738
Topic:RussiasReformsandIndustrialAdvance
Skill:Conceptual

3) CompareandcontrastindustrializationinJapanandRussia.
Answer: Bothrequiredstatedirectionduetolackofcapital,entrepreneurialexperience;both
reliedonimitationofWest,alsocertaincapitalizationfromWest;Russiamore
dependentonforeigncapital,morevulnerabletoforeigncontrolofindustry;duetolack
ofnaturalresources,Japaneseindustrializationmorefocusedonexportof
manufacturedgoods;Japaneseindustrializationaccompaniedbydevelopmentof
middleclass,butnomiddleclassinRussia.
Diff:2
PageRef:727-742
Topic:ProtestandRevolutioninRussia
Skill:Conceptual

528

4) WhydidrevolutionoccurinRussiainconsequenceofindustrialization,butnotinJapan?
Answer: Russianpoliticalreformswerelesssincere,lessfar-reaching;Westerntraditionsof
politicalradicalismpresentinRussia;workingclassabletoachieveunionization,
politicalradicalisminRussia,butnotinJapan;Japanmoresecularsociety,lessaffected
byreligiousconservatism;aristocracyofJapanreformulatedinindustrialrevolutionas
newelitewithentrepreneurs,samurai;nomiddle-classdevelopmentinRussia.
Diff:2
PageRef:727-742
Topic:ProtestandRevolutioninRussia
Skill:Conceptual

5) WhydidJapanratherthanChinaassumeleadershipinAsiainthefaceofthechallengeofthe
West?
Answer: PossessedflexibilitythatChinalacked;knewbenefitsofimitation;slowercentralization
ofgovernmentallowedmoreautonomousmerchanttradition;feudalconditionslimited
governmentcontrolsandpermittedinitiative;notcrushedbymassivepopulation
increaseaswasChina;Chinasuffereddynasticcrisisinthe19thcenturywhileJapan
maintainedpoliticalvigorinthefirsthalfofthe19thcentury.
Diff:2
PageRef:737
Topic:Japan:TransformationWithoutRevolution
Skill:Conceptual

529

Chapter33 DescentintotheAbyss:WorldWarIandthe
CrisisoftheEuropeanGlobalOrder
Multiple-ChoiceQuestions
1) ApproximatelyhowlongdidWorldWarIlast?
A) Aboutfouryears
B) Abouttwoyears
C) Aboutsixyears
D) Aboutoneyear
Answer: A
Diff:1
PageRef:755-763
Topic:WorldWarI
Skill:Factual

2) WhichofthefollowingnationsdidNOTcomprisetheTripleEntenteallianceinthetheearly
1900s?
A) Germany
B) Britain
C) France
D) Russia
Answer: A
Diff:1
PageRef:755
Topic:WorldWarI
Skill:Factual

3) WhichofthefollowingwasNOTaFrenchcolonyinnorthAfricaattheoutbreakofWWI?
A) Libya
B) Morocco
C) Tunisia
D) Algeria
Answer: A
Diff:1
PageRef:755
Topic:WorldWarI
Skill:Factual

530

4) WhatwastheregionofEuropethatproducedthemostdiplomaticcrisespriortoWorldWarI?
A) Scandinavia
B) TheBalkans
C) Italy
D) Spain
Answer: B
Diff:1
PageRef:757
Topic:WorldWarI
Skill:Factual

5) WhattwoEuropeanpowersweredirectlyinvolvedintheBalkandiplomacy?
A) GermanyandRussia
B) RussiaandBritain
C) RussiaandAustria-Hungary
D) FranceandAustria-Hungary
Answer: C
Diff:2
PageRef:757
Topic:WorldWarI
Skill:Factual

6) WhichofthefollowingwasNOTaneventleadingtotheoutbreakofWorldWarI?
A) TheassassinationoftheAustrianArchdukebyaSerbiannationalist
B) AustriasdeclarationofwaronSerbia
C) ThemobilizationoftheRussianarmy
D) FrancesinvasionofBelgium
Answer: D
Diff:2
PageRef:757
Topic:WorldWarI
Skill:Factual

7) GermanysleaderscountedonWHATtohelpthemoverwhelmtheBelgiansandFrench?
A) Theircountryssuperbrailwaysystem
B) Small,butlightning-quickarmies
C) Submarinewarfare
D) Diplomacy
Answer: A
Diff:2
PageRef:758
Topic:WorldWarI
Skill:Factual

531

8) WhichofthefollowingisNOTareasonforthequicksuppressionoftheRussianarmies
duringthefirstweeksofthewar?
A) Aristocraticgenerals
B) Uncodedbattlecommands
C) DestructionoftheRussianfleetatLeningrad
D) Ineffectiveartillerycover
Answer: C
Diff:2
PageRef:758-759
Topic:WorldWarI
Skill:Factual

9) Asadirectconsequenceofthewar
A) womensparticipationinthelaborforceincreasedgreatly.
B) womensparticipationinthelaborforceincreasedonlyslightly.
C) womensparticipationinthelaborforcedecreasedsignificantly.
D) womensparticipationinthelaborforcestayedbasicallyatthesamelevels.
Answer: A
Diff:2
PageRef:760
Topic:WorldWarI
Skill:Factual

10) WhichofthefollowingcountriesdidNOThavecoloniesoutsideofEuropeattheoutbreakof
thewar?
A) France
B) Britain
C) Austria-Hungary
D) Italy
Answer: C
Diff:2
PageRef:760
Topic:WorldWarI
Skill:Factual

11) TheseawarfareduringWorldWarIconsistedlargelyof
A) majorsurfacebattlesbetweenthefleetsofBritainandGermany.
B) asinglemajorbattleinwhichtheGermanfleetdestroyedtheRussiannavy.
C) Germansubmarinewarfare.
D) theBritishattempttodestroytheRussianfleetintheMediterranean.
Answer: C
Diff:1
PageRef:761
Topic:WorldWarI
Skill:Factual

532

12) By1916,conflictontheWesternFront
A) hadbecomeashiftinggameofrapidmaneuverwithfewmajorbattles.
B) hadresultedinthesurrenderofFranceandtheestablishmentoftheVichygovernment.
C) hadresultedinvictoryfortheBritishandFrenchtroopswhopushedtheexhausted
enemytothebordersofGermany.
D) hadsettledintoadeadlystalemateinwhichhundredsofthousandsofliveswere
expendedforafewfeetoftrench.
Answer: D
Diff:2
PageRef:758
Topic:WorldWarI
Skill:Conceptual

13) OntheAustro-Hungarianfront,theprimarycombatantswereItalyand
A) Germany.
B) Russia.
C) Austria-Hungary.
D) France.
Answer: C
Diff:1
PageRef:759
Topic:WorldWarI
Skill:Factual

14) Between1914and1917,warfareontheEasternFront
A) pittedtheforcesofRussian,andAustria-HungaryagainsttheinvadingGermans.
B) includedpartsofRussiaandtheBalkans.
C) resultedintheSerbianknockoutoftheAustrianforces.
D) featuredbloodytrenchwarfareinwhichalmostnolandchangedhands.
Answer: B
Diff:2
PageRef:759
Topic:WorldWarI
Skill:Conceptual

533

15) WhichofthefollowingwasNOTafeatureofwaronthehomefrontbetween1914and1919?
A) Governmentsorganizedthemajorsectorsoftheeconomytorationresourcesand
production
B) Executivebranchesofgovernmentincreasinglytookoverfromparliaments
C) Governmentscontrolledpublicopinionthroughmanipulationofmassmediasuchas
newspapers
D) Strictgovernmentregulationpreventedmaterialshortagesandfamine
Answer: D
Diff:2
PageRef:759
Topic:WorldWarI
Skill:Conceptual

16) WhichofthefollowingstatementsconcerningtheglobalaspectsofWorldWarIismost
accurate?
A) TheBritishdominions,Canada,Australia,andNewZealand , remainedaloofand
virtuallyuntouchedbythewar.
B) By1914,theUnitedStateshadnotenteredthescrambleforcolonialpossessions.
C) Americanbusinessmenpriorto1917profitedbysellinggoodstobothsidesandby
takingadvantageofEuropeandistractionstoseizenewworldmarkets.
D) TheUnitedStatesaggressivelyenteredthewarin1914todemonstrateitsnewposition
asaworldpower.
Answer: C
Diff:2
PageRef:761
Topic:WorldWarI
Skill:Conceptual

17) WhatnationprofitedmostbywarfareinAsiabetween1914and1919?
A) Japan
B) China
C) NewZealand
D) TheUnitedStates
Answer: A
Diff:1
PageRef:761-762
Topic:WorldWarI
Skill:Factual

534

18) DuringWorldWarI,theOttomanEmpire
A) vainlyattemptedtoretainitsneutralityinwhattheTurksperceivedwasaChristian
conflict.
B) longattachedtoGermanmilitaryadvisors,joinedGermanyinthewareffort.
C) usedtheopportunitytoreassertTurkishdominanceovertheArabregions.
D) launchedassaultsindifferentlyagainstthecolonialpossessionsofBritain,France,and
Germany.
Answer: B
Diff:2
PageRef:761
Topic:WorldWarI
Skill:Conceptual

19) TheBritishpromisedsupportforaJewishsettlementintheMiddleEastinthe
A) SinaiResolution.
B) ExodusPact.
C) ChamberlainManifesto.
D) BalfourDeclaration.
Answer: D
Diff:2
PageRef:772
Topic:WorldWarI
Skill:Factual

20) By1917thewarontheEasternFront
A) hadstagnatedintoastalemateinwhichneithersidehadanadvantage.
B) ledtoamajorrevolutioninRussiathattoppledthetsaristgovernment.
C) wasresolvedbyapeacetreatybetweenthecombatants.
D) resultedinamassiveRussianoffensivefueledbythenumericalsuperiorityofRussian
armies.
Answer: B
Diff:2
PageRef:759
Topic:WorldWarI
Skill:Conceptual

535

21) InwhatyeardidtheGermanforcesontheEasternFrontsueforpeace?
A) 1916
B) 1917
C) 1918
D) 1919
Answer: C
Diff:2
PageRef:763
Topic:WorldWarI
Skill:Factual

22) Beforetheirsurrender,theGermangeneralsrunningthegovernment
A) installedanewciviliangovernmenttoshouldertheblameofdefeat.
B) murderedtheemperor.
C) issuedastatementacceptingblameforthepoliciesthathadledtoWorldWarI.
D) overthrewtheciviliangovernmentandestablishedamilitarydictatorship.
Answer: A
Diff:2
PageRef:763
Topic:WorldWarI
Skill:Conceptual

23) TheseriesoftreatiesthatendedWorldWarIwerenegotiatedat
A) Potsdam.
B) Brest-Litovsk.
C) London.
D) Versailles.
Answer: D
Diff:1
PageRef:764
Topic:WorldWarI
Skill:Factual

536

24) WhichofthefollowingwasNOTincludedinthefinalsetoftreatiesthatendedWorldWarI?
A) ALeagueofNationswasformed,buttheUnitedStatesrefusedtojoin.
B) RussiawasrewardedforitsservicetotheAlliesbythegrantofsubstantialterritoriesin
PolandandtheBalticrepublics.
C) Germanywasforcedtoacceptblameforthewarandtopayhugereparationstothe
victoriousAllies.
D) Austria-HungarywasdividedupintoaGermanicAustriaaswellindependentstatesof
Hungary,Czechoslovakia,andYugoslavia.
Answer: B
Diff:2
PageRef:764-765
Topic:WorldWarI
Skill:Conceptual

25) ApproximatelyhowmanypeoplediedasaresultofthecarnageofWorldWarI?
A) Onemillion
B) Twomillion
C) Fivemillion
D) 10million
Answer: D
Diff:1
PageRef:763
Topic:WorldWarI
Skill:Factual

26) AsaresultofitsparticipationinWorldWarI,theOttomanEmpire
A) effectivelycollapsed.
B) recovereditscontrolovertheArabregionsoftheempire,includingEgypt.
C) wasrewardedbythegrantofsubstantialterritoriesintheBalkans.
D) recoveredlandpreviouslylosttoRussia.
Answer: A
Diff:1
PageRef:771
Topic:WorldWarI
Skill:Conceptual

537

27) WhichofthefollowingwasNOTcarvedfromtheOttomanEmpireattheendofWorldWarI?
A) Turkishrepublic
B) Greece
C) BritishmandatesinPalestineandIraq
D) FrenchmandatesinSyriaandLebanon
Answer: B
Diff:2
PageRef:772
Topic:WorldWarI
Skill:Factual

28) WhatwastheDinshawaiincident?
A) ApartyofEgyptianguerillastoreupmuchoftheEgyptianrailwaysystem.
B) AgroupofBritishofficerswereimprisonedinagreatholeinKhartoum.
C) BritishofficershuntingpigeonsaccidentallyshotthewifeofaMuslimprayerleaderand
weresubsequentlyassaulted.
D) TheBritishtoredownanIslamicmosquetoconstructacathedral.
Answer: C
Diff:2
PageRef:770
Topic:WorldWarIandtheNationalistAssault
Skill:Factual

29) WhatwastheoutcomeoftheDinshawaiincidentinEgypt?
A) TheBritishwithdrewfromEgyptpriortoWorldWarI.
B) TheBritishwereforcedtosuppressarevoltledbytheMahdiintheSudan.
C) Masssupport,includingtheemergenceofayan allegiance,forthenationalcausegrewin
Egypt.
D) Islamicmullahsbegantopreachagainstthenationalistmovement.
Answer: C
Diff:2
PageRef:771
Topic:WorldWarIandtheNationalistAssault
Skill:Conceptual

30) By1913,theBritish
A) hadgrantedfullindependencetoEgypt.
B) hadeliminatedresistancetotheirregimeinEgypt.
C) grantedaconstitutiontoEgyptiannationalists.
D) hadwithdrawnfromEgypt.
Answer: C
Diff:2
PageRef:771
Topic:WorldWarIandtheNationalistAssault
Skill:Conceptual

538

31) PriortoWorldWarI,whatwastheprimarydifferencebetweenEgyptianandtheIndian
nationalistmovements?
A) InIndiaasinglepoliticalpartydominatedthenationalistmovement,butinEgypta
varietyofrivalpartiesproliferated.
B) TheEgyptiannationalistmovementwasdominatedbypeasantry,whileinIndia
Western-educatedlawyersranthemovement.
C) WhereasIndiahadbeendominatedbytheBritishsincethe18thcentury,Europeans
playednoroleinEgyptiancolonialism.
D) Egyptlackedaneducatedelitecapableofassumingleadershipofthenationalist
movement.
Answer: A
Diff:2
PageRef:770
Topic:WorldWarIandtheNationalistAssault
Skill:Conceptual

32) WhatgroupwithinEgypttookupthecauseofnationalindependenceanddecolonization?
A) Theayan
B) TheTurco-Egyptianpoliticalelite
C) Sonsoftheeffendi
D) Thepeasants
Answer: C
Diff:2
PageRef:770
Topic:WorldWarIandtheNationalistAssault
Skill:Factual

33) WhichofthefollowinggroupsdidNOTbenefitfromtheBritisheconomicreformsinEgypt?
A) Theayan
B) TheEgyptianbourgeoisie
C) Thepeasants
D) TheTurco-Egyptianpoliticalelite
Answer: C
Diff:2
PageRef:770
Topic:WorldWarIandtheNationalistAssault
Skill:Conceptual

539

34) WhatBritishpoliticalofficerdominatedgovernmentpolicyinEgyptafter1882andpushedfor
economicreformsintendedtoreducethekhedivaldebt?
A) LordAfton
B) LordCromer
C) LordBentinck
D) GeneralGordon
Answer: B
Diff:2
PageRef:770
Topic:WorldWarIandtheNationalistAssault
Skill:Factual

35) BritishoccupationofEgyptmeantdoublecolonization bytheBritishand


A) theFrench.
B) theArabs.
C) theGermans.
D) theTurks.
Answer: D
Diff:2
PageRef:770
Topic:WorldWarIandtheNationalistAssault
Skill:Factual

36) TheBritishfirstoccupiedEgyptfollowingtheAhmadOrabirevoltin
A) 1805.
B) 1867.
C) 1882.
D) 1914.
Answer: C
Diff:2
PageRef:769-770
Topic:WorldWarIandtheNationalistAssault
Skill:Factual

37) EgyptistheonecountryintheAfro-Asianworldinwhich
A) decolonizationoccurredinthe19thcentury.
B) decolonizationfailedtooccuruntilthe1980s.
C) EuropeancountriesfailedtoestablishacolonialbaseuntilafterWorldWarI.
D) theemergenceofnationalismprecededEuropeanconquestanddomination.
Answer: D
Diff:2
PageRef:769
Topic:WorldWarIandtheNationalistAssault
Skill:Conceptual

540

38) Whichofthefollowingstatementsconcerningtheleadershipofthedecolonizationmovement
inIndiajustpriortoWorldWarIismostcorrect?
A) LeadershipwasassumedbymoreradicalmembersoftheCongressPartysuchasTilak
justbefore1914.
B) TheCongressPartylostitsleadershiproletotheSocialistParty,whichwasmorewilling
tocourtthemassesoftheIndianpeasantry.
C) Tilaksremovalandtherepressioncampaignsagainstterrorists,alongwithBritish
reforms,strengthenedthehandsoftheWestern-educatedmoderatesinCongress.
D) Itisdifficult,ifnotimpossible,toidentifyleadershipinthefragmentedCongressPartyof
1914.
Answer: C
Diff:2
PageRef:768
Topic:WorldWarIandtheNationalistAssault
Skill:Conceptual

39) WhatBritishreformin1909providededucatedIndianswithexpandedopportunitiesbothto
electandtoserveonlocalandall-Indialegislativecouncils?
A) TheHalstead-Colemanreforms
B) TheCornwallisCommission
C) TheBentinckreforms
D) TheMorley-Mintoreforms
Answer: D
Diff:2
PageRef:768
Topic:WorldWarIandtheNationalistAssault
Skill:Factual

40) WherewastheterroristmovementmostconcentratedinIndiabeforeWorldWarI?
A) Deccanplateau
B) Punjab
C) Bengal
D) Bombay
Answer: C
Diff:3
PageRef:768
Topic:WorldWarIandtheNationalistAssault
Skill:Factual

541

41) WhowasthefirstIndianleaderwithagenuinemassfollowing?
A) J.Nehru
B) M.K.Gandhi
C) M.A.Jinnah
D) B.G.Tilak
Answer: D
Diff:2
PageRef:768
Topic:WorldWarIandtheNationalistAssault
Skill:Factual

42) AlargeportionofthegovernmentofIndiasbudgetwenttoallofthefollowingEXCEPT
A) highsalariesandpensionsofBritishadministrators.
B) thepurchaseofrailwayequipmentandsteelfromGreatBritain.
C) thehugeIndianarmy,oftenengagedoutsideofIndia.
D) statesupportforthedevelopmentofIndianindustries.
Answer: D
Diff:2
PageRef:768
Topic:WorldWarIandtheNationalistAssault
Skill:Conceptual

43) Bythelastyearsofthe19thcentury,theCongressPartywasappealingto
A) investorsandbusinessmen.
B) peasants.
C) laborers.
D) untouchables.
Answer: A
Diff:2
PageRef:767
Topic:WorldWarIandtheNationalistAssault
Skill:Factual

44) WhichofthefollowingstatementsconcerningthepositionoftheearlyCongressPartyismost
accurate?
A) ItwascommittedtorelievingthepovertyoftheIndianmasses
B) ItdemandedtheimmediatedecolonizationofIndia
C) ItwasloyaltotheBritishrulersandprimarilyconcernedwithinterestsoftheIndianelite
D) ItwascloselyinvolvedwiththedevelopmentofastronglabormovementwithinIndian
industry
Answer: C
Diff:2
PageRef:767
Topic:WorldWarIandtheNationalistAssault
Skill:Conceptual

542

45) In1885,regionalassociationsofWestern-educatedIndianscametogethertoformthe
A) IndianSocialistWorkersParty.
B) LiberalParty.
C) RevolutionaryNationalistParty.
D) IndianNationalCongressParty.
Answer: D
Diff:2
PageRef:767
Topic:WorldWarIandtheNationalistAssault
Skill:Factual

46) WhatnationalistmovementpioneeredpatternsofdecolonizationandEuropeanretreat?
A) Kenya
B) India
C) Vietnam
D) Algeria
Answer: B
Diff:1
PageRef:767
Topic:WorldWarIandtheNationalistAssault
Skill:Factual

47) Whatmadecolonialregimesparticularlyvulnerabletochallengesfromwithin?
A) Thegrowingindustrializationofcolonialsocieties
B) Thefactthatcolonialgovernmentswerebuiltincollaborationwithindigenouselite
groups
C) TheirdependenceonEuropeanmilitaryforces
D) ThedependenceofplantationeconomiesontheWest
Answer: B
Diff:2
PageRef:765
Topic:WorldWarIandtheNationalistAssault
Skill:Conceptual

543

48) WhichofthefollowingstatementsconcerningtheearlynationalistmovementsofAfricais
mostaccurate?
A) French-speakingwestAfricanstendedtoconcentratetheireffortsatpolitical
representationwithintheircolonies.
B) Bythemid-1920s,racistviewsofAfricansocietywerebecomingmorestrident,andmost
EuropeansrefusedtocountenancetheconceptofaprogressiveAfricanculture.
C) Pan-Africanism,linkingallAfricansinasinglenationalmovementforindependence,
wasthemostsuccessfulapparatusforachievingdecolonization.
D) Theearlyleadershipofpan-AfricanorganizationswasmoreoftenAmericanandWest
IndianthanAfrican.
Answer: D
Diff:3
PageRef:775
Topic:WorldWarIandthePostwarCrisesoftheEuropeanEmpires
Skill:Conceptual

EssayQuestions
1) DiscusstheaftermathofWorldWarIanditseffectonEuropeansociety.
Answer: Newterritorytosomecountriesandlossestoothers;collapseofempires;creationof
newstates;assignmentofblametoGermany;reparationstoBritainandFrance;pariah
statusforRussia;generalweakeningofEurope,deathsanddestructionofindustrial
property;somechangesinstructureofcolonialism.
Diff:2
PageRef:763-777
Topic:WorldWarI
Skill:Conceptual

2) HowdidWorldWarIaffectsocietiesoutsideofwesternEurope?
Answer: Africa:littleactualfighting,butparticipationinwarexposedcoloniestonationalism,
createdpan-Africanmovementin1919,ledtodecolonization.U.S.:emergedaswar
powerthroughintervention,gainedcentralroleinglobaltradenetworkthrough
creditorstatus.Japan:emergedasimperialpowerinAsia,notamajorfactorinpeace
negotiations,ledtodistrustofWest.India:likeAfricaexposedtoWesternnationalism,
independenceleaderssupportedwarinhopesoffreedomafter,frustratedbyWestern
failuretodecolonize.
Diff:2
PageRef:765-777
Topic:WorldWarI
Skill:Conceptual

3) Discussthechangesingenderrolesbroughtaboutbythewar.
Answer: Womensparticipationintheworkforceincreasedgreatly;betterwages;higher
confidence;broaderliberation;increasedpoliticalactivismandtherighttovoteinsome
countries.
Diff:2
PageRef:759-760
Topic:WorldWarI
Skill:Factual

544

4) DiscussGhandisapproachtotheremovaloftheBritishfromIndia.
Answer: Thenon-violentapproachofsatyagrahaencouragedpeacefulboycotts,strikes,
noncooperation,andmassdemonstrations
Diff:2
PageRef:769
Topic:WorldWarIandtheNationalistAssault
Skill:Conceptual

5) DiscussZionismandhowWorldWarIhelpedmakepossiblethecreationofaJewish
homeland.
Answer: ZionismmovementforJewishhomeland,in1917BritishforeignsecretaryLordBalfour
promisedzionistsahomelandinPalestineafterthewar,WorldWarIdestroyedthe
OttomanEmpireandallowedincreasedJewishmigrationtoPalestine.
Diff:2
PageRef:772
Topic:WorldWarIandtheNationalistAssault
Skill:Conceptual

545

Chapter34 TheWorldBetweenWars:Revolutions,
DepressionsandAuthoritarianResponse
Multiple-ChoiceQuestions
1) Themid-1920sinwesternEuropecouldbestbedescribedasaperiodof
A) waranddestruction.
B) stabilityandoptimism.
C) depressionandunemployment.
D) internalpoliticalunanimity.
Answer: B
Diff:2
PageRef:781-782
Topic:TheGlobalGreatDepression
Skill:Conceptual

2) WhendidthecommunistrevolutioninRussiatakeplace?
A) 1917
B) 1918
C) 1919
D) 1920
Answer: A
Diff:2
PageRef:780
Topic:TheGlobalGreatDepression
Skill:Factual

3) TheU.S.economy
A) wasinseveredepressionthroughoutthe1920s.
B) sufferedbecausecorporationsrefusedtoinnovate.
C) boomedthroughoutmuchofthe1920s.
D) boomedonceHenryFordintroducedelectricity
Answer: C
Diff:2
PageRef:783
Topic:TheGlobalGreatDepression
Skill:Conceptual

546

4) WhichofthefollowingstatementsconcerningtheeconomyofEuropeduringtheRoaring
20sisNOTaccurate?
A) Theworstinflationarypressureswereresolved,thoughatthecostofwipingoutthe
valueofsavingsformanypropertiedgroups.
B) Industrialproductionboomed.
C) Massconsumptionstandards,basedonreadyavailabilityofnewproducts,rose.
D) TheUnitedStateslaggedbehindtheEuropeannationsinbothindustrialproductionand
massconsumption.
Answer: D
Diff:2
PageRef:781-783
Topic:TheGlobalGreatDepression
Skill:Conceptual

5) WhatcountrydidBenitoMussolinirule?
A) Britain
B) theU.S.
C) Italy
D) Germany
Answer: C
Diff:3
PageRef:785
Topic:TheGlobalGreatDepression
Skill:Factual

6) WhichofthefollowingstatementsmostaccuratelysummarizesthestateofMexicanpolitical
organizationaftertherevolution?
A) Amultitudeofpoliticalpartiessoconfusedtheelectoralscenethatnomeaningful
politicalcompromisecouldbeachieved.
B) Mexicowasimmediatelydominatedbylaborpartieswhooriginatedinthenorthern
partsofMexico.
C) Therevolutionaryleadershipinstitutionalizedtheregimebycreatingaone-partysystem
underPRI.
D) Presidentsruledmuchasthecaudillosbeforethemwithoutlimitationofpowersorterm
ofoffice.
Answer: C
Diff:3
PageRef:789
Topic:Revolution:TheFirstWaves
Skill:Conceptual

547

7) WhoweretheCristeros?
A) MarxistrevolutionarieswithintheMexicanlaborforce
B) RadicalswhosoughttooverthrowtheinfluenceofCatholicisminMexico
C) Conservativepeasantsbackedbythechurchwhofoughttostoptheslidetoward
secularism
D) Membersoftherevolutionaryelitewhodesiredtheestablishmentofaparliamentary
democracy
Answer: C
Diff:2
PageRef:789
Topic:Revolution:TheFirstWaves
Skill:Factual

8) Whichofthefollowingwasincludedinthepolicyofindigenismthatwasincorporatedinto
manyofthepost-revolutionaryreformsinMexico?
A) TheremovalofallevidenceofSpanishheritage
B) TheMexicanmuralistmovementfeaturingworksofDiegoRiveraandJoseClemente
Orozco
C) ActiveattemptstoEuropeanizeMexicanculture
D) IntentionalemulationofSovietsocialistrealism
Answer: B
Diff:2
PageRef:788
Topic:Revolution:TheFirstWaves
Skill:Conceptual

9) WhichofthefollowingreformswasNOTincludedintherevolutionaryconstitutionof1917in
Mexico?
A) StatetakeoverofpropertybelongingtotheCatholicchurch
B) Limitedforeignownershipofkeyresources
C) Landreform
D) Guaranteedrightsforworkers
Answer: A
Diff:3
PageRef:788
Topic:Revolution:TheFirstWaves
Skill:Conceptual

548

10) ThecommunalholdingscreatedaspartoftheprogramoflandreformfollowingtheMexican
Revolutionwerecalled
A) playas.
B) tierralibertad.
C) lascasas.
D) ejidos.
Answer: D
Diff:2
PageRef:788
Topic:Revolution:TheFirstWaves
Skill:Factual

11) In1920whowaselected,bringingtheMexicanRevolutiontoaclose?
A) EmilianoZapata
B) VictorianoHuerta
C) AlvaroObregon
D) FranciscoMadero
Answer: C
Diff:2
PageRef:788
Topic:Revolution:TheFirstWaves
Skill:Factual

12) WhatwastheprimarygoalofZapatasforceswithintheMexicanRevolution?
A) ThepresidencyforZapata
B) Industrialization
C) ExtensionoftheplantationeconomyinOaxaca
D) Sweepinglandreform
Answer: D
Diff:2
PageRef:788
Topic:Revolution:TheFirstWaves
Skill:Conceptual

13) WhichofthefollowingpoliticalandmilitaryleaderswasNOTinvolvedintheMexican
Revolutionin1910?
A) JuanJoseArevalo
B) FranciscoMadero
C) EmilianoZapata
D) PanchoVilla
Answer: A
Diff:2
PageRef:788
Topic:Revolution:TheFirstWaves
Skill:Factual

549

14) WhatpoliticianfirstchallengedtheMexicangovernmentin1910?
A) FranciscoMadero
B) AugustinIturbide
C) BenitoJuarez
D) PorfirioDiaz
Answer: A
Diff:2
PageRef:788
Topic:Revolution:TheFirstWaves
Skill:Factual

15) ThemanwhohadbeeninchargeoftheMexicangovernmentsince1876atthetimeofthe
MexicanRevolutionwas
A) FranciscoMadero.
B) AugustinIturbide.
C) BenitoJuarez.
D) PorfirioDiaz.
Answer: D
Diff:2
PageRef:787
Topic:Revolution:TheFirstWaves
Skill:Factual

16) TheMexicanRevolutionbeganin
A) 1901.
B) 1910.
C) 1914.
D) 1919.
Answer: B
Diff:2
PageRef:788
Topic:Revolution:TheFirstWaves
Skill:Factual

17) AttheendofWorldWarI,whatnationemergedasthedominantforeignpowerinLatin
America?
A) Britain
B) Germany
C) TheUnitedStates
D) Spain
Answer: C
Diff:1
PageRef:787
Topic:Revolution:TheFirstWaves
Skill:Factual

550

18) TwoeventssetinmotiontrendsthatwoulddeterminemuchofLatinAmericassubsequent
history:WorldWarIand
A) WorldWarII.
B) theMexicanRevolution.
C) thePero nera.
D) thecommunistrevolutioninCuba.
Answer: B
Diff:1
PageRef:787
Topic:Revolution:TheFirstWaves
Skill:Factual

19) ThelastManchuemperorabdicatedin
A) 1912.
B) 1923.
C) 1939.
D) 1945.
Answer: A
Diff:2
PageRef:793
Topic:Revolution:TheFirstWaves
Skill:Factual

20) Thebest-positionedcontendersforpowerintheaftermathoftheabdicationoftheQingwere
the
A) merchantsandbankersofcoastalcities.
B) warlordsormilitarycommanders.
C) universitystudents.
D) secretsocieties.
Answer: B
Diff:2
PageRef:793
Topic:Revolution:TheFirstWave
Skill:Factual

551

21) WhatforeignpowerplayedamajorfactorinChinesedomestichistoryfromthemid-1890sto
1945?
A) TheUnitedStates
B) Vietnam
C) Britain
D) Japan
Answer: D
Diff:1
PageRef:794
Topic:Revolution:TheFirstWave
Skill:Factual

22) TheleaderoftheRevolutionaryAlliance,aloosecoalitionofanti-Qingpoliticalgroupsthat
spearheadedthe1911revolt,was
A) YuanShikai.
B) ChiangKai-shek.
C) SunYat-sen.
D) MaoZedong.
Answer: C
Diff:1
PageRef:794
Topic:Revolution:TheFirstWave
Skill:Factual

23) WhathappenedtotheparliamentarygovernmentestablishedinChinabytheRevolutionary
Alliancein1912?
A) ItwasoverthrownbyaCommunistinsurrection.
B) SunYat-senresignedthepresidencyinfavorofthenorthernwarlord,YuanShikai.
C) TheparliamentarygovernmentunderthepresidencyofSunYat-Seneffectively
governednorthernChinafromthecapitalatBeijingfor20years.
D) TheparliamentarygovernmentwasoverthrownbytheBoxerrebellion.
Answer: B
Diff:2
PageRef:794
Topic:Revolution:TheFirstWave
Skill:Conceptual

552

24) JapangainedinfluenceinChinabyseizingtheconcessionaryareasofwhatEuropeanpower
duringWorldWarI?
A) Britain
B) TheUnitedStates
C) Russia
D) Germany
Answer: D
Diff:1
PageRef:794
Topic:Revolution:TheFirstWave
Skill:Factual

25) TheinitialgoaloftheMayFourthmovementwas
A) Communistinsurrection.
B) riddingChinaofallWesterninfluences.
C) thecreationofaliberaldemocracyalongWesternlines.
D) ChinesesubjectiontoJapan.
Answer: C
Diff:2
PageRef:795
Topic:Revolution:TheFirstWave
Skill:Conceptual

26) MostofthemembersoftheearlyMayFourthmovementwere
A) warlords.
B) peasants.
C) Communists.
D) intellectualsandstudents.
Answer: D
Diff:2
PageRef:794-795
Topic:Revolution:TheFirstWave
Skill:Factual

27) WhatmadeChinesethinkersawareofapossibleMarxistsolutiontoChinasills?
A) TheRussianrevolutionof1905
B) TheRussianrevolutionof1917
C) FrederickEngelssvisittoChinain1912
D) TheJapaneseexample
Answer: B
Diff:1
PageRef:795
Topic:Revolution:TheFirstWave
Skill:Conceptual

553

28) ThemanwhoheadedthestudycircleattheUniversityofBeijingandwhofirstgaveserious
attentiontoMarxistphilosophywas
A) ChiangKai-shek.
B) LiBo.
C) LiDazhao.
D) YuanShikai.
Answer: C
Diff:2
PageRef:795
Topic:Revolution:TheFirstWave
Skill:Factual

29) HowdidearlyChineseMarxistphilosophydifferfromLenins?
A) Chinesephilosophersemphasizedtheroleoftheproletariatintherevolution.
B) Chinesethinkersstressedthegradualistapproachtopoliticalchange.
C) SunYat-sentaughtthattherevolutioncouldonlyoccurafterthecomplete
industrializationofChina.
D) ThestudycircleattheUniversityofBeijingsawthepeasantsasthevanguardof
revolution.
Answer: D
Diff:2
PageRef:795
Topic:Revolution:TheFirstWave
Skill:Conceptual

30) EarlyMarxistphilosophersinChinacharacterizedallofChinassocietyas
A) proletarian.
B) peasant.
C) bourgeois.
D) Westernized.
Answer: A
Diff:2
PageRef:795
Topic:Revolution:TheFirstWave
Skill:Conceptual

554

31) WhichofthefollowingorganizationsandpublicationswasNOTassociatedwiththeearly
MarxistmovementinChina?
A) NewYouth
B) SocialistYouthCorps
C) CommunistYouthCorps
D) Guomindang
Answer: D
Diff:2
PageRef:795-796
Topic:Revolution:TheFirstWave
Skill:Factual

32) In1919,SunYat-senreturnedtoChinaandattemptedtounifythevariouspolitical
organizationsstrugglingforpoliticalinfluenceinChinabyreorganizingtherevolutionary
movementunderwhatname?
A) TheSocialistYouthCorps
B) TheMayFourthmovement
C) TheRedArmy
D) TheNationalistPartyofChina
Answer: D
Diff:1
PageRef:796
Topic:Revolution:TheFirstWave
Skill:Factual

33) WhichofthefollowinggroupswasNOTalliedwiththeNationalistPartyby1924?
A) TheUnitedStates
B) Urbanbusinessmenandmerchantsofthesoutherncoastalcities
C) TheCommunists
D) Localwarlordsandthecriminalunderworld
Answer: A
Diff:2
PageRef:796
Topic:Revolution:TheFirstWave
Skill:Conceptual

555

34) WhohelpedfindtheWhampoaMilitaryAcademyin1924?
A) theUnitedStates.
B) Britain.
C) SovietRussia.
D) Japan.
Answer: C
Diff:2
PageRef:796
Topic:Revolution:TheFirstWave
Skill:Factual

35) WhatwasChiangKai-sheksinitialrolewithintheChineseNationalistParty?
A) DelegatetotheLeagueofNations
B) RepresentativeattheTreatyofVersailles
C) FamilyheadoftheGreenGang
D) HeadoftheWhampoaMilitaryAcademy
Answer: D
Diff:2
PageRef:796
Topic:Revolution:TheFirstWave
Skill:Factual

36) TheNationalistPartysgreatestfailurewasitsinabilityto
A) createamilitarywingoftheparty.
B) dealwiththesocialandeconomiccrisisoftheChinesepeasantry.
C) industrializeChina.
D) riditselfofitstiestotheQingdynasty.
Answer: B
Diff:2
PageRef:796-797
Topic:Revolution:TheFirstWave
Skill:Factual

37) WhatearlyCommunistleaderemphasizedtheroleofthepeasantryinrevolutionarysolutions
toChineseproblems?
A) SunYat-sen
B) ChiangKai-shek
C) MaoZedong
D) YuanShikai
Answer: C
Diff:2
PageRef:797
Topic:Revolution:TheFirstWave
Skill:Factual

556

38) FollowingSunYat-sensdeathin1925,whatgroupseizedcontroloftheNationalistParty?
A) UrbanbusinessmenandmerchantsofthesouthernChinesecoast
B) Communists
C) Warlordsandmilitarychiefs
D) Peasants
Answer: C
Diff:2
PageRef:797
Topic:Revolution:TheFirstWave
Skill:Factual

39) WhoemergedastheheadoftheNationalistPartyanddefactorulerofChinabythelate
1920s?
A) SunYat-sen
B) MaoZedong
C) ChiangKai-shek
D) LiDizhao
Answer: C
Diff:2
PageRef:797
Topic:Revolution:TheFirstWave
Skill:Factual

40) OpenwarfarebeganbetweentheCommunistsandNationalistPartyfollowing
A) theCommunistassaultonBeijing.
B) theinterventionofJapanonthesideoftheNationalists.
C) theannouncementofSovietRussiansupportfortheNationalistcause.
D) theNationalistslaughteroftheirformeralliesinShanghai.
Answer: D
Diff:2
PageRef:797
Topic:Revolution:TheFirstWave
Skill:Conceptual

41) WhichofthefollowinggroupsdidNOTinitiallysupporttheNationalistsintheircontestwith
theCommunistsforcontrolofChina?
A) Urbanbusinessmenandmerchants
B) Urbanworkers
C) Intellectualsandstudents
D) Rurallandlords
Answer: B
Diff:3
PageRef:797
Topic:Revolution:TheFirstWave
Skill:Conceptual

557

42) WhatimpactdidtheNationalistsmashingoftheworkersmovementhaveonthe
developmentoftheCommunistPartyinChina?
A) ItdestroyedtheearlyCommunistmovement,leavingonlysmallcellsofintellectualsand
students.
B) TheslaughteroftheworkersinShanghaihadlittleimpact,becausethegreat
concentrationofworkerswasinBeijing.
C) ItpropelledMaotoleadership.
D) ItforcedtheCommuniststobecomeincreasinglydependentontheSovietUnion.
Answer: C
Diff:2
PageRef:797
Topic:Revolution:TheFirstWave
Skill:Conceptual

43) TheLongMarchrefersto
A) thehumiliatingexportationofChineseprisonerstoJapan.
B) themarchof90,000communistsfromHunantoShaanxiprovince.
C) theNationalistPartyprogressionfromminormovementtoascendancyinChina.
D) ChiangKai-sheksassaultonthecommuniststrongholdinBeijing.
Answer: B
Diff:2
PageRef:797
Topic:Revolution:TheFirstWave
Skill:Factual

44) ThecenteroftheCommunistPartyinChinafrom1934tothemid -1940swas


A) Shaanxiprovince.
B) Hunanprovince.
C) Beijing.
D) Macao.
Answer: A
Diff:2
PageRef:797
Topic:Revolution:TheFirstWave
Skill:Factual

558

45) WhatinterruptedChiangKai-sheksdestructionoftheChineseCommunistPartyinthelate
1930s?
A) Russianintervention
B) ThedeathofSunYat-sen
C) JapaneseinvasionoftheChinesemainland
D) TheinterventionoftheUnitedStates
Answer: C
Diff:1
PageRef:797
Topic:Revolution:TheFirstWave
Skill:Conceptual

46) InwhatcitydidtheRussianrevolutionof1917begin?
A) St.Petersburg
B) Moscow
C) Novgorod
D) Odessa
Answer: A
Diff:1
PageRef:789
Topic:Revolution:TheFirstWave
Skill:Factual

47) The1917revolutionsawcitiesrunbycouncilsofworkersor
A) glasnost.
B) soviets.
C) commissars.
D) kulaks.
Answer: B
Diff:1
PageRef:789
Topic:Revolution:TheFirstWave
Skill:Factual

48) IntheeightmonthsfollowingthebeginningoftheRussianrevolutioninMarch1917,what
termcouldbestbeusedtodescribetherevolutionarygovernment?
A) Liberal
B) Anarchist
C) Communist
D) Conservative
Answer: A
Diff:2
PageRef:789
Topic:Revolution:TheFirstWave
Skill:Conceptual

559

49) WhowastheleaderoftheRussiangovernmentaftertheMarchRevolution?
A) JosephStalin
B) Bakunin
C) AlexanderKerensky
D) NicholasRomanov
Answer: C
Diff:1
PageRef:789
Topic:Revolution:TheFirstWave
Skill:Factual

50) WhatgroupsucceededtheinitialrevolutionarygovernmentofRussia?
A) Bakuninsanarchists
B) Kerenskysliberals
C) LeninsBolsheviks
D) MolotovsMensheviks
Answer: C
Diff:2
PageRef:790
Topic:Revolution:TheFirstWave
Skill:Factual

51) WhatwasLeninssolutiontoRussianparticipationinWorldWarI?
A) HelaunchedamassiveoffensivecampaignthatcarriedRussianforcesdeepwithin
Germany.
B) HeimmediatelydemandedthathisBritishandFrenchalliessendhumanitarian,
economic,andmilitaryaidtotheeasternfront.
C) HenegotiatedapeacetreatywiththeGermansandsurrenderedhugeamountsofland
onRussiaswesternborder.
D) HesuccessfullyachievedasignificantroleattheVersaillespeacenegotiations.
Answer: C
Diff:2
PageRef:790
Topic:Revolution:TheFirstWave
Skill:Conceptual

560

52) Leninspowerin1917wasbasedonhiscontrolofthe
A) Duma.
B) zemstvos.
C) tsar.
D) CouncilofPeoplesCommissars.
Answer: D
Diff:2
PageRef:790
Topic:Revolution:TheFirstWave
Skill:Factual

53) WhichofthefollowingstatementsconcerningthepoliticalsituationinRussiaafterLenins
seizureofpowerismostaccurate?
A) LeninhopedtoestablishamajoritypartybasedalmostentirelyontheRussian
peasantry.
B) Thefirstparliamentaryelectionfollowingtherevolutionreturnedamajorityforthe
SocialRevolutionaryParty,notLeninsBolsheviks.
C) Leninremainedcompletelydedicatedtotheprinciplesofparliamentarygovernment,a
positionthatgarneredhimthesupportofRussianliberals.
D) Leninabandonedpowerinthefaceofapopularplebisciteforanotherformof
government.
Answer: B
Diff:2
PageRef:790
Topic:Revolution:TheFirstWave
Skill:Conceptual

54) WhatresponsedidtheRussianrevolutionprovokeelsewhereinEurope?
A) Britain,France,theU.S.,andJapanallinvadedRussia.
B) RussiawasimmediatelyalliedwithBritain,France,andtheU.S.againstcontinued
Germanaggression.
C) Francesupportedtherevolution;BritainandtheU.S.remainedneutral.
D) None
Answer: A
Diff:2
PageRef:791
Topic:Revolution:TheFirstWave
Skill:Conceptual

561

55) ThefirstheadoftheRedArmywas
A) Lenin.
B) Bakunin.
C) Trotsky.
D) Molotov.
Answer: C
Diff:2
PageRef:791
Topic:Revolution:TheFirstWave
Skill:Factual

56) LeninsfirstattemptatimprovingtheRussianeconomyin1921wascalledthe
A) FirstFiveYearPlan.
B) NewEconomicPolicy.
C) MarchonHunger.
D) ACT.
Answer: B
Diff:2
PageRef:791
Topic:Revolution:TheFirstWave
Skill:Factual

57) WhichofthefollowingstatementsconcerningtheSupremeSovietismostaccurate?
A) ItwaselectedbyatinyminorityoftheRussianpopulation
B) Itfeaturedsomanypoliticalpartiesthatitwasvirtuallypowerless
C) Competitioninelectionswasnormallyprohibited,whichmeantthattheCommunist
Partyeasilycontrolledthebody
D) Italmostnevermet
Answer: C
Diff:2
PageRef:791
Topic:Revolution:TheFirstWave
Skill:Conceptual

562

58) HowdidStalinsviewofcommunismdifferfromthatofLenin?
A) LeninwasonlyinterestedintheRussianrevolutionanddidnotvisualizeanyfurther
revolutionaryprocess.
B) LeninwasmoreinterestedinincludingabroadswathoftheRussianpopulationinthe
Communistmovement.
C) StalinrepresentedastronglynationalistversionofCommunismandconcentratedon
socialisminonecountry.
D) StalinwasnotamemberoftheCommunistParty.
Answer: C
Diff:2
PageRef:793
Topic:Revolution:TheFirstWave
Skill:Conceptual

59) TheinternationalofficeestablishedtoencourageCommunistpartiesintheWestwascalled
A) WesternInternational.
B) Comintern.
C) Leninistainternationale.
D) Eurored.
Answer: B
Diff:1
PageRef:793
Topic:Revolution:TheFirstWave
Skill:Factual

EssayQuestions
1) DiscussthetransitioninrevolutionaryRussiafromliberalismtocommunism.
Answer: In1917developmentofliberalgovernmentunderKerensky;attemptedtoestablish
parliamentarydemocracy,butlackedsubstantialmiddleclass;secondrevolution
broughtworkersgroupsandBolshevikstopowerunderLenin;becauseoflackof
popularmajority,forcedtoshutdownparliament,establishsingle-partyrule;
constructedpowerfulnewarmyunderpoliticalleadership;establishedstatecontrolof
economicpolicieswithNEP;createdUnionofSovietSocialistRepublicswith
governmentsdominatedbysmallcadreofCommunists;pseudo-parliament,the
SupremeSoviet,created;controlledbyparty.
Diff:2
PageRef:789-791
Topic:Revolution:TheFirstWave
Skill:Conceptual

563

2) Whatarethecharacteristicsof20thcenturyrevolutions?
Answer: Occurinsocietieswherechanges,industrialorpolitical,arealreadyunderway;involve
priorintellectualbuildup,liberalism,nationalism,religiousfundamentalism,or
socialism;theeventualdeclineofliberalismintherevolutionarymovement;theneedto
cometotermswithWesterninfluence.
Diff:2
PageRef:792
Topic:Revolution:TheFirstWave
Skill:Conceptual

3) InwhatwaysdidStalinismaltertheoriginalconceptsofSovieteconomyandgovernment?
Answer: Stalinlessorientedtogoalsofinternationalcommunism,moretiedtosocialisminone
country,moredirectinterventionineconomy,collectivizationofagriculture,five-year
plansforindustrialdevelopment;moreruthlesseliminationofpoliticaloreconomic
enemies;createdtotalitariangovernment,secretpolice,purgesofenemies,totalcontrol
ofbureaucracy.
Diff:2
PageRef:789-793
Topic:Revolution:TheFirstWaves
Skill:Conceptual

564

Chapter35 ASecondGlobalConflictandtheEndofthe
EuropeanGlobalOrder
Multiple-ChoiceQuestions
1) WorldWarIIofficiallybeganinwhatyear?
A) 1940
B) 1941
C) 1939
D) 1935
Answer: C
Diff:2
PageRef:818
Topic:WorldWarII
Skill:Factual

2) WhichofthefollowingcountrieswasNOTamemberoftheAxispowers?
A) TheSovietUnion
B) Germany
C) Italy
D) Japan
Answer: A
Diff:2
PageRef:818
Topic:WorldWarII
Skill:Factual

3) After1937,thegovernmentofJapanwasdominatedby
A) socialistswhogainedpowerintheaftermathofthedepression.
B) laborunionswhosepositionwasstrengthenedbytheircontrolofindustry.
C) theemergingestateofmiddle-classliberalsintentonabroaderfranchise.
D) amilitaryregimededicatedtotheultra-nationalistgoals.
Answer: D
Diff:2
PageRef:817
Topic:WorldWarII
Skill:Factual

565

4) In1931,theJapanesearmymarchedinto__________anddeclareditanindependentstate.
A) Korea
B) Vietnam
C) thePhilippines
D) Manchuria
Answer: D
Diff:2
PageRef:817
Topic:WorldWarII
Skill:Factual

5) AdolphHitlerwasthepoliticalandideologicalleaderofthe
A) SocialDemocraticParty.
B) NationalSocialistParty.
C) ChristianDemocraticParty.
D) ConservativeUnion.
Answer: B
Diff:1
PageRef:817
Topic:WorldWarII
Skill:Factual

6) HitlerpromisedtheGermanpeopleallofthefollowingEXCEPT
A) toputGermansbacktowork.
B) toridGermanyoftheJewishrace.
C) torestorepoliticalstability
D) toremilitarizeGermany.
Answer: B
Diff:1
PageRef:817
Topic:WorldWarII
Skill:Factual

7) InwhichyeardidGermanyrearm?
A) 1935
B) 1936
C) 1937
D) 1938
Answer: A
Diff:2
PageRef:885
Topic:WorldWarII
Skill:Conceptual

566

8) WhowastheleaderoffascistItaly?
A) BenedettoCroce
B) ErnestoMomigliano
C) BenitoMussolini
D) BenitoJuarez
Answer: C
Diff:1
PageRef:817
Topic:WorldWarII
Skill:Factual

9) WhichofthefollowingcountriescontributedtoSpainsrepublicansduringtheSpanishCivil
War?
A) Mexico
B) TheSovietUnion
C) Italy
D) Germany
Answer: B
Diff:1
PageRef:817
Topic:WorldWarII
Skill:Factual

10) ThiscitybecamethecapitaloftheChineseNationalistsformuchofWWII.
A) Chongqing
B) Taipei
C) Nanjing
D) Cuomintang
Answer: A
Diff:1
PageRef:820
Topic:WorldWarII
Skill:Factual

11) TheTripartitePact,signedin1940,includedallofthefollowingcountriesEXCEPT
A) Russia
B) Japan
C) Germany
D) Italy
Answer: A
Diff:1
PageRef:820
Topic:WorldWarII
Skill:Factual

567

12) Inordertoavoidatwo-frontwar,Hitlersignedanonaggressionpactwiththiscountryin
1939.
A) Spain
B) France
C) Italy
D) TheSovietUnion
Answer: D
Diff:1
PageRef:820
Topic:WorldWarII
Skill:Factual

13) TheNaziinvasionofthiscountryin1939putanendtotheideaofappeasement.
A) Poland
B) Czechoslovakia
C) France
D) Sweden
Answer: A
Diff:1
PageRef:820
Topic:WorldWarII
Skill:Factual

14) Germanyswareffortwasbasedontheconceptof
A) blitzkrieg.
B) horse-mountedinfantry.
C) trenchwarfare.
D) Anschloss.
Answer: A
Diff:1
PageRef:821
Topic:WorldWarII
Skill:Factual

15) In1940,theDutchportof__________wasvirtuallyleveledbytheGermans,killingover40,000
civilians.
A) Flanders
B) Helsinki
C) Rotterdam
D) Brisbane
Answer: C
Diff:1
PageRef:822
Topic:WorldWarII
Skill:Factual

568

16) WhichofthefollowingdidNOTcontributetothefallofFrancetotheGermans?
A) TherefusalofEnglishtroopstofightwiththeFrench
B) Dividedandweakleadership
C) DelaysoverrearmingtheFrenchforces
D) AthoroughlydemoralizedFrenchpopulation
Answer: A
Diff:1
PageRef:822
Topic:WorldWarII
Skill:Factual

17) ThiscitywasthecapitaloftheNazispuppetregimeinFrance.
A) Toulose
B) Vichy
C) Paris
D) St.Gabrielle
Answer: B
Diff:1
PageRef:822
Topic:WorldWarII
Skill:Factual

18) BritishvictoryintheBattleofBritainwasduetoallofthefollowingEXCEPT
A) thebraveryoftheroyalfamily.
B) thehighmoraleofthecitizens.
C) radar.
D) theV2rocket.
Answer: D
Diff:1
PageRef:822
Topic:WorldWarII
Skill:Factual

19) AllofthefollowinggroupsweretargetedbyHitlerinthemassexecutionsthatwouldbecome
knownasthefinalsolutionEXCEPT
A) musicians.
B) homosexuals.
C) Gypsies.
D) Jews.
Answer: A
Diff:1
PageRef:823
Topic:WorldWarII
Skill:Factual

569

20) Atthisconferencein1942,theNazihighcommandfinalizedplansforthedestructionofthe
Jewishrace.
A) Krakow
B) Auschwitz
C) Copenhagen
D) Wannsee
Answer: D
Diff:1
PageRef:823
Topic:WorldWarII
Skill:Factual

21) ApproximatelyhowmanypeoplewerekilledbytheNazisduringtheHolocaust?
A) 12million
B) Twomillion
C) Eightmillion
D) 20million
Answer: A
Diff:1
PageRef:823
Topic:WorldWarII
Skill:Factual

22) A1944AlliedlandinginthiscountrycreatedaEuropeanfrontagainsttheGermans.
A) Belgium
B) France
C) Sicily
D) Spain
Answer: B
Diff:1
PageRef:824
Topic:WorldWarII
Skill:Factual

23) HItlerslast-ditchefforttorepeltheAlliedarmiesbecameknownas
A) theBattleofParis.
B) theBattleoftheArdennes.
C) theBattleoftheBulge.
D) theBattleofBritain.
Answer: C
Diff:1
PageRef:824
Topic:WorldWarII
Skill:Factual

570

24) InwhatyeardidtheJapaneseattackPearlHarbor,thusbringingtheUnitedStatesintoWorld
WarII?
A) 1939
B) 1941
C) 1943
D) 1945
Answer: B
Diff:1
PageRef:824
Topic:WorldWarII
Skill:Factual

25) ThiscountrychoseapathofneutralityandcooperationwithJapaninthePacifictheaterof
WWII.
A) Australia
B) ThePhilippines
C) Indonesia
D) Thailand
Answer: D
Diff:1
PageRef:824-825
Topic:WorldWarII
Skill:Factual

26) Universallyrecognizedasthegreatestnavalbattleinhistory,theJapanesefleetwaseffectively
putoutofcommissionat
A) Midway
B) Tarawa
C) CoralSea
D) IwoJima
Answer: A
Diff:1
PageRef:825
Topic:WorldWarII
Skill:Factual

571

27) ThisAmericangeneralorderedmassaerialbombingofhighlyvulnerableJapanesecities.
A) McArthur
B) Patton
C) LeMay
D) Eisenhower
Answer: C
Diff:1
PageRef:826
Topic:WorldWarII
Skill:Factual

28) WhichofthefollowingstatementsconcerningwarfareintheEuropeantheaterduringWorld
WarIIismostaccurate?
A) Francemountedafanaticdefenseofitshometerritories,onlysuccumbingtotheNazi
advancein1944.
B) Bythesummerof1940,mostofFrancelayinGermanhands,whileasemi -fascist
collaborativeregimeruledinVichy.
C) BritishresistancecrumbledbeforetheairassaultsofGermany,andanamphibious
assaultknockedtheBritishfromthewar.
D) From1939on,thechiefresistancetotheGermanadvancewasprovidedbyAmerican
forces.
Answer: B
Diff:2
PageRef:822
Topic:WorldWarII
Skill:Conceptual

29) ThebalanceofthewarinEuropeshiftedin1941whenGermanyinvaded
A) France.
B) Britain.
C) Italy.
D) theSovietUnion.
Answer: D
Diff:1
PageRef:823
Topic:WorldWarII
Skill:Factual

572

30) JapanssurrenderinthePacificwasprecipitatedby
A) theuseofatomicweaponsonthecitiesofNagasakiandHiroshimabytheU.S.
B) amassivelandandseaassaultontheJapanesehomeislands.
C) thelossofthePhilippinestotheU.S.
D) theBritishadvancethroughMalayaintoChina.
Answer: A
Diff:1
PageRef:826-827
Topic:WorldWarII
Skill:Factual

31) Allofthefollowingwereconferencesheldamongthealliedpowerstodeterminethefateof
EuropeafterthedefeatofGermanyEXCEPT
A) Potsdam.
B) Yalta.
C) Sarajevo.
D) Teheran.
Answer: C
Diff:2
PageRef:826
Topic:WorldWarII
Skill:Factual

32) WhichofthefollowingwasNOTaresultofthepeacetreatiessignedfollowingWorldWarII?
A) TheUnitedStatesoccupiedJapan
B) Germanywasdividedintofourzonesofoccupation
C) TheSovietUniontookmuchofeasternPoland,whilethePoleswerecompensatedby
receivingpartofeasternGermany
D) Germanindustrialpowerwasdestroyed.
Answer: D
Diff:2
PageRef:829
Topic:WorldWarII
Skill:Conceptual

33) ThisinstitutionwascreatedasaresultofWWII.
A) TheLeagueofNations
B) TheUnitedNations
C) TheWorldBank
D) TheInternationalMonetaryFund
Answer: B
Diff:2
PageRef:828
Topic:WorldWarII
Skill:Factual

573

34) WhichofthefollowingcountrieswasNOTachartermemberoftheUN?
A) China
B) TheSovietUnion
C) Britain
D) France
Answer: A
Diff:2
PageRef:828
Topic:WorldWarII
Skill:Factual

35) WhichofthefollowingstatementsconcerningthecreationofthestateofIsraelin1948isNOT
correct?
A) ArabstatesborderingIsraelattackedthenewnation,butfailedtodefeattheIsraelis.
B) TheUnitedStatessupportedthecreationofIsraelin1948,buttheSovietUnionopposed
itsformation.
C) ThepartitionofPalestineintoJewishandArabicstateswascarriedoutintheUnited
Nations.
D) TheArab-Israeliwarof1948createdhundredsofthousandsofArabrefugeesfrom
Palestine.
Answer: B
Diff:2
PageRef:836
Topic:WorldWarII
Skill:Conceptual

36) WhichofthefollowingstatementsconcerningZionismfollowingWorldWarIIismost
accurate?
A) ZioniststurnedtoviolentattemptstoejecttheBritishfromPalestineinresponsetothe
BritishattemptstolimitimmigrationtotheMiddleEast.
B) TheZionistmovementturnedtopeacefuldemonstrationsandboycottsonthemodelof
theIndiannationalistmovementandrefusedtoparticipateinviolence.
C) TheZionistmovement,frustratedbythefailuretoachieveanindependentnation,
weakenedafterWorldWarII.
D) TheZionistmovementwaseliminatedafterWorldWarIIbythecombinedactionofthe
PalestinianArabsandtheBritish.
Answer: A
Diff:2
PageRef:836
Topic:WorldWarII
Skill:Conceptual

574

37) TheAfrikanerNationalPartyinSouthAfricaestablishedarigidsystemofracialsegregation
called
A) Boerprejudice.
B) voortrekker.
C) apartheid.
D) swartzfrei.
Answer: C
Diff:1
PageRef:835
Topic:WorldWarII
Skill:Factual

38) Inwhatnationwerewhitesettlersabletoretaintheirpositionofsupremacy?
A) Kenya
B) Algeria
C) SouthernRhodesia
D) SouthAfrica
Answer: D
Diff:1
PageRef:834
Topic:WorldWarII
Skill:Factual

39) WhichofthefollowingstatementsconcerningtheAlgerianindependencemovementismost
accurate?
A) AlgeriawonitsindependencefromFranceinapeacefulmovementledbywhitesettlers
inthecolony.
B) DecolonizationinAlgeriawasviolent,aswhitesettlersresistedindependencethrough
theOASsupportedbypowerfulelementswithintheFrenchmilitary.
C) IndependenceinAlgeriawasachievedasaresultofthemilitaryvictoryoftheFLNover
theFrencharmy.
D) UnliketherestofAfrica,Algeriawasneverdecolonizedandremainedaprovinceof
France.
Answer: B
Diff:2
PageRef:834
Topic:WorldWarII
Skill:Conceptual

575

40) FailureoftheBritishtodealwiththeleaderofthelargestnationalistparty,JomoKenyatta,led
toaviolentandprolongedrevolutionin
A) Nigeria.
B) Ghana.
C) SouthAfrica.
D) Kenya.
Answer: D
Diff:2
PageRef:833-834
Topic:WorldWarII
Skill:Factual

41) InwhattypeofAfricancolonieswastheprocessofdecolonizationmostviolent?
A) Thosethatwereleastindustrialized
B) ThosecolonizedbytheGermans
C) ThosewithlargenumbersofwhiteEuropeansettlers
D) Truecolonies
Answer: C
Diff:2
PageRef:832
Topic:WorldWarII
Skill:Factual

42) BywhatdecadehadtheEuropeancolonizationofmostofblackAfricacometoanend?
A) 1950s
B) 1960s
C) 1970s
D) 1980s
Answer: B
Diff:2
PageRef:832
Topic:WorldWarII
Skill:Factual

43) WhatradicalAfricanleaderhelpedtoachieveindependenceinGhana?
A) JuliusNyerere
B) JomoKenyatta
C) KwameNkrumah
D) NelsonMandela
Answer: C
Diff:2
PageRef:832
Topic:WorldWarII
Skill:Factual

576

44) Whichofthefollowingnationsachievedindependencewithoutviolence?
A) Indonesia
B) Philippines
C) Indochina
D) India
Answer: B
Diff:2
PageRef:831
Topic:WorldWarII
Skill:Factual

45) WhatwasthesolutiontothedivisioninIndiabetweenMuslimsandHindusin1947?
A) TheBritishestablishedasinglegovernmentwithaHindumajority,butwithspecific
officesreservedforMuslims.
B) ThegovernmentofIndiawasdividedbetweentwohousesoftheIndianparliament,one
forMuslims,oneforHindus.
C) TheBritishsimplywithdrewfromIndiawithoutanypoliticalsettlementoftheproblem
ofreligiousdivision.
D) TheBritishdecidedtodivideIndiaintotwonations: aMuslimPakistanandasecular,
butHindu-dominated,India.
Answer: D
Diff:2
PageRef:831
Topic:WorldWarII
Skill:Conceptual

46) OfallthenationalistpartiesinIndia,theBritishweremostcloselyalliedwith
A) theMuslimLeague.
B) Congress.
C) theQuitIndiamovement.
D) theCommunists.
Answer: A
Diff:1
PageRef:831
Topic:WorldWarII
Skill:Factual

577

47) HowdidtheIndianCongressPartyandnationalistleadersrespondtoBritishparticipationin
WorldWarII?
A) AsinWorldWarI,theCongressPartyandnationalistleaderssuchasGandhiled
popularralliesinfavoroftheBritishwareffort.
B) Nationalistleadersofallpartiesopposedthewareffort.
C) Congressopposedthewareffortanditsleaderswerejailed,buttheMuslimLeague
ralliedtotheBritishcause.
D) TheMuslimLeagueandtheCommunistsopposedtheBritishwareffortasameansof
establishingindependence.
Answer: C
Diff:2
PageRef:831
Topic:WorldWarII
Skill:Conceptual

48) AJewishhomelandtobecalledIsraelwasestablishedin1948
A) inPalestine.
B) inEgypt.
C) inSaudiArabia.
D) inSyria.
Answer: A
Diff:2
PageRef:836
Topic:WorldWarII
Skill:Factual

49) WhatdocumentduringWorldWarIIincludedaclausethatrecognizedtherightofallpeople
tochoosetheformofgovernmentunderwhichtheylive?
A) TheMarshallPlan
B) TheAtlanticCharter
C) TheBalfourDeclaration
D) TheTrumanDoctrine
Answer: B
Diff:2
PageRef:830
Topic:WorldWarII
Skill:Factual

578

50) Whichofthefollowingnations,createdintheaftermathofWorldWarI,lostitsindependence
followingWorldWarII?
A) Czechoslovakia
B) Yugoslavia
C) Greece
D) Latvia,Lithuania,Estonia
Answer: D
Diff:2
PageRef:829
Topic:WorldWarII
Skill:Factual

EssayQuestions
1) Definetotalwar.HowdidtheWorldWarsofthe20thcenturydemonstratetheapplication
oftotalwar?
Answer: Totalwar:warresultingfromtheimpactofindustrializationonmilitaryeffort,
reflectingboththetechnologicalinnovationandtheorganizationalcapacitythat
accompaniedtheindustrialeconomyfromitsearlystages.WorldWarIsawthe
applicationofmoredestructivetechnology,machineguns,poisongas,battleships,
submarines,airplanes,tanks;governmentcontrolofeconomicactivity,altered
managementpolicy,laborrelations,andpersonalconsumptionoptions;controlof
media;WorldWarIIsimplyintensifiedallofabove;mostobvioustechnological
advancewasatomicweapons.
Diff:2
PageRef:819
Topic:WorldWarII
Skill:Conceptual

2) InwhatwaysdidthesettlementofWorldWarIIrepudiatetheVersailles
treatiesthatendedWorldWarI?Inwhatwaysdidthesettlementaffirmthe
conceptsincludedintheVersaillestreaties?
Answer: Repudiation:creationofsmallrepublicsineastEuropelargelyundoneby
dominanceofSovietUnionfollowingWorldWarII;directlycontrolledmost,
tookoverBalticrepublics;U.S.isolationended;Japanwhowasonthe
victorioussideinWorldWarIoccupiedbyU.S.Affirmation:
internationalizationaffirmedwithcreationofUnitedNationstoreplacefailed
LeagueofNations;destructionofGermanyaccomplishedbycreationoffour
occupationzones,althougheconomyleftlargelyintact.
Diff:2
PageRef:827-829
Topic:WorldWarII
Skill:Conceptual

579

Chapter36 WesternSocietyandEasternEuropein
the Decades of the Cold War
Multiple-ChoiceQuestions
1) Whatcrisisemergedin1956thatdemonstratedthediminishedpowersofEuropeannationsin
worldaffairs?
A) PortugalattemptedtoreassertitscontroloverGoa.
B) TheNetherlandssoughttoestablishcolonialruleinSouthAfrica.
C) BritainandFranceattemptedforciblytohaltEgyptsnationalizationoftheSuezCanal.
D) FrancewasdrivenfromLibya.
Answer: C
Diff:2
PageRef:845
Topic:AfterWorldWarII:InternationalSettingfortheWest
Skill:Factual

2) WhatFrenchleadernegotiatedAlgeriasindependencein1962?
A) KingCharlesV
B) MarshalPetain
C) GeneralBoulanger
D) CharlesdeGaulle
Answer: D
Diff:1
PageRef:845
Topic:AfterWorldWarII:InternationalSettingfortheWest
Skill:Factual

3) InwhatyeardidtheFrenchrelinquishtheircolonyinVietnam?
A) 1947
B) 1954
C) 1973
D) 1981
Answer: B
Diff:2
PageRef:845
Topic:AfterWorldWarII:InternationalSettingfortheWest
Skill:Factual

580

4) WhatphrasedidWinstonChurchillcointodescribethedivisionbetweenfreeandrepressed
societiesafterWorldWarII?
A) Theredmenace
B) Theironcurtain
C) TheBerlinWall
D) Thecoldwar
Answer: B
Diff:1
PageRef:845
Topic:TheColdWarandDecolonization,1945-1989
Skill:Factual

5) WherewasthefocalpointofthecoldwarinEuropeimmediatelyafterWorldWarII?
A) Hungary
B) Czechoslovakia
C) France
D) Germany
Answer: D
Diff:2
PageRef:846
Topic:TheColdWarandDecolonization,1945-1989
Skill:Factual

6) Whichofthefollowingcountrieswasnotamemberoftheeasternbloc?
A) Poland
B) Hungary
C) Turkey
D) Romania
Answer: C
Diff:2
PageRef:845
Topic:TheColdWarandDecolonization,1945-1989
Skill:Factual

7) AprogramofloansthatwasdesignedtoaidwesternEuropeannationsrebuildafterWWIIs
devastationwasthe
A) DreyfusPlan.
B) McArthurPlan.
C) MarshallPlan.
D) ChurchillPlan.
Answer: C
Diff:2
PageRef:846
Topic:TheColdWarandDecolonization,1945-1989
Skill:Factual

581

8) U.S.oppositiontoSovietaggressioninwesternEuropewaspredicatedon
A) amassivesuperiorityinmenunderarms.
B) thetechnologicalsuperiorityofittanksandartillery.
C) anuclearumbrella.
D) thepoliticaldivisionwithintheSovietUnion.
Answer: C
Diff:1
PageRef:846
Topic:AfterWorldWarII:InternationalSettingfortheWest
Skill:Factual

9) WhichofthefollowingstatementsconcerningtheU.S.militaryspendingismostaccurate?
A) AfterWorldWarII,U.S.militaryspendingcontinuouslydeclined.
B) UnderDemocraticpresidents,thepercentageofU.S.resourcesdevotedtothemilitary
increasedwhileunderRepublicanpresidentsthesameexpendituresdecreased.
C) TheU.S.abandonedmilitarypreeminencetothegrowingpowerofFrance.
D) Regardlessofthepartyinpoliticalpower,thepercentageoftheU.S.budgetgoingtothe
militaryremainedstablefromthe1950stothe1980s.
Answer: D
Diff:2
PageRef:847
Topic:AfterWorldWarII:InternationalSettingfortheWest
Skill:Conceptual

10) WhichofthefollowingwasconsistentwiththepoliticalviewpointoftheChristianDemocrats?
A) Totalitariangovernment
B) Democraticinstitutionsandmoderatesocialreform
C) Abolitionoftradeunions
D) Nationalizationofallindustries
Answer: B
Diff:2
PageRef:847
Topic:AfterWorldWarII:InternationalSettingfortheWest
Skill:Conceptual

582

11) Whichofthefollowingstatementsconcerningthedevelopmentofnewgovernmentsin
EuropeafterWorldWarIIismostaccurate?
A) ExceptforGermanyandItaly,politicalstabilitywasrestoredthroughtheinstitutionof
moreauthoritariangovernments.
B) NewconstitutionsestablishedinwesternEuropeuniformlyestablishedeffective
parliamentswithuniversal(includingfemale)suffrage.
C) ConstitutionsformedafterWorldWarIIwerenoteworthyprimarilyforthelackof
durability.
D) Aslateasthe1980sseveralnationsclungtosemi -fascist,authoritarianregimesruledby
strongmen.
Answer: B
Diff:2
PageRef:848
Topic:AfterWorldWarII:InternationalSettingfortheWest
Skill:Conceptual

12) WhichofthefollowingstatementsconcerningtheGermangovernmentafterWorldWarIIis
mostaccurate?
A) GermanyremaineddividedamongthreeWesternpowersuntil1980.
B) Duringthecoldwar,France,Britain,andtheUnitedStatesmergedtheirterritoriesto
formtheFederalRepublicofGermany.
C) AfterWorldWarII,theWeimarRepublicwasrestoredinGermany.
D) GermanyfellunderthedirectgovernmentoftheSovietUnionalongwiththerestof
easternEurope.
Answer: B
Diff:2
PageRef:847
Topic:AfterWorldWarII:InternationalSettingfortheWest
Skill:Conceptual

13) Thecreationofthewelfarestate
A) wasaccomplishedonlyintheUnitedStates.
B) wascarriedoutaspartoftheprogramofconservatisminEuropeangovernments
followingWorldWarII.
C) wasnecessitatedbytherecurrenceoftheDepressionfollowingWorldWarII.
D) resultedfromtheleftwardshiftofthepoliticalspectruminEuropefollowingWorldWar
II.
Answer: D
Diff:2
PageRef:848
Topic:AfterWorldWarII:InternationalSettingfortheWest
Skill:Conceptual

583

14) WhichofthefollowingsocialinsuranceprogramswasNOTtypicalofthewelfarestate?
A) State-runmedicalfacilities
B) Unemploymentinsurance
C) Thevalue-addedtaxscheme
D) Familyassistance
Answer: C
Diff:2
PageRef:848-849
Topic:AfterWorldWarII:InternationalSettingfortheWest
Skill:Conceptual

15) WhichofthefollowingstatementsconcerningtheEuropeanwelfarestateismostaccurate?
A) Theimpositionofthewelfarestatewasaccompaniedbytheeliminationoftheprivate
sectorinmostEuropeannations.
B) Middle-classpeople,ingeneral,failedtorealizeanybenefitsfromthewelfarestate.
C) Thewelfarestatecushionescitizensagainstmajorexpensesandunusualhardships,
thoughitdidnotrearrangeoverallsocialstructure.
D) Allofthetaxschemesintroducedbythewelfarestatewereintendedtoredistribute
incomefromthewealthytothepoor.
Answer: C
Diff:2
PageRef:849
Topic:AfterWorldWarII:InternationalSettingfortheWest
Skill:Conceptual

16) WhichofthefollowingwasNOTaneffectofthewelfarestate?
A) Itcushionedcitizensagainstbigexpensesandunusualhardships,ratherthan
rearrangingthesocialstructure.
B) Itprotectedthepurchasingpoweroftheverypooragainstcatastropheandcontributed
toimprovedhealthconditions.
C) Itincreasedcontactsbetweengovernmentandcitizenandproducedahostofnew
regulations.
D) Immediatelyuponitscreation,itgeneratedastormofpoliticalprotestfromconservative
politicalfactions.
Answer: D
Diff:2
PageRef:848-849
Topic:AfterWorldWarII:InternationalSettingfortheWest
Skill:Conceptual

584

17) Whichofthefollowingparalleledthedevelopmentofthewelfarestate?
A) Increasedmilitaryspending
B) Increasedgovernmentroleineconomicpolicy
C) Increasedpoliticalconservatism
D) Decreasedgovernmentspendingoverall
Answer: B
Diff:2
PageRef:849
Topic:AfterWorldWarII:InternationalSettingfortheWest
Skill:Conceptual

18) WelfarecommitmentsbecamefarandawaythelargestcomponentofWesterngovernment
budgetsoutsideof
A) Sweden
B) theUnitedStates
C) Britain
D) France
Answer: B
Diff:3
PageRef:849
Topic:AfterWorldWarII:InternationalSettingfortheWest
Skill:Factual

19) Atechnocratwas
A) anewpoliticalpartisanoftheChristianDemocraticParty.
B) anopponentofthereplacementofmenbymachines.
C) anewbreedofbureaucrattypifiedbytraininginengineeringoreconomics.
D) apersonwhopromotedgovernmentbythemagnatesofindustry.
Answer: C
Diff:1
PageRef:849
Topic:AfterWorldWarII:InternationalSettingfortheWest
Skill:Factual

20) Whatupsetthepatternofpoliticalcompromisearoundthepatternsofparliamentary
democracyandthewelfarestateinthe1960s?
A) MilitaryrevolutionsinFranceandSpain
B) ProtestoncollegecampusesinEuropeandtheU.S.
C) TheelectionofafascistgovernmentinPortugal
D) ThewidespreaddepressionthatafflictedEurope
Answer: B
Diff:2
PageRef:849
Topic:AfterWorldWarII:InternationalSettingfortheWest
Skill:Factual

585

21) WhichofthefollowingrepresentedanewpoliticalconcernintheWestfollowingtheupheaval
ofthe1960s?
A) Communism
B) Socialism
C) TheGreenmovement
D) Conservatism
Answer: C
Diff:1
PageRef:850
Topic:AfterWorldWarII:InternationalSettingfortheWest
Skill:Factual

22) WhichofthefollowingnationswasNOTinvolvedintheoriginalEuropeanEconomic
Community?
A) Britain
B) Italy
C) France
D) Germany
Answer: A
Diff:3
PageRef:850
Topic:AfterWorldWarII:InternationalSettingfortheWest
Skill:Factual

23) TheEuropeanEconomicCommunityisagoodexampleof
A) Europescontinuednationalstrife.
B) cooperationbetweenEuropeannationsandawillingnesstocreateasingleEuropean
economy.
C) theneedforEuropetodevelopasingleforeignpolicyindependentoftheU.S.
D) thecontinuedeconomicdependenceoftheEuropeannationsonthecapitalderivedfrom
theU.S.
Answer: B
Diff:2
PageRef:850
Topic:AfterWorldWarII:InternationalSettingfortheWest
Skill:Conceptual

586

24) WhichofthefollowingstatementsmostaccuratelydescribestheEuropeaneconomyofthe
post-1950s?
A) TheEuropeaneconomystagnatedshortlyafterthewithdrawaloftheU.S.from
Europeanaffairs.
B) Overallgrowthingrossnationalproductsurpassedtheratesofanyextendedperiod
sincetheIndustrialRevolutionbegan.
C) Withtheexceptionofamajordepressionfrom1958to1964,theEuropeaneconomy
grew.
D) WhilesouthernEuropeannations,longtheleastdevelopedofthecontinent,
demonstratedremarkablegrowth,northernEuropeslidintoeconomicrecession.
Answer: B
Diff:2
PageRef:851
Topic:AfterWorldWarII:InternationalSettingfortheWest
Skill:Conceptual

25) WhichofthefollowingwasNOTtypicaloftheEuropeaneconomyafter1950?
A) Rampantconsumerism
B) Risingper-capitadisposableincome
C) Highlevelsofunemployment
D) Difficultieswithimmigrantlaborers
Answer: C
Diff:2
PageRef:851
Topic:AfterWorldWarII:InternationalSettingfortheWest
Skill:Conceptual

26) WhichofthefollowingstatementsmostaccuratelydescribesthesituationofEuropean
peasantryinthesocialstructureoflate20th-centuryEurope?
A) TheEuropeanpeasantrydisappearedfollowingWorldWarII.
B) TheEuropeanpeasantrydecreasedinsizeandbecameincreasinglycommercialized.
C) TheEuropeanpeasantrycontinuedtoremainexclusivelyruralandrejectedtwentieth
centuryurbanculture.
D) AloneamongEuropeansocialclasses,thepeasantryremainedindependentofthestate
bureaucracies.
Answer: B
Diff:3
PageRef:856
Topic:CultureandSocietyintheWest
Skill:Conceptual

587

27) WhichofthefollowingwasNOTarightachievedbywomenintheWestinthelater20th
century?
A) Voting
B) Familyrights,accesstodivorce,abortion,birthcontrol
C) Accesstohighereducation
D) Paymentequaltomalesforequalwork
Answer: D
Diff:2
PageRef:856
Topic:CultureandSocietyintheWest
Skill:Conceptual

28) WhatworkbySimonedeBeauvoirsignifiedthebeginningofthenewfeminismin1949?
A) PatriarchalSociety
B) TheFeminineMystique
C) TheSecondSex
D) Mine,Yours,andOurs
Answer: C
Diff:2
PageRef:858
Topic:CultureandSocietyintheWest
Skill:Factual

29) WhowroteTheFeminineMystique?
A) BettyFriedan
B) GerdaLerner
C) JeanneStemple
D) SojournerTruth
Answer: A
Diff:2
PageRef:858
Topic:CultureandSocietyintheWest
Skill:Factual

588

30) WhichofthefollowingstatementsconcerningWesterncultureduringthelater20thcenturyis
mostaccurate?
A) Inthefinearts,mostartistsattemptedareturntotheclassicalformsoftheRenaissance
andtheancientworld.
B) Morethananyotherscientificfield,thesocialsciencesabandonedthetraditional
emphasisonconsistencyinhumanandsocialbehavior.
C) Inthefieldofphysics,modernscientistswereunabletogobeyondthestartling
discoveriesofthe19thcentury.
D) Westerncultureinthe20thcentury,bothinartandinscience,becamelargelyrelative
ratherthanobjective.
Answer: D
Diff:3
PageRef:858-859
Topic:CultureandSocietyintheWest
Skill:Conceptual

31) WhichofthefollowingnationsdidNOTremainindependentofdirectSovietcontrolby1948?
A) Greece
B) Albania
C) Yugoslavia
D) Poland
Answer: D
Diff:2
PageRef:862
Topic:TheSovietUnionasaSuperPower
Skill:Factual

32) AllofthefollowingpolicieswerefollowedbySoviet-sponsoredregimesineasternEurope
EXCEPT
A) attacksonpotentialpoliticalrivals,includingtheCatholicchurch.
B) theinitiationoffive-yearplans.
C) theestablishmentofparliamentarydemocracies.
D) collectivizationofagriculture.
Answer: C
Diff:1
PageRef:862
Topic:TheSovietUnionasaSuperPower
Skill:Conceptual

589

33) DespitethelooseningofSovietcontrolovereasternEuropefollowingStalinsdeath,what
aspectsofSovietdominationcontinuedtobeenforced?
A) Single-partydominanceandmilitaryalignmentwiththeSovietUnion
B) Centralizedeconomicplanning
C) TotalrejectionofCatholicism
D) Agriculturalcollectivization
Answer: A
Diff:3
PageRef:863
Topic:TheSovietUnionasaSuperPower
Skill:Conceptual

34) TheindependentlabormovementinPolandthatchallengedSovietdominancewascalled
A) Comintern.
B) Solidarity.
C) Izvestia.
D) Pravda.
Answer: B
Diff:2
PageRef:862
Topic:TheSovietUnionasaSuperPower
Skill:Factual

35) WhichofthefollowingstatementsconcerningtheOrthodoxchurchunderStalinsregimeis
mostaccurate?
A) TheregimeappointedtheOrthodoxchurchtheonlyapprovedreligionwithinthestate.
B) TheOrthodoxchurchbecametheprimaryinstrumentoftheregimespolicyofeducation.
C) TheOrthodoxchurchceasedtoexistduringtheStalinistregime.
D) LoyaltiestoOrthodoxypersisted,buttheywereconcentratedinalargelyelderly
minority.
Answer: D
Diff:2
PageRef:863
Topic:TheSovietUnionasaSuperPower
Skill:Conceptual

590

36) WhatRussianauthorofGulagArchipelagowasexiledtotheWest,butfoundlifetheretoo
materialistic?
A) Turgenev
B) Tolstoy
C) Solzhenitsyn
D) Karamazov
Answer: C
Diff:1
PageRef:864
Topic:TheSovietUnionasaSuperPower
Skill:Factual

37) WhichofthefollowingwasNOTtypicaloftheSovieteconomy?
A) Controlofvirtuallyalleconomicsectors
B) Emphasisonheavyindustrialgoods
C) Needtocreateamassivearmamentsindustry
D) Readyavailabilityofconsumergoods
Answer: D
Diff:2
PageRef:864
Topic:TheSovietUnionasaSuperPower
Skill:Conceptual

38) WhichofthefollowingwasNOTasourceofpressureontheSovietfamily?
A) ReligiousconstraintsimposedbytheOrthodoxChurch
B) Crowdedhousing
C) Lossoftiestoextendedfamilylifeofthecountryside
D) Motherswhoworkedlonghoursawayfromthehome
Answer: A
Diff:2
PageRef:865
Topic:TheSovietUnionasaSuperPower
Skill:Conceptual

39) InwhatwaydidthesocialorganizationoftheindustrializedSovietUnioncome
toresemblethatoftheWest?
A) Emphasisonserviceindustriesandmanagerialhierarchies
B) Thedivisionofurbansocietybetweenworkersandamanagerialmiddleclass
C) Theabsenceofatrueproletariat
D) Lackofapeasantry
Answer: B
Diff:3
PageRef:865
Topic:TheSovietUnionasaSuperPower
Skill:Conceptual

591

40) Bythe1970s,theRussianrateofpopulationgrowthwas
A) muchlessthanthatoftheWest.
B) aboutthesameasthatofChina.
C) aboutthesameasthatoftheWest.
D) muchgreaterthantheWest.
Answer: C
Diff:2
PageRef:865
Topic:TheSovietUnionasaSuperPower
Skill:Factual

41) WomeninRussianindustrializedsociety
A) rapidlyreachedthesamestatusasmales.
B) werelesslikelytobeintheworkforcethanwomenintheWest.
C) dominatedsomeprofessions,suchasmedicine.
D) wereaffordedthesametypeofdomesticidealizationtypicalofwomenintheWest.
Answer: C
Diff:2
PageRef:865
Topic:TheSovietUnionasaSuperPower
Skill:Conceptual

42) ImmediatelyafterStalinsdeathin1953,whatformofgovernmentwasestablished?
A) Aliberaldemocracy
B) Arulingcommitteeratherthansingle-manrule
C) AutocracyunderLeonidBrezhnev
D) RulebytheSupremeSoviet
Answer: B
Diff:2
PageRef:866
Topic:TheSovietUnionasaSuperPower
Skill:Factual

43) Whatleaderemergedtotakeprimarypowerin1956?
A) JosephStalin
B) MikhailGorbachev
C) YuriAndropov
D) NikitaKhrushchev
Answer: D
Diff:1
PageRef:866
Topic:TheSovietUnionasaSuperPower
Skill:Factual

592

44) WhichofthefollowingwasaSovietsuccessduringtheyearsofKhrushchevsdominance?
A) TheCubanmissilecrisis
B) ThedevelopmentofagricultureinSiberia
C) ThelaunchingofSputnik
D) Increasesinoverallagriculturalproductivity
Answer: C
Diff:2
PageRef:866
Topic:TheSovietUnionasaSuperPower
Skill:Conceptual

45) WhichofthefollowingstatementsconcerningtheSovietmilitaryfollowingStalinsdeathis
mostaccurate?
A) OveralltheSovietUnionplayedacautiousdiplomaticgame,almostneverengagingin
warfarebutmaintainingahighlevelofpreparedness.
B) ThecontinuedgrowthoftheSovietmilitaryledtoanincreasingwillingnessonthepart
ofRussianrulerstoengageindirectmilitaryinterventionaroundtheglobe.
C) FollowingStalinsdeath,subsequentSovietrulersenactedapolicyofprogressive
demilitarization.
D) FollowingWorldWarII,theSovietmilitaryplayednosignificantroleinsubsequent
Russianhistory.
Answer: A
Diff:2
PageRef:866
Topic:TheSovietUnionasaSuperPower
Skill:Conceptual

46) WhatRussianleadersignificantlyalteredpolitical,diplomatic,andeconomicpoliciesinthe
SovietUnionafter1985?
A) YuriAndropov
B) LeonidBrezhnev
C) MikhailGorbachev
D) YuriGagarin
Answer: C
Diff:1
PageRef:868
Topic:TheExplosionofthe1980sand1990s
Skill:Factual

593

47) Thetermperestroikarefersto
A) anewfreedomtocommentandcriticizetheSovietgovernment.
B) economicrestructuringandmoreleewayforprivateownership.
C) theSovietspaceprogram.
D) theestablishmentofaliberaldemocracy.
Answer: B
Diff:2
PageRef:868-869
Topic:TheExplosionofthe1980sand1990s
Skill:Conceptual

EssayQuestions
1) WhateventsledtotheColdWar?
Answer: SovietoccupationofeasternEurope,PresidentTrumanwasaggressivetowardsthe
SovietUnion,MarshallPlan,unificationofGermany,andtheestablishmentofNATO
andtheWarsawPact.
Diff:2
PageRef:845-847
Topic:TheColdWar
Skill:Conceptual

2) HowdidwesternEuropechangeafterWorldWarII?
Answer: Thespreadofliberaldemocracy,movementtowardsawelfarestate,establishmentof
theEuropeanCommonMarketandamovetowardsEuropeanUnion,unprecedented
economicexpansion,andimmigration.
Diff:2
PageRef:847-853
Topic:TheResurgenceofWesternEurope
Skill:Conceptual

3) HowdidcultureandsocietyintheWestchangeduringthedecadesoftheColdWar?
Answer: Workersbecamemoreaffluentandthemiddleclassgrew,awomensmovement
changedthesocial,economicandpoliticalpositionofwomen,growingpermissiveness,
culturalinfluenceoftheUS,andanupsurgebytheyoungergeneration.
Diff:2
PageRef:855-860
Topic:CultureandSocietyintheWest
Skill:Conceptual

4) WhatwerethemaineventsineasternEuropeafterWorldWarII?
Answer: SovietUnionbecameasuperpower,theSovietsestablishedcontrolofeasternEurope,
rapidindustrializationbutfewerconsumergoodsthanintheWest,andWesternideas
andcultureincreasinglybegantopenetrateeasternEurope.
Diff:2
PageRef:860-869
Topic:EasternEuropeafterWorldWarII:ASovietEmpire
Skill:Conceptual

594

5) WhatdidMikhailGorbachevintendbythepoliciesofglasnostandperestroika?
Answer: TheSovieteconomyandsocietyhadstagnatedandglasnostwasanattempttoallowa
moresearchingenquiryintotheproblemsoftheSovietsocietyandperestroikawasthe
introductionofsmallscalemarketmechanismsintotheeconomy.Theideawasto
preservecommunism.
Diff:2
PageRef:868-869
Topic:ANewWaveofSovietReform
Skill:Conceptual

595

Chapter37 LatinAmerica:RevolutionandReactionin
the21stCentury
Multiple-ChoiceQuestions
1) Whatdistinguishesthoseregionsreferredtoasthethirdworldfromothersocieties?
A) Lackofstablepoliticalinstitutions
B) Lackofindustrialization
C) Lackofmajorreligioussystems
D) Isolationintheworldcommercialnetwork
Answer: B
Diff:2
PageRef:874
Topic:Introduction
Skill:Conceptual

2) Thesecondworldrefersto
A) theindustrializedWest.
B) LatinAmerica.
C) industrializedcommunistnations.
D) unindustrializednations.
Answer: C
Diff:2
PageRef:874
Topic:Introduction
Skill:Conceptual

3) WhichofthefollowingnationsisNOTpartofthefirstworld?
A) TheUnitedStates
B) Japan
C) Britain
D) TheSovietUnion
Answer: D
Diff:2
PageRef:874
Topic:Introduction
Skill:Conceptual

596

4) InwhichoftheregionsoftheworlddoesLatinAmericabelong?
A) Firstworld
B) Secondworld
C) Thirdworld
D) Fourthworld
Answer: C
Diff:1
PageRef:874
Topic:Introduction
Skill:Factual

5) WhichofthefollowingstatementsconcerningchangeinLatinAmericainthe20thcenturyis
mostaccurate?
A) Theregionhasgrappledwithsocialjustice,culturalautonomyandeconomicsecurity.
B) WhiletheLatinAmericaneconomyhadremainedrelativelyunchanged,thepolitical
andsocialorderhasbeenrevolutionized.
C) Themostsignificantchangehasbeenintheoverthrowoftheoldsocialhierarchybased
onraceandcolor.
D) TheelitesofLatinAmericanowarebasedontheincreasinglydominantlabormovement
thataccompaniedindustrializationthroughoutLatinAmerica.
Answer: A
Diff:2
PageRef:874
Topic:Introduction
Skill:Conceptual

6) InLatinAmericathestrugglefordecolonizationhasbeenprimarily
A) oneofeconomicsubordination.
B) oneofeconomicexploitation.
C) oneofeconomicbrinksmanship.
D) oneofeconomicdisengagement.
Answer: D
Diff:2
PageRef:874
Topic:Introduction
Skill:Factual

597

7) WhichofthefollowingwasakeyfigureinLatinAmericaafterWorldII?
A) JuanPeron
B) EmilianoZapata
C) ChristopherColumbus
D) PanchoVilla
Answer: A
Diff:2
PageRef:875
Topic:Introduction
Skill:Conceptual

8) TheGuatemalanreformsofJuanJoseArevalothatbroughthisgovernmentintodirectconflict
withforeigninterestsdoNOTinclude
A) oustingtheCatholicchurch.
B) landreform.
C) improvementintherightsandconditionsofworkers.
D) aproposedincometax.
Answer: A
Diff:2
PageRef:879
Topic:RadicalOptionsinthe1950s
Skill:Factual

9) WhatcorporationinGuatemalabecamethecenterpieceofthedisputebetweentheUnited
StatesandreformgovernmentsintheCentralAmericancountry?
A) U.S.Steel
B) FordMotorCompany
C) ContinentalCanCompany
D) UnitedFruitCompany
Answer: D
Diff:2
PageRef:879
Topic:RadicalOptionsinthe1950s
Skill:Factual

598

10) WhichofthefollowingstatementsconcerningtherevolutioninGuatemalain1954ismost
accurate?
A) ThecommunistArbenzgovernmentthatruledGuatemalawasoverthrownbyliberal
reformersunderArevalo.
B) Communistrevolutionariesoverthrewtheconservativemilitarygovernmentdespite
activeU.S.support.
C) TheU.S.CentralIntelligenceAgencyaidedconservativedissidentsinoverthrowingthe
nationalisticArbenzgovernment.
D) TheU.S.-supportedregimethatreplacedtheArbenzgovernmentintroducedsignificant
landreformandlimitedforeignownershipofGuatemalanindustry.
Answer: C
Diff:2
PageRef:879
Topic:RadicalOptionsinthe1950s
Skill:Conceptual

11) AfterthecoupthattoppledArbenzsgovernment,Guatemalawasvirtuallyrunbyallofthe
followingEXCEPT
A) coffeeplanters.
B) foreigncompanies.
C) theSovietUnion.
D) themilitary.
Answer: C
Diff:2
PageRef:879
Topic:RadicalOptionsinthe1950s
Skill:Factual

12) Byabout1950,whatportionofCubasimportscamedirectlyfromtheUnitedStates?
A) Three-fourths
B) One-half
C) 20percent
D) 10percent
Answer: A
Diff:2
PageRef:879
Topic:RadicalOptionsinthe1950s
Skill:Factual

599

13) AllofthefollowingwerereformpromisesoftheBatistagovernmentEXCEPT
A) nationalizationofnaturalresources.
B) fullemployment.
C) theoverthrowoftheUnitedStates.
D) landreform.
Answer: C
Diff:2
PageRef:879
Topic:RadicalOptionsinthe1950s
Skill:Conceptual

14) WhowastheleaderoftheCubanrevolutionthatoverthrewFulgencioBatista?
A) HortensioHerrera
B) CristobalColon
C) JuanPeron
D) FidelCastro
Answer: D
Diff:1
PageRef:879
Topic:RadicalOptionsinthe1950s
Skill:Factual

15) TheCubanrevolutionwasreferredtobytherevolutionariesas
A) TierrayLibertad.
B) the26thofJulyMovement.
C) theDecembristrevolt.
D) VotaAsi.
Answer: B
Diff:2
PageRef:879
Topic:RadicalOptionsinthe1950s
Skill:Factual

16) ThebeardedrebelsunderCastrowerealsoknownas
A) gauchos.
B) barbudos.
C) loslobos.
D) cubanos.
Answer: B
Diff:2
PageRef:880
Topic:RadicalOptionsinthe1950s
Skill:Factual

600

17) WhatArgentinerevolutionaryassistedintheoverthrowoftheCubangovernmentin1956?
A) CarlosObregon
B) ErnestoCheGuevara
C) FulgencioBatista
D) EnriqueRamirez
Answer: B
Diff:2
PageRef:879
Topic:RadicalOptionsinthe1950s
Skill:Factual

18) WhichofthefollowingstatementsmostaccuratelydescribestheoutcomeoftheCuban
revolution?
A) DespitethesuccessfuloverthrowofBatista,therevolutionarygovernmentfailedtoenact
significantreformsintheplantationeconomy.
B) TherevolutionarygovernmenteventuallyannounceditsadoptionofMarxist-Leninist
leanings,brokeoffrelationswiththeU.S.,andintroducedsweepingsocialistreforms.
C) Thelargelyliberalgovernmentthatresultedfromtherevolutionreturnedtothe
constitutionof1940andcloserrelationshipswiththeU.S.
D) AfterabriefsojournintheU.S.,Batistawasabletoreturntopowerwiththesupportof
theU.S.military.
Answer: B
Diff:2
PageRef:880
Topic:RadicalOptionsinthe1950s
Skill:Conceptual

19) TherevolutionarygovernmentofCubatradedeconomicdependencyontheU.S.for
A) economicautonomywithasuccessfulprogramofindustrialization.
B) increasingpoliticalandeconomictieswithJapan.
C) increasingeconomicdependencyontheSovietUnion.
D) asignificantshareoftheworldspetroleummarket.
Answer: C
Diff:2
PageRef:880
Topic:RadicalOptionsinthe1950s
Skill:Conceptual

601

20) ThesweepingreformsofCastrosCubahaveimprovedallofthefollowingEXCEPT
A) basicfreedom.
B) education.
C) health.
D) housing.
Answer: A
Diff:2
PageRef:880
Topic:RadicalOptionsinthe1950s
Skill:Factual

21) WheredidCheGuevaradiein1967?
A) Armenia
B) Turkey
C) Russia
D) Bolivia
Answer: D
Diff:2
PageRef:881
Topic:RadicalOptionsinthe1950s
Skill:Factual

22) WhichofthefollowingwereNOTavenuesofreformforLatinAmericaduringthe1960sand
1970s?
A) TheCatholicchurch
B) Politicalstability
C) Ahigh-techmilitary
D) Socialjustice
Answer: C
Diff:2
PageRef:881
Topic:TheSearchforReformandtheMilitaryOption
Skill:Factual

23) TheemergencewithintheRomanCatholichierarchyofacombinationoftheologyand
socialistprinciplesinanefforttobringaboutimprovedconditionsforthepoorwascalled
A) liberationtheology.
B) socialisminonecountry.
C) TierrayLibertad.
D) thecommunistprimer.
Answer: A
Diff:2
PageRef:881
Topic:TheSearchforReformandtheMilitaryOption
Skill:Factual

602

24) Liberaltheologiansstressed__________asaformofpersonalsalvation.
A) socialequality
B) confession
C) materialwealth
D) physicalprowess
Answer: A
Diff:2
PageRef:881
Topic:TheSearchforReformandtheMilitaryOption
Skill:Factual

25) WhichofthefollowingisNOTtrueinregardstoofficersoftheLatinAmericanmilitary?
A) Theybegantoseethemselvesasabovetheselfishinterestsofpoliticalparties.
B) Theybegantothinkofthemselvesasthetruerepresentativesofthenation.
C) Theybelievedinthevalueofhierarchyandusuallykepttheirplaceinsociety.
D) Theybelievedtheywerethebestequippedtosolvetheirnationsproblems.
Answer: C
Diff:2
PageRef:882-883
Topic:TheSearchforReformandtheMilitaryOption
Skill:Factual

26) WhichofthefollowingcountrieswasNOTaffectedbyamilitarycoupduringthe1960sand
1970s?
A) Chile
B) Brazil
C) Argentina
D) Mexico
Answer: D
Diff:2
PageRef:883-884
Topic:TheSearchforReformandtheMilitaryOption
Skill:Factual

27) InArgentina,violentoppositiontomilitaryruleledtoacounteroffensiveknownas
__________.
A) thegrange.
B) thetroubles.
C) thedirtywar.
D) theredcommand.
Answer: C
Diff:2
PageRef:884
Topic:TheSearchforReformandtheMilitaryOption
Skill:Conceptual

603

28) ThecommonthreadrunningthroughallofthemilitaryregimesofLatinAmericawas
A) theywereallsupportedbytheSovietUnion.
B) theywereallsupportedbytheworkingpopulations.
C) theywereallreform-minded.
D) theywereallnationalistic.
Answer: D
Diff:2
PageRef:884
Topic:TheSearchforReformandtheMilitaryOption
Skill:Conceptual

29) WhichofthefollowingstatementsconcerningmilitarygovernmentsinLatinAmericaafter
1960ismostaccurate?
A) Militarygovernmentstendedtofavorlaborandtheworkingclassesattheexpenseofthe
traditionaloligarchy.
B) Politicalrepressionandtorturewereoftenusedtosilencecritics.
C) Noneofthemilitarygovernmentswassuccessfulinintroducingsocialoreconomic
reforms.
D) MilitarygovernmentswereuniformlysurrogatesforconservativesinLatinAmerican
society.
Answer: B
Diff:2
PageRef:883-884
Topic:TheSearchforReformandtheMilitaryOption
Skill:Conceptual

30) Whichofthefollowingstatementsmostaccuratelyreflectstheprogressofdemocratizationin
LatinAmericaduringthe1980s?
A) DespitereturntodemocraticgovernmentinmanyLatinAmericancountries,problems
withpopulistmovements,threatsfrommilitaryleaders,inflation,andthedrugtrade
weakenedthenewregimes.
B) OncedemocraticgovernmentswererestoredinmuchofLatinAmerica,theinfluenceof
theU.S.intheregionbegantowane.
C) ThereturnofdemocracytoLatinAmericawassouniversalthatmilitarygovernments
ceasedtoexist.
D) Democraticgovernmentsinthe1980sceasedtobetroubledbytheexistenceofleftist,
guerrillamovements.
Answer: A
Diff:2
PageRef:884
Topic:TheSearchforReformandtheMilitaryOption
Skill:Conceptual

604

31) TheUnitedStatesdemonstrateditscontinuingpowerintheLatinAmericanregioninits
invasionof__________in1989
A) Panama
B) CostaRica
C) ElSalvador
D) Nicaragua
Answer: A
Diff:2
PageRef:888
Topic:TheSearchforReformandtheMilitaryOption
Skill:Factual

32) LatinAmericangovernmentsinthelastdecadesofthe20thcenturyfacedallofthefollowing
problemsEXCEPT
A) largeforeignloans
B) highratesofinflation
C) overpopulation
D) criminalactivityduetothedrugtrade
Answer: C
Diff:2
PageRef:884
Topic:TheSearchforReformandtheMilitaryOption
Skill:Factual

33) HowmanydirectU.S.militaryinterventionswerethereintheCaribbeanandCentralAmerica
before1933?
A) Two
B) Seven
C) 16
D) Over30
Answer: D
Diff:2
PageRef:885
Topic:TheSearchforReformandtheMilitaryOption
Skill:Factual

605

34) WhatNicaraguanpoliticianledanoppositiontoU.S.occupationuntilhisassassinationin
1934?
A) AugustoSandino
B) VioletaChamorro
C) DanielOrtega
D) LuizInacioSilva
Answer: A
Diff:2
PageRef:886
Topic:TheSearchforReformandtheMilitaryOption
Skill:Factual

35) ConservativegovernmentsanddictatorshipsestablishedintheaftermathofU.S.military
interventioninLatinAmericawerereferredtoas
A) UncleSamsAmigos.
B) C.I.A.heaven.
C) BananaRepublics.
D) BurritoBandits.
Answer: C
Diff:1
PageRef:886
Topic:TheSearchforReformandtheMilitaryOption
Skill:Factual

36) WhatU.S.presidentintroducedtheGoodNeighborPolicythatpromisedtodealmorefairly
withLatinAmericancountriesandtohaltdirectmilitaryintervention?
A) TheodoreRoosevelt
B) CalvinCoolidge
C) FranklinRoosevelt
D) HarryTruman
Answer: C
Diff:2
PageRef:887
Topic:TheSearchforReformandtheMilitaryOption
Skill:Factual

606

37) WhatledtotheU.S.returntomoreaggressivepoliciesincludingdirectmilitaryintervention
followingWorldWarII?
A) Thedesiretocontaincommunismandthecoldwar
B) ThediscoveryofuraniuminMexico
C) TheincreasinginterventionofJapanintoLatinAmericaneconomies
D) TheallianceofmanyLatinAmericancountrieswithfascistgovernmentsduringthewar
Answer: A
Diff:2
PageRef:888
Topic:TheSearchforReformandtheMilitaryOption
Skill:Conceptual

38) WhichofthefollowingwasNOTaUSmethodofinfluencewithinLatinAmericaafterWorld
WarII?
A) Participationinregionalorganizations
B) Supportofgovernmentsthatexpressedpro-democraticsentiments
C) Covertunderminingofunfriendlycountries
D) Directannexation
Answer: D
Diff:2
PageRef:888
Topic:TheSearchforReformandtheMilitaryOption
Skill:Factual

39) Duringthe1970sand1980s,U.S.policytowardsLatinAmericawasoften
A) hostile.
B) pragmatic.
C) hands-off.
D) dictatorial.
Answer: B
Diff:2
PageRef:888
Topic:TheSearchforReformandtheMilitaryOption
Skill:Factual

40) PresidentJimmyCarter
A) invadedGrenada.
B) invadedPanama.
C) gavecontrolofthePanamaCanaltoPanama.
D) invadedCostaRica
Answer: C
Diff:2
PageRef:888
Topic:TheSearchforReformandtheMilitaryOption
Skill:Factual

607

41) Profoundculturalvariationsonwhatismoralandproperleadtoahuman-rightspositionof
A) absolutism.
B) relativism.
C) universalism.
D) sectionalism.
Answer: B
Diff:2
PageRef:886-887
Topic:HumanRightsinthe20thCentury
Skill:Conceptual

42) WhichofthefollowingstatementsconcerningwomeninLatinAmericanpoliticsandsocietyis
mostaccurate?
A) NowhereinLatinAmericadidwomenachievetherighttovotebefore1955.
B) Womentendedtojointhenationalpoliticalparties,wheretraditionalprejudicesagainst
womeninpubliclifelimitedtheirabilitytoinfluenceprograms.
C) WomencontinuedtobeexcludedfromtheLatinAmericanindustrializedlaborforce,
althoughtheyplayedamajorroleinagriculturalproduction.
D) Bythemid-1980s,LatinAmericanwomencontinuedtoholdsocialandpoliticalstatus
moresimilartootherareasofthethirdworldthanwesternEuropeandNorthAmerica.
Answer: B
Diff:3
PageRef:888
Topic:SocietiesinSearchofChange
Skill:Conceptual

43) WhichofthefollowingfactorsdidNOTcontributetosignificantchangeinLatinAmerican
womensroles?
A) LatinAmericanmen
B) Feministorganizations
C) Suffragemovements
D) Internationalpressures
Answer: A
Diff:3
PageRef:888-889
Topic:SocietiesinSearchofChange
Skill:Factual

608

44) WhichofthefollowingwasNOTa1980sreasonfortheincreasedmovementofpeopleswithin
thehemisphere?
A) Workersseekingjobs
B) Thedemandsforcheaplabor
C) Theflightofpoliticalrefugees
D) Tourism
Answer: D
Diff:3
PageRef:889
Topic:SocietiesinSearchofChange
Skill:Factual

45) WhichofthefollowingstatementsconcerningLatinAmericanpopulationismostaccurate?
A) Between1950and1985,LatinAmericanpopulationremainedstagnantduetopoor
healthconditionsandconstantinternalwarfare.
B) Despiteimprovements,LatinAmericaspopulationcontinuedtoincreasemoreslowly
thanthatofNorthAmerica.
C) AlmostallpopulationincreaseinLatinAmericacanbeattributedtoimmigrationof
Europeanlaborers.
D) Since1950,LatinAmericanpopulationhasmorethandoubled,whileNorthAmerican
populationhasgrownmoreslowly.
Answer: D
Diff:2
PageRef:889
Topic:SocietiesinSearchofChange
Skill:Conceptual

46) WhichofthefollowingtypesofpopulationmovementwasNOTtypicalofLatinAmericain
thelate20thcentury?
A) MovementoflaboracrossinternationalboundariesinLatinAmerica
B) MovementoflaborfromLatinAmericatotheU.S.
C) Flightofpoliticalrefugees
D) Movementofpopulationawayfromthecitiestothecountryside
Answer: D
Diff:2
PageRef:889
Topic:SocietiesinSearchofChange
Skill:Conceptual

609

47) ThepercentageofLatinAmericanpeoplelivingincitiesis
A) greaterthanwesternEurope.
B) lessthanAsia.
C) lessthanAfrica.
D) lessthanEurope,butgreaterthanAfricaandAsia.
Answer: D
Diff:2
PageRef:890
Topic:SocietiesinSearchofChange
Skill:Conceptual

48) Thetermfavelasrefersto
A) largecoffeeestates.
B) shantytowns.
C) unemployment.
D) illegalbordercrossings.
Answer: B
Diff:2
PageRef:890
Topic:SocietiesinSearchofChange
Skill:Factual

49) Withrespecttoreligion,thevastmajorityofLatinAmericans
A) arestillnominallyRomanCatholics.
B) practicereligionsbroughtwithslavepopulationsfromAfrica.
C) followindigenousIndianreligionsthatpredatethearrivaloftheSpaniards.
D) haveconvertedtosomeformofProtestantism.
Answer: A
Diff:1
PageRef:890
Topic:SocietiesinSearchofChange
Skill:Conceptual

50) WhichofthefollowingisNOTaLatinAmericanliteraryfigureofthelate20thcentury?
A) PabloNeruda
B) GabrielGarciaMarquez
C) PanchoVilla
D) JorgeLuisBorges
Answer: C
Diff:3
PageRef:891
Topic:SocietiesinSearchofChange
Skill:Factual

610

EssayQuestions
1) DescribetheCubanrevolutionofthe1950s.
Answer: Cubanrevolutionwaslaunchedagainstlong-standingpersonalruleofBatista;
nationalisticresponsetoforeigncontrolofinternalresourcesandeconomy;itinvolved
demandsforredistributionofland;itwaswonbyguerillawarfareagainstnational
militaryforces.Theoutcomeoftherevolutionwastheestablishmentof
Marxist-Leninistsocialistgovernmentandactualgovernmentcontrolledbysingle
party.
Diff:2
PageRef:879-881
Topic:Revolution:TheFirstWaves
Skill:Conceptual

2) Defineauthoritarianreformer.HowhaveauthoritarianreformshadaroleinLatinAmerica
since1945?
Answer: AuthoritarianreforminLatinAmericawastypifiedbymobilizationofsupportfrom
labor,poor;strongnationalismparticularlydirectedagainstforeignownershipof
resources;oftenledbymilitaryfigureswhowishedtoretainstructureofgovernment.
AfterWorldWarII,PeroninArgentina,PRIinMexicoandVargasinBrazil.
Diff:2
PageRef:875-876
Topic:EconomicandPoliticalChangesinLatinAmerica
Skill:Conceptual

3) Compareandcontrasttheradicalrevolutionsofthe1950sinCubaandGuatemala.
Answer: Comparison:bothbasedonpopulistappealoflaboringgroups,nationalist
expropriationofforeigncapital,landredistribution.Contrasts:Guatemalanreform
movementunderArevaloandArbenzhaltedbyinterventionofU.S.Central
IntelligenceAgencyinsupportofbusinessinterests;CubanrevoltunderCastroresulted
increationofMarxist-LeniniststatewitheconomydependentonSoviet
Union;succeededwhereGuatemalathwartedbyU.S.intervention.
Diff:2
PageRef:877-881
Topic:RadicalOptionsinthe1950s
Skill:Conceptual

4) DiscusstheroleofthemilitaryinLatinAmericanpoliticaldevelopmentduringthelate20th
century.
Answer: Militarycametoforefrontduring1960sinfearofCubanexpansionofMarxist
revolution;madepossiblebyfailureofpopulistandliberalgovernmentstosupply
reforms;oftensupportedinitiallybyU.S.asmeansofopposingcommunism;tendedto
conservatism;imposedbureaucraticauthoritarianregimesbasedonmilitarychainof
command;heavyuseoftorture,politicalrepressionofopposition;governmentslasted
untilmid-1980s.
Diff:2
PageRef:881-884
Topic:TheSearchforReformandtheMilitaryOption
Skill:Conceptual

611

Chapter38 Africa,theMiddleEast,andAsiaintheEra
of Independence
Multiple-ChoiceQuestions
1) WhichcountrydidIndiraGandhigovern?
A) Britain
B) Nigeria
C) India
D) China
Answer: C
Diff:1
PageRef:894
Topic:Introduction
Skill:Conceptual

2) Anothertermforthelong-standingdivisionsbetweendifferentclassesandethnicgroupsis
A) communism
B) socialism
C) communalism.
D) ethniccleansing.
Answer: C
Diff:1
PageRef:897
Topic:Introduction
Skill:Factual

3) WhyhaveethnicrivalriesandcommunalviolencebeenendemicindecolonizedAfricanand
Asianstates?
A) ThelevelofcivilizationinAfricaandAsiawasmoreprimitiveatthetimeof
colonization.
B) TriballifeinAfricaandAsiawastraditionallymoreviolentthanothercultures.
C) Theintroductionofslaverybywhitesinthe19thcenturybrutalizedAfricanandAsian
culture.
D) EuropeanshastilycolonizedAfricaandAsiaandestablishedboundarieswithout
referencetoethnicgroupsorculturalhomogeneity.
Answer: D
Diff:2
PageRef:897-898
Topic:TheChallengesofIndependence
Skill:Conceptual

612

4) ThefragmentationofPakistanresultedinthecreationoftheindependentnationof
A) Myanmar.
B) SriLanka.
C) Bangladesh.
D) Bhutan.
Answer: C
Diff:1
PageRef:898
Topic:TheChallengesofIndependence
Skill:Factual

5) WhatwasthemostformidablebarriertoeconomicgrowthinpostcolonialAfricaandAsia?
A) Lackofcapital
B) Lackoftechnology
C) Lackofeducationalinstitutions
D) Rapidpopulationgrowth
Answer: D
Diff:2
PageRef:898
Topic:TheChallengesofIndependence
Skill:Conceptual

6) EuropeancolonizerscontributedtoAfricanpopulationgrowthbyallofthefollowingmeans
EXCEPT
A) theintroductionofnewfoodsourcesfromtheNewWorld.
B) bybringinganendtolocalwars.
C) byintroducingrailwaylinesthatcutdownonregionalfamines.
D) byencouragingimmigrationoflargenumbersofwhites.
Answer: D
Diff:1
PageRef:898-899
Topic:TheChallengesofIndependence
Skill:Conceptual

7) Whichofthefollowingstatementsconcerningpopulationgrowthinthethirdworldismost
accurate?
A) PopulationinAsiahasactuallybeguntodeclineinrecentdecades.
B) TherateofpopulationgrowthofAsiaismuchhigherthanthatofAfrica.
C) TherateofpopulationgrowthofAfricaisgreaterthanthatofAsia.
D) ThepopulationofAfricahasactuallybeguntodeclineinrecentdecades.
Answer: C
Diff:2
PageRef:899
Topic:TheChallengesofIndependence
Skill:Conceptual

613

8) WhichofthefollowingisNOTafactorinthehighbirthrateofthirdworldnations?
A) Theabilitytoimportfoodtofeedtheincreasedpopulation
B) Resistancetobirthcontrol
C) Socialstatussymbolsattachedtomalevirilityandtheabilityofwomentohavemale
children
D) TheneedtoextendfamilylineagesinAfrica
Answer: A
Diff:2
PageRef:899-900
Topic:TheChallengesofIndependence
Skill:Conceptual

9) WhichofthefollowingregionsdoesNOTcontainartificialnations?
A) NorthAmerica
B) Asia
C) Africa
D) TheMiddleEast
Answer: A
Diff:2
PageRef:904-905
Topic:TheChallengesofIndependence
Skill:Factual

10) Oneofthechiefby-productsofpopulationgrowthinthirdworldnationshasbeen
A) industrialization.
B) massmigrationstourbanareas.
C) impositionofeffectivestatebirthcontrolprograms.
D) intensiveprogramsoflandredistribution.
Answer: B
Diff:2
PageRef:900
Topic:TheChallengesofIndependence
Skill:Conceptual

11) Whatdidthirdworldcitieslackthathadmadepossibletheabsorptionofasimilarmigrant
influxintheWest?
A) Expandingindustrialsectors
B) Slums
C) Lowpopulationgrowthinthecountryside
D) Competitionforjobs
Answer: A
Diff:2
PageRef:900
Topic:TheChallengesofIndependence
Skill:Conceptual

614

12) Whichofthefollowingstatementsconcerningtheurbanpoorinthirdworldcitiesismost
accurate?
A) Despitetheirconditionandlargenumbers,theurbanpoorofthethirdworldhavenot
hadapoliticalimpact.
B) Developmentspecialistshaveconcludedthatslumsprovidetheonlyurbanhousingthe
poorarelikelytofind.
C) Citiesinthethirdworld,fueledbytheexistenceofcheaplaborsupply,havebecomethe
mostproductivecentersoftheeconomy.
D) Thirdworldcitiesgenerallydisplaythemarkingsofcarefulurbanplanningintheir
programsofexpansion.
Answer: B
Diff:2
PageRef:900-901
Topic:TheChallengesofIndependence
Skill:Conceptual

13) WhichofthefollowingisNOTaproblemintheruralenvironmentofthirdworldcountries?
A) Deforestation
B) Depletionofsoils
C) Insufficientlaborsupply
D) Industrialpollution
Answer: C
Diff:1
PageRef:901
Topic:TheChallengesofIndependence
Skill:Conceptual

14) Whichofthefollowingstatementsismostcorrectconcerningwomensrolesinthethird
world?
A) Therearemanyopportunitiesforwomenateverylevel.
B) Womenareneverallowedtoholdofficeorpoliticalpower.
C) Womeninpositionsofpoliticalauthorityarewell-connected.
D) Womenareuniversallydeniedtherighttovote.
Answer: C
Diff:1
PageRef:902
Topic:TheChallengesofIndependence
Skill:Conceptual

615

15) Onwhathavethirdworldcountriestraditionallydependedtofinanceindustrialization?
A) Developmentofbankingsystems
B) Saleofresourcesleftbehindbycolonizers
C) Profitsofpreviousindustrialization
D) Saleofcashcropsandminerals
Answer: D
Diff:2
PageRef:902
Topic:TheChallengesofIndependence
Skill:Conceptual

16) Inwhatexportcommodityhavesomethirdworldnationsbeenabletoimprovetheterms
underwhichtheyparticipateintheglobaleconomy,atleastforperiodsoftime?
A) Oil
B) Coffee
C) Bauxite
D) Cocoa
Answer: A
Diff:1
PageRef:903
Topic:TheChallengesofIndependence
Skill:Factual

17) Neocolonialeconomyrefersto
A) EuropesconquestofnewcoloniesinAfricaandAsia.
B) JapansconquestofmuchofAsiaduringWorldWarII.
C) theglobaleconomydominatedbytheindustrialnations.
D) thecreationofcoloniesbyIndiaandthemoreadvancednationsofAfricainthelast
severaldecades.
Answer: C
Diff:2
PageRef:905
Topic:TheChallengesofIndependence
Skill:Conceptual

616

18) WhichofthefollowingisNOTadrawbacktoacceptinginvestmentcapitalfromfirstand
secondworldnations?
A) Excessivelyhighratesofinterest
B) Requiredmilitaryalliances
C) Requirementsforremovalofstatesubsidiesonfoodandotheressentialitems
D) Commitmentstobuyproductsofinvestors
Answer: A
Diff:2
PageRef:906
Topic:TheChallengesofIndependence
Skill:Conceptual

19) WhatwasKwameNkrumahsresponsetothefailureofhisprogramsofsocialreformand
economicuplift?
A) HebecameincreasinglyindebtedtothepowersofEuropeandtheUnitedStates.
B) Heforciblycrushedalloppositionpartiesandassumeddictatorialpowers.
C) HeabandonedAfricancultureandincreasinglyimposedmandatoryWesternization.
D) Heresignedtheprimeministrytohispoliticalopponentsandestablishedaguerilla
movementintheruralcountryside.
Answer: B
Diff:2
PageRef:906
Topic:PathstoEconomicGrowthandSocialJustice
Skill:Conceptual

20) KwameNkrumahwasthefirstpost-colonialprimeministerof
A) India.
B) Nigeria.
C) China.
D) Ghana.
Answer: D
Diff:2
PageRef:906
Topic:PathstoEconomicGrowthandSocialJustice
Skill:Conceptual

617

21) KwameNkrumahspoliticalandeconomicprograms
A) rejuvenatedGhanaslaggingindustrializationwithmassiveinjectionsfromtheWest.
B) resultedinthecreationofthemostdemocraticgovernmentinAfrica.
C) ledtofaileddevelopmentschemesandhiseventualousterfrompowerin1966.
D) representedthemostthoroughWesternizationprograminAfrica.
Answer: C
Diff:2
PageRef:907
Topic:PathstoEconomicGrowthandSocialJustice
Skill:Conceptual

22) OneofthemostcommonelementsofAfricanandAsiangovernmentssincedecolonizationis
A) thecreationofliberaldemocracies.
B) militarytakeovers.
C) communism.
D) effectiveindustrialization.
Answer: B
Diff:2
PageRef:908
Topic:PathstoEconomicGrowthandSocialJustice
Skill:Factual

23) WhichofthefollowingcountrieshasNOTexperiencedamilitarytakeoverofitsgovernment?
A) Ghana
B) Vietnam
C) Nigeria
D) India
Answer: D
Diff:1
PageRef:908
Topic:PathstoEconomicGrowthandSocialJustice
Skill:Factual

618

24) WhichofthefollowingreasonswasNOTafactorinexplainingthefrequencyofmilitary
takeoverinthirdworldnations?
A) Regimentationrenderedsoldiersmoreresistanttodivisionbyreligiousandethnic
rivalries.
B) MilitaryforcesinthirdworldnationsoftenreceivedsupportfromtheSovietbloc.
C) Themilitarypossessedamonopolyofforceessentialinrestoringorderduringpolitical
crises.
D) Militarypersonnelpossessedsometechnicaltrainingthatwasoftenlackingamong
civiliannationalistleaders.
Answer: B
Diff:2
PageRef:908
Topic:PathstoEconomicGrowthandSocialJustice
Skill:Conceptual

25) Amongtheworstexamplesofmilitaryregimesinthirdworldnationswas
A) Uganda.
B) India.
C) Kenya.
D) Zambia.
Answer: A
Diff:3
PageRef:908
Topic:PathstoEconomicGrowthandSocialJustice
Skill:Factual

26) TheMuslimBrotherhood,anEgyptianreformmovementfoundedin1928,wasledby
A) GamalAbdulNasser.
B) AhmadOrabi.
C) theKhediveFarouk.
D) Hasanal-Banna.
Answer: D
Diff:2
PageRef:909
Topic:PathstoEconomicGrowthandSocialJustice
Skill:Factual

619

27) TheMuslimBrotherhoodembracedallofthefollowingEXCEPT
A) afundamentalistapproachtoIslam.
B) thepromotionoftradeunions.
C) non-violence.
D) landreform.
Answer: C
Diff:2
PageRef:909
Topic:PathstoEconomicGrowthandSocialJustice
Skill:Conceptual

28) TheKhediveFaroukwastoppledfrompowerin1952byacoupledby
A) theFreeOfficersMovement.
B) theMuslimBrotherhood.
C) theMahdists.
D) BlackSeptember.
Answer: A
Diff:2
PageRef:910
Topic:PathstoEconomicGrowthandSocialJustice
Skill:Factual

29) ThemanwhoemergedasheadoftheEgyptiangovernmentfollowingthe1952coupwas
A) GamalAbdulNasser.
B) AhmadOrabi.
C) AnwarSadat.
D) Hasanal-Banna.
Answer: A
Diff:2
PageRef:910
Topic:PathstoEconomicGrowthandSocialJustice
Skill:Factual

30) ThemilitarygovernmentofEgyptafter1952attemptedallofthefollowingreformsEXCEPT
A) landredistributionschemeslimitingtheamountoflandasingleindividualcouldown.
B) state-financededucationthroughthecollegelevel.
C) statesubsidiestolowerthepriceoffoodstaples.
D) anisolationistforeignpolicytominimizemilitaryexpenses.
Answer: D
Diff:2
PageRef:910
Topic:PathstoEconomicGrowthandSocialJustice
Skill:Conceptual

620

31) TheEgyptiangovernmentwasabletoforcetheBritishandtheirFrenchalliesoutoftheSuez
Canalzonein
A) 1952.
B) 1956.
C) 1961.
D) 1974.
Answer: B
Diff:2
PageRef:910
Topic:PathstoEconomicGrowthandSocialJustice
Skill:Factual

32) ThecornerstoneofEgyptiandevelopmentafter1952was
A) theAswandamproject.
B) theconstructionoftheSuezCanal.
C) theirrigationoftheSinai.
D) theconstructionofaMediterraneanportatKhartoum.
Answer: A
Diff:1
PageRef:911
Topic:PathstoEconomicGrowthandSocialJustice
Skill:Factual

33) InwhichofthefollowingwaysdidAnwarSadatalterEgyptianpoliciesestablishedbythe
militarygovernmentafter1952?
A) Heincreasedstatecontroloftheeconomy
B) HecreatedstrongertieswiththeSovietUnionasameansofincreasingforeign
investmentinEgypt
C) HeendedthecostlyconfrontationwithIsraelafter1973
D) HeincreasedEgyptiansupportforArabrevolutionarymovements
Answer: C
Diff:2
PageRef:911
Topic:PathstoEconomicGrowthandSocialJustice
Skill:Conceptual

621

34) InwhichofthefollowingwayswasIndiasimilartoEgyptfollowingdecolonization?
A) Levelofindustrialization
B) Emphasisonsocialismandstateintervention
C) Militarytakeoverofgovernment
D) Sizeofthemiddleclass
Answer: B
Diff:2
PageRef:912
Topic:PathstoEconomicGrowthandSocialJustice
Skill:Conceptual

35) Theworldslargestfunctioningdemocracyis
A) Canada.
B) theUnitedStates.
C) India.
D) China.
Answer: C
Diff:2
PageRef:912
Topic:PathstoEconomicGrowthandSocialJustice
Skill:Factual

36) Inthefirstdecadesofindependence,Indiawasgovernedby
A) M.K.Gandhi.
B) M.A.Jinnah.
C) M.A.Bhutto.
D) J.Nehru.
Answer: D
Diff:2
PageRef:912
Topic:PathstoEconomicGrowthandSocialJustice
Skill:Factual

37) Perhapsmorethananyotherthirdworldnation,Indiahasbeensuccessfulat
A) controllingpopulationgrowth.
B) raisinglivingstandards.
C) equitablelandredistribution.
D) preservingcivilrightsanddemocracy.
Answer: D
Diff:2
PageRef:912
Topic:PathstoEconomicGrowthandSocialJustice
Skill:Conceptual

622

38) Whowasbroughttopowerin1979inIranthrougharadicalrevolution?
A) SaddamHussein
B) HosniMubarak
C) AyatollahKhomeini
D) GamalAbdulNasser
Answer: C
Diff:1
PageRef:912
Topic:PathstoEconomicGrowthandSocialJustice
Skill:Factual

39) Inmanyrespects,theIranianrevolutionof1979ismostlike
A) themilitarycoupinEgyptin1952.
B) Gandhisnon-violentresistancetotheBritishRaj.
C) KwameNkrumahsgovernment.
D) theMahdistrevolutionintheSudaninthe1880s.
Answer: D
Diff:2
PageRef:912
Topic:PathstoEconomicGrowthandSocialJustice
Skill:Conceptual

40) Iran,unlikeotherareasofthethirdworld,
A) hadnotbeenformallycolonizedbyEuropeanpowers,buthadbeenreducedtoan
informalsphereofinfluence.
B) didnothaveproblemswithinequitablelanddistribution.
C) possessedasubstantialWestern-educatedmiddleclass.
D) washeavilyindustrializedandnotdependentontheexportofcashcropsormineral
wealth.
Answer: A
Diff:2
PageRef:913
Topic:PathstoEconomicGrowthandSocialJustice
Skill:Conceptual

41) DevelopmentschemesinIranwereforestalledbecauseof
A) internalbickeringamongthemullahsandayatollahs.
B) alengthyandexhaustingborderwarwithneighboringIraq.
C) IransinvasionofIsrael.
D) theradicalsecularizationofIransgovernment.
Answer: B
Diff:2
PageRef:913
Topic:PathstoEconomicGrowthandSocialJustice
Skill:Factual

623

42) WhatEuropeancolonizer,otherthanBritain,wasabletoholdontoitscoloniesinAfricainto
themid-1970s?
A) France
B) Germany
C) TheUnitedStates
D) Portugal
Answer: D
Diff:2
PageRef:914-915
Topic:PathstoEconomicGrowthandSocialJustice
Skill:Factual

43) From1948,SouthAfricanpoliticsweredominatedby
A) theNationalistParty.
B) theblackleadershipoftheZulunation.
C) Britishadministrators.
D) aUNmandategovernmentdominatedbytheU.S.
Answer: A
Diff:2
PageRef:915
Topic:PathstoEconomicGrowthandSocialJustice
Skill:Factual

44) WhichofthefollowingwasnotanintendedroleofapartheidinSouthAfrica?
A) Tospreadtheprofitsfromthecountrysmineralwealthtowhereitwoulddothemost
good
B) Economicdominanceforthewhitemajority
C) Monopolyofpoliticalpower
D) Toimposeasystemofextremesegregationonallracesofthecountry
Answer: A
Diff:2
PageRef:915
Topic:PathstoEconomicGrowthandSocialJustice
Skill:Factual

45) WhichofthefollowinggroupswasflourishingundertheruleoftheIranianshahs?
A) Foreigninvestors
B) Theayatollahs
C) Themullahs
D) Smallbazaarmerchants
Answer: A
Diff:2
PageRef:913
Topic:PathstoEconomicGrowthandSocialJustice
Skill:Factual

624

46) WhatwerethehomelandsestablishedbythegovernmentofSouthAfrica?
A) Areasreservedforthewhiteminority
B) Areasdesignatedforthemainethno-linguisticgroupsofindigenouspeopleswithin
SouthAfrica
C) AreasoutsideoftheboundariesofSouthAfricadesignatedforemigrationofindigenous
peoples
D) AreasofSouthAfricainwhichlandredistributionamongtheBoerpopulationhastaken
place
Answer: B
Diff:2
PageRef:915
Topic:PathstoEconomicGrowthandSocialJustice
Skill:Conceptual

47) WhichofthefollowingmethodswasNOTusedbytheSouthAfricangovernmenttosuppress
dissentamongtheblackpopulation?
A) Arrestofoppositionleaders
B) Favoritismshowntosomeleadersinordertodivideopponentsofapartheid
C) Useofspiesandpoliceinformers
D) Useofstateprogramstoimprovetheconditionsoftheblacktownships
Answer: D
Diff:1
PageRef:915
Topic:PathstoEconomicGrowthandSocialJustice
Skill:Conceptual

48) OneofthesignsthatthewhitemajoritywaswillingtonegotiatethefutureofSouthAfrican
politicsandsocietywasthefreeingof
A) SteveBiko.
B) JuliusNyerere.
C) NelsonMandela.
D) JomoKenyatta.
Answer: C
Diff:2
PageRef:915
Topic:PathstoEconomicGrowthandSocialJustice
Skill:Factual

625

49) EnfranchisementofalladultSouthAfricansoccurredinwhatyear?
A) 1989
B) 1996
C) 1994
D) 1990
Answer: C
Diff:1
PageRef:915
Topic:PathstoEconomicGrowthandSocialJustice
Skill:Factual

EssayQuestions
1) Whatproblemstendtobetypicalofallthirdworldnations?
Answer: Populationpressure, growingpopulationsoverwhelmnationalresources,restrictability
toachieveeconomicindependence;lackofindustrializationcomplicatedbylackof
investmentcapital;continueddependenceonexportofagriculturalproducts,mineral
wealth;fluctuationofmarketvalueofexportproducts;continueddependenceonglobal
tradenetworkdominatedbyindustrialnationsoffirstandsecondworlds;dramatic
populationshiftstourbanregionsthatcannotprovideemploymentorhousing;creation
ofpotentiallyvolatileunderemployedpopulationsincities.
Diff:2
PageRef:894-906
Topic:TheChallengesofIndependence
Skill:Conceptual

2) Howhavewomenfaredinthenewlyindependentnationsofthethirdworld?
Answer: Greaterpoliticalrights, suffragealmostinvariablygranted;greaterlegalrightssuchas
accesstodivorce;actualbenefitsofabovelimitedbytraditionalsocialroles;fewwomen
reachedpositionsofpower;thosethatdidwererelatedtopowerfulmales;noentryinto
administrations;continuedsubjectioninhouseholds;continuedexistenceofdomestic
seclusionandveiling,particularlyinIslamicsocieties;spreadofreligious
fundamentalismhasintensifiedwomenssubordinatestatusinmostsocieties.
Diff:2
PageRef:901-902
Topic:TheChallengesofIndependence
Skill:Conceptual

3) CompareandcontrastthepostcolonialgovernmentsofIndiaandEgypt.
Answer: Contrasts:nomilitaryinterventioninIndia,retentionofcivilianrule;Indiahadalarger
industrialandscientificsector,alsobettertransportandcommunicationinfrastructure;
IndiahadlargermiddleclassthanEgypt;Indiastateinterventionineconomylessdirect
thaninEgypt;greateraccesstointernationalcapitalization.Similarities:bothnations
typifiedbyoverwhelmingpopulationgrowththatateupmuchofgains;bothengaged
instatestimulationofeconomy,statefinancededucation,landredistribution(although
largelyunsuccessful).
Diff:2
PageRef:906-912
Topic:PathstoEconomicGrowthandSocialJustice
Skill:Conceptual

626

4) InwhatsensewastheIranianrevolutionof1979athrowbacktothefundamentalistrevoltsof
the19thcentury?
Answer: RevolutionaimedatWesternizedregimethatwasdemonstrablyun -Islamicalthough
composedofindigenousrulers;Khomeiniclaimedtobedivinelyinspiredleaderfor
returntopureformsofIslamtypicalofthedaysoftheProphet;promisedrebelsinstant
paradiseshouldtheyfallduringrevolution;attemptedtospreadIslamicrevolutionto
otherneighboringregimes;continuedconflictofShiaversionsofIslamversusSunni
regimes.
Diff:2
PageRef:912-914
Topic:PathstoEconomicGrowthandSocialJustice
Skill:Conceptual

5) Inwhatsensehastheprocessofdecolonizationbeenapositivemovement?
Answer: Despiteproblems,decolonizationdidresultinindependencefromAsian,European,
andAmericanimperialists;insomecases,notablyIndia,democraticregimeswere
establishedandachievedpoliticalsuccess;althoughindustrializationhasbeenslowto
occur,conditionsofindustrializationnoworsethanconditionsinEuropeduring18th
and19thcenturies,urbansqualor,underemployment,poorhealthcare;difficultto
overcomeburdensofexcessivepopulation;conditionsoflimitedcapitalization,
dependencycreatedbyexternalforcesandimperialists.
Diff:2
PageRef:916-918
Topic:Conclusion:ThePostcolonialExperienceinHistoricalPerspective
Skill:Conceptual

627

Chapter39 RebirthandRevolution:NationBuildingin
EastAsiaandthePacificRim
Multiple-ChoiceQuestions
1) WhowonthecivilwarinChinain1949?
A) Thecommunists
B) Vietnam
C) TheLiberalParty
D) Japan
Answer: A
Diff:2
PageRef:922
Topic:Introduction
Skill:Conceptual

2) WhoheadedtheAmericanoccupationgovernmentofJapan?
A) HapArnold
B) OmarBradley
C) DouglasMacArthur
D) DwightD.Eisenhower
Answer: C
Diff:1
PageRef:923
Topic:EastAsiainthePostwarSettlement
Skill:Factual

3) AmericansintroducedallofthefollowingreformstoJapanduringtheiroccupationEXCEPT
A) givingwomenthevote.
B) abolishingShintoismasastatereligion.
C) outlawinglaborunions.
D) makingtheemperorasymbolicfigurehead.
Answer: C
Diff:2
PageRef:923
Topic:EastAsiainthePostwarSettlement
Skill:Conceptual

628

4) WhatpartymonopolizedJapanesegovernmentintothe1990s?
A) Socialist
B) LiberalDemocratic
C) Labor
D) Communist
Answer: B
Diff:2
PageRef:923
Topic:EastAsiainthePostwarSettlement
Skill:Factual

5) InwhatyeardidtheAmericanoccupationofJapancometoanend?
A) 1945
B) 1947
C) 1952
D) 1955
Answer: C
Diff:2
PageRef:923
Topic:EastAsiainthePostwarSettlement
Skill:Factual

6) InwhatwaywastherestorationofanindependentKoreacomplicated?
A) KoreahadbecomeacolonyofChina,whichrefusedtorestoreindependence.
B) KoreawasdividedintozonescontrolledbytheU.S.andtheSovietUnion.
C) Koreasgovernmentwasclaimedbysurvivingmembersoftheoldmonarchy.
D) Koreahadnopriorexperienceasanindependentgovernment.
Answer: B
Diff:2
PageRef:924
Topic:EastAsiainthePostwarSettlement
Skill:Conceptual

7) WhowasthepoliticalleaderoftheSoviet-dominatedPeoplesDemocraticRepublicofKorea?
A) SyngmanRhee
B) LeeHoPark
C) KimIl-Sung
D) SoKimChung
Answer: C
Diff:2
PageRef:924
Topic:EastAsiainthePostwarSettlement
Skill:Factual

629

8) ThefirstleaderoftheU.S.-dominatedRepublicofKoreawas
A) SyngmanRhee
B) LeeHoPark
C) KimIl-Sung
D) SoKimChung
Answer: A
Diff:2
PageRef:924
Topic:EastAsiainthePostwarSettlement
Skill:Factual

9) InwhatyeardidtheconflictbetweenNorthandSouthKoreacometoatemporaryconclusion
withthesigningofanarmistice?
A) 1947
B) 1950
C) 1953
D) 1964
Answer: C
Diff:2
PageRef:925
Topic:EastAsiainthePostwarSettlement
Skill:Factual

10) WhocommandedtheUnitedNationstroopswhoparticipatedintheKoreanconflictonbehalf
oftheRepublicofKorea?
A) HapArnold
B) DouglasMacArthur
C) OmarBradley
D) DwightD.Eisenhower
Answer: B
Diff:2
PageRef:924
Topic:EastAsiainthePostwarSettlement
Skill:Factual

630

11) WhichofthefollowingstatementsmostaccuratelyreflectsthesituationinKoreafollowingthe
KoreanWar?
A) NorthernandsouthernKoreawererapidlyreunitedunderasingle,authoritarian
governmentcontrolledbytheSovietUnion.
B) NorthernKoreathrewoffitstieswithChinaandtheSovietUnionandsoughtacloser
relationshipwiththeU.S.
C) Korearemaineddividedwithrelativelyauthoritariangovernmentsinbothhalvesofthe
dividednation.
D) SouthernKoreabecamefullydemocratic,butmovedclosertopoliticalneutralityduring
thecoldwar.
Answer: C
Diff:2
PageRef:925
Topic:EastAsiainthePostwarSettlement
Skill:Conceptual

12) WhatChineseleaderestablishedanautocraticgovernmentonTaiwanin1948?
A) SunYatsen
B) ShiZilin
C) KimIl-Sung
D) ChiangKai-shek
Answer: D
Diff:2
PageRef:925
Topic:EastAsiainthePostwarSettlement
Skill:Factual

13) WhichofthefollowingeconomicpowersofthePacificRimremainedaBritishcolonylong
afterWorldWarII?
A) Malaya
B) Indonesia
C) ThePhilippines
D) HongKong
Answer: D
Diff:2
PageRef:925
Topic:EastAsiainthePostwarSettlement
Skill:Factual

631

14) WhichnationretainedalargeBritishnavalbaseuntil1971?
A) Malaya
B) Taiwan
C) Singapore
D) HongKong
Answer: C
Diff:3
PageRef:925
Topic:EastAsiainthePostwarSettlement
Skill:Conceptual

15) TheJapanesepoliticalsystemafter1955
A) wasmarkedbyradicalshiftsbetweenpartiesoftheleftandright.
B) wastypifiedbythedominanceofsocialism.
C) revivedmanyoftheoligarchicfeaturesofearlierpoliticaltradition.
D) wasintentonthedestructionofthebigbusinesscombines.
Answer: C
Diff:2
PageRef:925
Topic:Japan,Incorporated
Skill:Conceptual

16) WhatwastheonlyweaknessoftheleadershipoftheLiberalDemocraticPartyinJapaninthe
1980s?
A) Inabilitytoprovideeconomicgrowth
B) Corruption
C) AssociationwithmilitarypoliciesofWorldWarII
D) Adoptionofapolicyofnucleararmament
Answer: B
Diff:2
PageRef:926
Topic:Japan,Incorporated
Skill:Conceptual

17) WhatWesternlabelwasappliedtotheclosecoordinationofJapanesegovernmentand
businessforpromotionofeconomicgrowthandexportexpansion?
A) Japan,Incorporated
B) Business,Japan
C) Toyota,Inc.
D) SonyUnited
Answer: A
Diff:1
PageRef:926
Topic:Japan,Incorporated
Skill:Factual

632

18) WhichofthefollowingwasNOTafeatureofgovernmentinvolvementinJapaneseindustry?
A) Settingproductiongoals
B) Establishinginvestmentgoals
C) Limitingimports
D) Selectingtheheadsofthemajorcorporations
Answer: D
Diff:2
PageRef:926
Topic:Japan,Incorporated
Skill:Conceptual

19) WhichofthefollowingstatementsconcerningthedevelopmentofJapanesecultureinthe
postwareraismostaccurate?
A) Japaneseculturethrewoffitsconnectionswiththepastandwholeheartedlyembraced
Westernstylesinliterature,drama,andthearts.
B) Japaneseculturewasdefinedbyitsconservatismandretentionofoldformstotheextent
thatWesternforms,particularlyinartandarchitecture,wereunknown.
C) Asidefrominteriordecorationandfilm,Japanesecontributionstoworldculturewere
negligible.
D) BuddhismandShintoismwereofficiallybannedbecauseoftheirassociationwiththe
warlikeJapanesepast.
Answer: C
Diff:2
PageRef:926
Topic:Japan,Incorporated
Skill:Conceptual

20) WhichofthefollowingwasNOTafactorintheamazingeconomicgrowthofJapanfollowing
the1950s?
A) Cheaploansfortechnologicalinnovation
B) Educationalexpansion
C) Agrowingpopulationandareductionintheagriculturallaborforce
D) Arapidlygrowingmilitary-industrialcomplex
Answer: D
Diff:2
PageRef:927
Topic:Japan,Incorporated
Skill:Conceptual

633

21) Japanproducedadistinctiveeconomiccultureafterthe1950sthatincludedallofthefollowing
featuresEXCEPT
A) astrongtraditionofindependentunions.
B) managerswhodisplayedactiveinterestinsuggestionsbyemployees.
C) anetworkofpoliciesandattitudesthatreflectedoldertraditionsofgroupsolidarity.
D) willingnessamongmanagementtoabidebycollectivedecisionsandlessconcernfor
quickpersonalprofits.
Answer: A
Diff:2
PageRef:927
Topic:Japan,Incorporated
Skill:Conceptual

22) WhichofthefollowingrepresentsasignificantdifferencebetweenJapaneseandWestern
womeninthelater20thcentury?
A) WomeninJapanparticipatedactivelyinleisureactivitieswiththeirhusbands.
B) JapanesewomenhadhigherratesofdivorcethantheirWesterncounterparts.
C) TheJapanesefeministmovementwasconfinedtoasmallnumberofintellectuals.
D) JapanesewomenconcentratedlessondomesticdutiesthanwomenintheWest.
Answer: C
Diff:2
PageRef:927-928
Topic:Japan,Incorporated
Skill:Conceptual

23) Inthe1980stheJapanesegovernmentinvestedconsiderablemoneyinteaching
A) Japanesemotherstobreast-feed.
B) socialetiquetteatgeishahouses.
C) eatingwithchopsticks.
D) Western-styledancing.
Answer: C
Diff:2
PageRef:928
Topic:Japan,Incorporated
Skill:Factual

634

24) WhatnationotherthanJapaninthePacificRimwasthemostobviousexampleofthespread
ofneweconomicdynamism?
A) SouthKorea
B) NorthKorea
C) Vietnam
D) Malaya
Answer: A
Diff:1
PageRef:929
Topic:ThePacificRim:NewJapans?
Skill:Factual

25) WhatleadershipwastypicalofSouthKoreabetween1960andthelate1980s?
A) LiberalDemocratic
B) Socialist
C) Communist
D) Military
Answer: D
Diff:2
PageRef:929
Topic:ThePacificRim:NewJapans?
Skill:Factual

26) WhichofthefollowingstatementsmostaccuratelydescribesKoreaneconomicgrowthafter
1950?
A) Economicgrowthwaslimitedtoproductionofporcelainandsilksforexport.
B) KoreaneconomicgrowthwasmuchslowerthanthatofJapan.
C) Koreaneconomicgrowthwasdependentonsmall,government -financedcorporations
producinglimitedquantitiesofgoodswithlittleadvancedtechnology.
D) Koreawasablebythe1970stocompetesuccessfullyintheareasofsteel,automobiles,
andcheapconsumergoods.
Answer: D
Diff:2
PageRef:929
Topic:ThePacificRim:NewJapans?
Skill:Conceptual

635

27) Whichofthefollowingcompaniesexemplifiestheeconomicgrowthandpoliticalinfluenceof
Koreancorporations?
A) Hyundai
B) Sony
C) Mitsubishi
D) Magnavox
Answer: A
Diff:1
PageRef:929
Topic:ThePacificRim:NewJapans?
Skill:Factual

28) In1978theUnitedStatesgovernment
A) recognizedTaiwanasamost-favoredtradingpartner.
B) severeditsdiplomatictieswithTaiwanandrecognizedthelegitimacyofthecommunist
governmentofmainlandChina.
C) offeredsignificantmilitaryaidtoTaiwaninitscontinuingoppositiontocommunismin
Asia.
D) supportedTaiwansinvasionofQuemoyandMatsu.
Answer: B
Diff:2
PageRef:930
Topic:ThePacificRim:NewJapans?
Skill:Factual

29) Taiwansgreatesttradepartnerwas
A) mainlandChina.
B) theU.S.
C) Japan.
D) HongKong.
Answer: C
Diff:3
PageRef:930
Topic:ThePacificRim:NewJapans?
Skill:Factual

636

30) WhendidLeeKuanYew,primeministerofTaiwan,takeoffice?
A) 1955
B) 1965
C) 1975
D) 1985
Answer: B
Diff:2
PageRef:930
Topic:ThePacificRim:NewJapans?
Skill:Factual

31) WhatforeignpowerdidtheChineseCommunistsfightinthe1930sand1940s?
A) TheUnitedStates
B) Russia
C) Japan
D) Britain
Answer: C
Diff:2
PageRef:933
Topic:MaosChinaandBeyond
Skill:Conceptual

32) Whichofthefollowingstatementsismostaccurate?
A) AfterthecivilwarthatbroughttheCommuniststopower,thecountrywasrapidly
demilitarized.
B) ThePeoplesLiberationArmy,thesourceoftheCommunistrisetopower,gained
dominanceovertheCommunistParty.
C) MostofChinawasadministeredbycivilianbureaucratsrecruitedfromtheold
Confucianscholar-gentry.
D) ThearmyremainedclearlysubordinatetotheCommunistParty,withcadreadvisors
attachedtomilitarycontingents.
Answer: D
Diff:2
PageRef:935
Topic:MaosChinaandBeyond
Skill:Conceptual

637

33) WhichofthefollowingstatementsconcerningChineseforeignpolicyduringthe1950sand
1960sisNOTaccurate?
A) TheChineseintervenedmilitarilyintheconflictbetweenNorthandSouthKorea,forcing
theUnitedStatestosettleforastalemateapermanentdivisionofthecountry.
B) TheclosecollaborationbetweentheSovietUnionandChinathatmarkedtheearlyyears
ofMaosrulebrokedown.
C) Chinawasabletoachieveaclosealliancewiththeemergingthirdworldgovernmentin
IndiaasameansofunitingthemajorpowersofAsia.
D) Chinabecamethefirstnon-industrialnationtoexplodeanucleardevice.
Answer: C
Diff:2
PageRef:935
Topic:MaosChinaandBeyond
Skill:Conceptual

34) WhichofthefollowingstatementsconcerningChinesedomesticpoliciesduringthe1950sand
1960sismostaccurate?
A) Despitepledgesmadeduringthecivilwar,theCommunistPartyfailedtoundertake
substantiallandredistributionprograms.
B) Withtheintroductionofthefirstfive-yearplanin1953,theCommunistleadership
turnedawayfromthepeasantry.
C) IncreasinglyMaocametoembracetheoldConfucianconceptofabureaucraticeliteas
themeansofgovernment.
D) Maosprimarytrustcametorestinagroupofintellectualsassociatedwiththe
UniversityofBeijing.
Answer: B
Diff:2
PageRef:935
Topic:MaosChinaandBeyond
Skill:Conceptual

35) Maos1958programofpushingindustrializationthroughsmall -scaleprojectsintegratedinto


thepeasantcommuneswascalled
A) theMayFourthprogram.
B) ThousandFlowersBloom.
C) theGreatLeapForward.
D) theLongMarch.
Answer: C
Diff:2
PageRef:936
Topic:MaosChinaandBeyond
Skill:Factual

638

36) Unlikemostthirdworldcountries,Chinarespondedtoitspopulationprobleminthe
mid-1960sby
A) acceptingUNproposalsforbirthcontrolprograms.
B) limitingurbancouplestotwochildrenandruralcouplestoone.
C) refusingtoadoptanybirthcontrolprogramsinordertofostermorecommunistchildren.
D) allowingunrestrictedbirthsamongthepeasantry,butlimitingurbancouplestoone
child.
Answer: B
Diff:3
PageRef:936
Topic:MaosChinaandBeyond
Skill:Factual

37) By1960,Maolosthispositionasstatechairman
A) becauseofthegeneralandcatastrophicfailureoftheGreatLeapForward.
B) whenhewasassassinatedbyadisgruntledintellectual.
C) whenheproposedthedestructionofBuddhistmonasteriesthroughoutChina.
D) followingthedefeatofChineseforcesinVietnam.
Answer: A
Diff:2
PageRef:937
Topic:MaosChinaandBeyond
Skill:Conceptual

38) WhichofthefollowingpragmatistscametopowerfollowingMaosfallin1960?
A) LiDizhao
B) ChiangKai-shek
C) JiangQing
D) ZhouEnlai
Answer: D
Diff:3
PageRef:937
Topic:MaosChinaandBeyond
Skill:Factual

639

39) WhichofthefollowingprincipleswasNOTpartoftheChineseCommunistPartyattitudes
towardswomen?
A) Womenshouldachievelegalequality
B) Careeropportunitiesshouldbeopentowomen,whoshouldworkoutsideofthe
household
C) Virtueismoreimportantforwomenthanlearning
D) Womenholduphalfoftheheavens
Answer: C
Diff:2
PageRef:937
Topic:MaosChinaandBeyond
Skill:Conceptual

40) Maoslastcampaign,launchedin1965,wasthe
A) LongMarch.
B) CulturalRevolution.
C) GreatLeapForward.
D) MayFourthmovement.
Answer: B
Diff:2
PageRef:938
Topic:MaosChinaandBeyond
Skill:Factual

41) TheGangofFourwere
A) thetemporarilydismissedpragmatists,whosoughttoremoveMaofrompower.
B) Britain,Japan,theSovietUnion,andtheUnitedStates wereidentifiedbyMaoasChinas
chiefrivals.
C) JiangQingandthreeallieswhocontestedforpowerwiththepragmatistsonbehalfof
theagingMao.
D) universityintellectualsidentifiedascapitalistroadersduringtheCulturalRevolution.
Answer: C
Diff:2
PageRef:939
Topic:MaosChinaandBeyond
Skill:Factual

640

42) Duringthe1980s,allofthefollowingweretypicaloftheChinesegovernmentEXCEPT
A) encouragementofprivatemarketproductionforthepeasantry.
B) privateenterprisewithintheindustrialsector.
C) dominationbythepragmatists.
D) democraticreform.
Answer: D
Diff:2
PageRef:940
Topic:MaosChinaandBeyond
Skill:Conceptual

43) InwhichofthefollowingwayswerethepreconditionsforrevolutioninVietnamNOTsimilar
tothoseinChina?
A) PossessionofapolitybasedontheConfuciansystemofChina
B) Failureofimperialgovernmenttorespondtoforeigninterventionleadingtocomplete
collapseofdynasticsystem
C) ActualcolonizationbyaEuropeanpower
D) Radicalratherthangradualistsolutionstoreconstruction
Answer: C
Diff:2
PageRef:940
Topic:ColonialismandRevolutioninVietnam
Skill:Conceptual

44) Fromthe17thcentury,VietnamhadbeenpartofthecolonialsystemofwhatEuropean
power?
A) Britain
B) Portugal
C) Spain
D) France
Answer: D
Diff:1
PageRef:940
Topic:ColonialismandRevolutioninVietnam
Skill:Factual

641

45) TheVietnameserebellionthattoppledboththeTrinhandtheNguyendynastiesofthe18th
centurywasthe
A) Tayson.
B) Boxer.
C) Mekong.
D) ChuLai.
Answer: A
Diff:2
PageRef:940
Topic:ColonialismandRevolutioninVietnam
Skill:Factual

46) TheFrenchintervenedinthe18thcenturyVietnameserebellionandsupported
A) theChuLai.
B) theTayson.
C) NguyenAnh.
D) HongTrinh.
Answer: C
Diff:2
PageRef:940
Topic:ColonialismandRevolutioninVietnam
Skill:Factual

47) Theoutcomeofthe18th-centuryVietnameserebellionwas
A) theunificationofthecountryunderasingleemperoratHue.
B) theexpulsionoftheFrenchfromVietnam.
C) thevictoryoftheTaysonandthecreationofanewdynastyunderChuLai.
D) thedestructionofallrebelsbytheFrenchandtheestablishmentofdirectFrench
administration.
Answer: A
Diff:2
PageRef:940
Topic:ColonialismandRevolutioninVietnam
Skill:Conceptual

48) BywhatdecadedidtheFrenchmanagetoachievecontrolofallofVietnam?
A) 1810s
B) 1840s
C) 1870s
D) 1890s
Answer: D
Diff:2
PageRef:941
Topic:ColonialismandRevolutioninVietnam
Skill:Factual

642

49) InwhatwaywastheearlynationalistorganizationinVietnamsimilartootherthirdworld
nations?
A) ItwascomposedofaWestern-educatedmiddleclass
B) ItwasmodeledontheMarxismoftheCommunistPartyofRussia
C) Itwasdrawnalmostentirelyfromthepeasantry
D) Itsgoalswereentirelypeaceful
Answer: A
Diff:2
PageRef:941
Topic:ColonialismandRevolutioninVietnam
Skill:Conceptual

50) TheVietnamesenationalistsin1954decisivelydefeatedtheFrenchatthebattleof
A) Hue.
B) Hanoi.
C) QangDiep.
D) DienBienPhu.
Answer: D
Diff:2
PageRef:942
Topic:ColonialismandRevolutioninVietnam
Skill:Factual

51) InordertoopposetheCommunistgovernmentofNorthVietnam,theUnitedStatesdecidedto
propupapro-U.S.regimeintheSouthunder
A) HoChiMinh.
B) GeneralGiap.
C) NgoDinhDiem.
D) NguyenTrinh.
Answer: C
Diff:2
PageRef:943
Topic:ColonialismandRevolutioninVietnam
Skill:Factual

643

52) CommunistoppositiontotheU.S.-supportedgovernmentinSouthVietnamwasreferredtoas
the
A) VietCong.
B) VietMinh.
C) VNQDD.
D) Tayson.
Answer: A
Diff:1
PageRef:1943
Topic:ColonialismandRevolutioninVietnam
Skill:Factual

EssayQuestions
1) DiscusssimilaritiesinthepreconditionsofrevolutioninChinaandVietnam.
Answer: Bothsufferedheavilyfromtheassaultsandexploitivetermsofexchangeimposedby
theimperialistpowers;eachcontendedwithunderdevelopment,overpopulation,and
poverty;bothsawtheirancienttraditions,embodiedintheConfuciansystem,collapse
infaceofoutsideinfluenceandfailureofConfucian-styleelitestoorganizeresistance;
bothcountriesgainedlittle,ifanything,fromyearsofEuropeandomination;both
alreadyhad,priortoWesternincursion,astrongsenseofidentity,commonlanguage,
andunifyingpolity.
Diff:2
PageRef:933-945
Topic:Introduction
Skill:Conceptual

2) HowdidMaospoliciesconcerningthepeasantryimpacttheChinesestate?
Answer: Maodeclaredinfavorofpeasantrevolution;earlyemphasisonupliftofpeasantry,
redistributionofland;Maoremaineddistrustfulofsmallelites,continuedtofavor
peasantrevolution;ledtoseriesofpurgeslaterinMaosregime.
Diff:2
PageRef:933-940
Topic:Revolution:TheFirstWave
Skill:Conceptual

3) InwhatwaysdidMaosconceptofapeasantrevolutionleadtospecificshiftsindomestic
policyduringhisrule?
Answer: Following1949victoryoverChiangandNationalists,Maoenactedpeasantland
reforms;in1953CommunistPartyintroducedfirstfive -yearplantoindustrialize
accompaniedbyincreasedstateplanning,growthoftechnocraticbureaucracy;Mao
repudiateddevelopmentofelitebyMassLineapproachin1955,attackonintellectuals
in1957;GreatLeapForwardannouncedin1958intendedtomoveindustrializationto
ruralcommunes;failedby1960;lastattackonelitesinCulturalRevolutionin1965.
Diff:2
PageRef:933-940
Topic:MaosChinaandBeyond
Skill:Conceptual

644

4) InwhatwayshavetheCommunistregimesofVietnamandChinaretainedtiesto
thetraditionalculturesoftheregion?
Answer: Althoughtraditionalimperial,Confuciandynastiesweredestroyed,some
conceptstypicalofancientChinesecultureretained;stillbiasagainst
commercialandbusinessclasses;emphasisonnecessityofrulerstopromotethe
welfareofthemassofthepeople;ideologicalsystemsstresssecular,social
harmony;lackofreligiousemphasis;continuedsenseofculturalsuperiority;
reassertionofelitistthinkingandbureaucraticcontrolinChina;continued
patternsoffamilyandhouseholdfrompast.
Diff:2
PageRef:945
Topic:Conclusion:RevolutionsandCivilizationsinChinaandVietnam
Skill:Conceptual

5) HowdidtheexperienceofdecolonizationinVietnamdifferfromthatofmostthirdworld
nations?
Answer: Mostthirdworlddecolonizationachievedwithoutviolence;Vietnameseexperience
totallyviolent;FrenchrulepromotedVietnamesesenseofseparateidentity;Confucian
traditionregardedFrenchasbarbarians;anyVietnamesewhosupportedFrenchrule
wasregardedasatraitor;notraditionofpeacefulcolonialismasinmostthirdworld
nations;failureofConfucianempiretoresistforeignersledtocompleteabandonment;
leftnoculturaltraditiontodefend;ledtoradicalrevolutionarymeans;nostrong
religiousbasisasculturalunity;Frenchdestroyedbourgeoispoliticalorganizations.
Diff:2
PageRef:940-945
Topic:ColonialismandRevolutioninVietnam
Skill:Conceptual

645

Chapter40 TheEndoftheColdWarandtheShapeofa
NewEra:WorldHistory1990-2006
Multiple-ChoiceQuestions
1) WhichofthefollowingfactorsdidNOTplayaroleinthedemiseofcommunism?
A) Sovietleadership
B) ReassertionofeasternEuropeaninitiative
C) WesternEuropesthrivingEconomy
D) ThesteadfastnessofChinesepolicy
Answer: D
Diff:2
PageRef:949-950
Topic:TheEndoftheColdWar
Skill:Conceptual

2) Anewstrategicarmstreatycalled________wasnegotiatedbetweenthesuperpowersin1979.
A) SALTI
B) SALTII
C) START
D) STOP
Answer: B
Diff:2
PageRef:950
Topic:TheEndoftheColdWar
Skill:Factual

3) WhichofthefollowingwasnotasignificantchangeinChinesepolicyinthelate1970s?
A) Thedecisiontoparticipateintheworldeconomy
B) Thedecisiontorelaxpoliticalcontrols
C) ThedecisiontoadmitmoremarketforcesintoChina
D) ThedecisiontoallowcompetitivefreeenterprisewithinChina
Answer: B
Diff:2
PageRef:950
Topic:TheEndoftheColdWar
Skill:Factual

646

4) Inprotestagainstthe1979invasionofAfghanistanbytheSovietUnion,theU.S.
A) boycottedtheUnitedNations.
B) threatenednuclearwar.
C) invadedIraq.
D) boycottedthe1980MoscowWinterOlympics.
Answer: D
Diff:2
PageRef:950
Topic:TheEndoftheColdWar
Skill:Factual

5) WhatRussianleadersignificantlyalteredpolitical,diplomatic,andeconomicpoliciesinthe
SovietUnionafter1985?
A) YuriAndropov
B) LeonidBrezhnev
C) MikhailGorbachev
D) YuriGagarin
Answer: C
Diff:1
PageRef:951
Topic:TheExplosionofthe1980sand1990s
Skill:Factual

6) Thetermperestroikarefersto
A) anewfreedomtocommentandcriticizetheSovietgovernment.
B) economicrestructuringandmoreleewayforprivateownership.
C) theSovietspaceprogram.
D) theestablishmentofaliberaldemocracy.
Answer: B
Diff:2
PageRef:952
Topic:TheExplosionofthe1980sand1990s
Skill:Conceptual

7) Inwhichofthefollowingnationsdidoutrightviolenceleadtotheoverthrowofan
authoritarianCommunistleader?
A) Czechoslovakia
B) Poland
C) EastGermany
D) Romania
Answer: D
Diff:2
PageRef:953
Topic:TheExplosionofthe1980sand1990s
Skill:Factual

647

8) WhichofthefollowwasNOTafactorleadingtostagnationofindustrialproductioninthe
SovietUnion?
A) Shortageofrawresourcesandmaterials
B) Rigidcentralplanning
C) Healthproblems
D) Poorworkermorale
Answer: A
Diff:2
PageRef:951
Topic:TheEndoftheColdWar
Skill:Factual

9) AllofthefollowingwerereformsofMikhailGorbachevEXCEPT
A) relaxationofmediarestrictions.
B) thedividingoftheSovietUnionintoitsoriginalstates.
C) thereductionofnucleararms.
D) theendingoftheSoviet-Afghaniwar.
Answer: B
Diff:2
PageRef:952-953
Topic:TheEndoftheColdWar
Skill:Factual

10) In1988,Gorbachevencouragedanewparliamentwithinhiscountrycalled
A) theLeagueofStalin.
B) thePeoplesDomain.
C) theCongressofPeoplesDeputies.
D) theDiet.
Answer: C
Diff:2
PageRef:952
Topic:TheEndoftheColdWar
Skill:Factual

11) Inthiscountry,theCommunistPartyrenameditselfSocialistin1988.
A) Hungary
B) Bulgaria
C) Czechoslovakia
D) Romania
Answer: A
Diff:2
PageRef:953
Topic:TheEndoftheColdWar
Skill:Factual

648

12) Thiscountrypushedforeconomicliberalizationin1987butwasheldbackbytheSoviet
centralstate.
A) Hungary
B) Bulgaria
C) Albania
D) Romania
Answer: B
Diff:2
PageRef:953
Topic:TheEndoftheColdWar
Skill:Factual

13) Thiscountryinstalledanon-communistgovernmentin1988.
A) Hungary
B) Romania
C) Poland
D) Albania
Answer: C
Diff:2
PageRef:953
Topic:TheEndoftheColdWar
Skill:Factual

14) Germanreunificationoccurredinwhatyear?
A) 1991
B) 1990
C) 1988
D) 1995
Answer: A
Diff:2
PageRef:953
Topic:TheEndoftheColdWar
Skill:Factual

15) WhichofthefollowingisnotaminoritynationalityregionoftheformerYugoslaviannation?
A) Bosnia
B) Muria
C) Serbia
D) Croatia
Answer: B
Diff:2
PageRef:954
Topic:TheEndoftheColdWar
Skill:Factual

649

16) WhichofthefollowingisnotoneoftheBalticstates?
A) Estonia
B) Latvia
C) Lithuania
D) Ukraine
Answer: D
Diff:2
PageRef:956
Topic:TheEndoftheColdWar
Skill:Factual

17) In1991,BorisYeltsinemergedasheadofwhatmajorSovietRepublic?
A) Belarus
B) Russia
C) Ukraine
D) Georgia
Answer: B
Diff:2
PageRef:956
Topic:TheEndoftheColdWar
Skill:Factual

18) TheformerSovietUniongavewaytothe
A) SovietCommonwealth
B) UnitedStatesofRussia
C) CommonwealthofIndependentStates
D) RussianCommonwealth
Answer: C
Diff:2
PageRef:956
Topic:TheEndoftheColdWar
Skill:Factual

19) WhichofthefollowingwasNOTaconcernwithintheformerSovietUnion?
A) InvasionbytheWest
B) Economiccoordination
C) Controlofthemilitary
D) RelationshipsbetweentheformerSovietstates
Answer: A
Diff:2
PageRef:957
Topic:TheEndoftheColdWar
Skill:Factual

650

20) ThismansucceededYeltsinasleaderofRussia.
A) Tastin
B) Shovin
C) Putin
D) Estin
Answer: C
Diff:2
PageRef:956
Topic:TheEndoftheColdWar
Skill:Factual

21) Theendofthecoldwarwasassociatedwithwhatlargetrendintheworldattheendofthe
20thcentury?
A) Therapidindustrializationofthethirdworld
B) Thespreadofmultipartydemocracy
C) Theendingofworldhunger
D) ThedismantlingofWesternarmedforces
Answer: B
Diff:2
PageRef:957
Topic:TheEndoftheColdWar
Skill:Factual

22) WhichofthefollowingcountriesdidNOTexperiencedemocraticrestructuringinthe
mid-1970s?
A) Spain
B) Germany
C) Portugal
D) Greece
Answer: B
Diff:2
PageRef:957
Topic:TheEndoftheColdWar
Skill:Factual

651

23) Duringthe1990s,whichofthefollowingLatinAmericancountrieswasnotadaptingto
democracy?
A) Argentina
B) Cuba
C) Chile
D) Panama
Answer: B
Diff:2
PageRef:957
Topic:TheEndoftheColdWar
Skill:Factual

24) TheheaviestpopulatedcountryonthecontinentofAfricais
A) SouthAfrica.
B) Nigeria.
C) theIvoryCoast.
D) Tanzania.
Answer: B
Diff:2
PageRef:958
Topic:TheEndoftheColdWar
Skill:Factual

25) Aftertheendofthecoldwar,whichofthefollowingcountriesremainedasuperpower?
A) Russia
B) TheSovietUnion
C) TheUnitedStates
D) GreatBritain
Answer: C
Diff:2
PageRef:963
Topic:TheGreatPowersandNewDisputes
Skill:Factual

26) WhichcountrywasattackedbyIslamicmilitantsonSeptember11,2001?
A) TheUnitedStates
B) China
C) Russia
D) Romania
Answer: A
Diff:2
PageRef:966
Topic:TheGreatPowersandNewDisputes
Skill:Factual

652

27) WhichofthefollowingcountriesdidtheUSinvadein2003?
A) Iran
B) NorthKorea
C) Iraq
D) Russia
Answer: C
Diff:2
PageRef:967
Topic:TheGreatPowersandNewDisputes
Skill:Factual

28) ThePersianGulfWarof1991wasinresponsetotheIraqiinvasionof
A) Israel
B) Kuwait
C) Syria
D) Iran
Answer: B
Diff:2
PageRef:961
Topic:TheGreatPowersandNewDisputes
Skill:Factual

29) ThecommoncurrencyoftheEuropeanUnioniscalledthe
A) dollar.
B) euro.
C) dinero.
D) peso.
Answer: B
Diff:2
PageRef:963
Topic:TheGreatPowersandNewDisputes
Skill:Factual

30) WhobecamepresidentoftheU.S.in2001?
A) AlGore
B) GeorgeW.Bush
C) DickCheney
D) BillClinton
Answer: B
Diff:2
PageRef:966
Topic:TheGreatPowersandNewDisputes
Skill:Factual

653

31) TheattackontheNewYorksWorldTradeCenterin2001ledtotheinvasionofwhatcountry?
A) Iran
B) NorthKorea
C) Kuwait
D) Afghanistan
Answer: D
Diff:2
PageRef:967
Topic:TheGreatPowersandNewDisputes
Skill:Factual

32) TheformercountryofCzechoslovakiahasnowsplitintowhattwonations?
A) BosniaandSlovakia
B) MoldviaandSlovakia
C) TheCzechRepublicandSlovakia
D) CroatiaandSlovakia
Answer: C
Diff:2
PageRef:960
Topic:TheGreatPowersandNewDisputes
Skill:Factual

33) Ontwooccasionsduringthe1990s,theRussianmilitaryhadtoputdownrevoltsinthis
Muslimregion.
A) Ukraine
B) Modavia
C) Chechnya
D) Chernyenko
Answer: C
Diff:2
PageRef:960
Topic:TheGreatPowersandNewDisputes
Skill:Factual

34) WhatcountryexperiencedpoliticalprotestsinTinanmenSquareinJune1989?
A) Russia
B) Britain
C) China
D) TheUnitedStates
Answer: C
Diff:2
PageRef:958
Topic:Globalization
Skill:Factual

654

35) WhatisthenameofthedisputedterritoryaroundwhichIndiaandPakistanhaveclashed?
A) Kashmir
B) Nepal
C) Iraq
D) SouthIndia
Answer: A
Diff:2
PageRef:961
Topic:Globalization
Skill:Factual

EssayQuestions
1) WhatstrainsappearedincommunismineasternEuropethateventuallyledtoitscollapse?
Answer: Sovietleadershipwasconservative,warinAfghanistan,successofwesternEurope,
Chinesemovetowardsmarketeconomy,aggressivepolicyoftheUSunderReagan,and
inabilitytomaintainmilitaryspendingwithastagnatingeconomy.
Diff:2
PageRef:949-951
Topic:TheEndoftheColdWar
Skill:Conceptual

2) HowdidthereformsofMikhailGorbachevleadtothecollapseofcommunismineastern
Europe?
Answer: Glasnostintroducedagreaterfreedom,perestroikaintroducedmarketreformsbut
economystillstagnated,GorbachevallowedeasternEuropetomoveawayfromthe
SovietsandeventuallyoverthrowtheirproSovietregimesin1989,andunderthestrain
ofreformersandinternalrevoltofnationalitiestheSovietUnioncollapsedin1991.
Diff:2
PageRef:951-956
Topic:TheAgeofReform
Skill:Conceptual

3) Inwhatpartsoftheworlddiddemocracyincreasesincethe1970s?
Answer: AuthoritarianregimesinSpain,PortugalandGreececollapsedinthe1970s,
dictatorshipsinLatinAmericabegantocollapseinthe1980s,andcommunism
collapsedinthe1990s,
Diff:2
PageRef:957-960
Topic:TheSpreadofDemocracy
Skill:Conceptual

4) Whatpartsoftheworldcontinuedtoenduremilitaryconflictsattheendofthe20thcentury?
Answer: BreakupofYugoslavia,PersianGulfWar,Israeli-Palestineconflict,tensionsbetween
IndiaandPakistanoverKashmir,andgenocideinRwanda.
Diff:2
PageRef:961-962
Topic:EndemicandEthnicConflicts
Skill:Conceptual

655

5) WhywastheUSattackedonSeptember11,2001?
Answer: RiseofIslamicfundamentalism,hostilitytowardstheWestandparticularlytheUS,the
GulfWarandstationingofUStroopsinSaudiArabia,andUSsupportforIsraeland
authoritarianregimesintheMiddleEast.
Diff:2
PageRef:966-967
Topic:Anti-AmericanTerrorismandResponse
Skill:Conceptual

656

Chapter41 GlobalizationandResistance:WorldHistory
1990-2006
Multiple-ChoiceQuestions
1) NAFTAstandsforthe
A) NorthAtlanticFreeTradeAgreement.
B) NorthAmericanFreeTradeAgreement.
C) NorthAtlanticFreeTradeAmalgamation.
D) NorthAmericanFreeTradeAmalgamation.
Answer: B
Diff:2
PageRef:978
Topic:TheGreatPowersandNewDisputes
Skill:Factual

2) Theincreasedinterconnectednessofallpartsoftheworldiscalled
A) gradualism.
B) universalism.
C) globalization.
D) SpaceshipEarth.
Answer: C
Diff:2
PageRef:971
Topic:Globalization
Skill:Factual

3) WhichofthefollowingisNOTanewdevelopmentregardingtheglobalizationoftheEarth?
A) Advancesincreatingaprototypeuniversallanguage
B) Theendofthecoldwarledtonewopportunitiesforglobalcommunication
C) Advancedtechnicaldevelopments
D) Theadjustmentofpeopleseverywheretothenotionofglobalcommunications
Answer: A
Diff:2
PageRef:971-973
Topic:Globalization
Skill:Factual

657

4) Possiblythemostimportanttechnologicalinnovationincommunicationsinthe1990swould
be
A) thecellularphone.
B) theInternet.
C) thepersonalcomputer.
D) satellitetelevision.
Answer: A
Diff:2
PageRef:972
Topic:Globalization
Skill:Factual

5) TheGermanVolkswagonbugcarisproducedinwhatcountry?
A) TheUnitedStates
B) Mexico
C) Canada
D) Germany
Answer: B
Diff:2
PageRef:973
Topic:Globalization
Skill:Factual

6) Intheyear2000,whatwasthepercentageofAmericanhouseholdsinwhichEnglishwasnot
theprimarylanguage?
A) 50
B) 25
C) 10
D) Five
Answer: B
Diff:2
PageRef:974
Topic:Globalization
Skill:Factual

658

7) Thetextbookmentionsthiscompanyasthemoststrikinginternationalculturalinfluencesince
the1970s.
A) Nike
B) McDonalds
C) Ford
D) Sony
Answer: B
Diff:2
PageRef:974
Topic:Globalization
Skill:Conceptual

8) NorthernMexicoembracedtheAmericanholidayofHalloween,displacingwhattraditional
Catholicobservance?
A) Passover
B) Easter
C) AllSaintsDay
D) Theconfessional
Answer: C
Diff:2
PageRef:975
Topic:Globalization
Skill:Factual

9) Onthewhole,politicalinstitutionsglobalizedatwhatratecomparedtotechnologyor
business?
A) Ataboutthesamepace
B) Lessrapidly
C) Morerapidly
D) Politicalinstitutionsneverglobalize
Answer: B
Diff:2
PageRef:978
Topic:Globalization
Skill:Factual

659

10) WhichofthefollowingisNOTanissueraisedagainsteconomicglobalization?
A) Damagetolaborconditionsthroughuseofcheapworkers
B) Damagetotheenvironment
C) Itspaceistooslow
D) Rampantconsumerism
Answer: C
Diff:2
PageRef:976-977
Topic:Globalization
Skill:Conceptual

11) WhichofthefollowingisNOTatrendrunningcountertoglobalization?
A) Nationalism
B) Religiousdifferences
C) Ethniccompetition
D) TheInternet
Answer: D
Diff:2
PageRef:979-981
Topic:Globalization
Skill:Factual

12) Whatreligionspreadrapidlyinthe1990sthroughoutLatinAmerica?
A) Hinduism
B) Protestantfundamentalism
C) Islam
D) OrthodoxChristianity
Answer: B
Diff:2
PageRef:979
Topic:Globalization
Skill:Factual

13) AmajorconferenceinKyoto,Japan,setlimitson
A) greenhousegasemissions.
B) communism.
C) racism.
D) politicalfundamentalism
Answer: A
Diff:2
PageRef:984
Topic:Globalization
Skill:Conceptual

660

14) Bytheendofthelastcentury,whatpercentageofhumanityconsumedawhoppingfour -fifths


ofallmarketedgoodsandservices?
A) Four-fifths
B) One-half
C) Two-thirds
D) One-fifth
Answer: D
Diff:2
PageRef:981
Topic:Globalization
Skill:Conceptual

EssayQuestions
1) Whatisglobalizationandwhataretheprocessesthathaveledtoglobalization?
Answer: Globalizationistheincreasinginterconnectednessofallpartsoftheworld.Political,
demographic,culturalandtechnologicalchanges.Therehasbeenanintensificationof
internationalcontacts(moreextensivetrade,worldwidealliancesystems,cultural
exchanges).
Diff:2
PageRef:971-978
Topic:ANewPeriodinWorldHistory
Skill:Conceptual

2) Whattrendsinpoliticalorganizationandeconomicdevelopmentcanbeidentifiedwith
globalization?
Answer: MajorindustrialnationsliketheUnitedStatesandorganizationssuchastheWorld
BankandInternationalMonetaryFundhavepromotedglobalization.Economictrends
basedonindustrialization;gapsremainwidebasedonlevelofindustrializationand
sophisticationoftechnology;createdimmigration,impactonurbanization.
Diff:2
PageRef:973-978
Topic:ExtrapolatingTrendsinWorldSocieties
Skill:Conceptual

3) Inwhatwayshasglobalizationofferedevidenceofhumanprogress?Whatproblemshasit
produced?
Answer: Progress:advancesintechnologicalsophisticationandscientificknowledgeallows
abilitytomanagehumanandnaturalenvironment,withbettermeanstopreserve
healthandimprovelifeexpectancyandincreaseinconsumergoods.Problemsinclude
disparitiesinwealth,anddestructionofnationalculturesandtheenvironment.
Diff:2
PageRef:971-984
Topic:OptimismorPessimism
Skill:Conceptual

661

4) Theauthorspostulateseveralpotentialcausalfactorsimpactingcivilizationinthefuture,
includingpopulationgrowth,theexhaustionoffrontiers,andtechnologicaladvances
associatedwiththepostindustrialworld.Evaluatetheimpactsuchfactorsarelikelytohave
onthefuture.
Answer: In1960sand1970s,therewaswidespreadconcernthatpopulationgrowthwould
outstripproductionoffoodandresourcesandleadtoenvironmentaldisasterand
warfare;dropinbirthrateshascausedthisproblemtobediscountedby1990salthough
birthratesremaindangerouslyhigh;endoffrontiersmeansgreaterpotentialfor
friction,seennowinhostilitytowardimmigrants;nomorepopulationmigrations
possible,leadingtopotentialconflictoverspace.Technologicaldevelopments
(computers,geneticengineering,robotics)linkedtocreationofpostindustrialsociety
mosttypicalofadvancedindustrialstates;ledtoservice-orientedeconomywith
machinesperformingmostindustrialtasksandmuchofagriculturalproductionaswell;
fostersagenerallyoptimisticviewofindustrializedsociety,thoughcriticsemphasize
increaseininequalitiesinworldeconomy;othersarguethattechnological
transformationsarenotfundamentalbutmerelyreinforceexistingtrends.
Diff:2
PageRef:984-986
Topic:ForecastingbyaSingleCausalFactor
Skill:Conceptual

5) Discusstheeconomicandculturalaspectsofglobalization.
Answer: Speedoftransportationandcommunication,worldtrade,developmentofinternational
scientificcommunity;internationalartisticstyles,popularityofWesternfashions,fads,
andsports.
Diff:2
PageRef:971--981
Topic:Regions,Civilizations,andWorldForces
Skill:Conceptual

662

You might also like